Download as pdf or txt
Download as pdf or txt
You are on page 1of 498

CHARTERED ACCOUNTANCY PROFESSIONAL II

(CAP-II)

Compilation of Suggested Answers

Paper 7: Income Tax and VAT


(June 2001 - Dec 2019)
Updated as per amendments by Finance Act 2077/78

Education Division
The Institute of Chartered Accountants of Nepal
Publisher: The Institute of Chartered Accountants of Nepal
ICAN Marg, Satdobato, Lalitpur, P.O. Box: 5289
Tel: 977-1-5530832, 5530730, Fax: 977-1-5550774
E-mail: [email protected], Website: www.ican.org.np

© The Institute of Chartered Accountants of Nepal

This compilation of suggested answers is prepared by the Institute of Chartered Accountants of


Nepal. Permission of the Council of the Institute is essential for reproduction of any portion of this
paper.

All rights reserved. No part of this publication may be reproduced stored in a retrieval system, or
transmitted, in any form, or by any means, electronic, mechanical, photocopying, printing,
recording or otherwise, without prior permission, in writing, from the publisher.

The compilation of suggested answers is prepared by the Institute with a view to assist the students
of ICAN in their study. The suggested answers presented here are indicative and not exhaustive.
Students are expected to apply their knowledge and write the answer in the examinations taking the
suggested answers as guidance.

Due care has been taken to compile the suggested answers. In case students need any clarification,
creative feedbacks or suggestions for the further improvement on the material, any error or
omission on the material, they may report to Education Division of the Institute.

September 2020
Education Division
The Institute of Chartered Accountants of Nepal
Compilation of Suggested Answers Income Tax

Part A Income Tax


Basic Concept ............................................................................................................................................... 4
Chapter 1: Taxation System in Nepal ........................................................................................................... 4
Chapter 2: Preliminary .................................................................................................................................. 5
Residence Concept .............................................................................................................................. 11
Concept of Non-Business Chargeable Asset ....................................................................................... 16
Concept of Permanent Establishment ................................................................................................. 19
Concept of Tax-Exempt Entity ............................................................................................................ 22
Chapter 3: Basis of Taxation ....................................................................................................................... 24
Concept of Person chargeable to Tax ................................................................................................. 25
Concept of Adjusted taxable Income ................................................................................................... 26
Income Heads Chargeable to Income Tax .......................................................................................... 28
Identification of Source of Payments .................................................................................................. 30
Chapter 4: Calculation of Tax and Tax Rates ............................................................................................. 31
Tax on Repatriation of Income............................................................................................................ 31
Presumptive Tax.................................................................................................................................. 32
Tax Computation- General ................................................................................................................. 34
Chapter 5: Exemptions, Concessions, Reductions, Deductions, Set off ..................................................... 35
Exempt Amounts and Extent of Exemptions ........................................................................................ 35
Income Exempt from Tax under Section 11 ........................................................................................ 42
Concessions in Tax Rates.................................................................................................................... 44
Tax Holiday and Reduced Rates ......................................................................................................... 54
Set off of Losses ................................................................................................................................... 55
Others.................................................................................................................................................. 60
Chapter 6- Characterization, Allocation and Quantification....................................................................... 62
Quantification of Perquisites (Section 27) .......................................................................................... 62
Exchange Gain Loss ........................................................................................................................... 65
Joint Investment .................................................................................................................................. 66
Transfer Pricing, Income Splitting and GAAR ................................................................................... 69
Computation of Taxable Income................................................................................................................. 72
Mix of all Income Heads ..................................................................................................................... 72

© The Institute of Chartered Accountants of Nepal 1


CAP II Paper 7 Income Tax and VAT

Chapter 7: Income from Business ............................................................................................................... 81


Inclusions ............................................................................................................................................ 81
Determination of Assessable Income .................................................................................................. 81
Tax Liability of Specialized Business ................................................................................................ 160
Deduction of General Expenses and Concept of Non-deductibility.................................................. 164
Interest Expenses and Concept of ATI for Sec. 14 (2) ...................................................................... 168
Cost of Trading Stock........................................................................................................................ 173
Repair and Improvement Expenses ................................................................................................... 174
Pollution Control Cost ...................................................................................................................... 175
Depreciation ..................................................................................................................................... 176
Chapter 8: Income from Employment ...................................................................................................... 190
Chapter 9: Income from Investment ......................................................................................................... 247
Chapter 10: Tax Credits ............................................................................................................................ 265
Foreign Tax Credit ........................................................................................................................... 265
Relevant Matters in Taxable Income ........................................................................................................ 273
Chapter 11- Tax Accounting: Methods and Timing ................................................................................. 273
Chapter 12- Long Term Contract .............................................................................................................. 280
Chapter 13- Gain on Disposal of Asset and Liability ............................................................................... 283
Chapter 14- Other Matters ........................................................................................................................ 292
Taxpayers’ Right and Taxpayers’ Identification............................................................................... 292
Donations .......................................................................................................................................... 293
Special Provisions related to Natural Person & Entity .................................................................... 293
Retirement Funds, Finance Lease, etc. ............................................................................................. 295
Payment of Tax and Tax Return ............................................................................................................... 296
Chapter 15: Form, Place and Time of Payment ........................................................................................ 296
Time of Payment of Tax .................................................................................................................... 296
Chapter 15: Withholding Taxes and Collection of Advance Tax by Agent ............................................. 297
Collection of Advance Tax by Agent ................................................................................................. 297
Withholding Tax ................................................................................................................................ 297
Installment Tax.................................................................................................................................. 320
Tax Assessment ................................................................................................................................. 322
Collection of Tax in Arrears ............................................................................................................. 327

© The Institute of Chartered Accountants of Nepal 2


Compilation of Suggested Answers Income Tax

Income Returns ................................................................................................................................. 328


True False Questions................................................................................................................................. 332
Short Notes Questions............................................................................................................................... 337

© The Institute of Chartered Accountants of Nepal 3


CAP II Paper 7 Income Tax and VAT

Basic Concept

Chapter 1: Taxation System in Nepal


This Chapter includes:

• Concepts and History of Tax


• Objectives of Taxation
• Types of Taxes in Nepal
• Concept of Income and Meaning of Income Tax
• History of Income Tax in Nepal
• Defects of Previous Income Tax Acts
• Relations with Constitution and Components of tax Laws
• Income Tax Act, 2058: New Era of Income Tax

1. Distinguish between Direct Tax and Indirect Tax.


(Dec 2003, 3 Marks, CA Inter)
Answer
If a taxpayer knows the actual tax obligation and makes direct tax payment to government is
direct tax, example- income tax, land registration tax, etc. Under direct taxation, a taxpayer
known to tax authority is same as the taxpayer who actually bears tax liability.

If any tax is collected using an agent and the actual taxpayer does not know how much tax he
has paid, it is called indirect tax. Example, VAT, Excise Duty, etc. Under indirect taxation, a
taxpayer defined by the law is different than the taxpayer who actually bears tax liability.

© The Institute of Chartered Accountants of Nepal 4


Compilation of Suggested Answers Income Tax

Chapter 2: Preliminary
This Chapter includes:
• Extent of Income Tax Act, 2058
• Definition of Words and Legal Phrase
• Meaning of different words
2. Define the following terms as per Income Tax Act, 2058.
a. Company (June 2017, 2.5 Marks)
Answer:
Company means a company incorporated under prevailing company law and for the
purpose of tax, the term also includes:
1. Organized institutions established under prevailing law,
2. Any unincorporated association, committee, union or society, or registered or
unregistered group of persons other than Private firm and partnership, or a trust,
3. Partnership firm comprising 20 or more partners that are registered or not registered
under prevailing law, retirement fund, cooperative, unit trust, joint venture,
4. Foreign companies, and
5. Any other foreign entity prescribed by the Director-General

b. Natural person (June 2017, 2.5 Marks; Dec 2007, 2 Marks, CA Inter)
Answer:
Natural person means an individual, and the term also includes:
1. Married spouses electing Couple assessment under Sec. 50 (1) of the Act,
2. Widower or widow deemed to be treated as Couple under Sec. 50 (3) of the Act, and
3. Private Firm of an Individual.

c. Permanent establishment
(Dec 2015, 5 Marks; Dec 2009, 2 Marks, CA Inter; June 2003, 5 Marks, CA Inter)
Answer:
Permanent Establishment (PE) means any place where a person carries on its business,
and the term also includes the following:
1. Any place from where a person carries on business through an agent, who is not a
general agent of independent status (Agency PE),
2. Any place where a person has, is using or is installing main equipment or machinery
(Fixed Base PE),
3. one or more places of a country where a person provides technical, business or
consultancy services through employee or otherwise for more than 90 days in any 12
months period (Service PE)
4. a place where a person is engaged in a construction, assembly, or installation project
for 90 days or more, including a place where a person is conducting supervisory
activities in relation to such a project (Fixed Site PE)

© The Institute of Chartered Accountants of Nepal 5


CAP II Paper 7 Income Tax and VAT

d. Residential status of an individual (June 2019, 2.5 Marks; Dec 2015, 5 Marks)
An individual is resident of Nepal, if any of the following three conditions is satisfied:
1. His/her habitual place of abode is in Nepal,
2. S/he stays in Nepal for 183 days or more during any period of consecutive 365 days,
or
3. S/he is employee of Government of Nepal, deputed by the employer in any foreign
country during the Income Year.

Habitual Place of abode is such place where the person’s major economic activities are
carried out.

Income Tax Manual issued by Inland Revenue Department interprets a period of


consecutive 365 days as Income Year, as such, if any person stays in Nepal for 183 days
or more during any Income year, the person is resident of Nepal during the Income Year.

e. Payment (July 2015, 5 Marks)


“Payment” means the following acts:
1. the transfer by one person of an asset or money to another person or the transfer by
another person of a liability to the one person;
2. the creation by one person of an asset that on creation is owned by another person or
the decrease by one person of a liability owed by another person,
3. the provision by one person of services to another person; and
4. the making available of an asset or money owned by one person for use by another
person or the granting of use of such an asset or money to another person,

f. "Business Asset'", "Depreciable Asset" and "Trading Stock" (July 2015, 5 Marks)
Business Asset
“Business Asset” means an asset used in a business, but excludes trading stock or a
depreciable asset of a business.

Depreciable Asset
“Depreciable Asset” means an asset to the extent to which it is used in the production of
income from a business or investment and that is likely to lose value because of wear and
tear, obsolescence, or the passing of time. Provided that, the term shall not include any
trading stock.

Trading Stock
“Trading stock” means assets owned by a person that are intended to be sold in the
ordinary course of a business conducted by the person, work in progress on such assets,
and inventories of materials to be incorporated into such assets. The term excludes asset
denominated in foreign currency.

© The Institute of Chartered Accountants of Nepal 6


Compilation of Suggested Answers Income Tax

g. Final Withholding Tax


(Dec 2012, 3 marks, CA Inter; June 2006, 2 Marks, CA Inter)
“Final Withholding Payments” means dividend, rent, gain, interest and payments made to
non-resident person that are subject to withholding taxes as referred to in Section 92.

The taxes paid on final withholding payments are generally understood as Final
withholding tax.

h. Retirement Fund (Dec 2012, 2 Marks, CA Inter)


“Retirement Fund” means an entity established only with the objective of accepting and
investing retirement fund contributions for the purpose of making retirement fund
payments to the entity's beneficiary natural persons or their dependents.

i. Resident Person (June 2011, 5 Marks, CA Inter; June 2003, 5 Marks, CA Inter)
“Resident Person” mean the following persons in respect of an income year:
1. In respect of a natural person:
(a) Place of habitual abode (normal place of abode) is in Nepal,
(b) Who is present in Nepal for 183 days or more in a period of 365 consecutive days,
or
(c) who is an employee of Government of Nepal deputed by the employer in any foreign
country at any time during an income year.
2. Partnership firm.
3. In respect of a trust, such trust which:
a. Is established in Nepal,
b. The trustee of the trust is a resident person in the income year; or
c. The trust is controlled in the income year by a resident person, or by a group of
persons including him, directly or indirectly through of one or more interposed
entities.
4. In respect of a company, a company:
a. Which is incorporated under the law of Nepal, or
b. the management of which is effective in Nepal in an income year
5. Government of Nepal or Province government
6. Rural Municipality, Urban Municipality, or District Coordination Committee
7. In respect of such entity of a foreign government or a political subdivision of the foreign
government, such an entity-
a. which is established under the law of Nepal, or
b. the management of which is effective in Nepal in an income year
8. An institution or entity established under a treaty or agreement, and
9. A foreign permanent establishment situated in Nepal of a non-resident person.

© The Institute of Chartered Accountants of Nepal 7


CAP II Paper 7 Income Tax and VAT

j. Disabled person (Dec 2010, 3Marks, CA Inter)


“Disabled person” means an individual who, by reason of mental or physical illness, is
incapable of managing their affairs.

k. Non-Business Chargeable Asset


(June 2004, 5 Marks, CA Inter; Dec 2010, 3Marks, CA Inter)
Non-Business chargeable Asset” means land, building and interest in any entity, or
securities, other than the following assets:
1. Business asset, depreciable asset or trading stock,
2. Private building of a natural person in the following conditions:
(a) having owned it for ten or more years continuously, and
(b) resided therein for ten or more years continuously or intermittently by such person.

Clarification: For the purpose of this Clause, “Private building” means a building and
lesser of land area occupied by the building or one ropani of such area occupied by
the building.

3. An interest of a beneficiary in a retirement fund,


4. Land, Land and Building and private building of a natural person that has been
disposed of at a price of less than 1 million rupees, or
5. Assets disposed of through transfer by any means within three generations, other than
sale and purchase.

l. Long Term Contract (Dec 2010, 3Marks, CA Inter)


“Long-Term Contract” means a contract as referred to in Section 26 with term more than
12 months.
For the purpose of Section 26, “long-term contract” means a contract of following
conditions
a. the term of which exceeds 12 months, and
b. that is either a contract for manufacture, installation, or construction, or, in relation
to each, the performance of related services; or a contract with a deferred return that
is not an excluded contract.

m. Entity (June 2010, 5 Marks, CA Inter)


“Entity” means the following institutions and organizations:
1. Partnership, trust, or company;
2. Rural Municipality, Urban Municipality, or District Coordination Committee;
3. Government of Nepal, Provincial Government or local government

© The Institute of Chartered Accountants of Nepal 8


Compilation of Suggested Answers Income Tax

4. Public International Organization established under any foreign government, or the


provincial or local government functioning under such a government, or through a
treaty; or
5. A permanent establishment of the institution or organization referred to in Sub-Clause
(1), (2), (3) and (4) that is not situated in the country where it is resident.

n. Exempt Entity (Dec 2011, 5 Marks, CA Inter)


“Exempt organization” means the following entities:
1. Following entities registered with the Department as an exempt organization:
a. Social, religious, educational or charitable organizations of a public character
established without profit motive,
b. An amateur sporting association formed for the purpose of promoting social or
sporting related amenities not involving the acquisition of gain by its member,
2. Political parties registered with the Election Commission.

Provided that, any entity, giving benefit to any person from the assets of, and amounts
derived by the entity except in pursuit of the entity’s function as per its objectives or as
payment for assets or services rendered to the entity by the person, is not exempt from tax.

o. Lease (June 2008, 1 Mark, CA Inter; June 2006, 2 Marks, CA Inter)


“Lease” means a temporary right of a person to use the assets, other than money, of
another person, and the term includes a license, rental agreement, option, royalty
agreement, or tenancy.

p. Service Fee (June 2008, 1 Mark, CA Inter)


“Service Fee” means any payment to any person, according to the market value, for the
service rendered by such a person and the term also includes commission, meeting fee
(allowance), management fee, or technical service fee.

q. Income Year (Dec 2007, 2 Marks, CA Inter)


“Income Year” means the period beginning on Shrawan 1 of a year and ending on the last
day of Ashad of the next year.

© The Institute of Chartered Accountants of Nepal 9


CAP II Paper 7 Income Tax and VAT

r. Turnover (Dec 2007, 2 Marks, CA Inter)


“Turnover (transaction)” means a turn-over equivalent to the total amount to be included
under Section 7, 8 or 9 for purposes of computing the income from business, employment
or investment during any income year.

s. Assessable Income (Dec 2007, 2 Marks, CA Inter)

Provision of Section 6:
Subject to Income Tax Act, the assessable income of a person for an Income year from any
employment, business, or investment, or windfall gain shall be as follows:
a. In respect of a resident person, the person’s income from business, employment,
investment, or windfall gain irrespective of the source of such income, and
b. In respect of a non-resident person, the person’s income from business, employment,
investment or windfall gain from income having source in Nepal.
Provided that, the assessable income does not include any income exempt under section 11
or 64 or both.

t. Underlying Ownership(entity) (Dec 2007, 2 Marks, CA Inter)

“Underlying Ownership” means ownership as referred to below:


1. In respect to an entity, the ownership created in the entity on the basis of an interest
held in it directly or indirectly by any natural person, or by an entity in which no natural
person has any interest, through one or more interposed entities.
2. In respect to an asset under the ownership of an entity, ownership of the asset
determined on the basis of the proportionate ownership of persons having underlying
ownership of the entity.

3. State the entities other than those registered under the Companies Act that is treated as
company under the Income Tax Act. (June 2009, 5 Marks)
Answer:
The following entities other than those registered under the Companies Act are treated as
company for the purpose of Income Tax Act:
a. Organized institutions established under prevailing law,
b. Any unincorporated association, committee, union or society, or registered or unregistered
group of persons other than Private firm and partnership, or a trust,
c. Partnership firm comprising 20 or more partners that are registered or not registered
under prevailing law, retirement fund, cooperative, unit trust, joint venture,
d. Foreign companies, and
e. Any other foreign entity prescribed by the Director-General

© The Institute of Chartered Accountants of Nepal 10


Compilation of Suggested Answers Income Tax

4. Distinguish between: (Dec 2005, 2.5 Marks, CA Inter)


• “Royalty” and “Payment for natural sources”
Royalty is lease payment for using or not using intangible assets. However, payment for
natural resource is the lease payment that may base on the quantity of resources extracted
from land, including water, minerals and living and non-living remains.

• “Market Value" and "Cost"


Market value is determined by the market forces and has no any connection with the cost.
Cost is the actual outflow of resources in respect of expense or asset.

• "Underlying ownership" and "Ownership"


Ownership may be direct ownership or underlying ownership. Underlying ownership is a
sub-set of ownership.

Residence Concept
5. Nepal Telecommunications Authority (NTA) hired Mr. Eric, a German Citizen as Rural
Communication Infrastructure Expert with effect from September 17, 2018. He came to
Nepal, 7 days before his joining date on September 10, 2018. He left Nepal on December
20, 2018 and came back to Nepal after Christmas leave on January 9, 2019, and continued
his service for the contract period. After the expiry of contract, he converted his visa
category into Tourist Visa, and went to Annapurna Base Camp for 17 days, and returned
to his home country on April 2, 2019. Comment about the residential status of Mr. Eric.
(June 2019, 5 Marks)
Answer:
Step 1: Conditions for being Resident
An individual is resident of Nepal, if any of the following three conditions is satisfied:
a. His/her habitual place of abode is in Nepal,
b. S/he stays in Nepal for 183 days or more during any period of consecutive 365 days, or
c. S/he is employee of Government of Nepal, deputed by the employer in any foreign country
during the Income Year.

Habitual Place of abode is such place where the person’s major economic activities are
carried out.

Income Tax Manual issued by Inland Revenue Department interprets a period of consecutive
365 days as Income Year, as such, if any person stays in Nepal for 183 days or more during
any Income year, the person is resident of Nepal during the Income Year. For the purpose of
determining 183-days, ‘travel for’ period and date of entry and exit regardless of time spent
in Nepal is counted.

© The Institute of Chartered Accountants of Nepal 11


CAP II Paper 7 Income Tax and VAT

Step 2: Testing of Given Facts with the Condition

Mr. Eric’s habitual place of abode is not easily determinable, as such, we will test his physical
presence in Nepal to determine his residential status in Nepal.

Computation of days stay during Income Year 2018/19 (2075/76):

• First stay, September 10 to December 20: 22 days in September plus 31 days in October
plus 30 days in November plus 20 days in December
• Subsequent Stay: January 9 to April 2: 23 days in January plus 28 days in February plus
31 days in March plus 2 days in April

Total day-stay in Nepal: 186 days during Income Year 2075/76.

Step 3: Conclusion

Therefore, Mr. Eric is resident in Nepal during Income Year 2075/76.

6. A company operating in Bhutan is engaged in production of Jam products. It conducted


its AGM at Thimpu and elected its board members. The majority of elected board
members represent from Nepal. During FY 20X4/X5, most of the board meetings took
place in Kathmandu. The company has earned Rs. 10,000,000 from Bhutan, Rs.
30,000,000 from India, Rs. 20,000,000 from Bangladesh and Rs. 10,000,000 from Nepal
during FY 20X4/X5. Decide the residential status and taxable income of the Company as
per the provision of Income Tax Act, 2058? (Dec 2018, 5 Marks)
Answer:
A company is resident of Nepal, if any of the following two conditions is satisfied:
i. Incorporation Test: in case it is incorporated under prevailing law of Nepal, or
ii. Effective Management Test: In case the effective management of the company is in
Nepal during any Income Year.

The company does not satisfy incorporation test.

To determine whether the company has effective management in Nepal or not, we need to
understand what effective management is.

As per Organization for Economic Cooperation and Development, the place of effective
management is the place where key management and commercial decisions that are
necessary for the conduct of the enterprise’s business are in substance made. The place of
effective management will ordinarily be where the most senior person or group of persons
(for example a board of directors) makes its decisions, the place where the actions to be
taken by the enterprise as a whole are determined; however, no definitive rule can be given
and all relevant facts and circumstances must be examined to determine the place of

© The Institute of Chartered Accountants of Nepal 12


Compilation of Suggested Answers Income Tax

effective management. An enterprise may have more than one place of management, but it
can have only one place of effective management at any one time.

In the given case, the fact that most of the board meetings are held in Kathmandu demonstrates
that Nepal is the place where key management and commercial decisions that are necessary
for the conduct of the enterprise’s business are in substance made. As such, the company’s
effective management is in Nepal during the Income Year.

Therefore, the company is resident.

7. Mrs. Aang Futi has stayed in Nepal in the following periods. Find out the residential
status of her for the income years 20X2/X3 and 20X3/X4. (June 2017, 5 Marks)
Period of Stay
20X1 Falgun 14 to Chaitra 11
20X2 Baishakh 13 to Ashad 26
20X2 Shrawan 3 to Bhadra 25
20X2 Marga 8 to Falgun 13
20X3 Jeshtha 18 to Bhadra 16

The number of days in the relevant months is as follows:


Year (BS) Month No. of Year (BS) Month No. of
Days Days

20X1 Falgun 30 20X2 Poush 30

Chaitra 30 Magh 29

20X2 Baishakh 31 Falgun 30

Jeshtha 32 Chaitra 30

Ashad 31 20X3 Baishakh 31

Shrawan 32 Jeshtha 32

Bhadra 31 Ashad 31

Ashwin 30 Shrawan 32

Kartik 30 Bhadra 31

Marga 29

© The Institute of Chartered Accountants of Nepal 13


CAP II Paper 7 Income Tax and VAT

Answer:
An individual is resident of Nepal, if any of the following three conditions is satisfied:
a. His/her habitual place of abode is in Nepal,
b. S/he stays in Nepal for 183 days or more during any period of consecutive 365 days,
or
c. S/he is employee of Government of Nepal, deputed by the employer in any foreign
country during the Income Year.

Habitual Place of abode is such place where the person’s major economic activities are
carried out.

Income Tax Manual issued by Inland Revenue Department interprets a period of consecutive
365 days as Income Year, as such, if any person stays in Nepal for 183 days or more during
any Income year, the person is resident of Nepal during the Income Year. For the purpose of
determining 183-days, ‘travel for’ period and date of entry and exit regardless of time spent
in Nepal is counted.

Computations of days stay during Income Year 20X1/X2:

a. First stay, 20X1 Falgun 14 to Chaitra 11: 17 days in Falgun and 11 days in Chaitra
b. Subsequent Stay:20X2 Baishakh 13 to Ashad 26: 19 days in Baisakh, plus 32 days in
Jestha plus 26 days in Ashad

Total day-stay in Nepal: 105 days during 20X1/X2

Computations of days stay during Income Year 20X2/X3:

a. First stay, 20X2 Shrawan 3 to Bhadra 25: 30 days in Shrawan plus 25 days in Bhadra
b. Subsequent Stay: 20X2 Marga 8 to Falgun 13: 22 days in Marga, plus 30 days in Poush,
plus 29 days in Magh plus 13 days in Falgun
c. Subsequent Stay: 20X3 Jeshtha 18 to Ashad end:15 days in Jestha plus 31 days in Ashad

Total day-stay in Nepal: 190 days during 20X2/X3

Therefore, Mrs. Ang Futi is non-resident during 20X1/X2, since none of the conditions for
being resident are satisfied and is resident during 20X2/X3, since she stayed for more than 183
days.

8. Discussing the relevant provisions of Income Tax Act, 2058, Determine the residential
status and taxability of incomes in Nepal of below persons (calculation of Tax Liability is
not required): (Dec 2016; 2.5+2.5=5)
• Mr. Shyam Khadka, a Nepali Citizen, had left Nepal on 1 Chaitra 20X2 to work as
st

a tour guide in United States of America. His Company in United States of America

© The Institute of Chartered Accountants of Nepal 14


Compilation of Suggested Answers Income Tax

sent him Nepal on 1st Baishak 20X3 with a group of tourist. He spent 30 days in Nepal,
and returns back to USA. His monthly remuneration in USA is equivalent to Nepalese
Rs 5,00,000 per month. Determine the residential status and taxability of income of
Mr. Shyam Khadka in Nepal.
Answer:
An individual is resident of Nepal, if any of the following three conditions is satisfied:
i. His/her habitual place of abode is in Nepal,
ii. S/he stays in Nepal for 183 days or more during any period of consecutive 365 days, or
iii. S/he is employee of Government of Nepal, deputed by the employer in any foreign
country during the Income Year.

As Mr. Shyam Khadka stayed in Nepal for more than 183 days during 20X2/X3 (full period
until Falgun and 30 days after Chaitra until Ashad), Mr. Khadka is resident of Nepal.

Mr. Khadka is liable to pay tax on his worldwide income under Sec. 1 (1) or 1 (2) of
Schedule 1 of the Act.

• Mr. Ram Bansal, an India citizen, has joined a Cement Industry in Nepal as a Chief-
Engineer Maintenance on 1st Mangsir 20X2 at a monthly gross remuneration of Rs.
1,50,000. While in India, he has been working for a similar cement Company for a
gross monthly remuneration equivalent to Nepalese Rs. 1,00,000 per month.
Determine the residential status and taxability of income of Mr. Ram Bansal in Nepal.
Answer:
An individual is resident of Nepal, if any of the following three conditions is satisfied:
i. His/her habitual place of abode is in Nepal,
ii. S/he stays in Nepal for 183 days or more during any period of consecutive 365 days, or
iii. S/he is employee of Government of Nepal, deputed by the employer in any foreign
country during the Income Year.
As Mr. Ram Bansal stayed in Nepal for more than 183 days during 20X2/X3 (full period
after 1st Mangsir 20X2, 9 Months, >270 days), Mr. Bansal is resident of Nepal.

Mr. Bansal is liable to pay tax on his worldwide income under Sec. 1 (1) or 1 (2) of
Schedule 1 of the Act.

9. Dr. Koirala, a senior cardiologist working in a private hospital of Kathmandu, left Nepal
for London on 1st Ashwin 20X1 of the Income Year. His visit to London was unofficial.
Before leaving Nepal, the following agreement was signed between him and the hospital.
• No salaries for non-duties
• Maximum leave period approved = one year
As per the agreement signed, the validity will remain until the end of Bhadra, 20X2.
However, he returned back to Nepal before the validity has expired as on 1 st Baisakh,

© The Institute of Chartered Accountants of Nepal 15


CAP II Paper 7 Income Tax and VAT

20X2 and joined the duty on the same date. He was drawing Rs. 40,000 as a gross salary
per month in Nepal during the year.
As on Ashad end 20X2, Dr. Koirala received £3,000 (after TDS of £2,000) in Nepal
through western union money transfer as consideration payment made by the London
hospital against the service provided by him. The exchange rate £ 1 = NPR 153.
You are required to determine the following with reference to Income Tax Act, 2058.
(June 2016, 5 Marks)
a. Residential Status for the Income Year 20X1/X2
Answer
An individual is resident of Nepal, if any of the following three conditions is satisfied:
i. His/her habitual place of abode is in Nepal,
ii. S/he stays in Nepal for 183 days or more during any period of consecutive 365 days, or
iii. S/he is employee of Government of Nepal, deputed by the employer in any foreign
country during the Income Year.

Test 1: Habitual place of abode cannot be easily determined

Test 3: Not a government employee

Test 2: He has stayed in Nepal for less than 183 days (5 Months, Shrawan 20X1, Bhadra
20X1, Baisakh 20X2, Jestha 20X2, Ashad 20X2. Day’s Stay test also not satisfied.

Therefore, Dr. Koirala is non-resident in Nepal.

b. Tax Liability
Answer
He received Rs. 200,000 from private hospital in Nepal (income having source in Nepal),
which is taxable @ 25%, i.e. tax liability is Rs. 50,000 for the year.

The amount received for the service provided in the UK is not taxable in Nepal, since he is
non-resident and the amount are derived is not having source in Nepal [Joint reading of
Sec. 6 and Sec. 67 (6)]

Concept of Non-Business Chargeable Asset


10. Mr. Rich has purchased a private building amounting to Rs. 2 crores on Kartik 23, 20X-
60 at Kathmandu. He sold such building on Rs. 3 crores on Poush 26, 20X-70. During
such period, Mr. Rich was gone abroad, intermittently for a period of 120 days. Will such
building be considered as "Non-Business Chargeable Assets"? What will be your answer
if such building was sold after two months (i.e. on Falgun 26, 20X-70)?
(Dec. 2014, 5 Marks)
Answer:
A building of a natural person is non-business chargeable asset, when it does not satisfy any
of the following conditions:

© The Institute of Chartered Accountants of Nepal 16


Compilation of Suggested Answers Income Tax

• Nature of Asset Test: It is a Business Asset, Trading Stock or Depreciable asset,


• Ownership and Residence Test: It is private building, which the owner has continuously
owned for 10 years and more and has resided for 10 years or more, either continuously. A
building and the area occupied by the building is defined as private building unless the
land area occupied by the building is more than 1 ropani,
• Disposal Value Test: The disposal value of building is less than Rs. 1 Million, and
• Three Generation Test: The building is transferred within three generation without
involving commercial transactions.

Testing the given building against the conditions above:


(a) Nature of Asset Test: The asset is not business asset, trading stock or depreciable asset.
(b) Ownership & Residence Test: The building is owned for more than 10 years (23rd Kartik
20X-60 to 26th Poush 20X-70), but the residence is for less than 10 years (just 120 days),
this test is also not satisfied.
(c) Disposal Value Test: The building is disposed off for value more than 1 Million, i.e. 30
Million
(d) Three Generation Test: It is not the case of transfer within three generation without
involving commercial transactions.

Conclusion
None of the above conditions is satisfied, that brings us to the conclusion that the building in
concern is a “non-business chargeable asset”.

Even if the building was sold after two months, none of the above conditions would be satisfied;
and there would be no change in our conclusion, i.e. the building remains “non-business
chargeable asset” within the definition of Income Tax Act, 2058.

11. Mr. Ramesh has sold a building with one ropani land at a consideration of Rs. 15,000,000
on Chaitra 31, 20X-66. This building was purchased by him on Falgun 20, 20X-61 at a
consideration of Rs. 5,000,000 and also incurred an expenditure of Rs. 1,000,000 for
extension of building. Calculate the amount of tax to be deducted 1at the time of the sale
of land.
Will your answer be different if Mr. Ramesh had purchased that building on Falgun 20,
20X-55? (June 2010, 5 Marks)
Answer:
A building of a natural person is non-business chargeable asset, when it does not satisfy any
of the following conditions:
• Nature of Asset Test: It is a Business Asset, Trading Stock or Depreciable asset,

1
It is amount of advance tax to be collected by Malpot Karyalaya, instead of amount of tax to be deducted.

© The Institute of Chartered Accountants of Nepal 17


CAP II Paper 7 Income Tax and VAT

• Ownership and Residence Test: It is private building, which the owner has continuously
owned for 10 years and more and has resided for 10 years or more, either continuously. A
building and the area occupied by the building is defined as private building unless the
land area occupied by the building is more than 1 ropani,
• Disposal Value Test: The disposal value of building is less than Rs. 1 Million, and
• Three Generation Test: The building is transferred within three generation without
involving commercial transactions.

Testing the given building against the conditions above:


(a) Nature of Asset Test: The asset is not business asset, trading stock or depreciable asset.
(b) Ownership & Residence Test: The building is owned for more less than 10 years (20th
Falgun 20X-61 to 31st Chaitra 20X-66), also the residence is for less than 10 years
(assuming resided from the date of purchase to date of sales), this test is also not satisfied.
(c) Disposal Value Test: The building is disposed off for value more than 1 Million, i.e. 15
Million
(d) Three Generation Test: It is not the case of transfer within three generation without
involving commercial transactions.

Conclusion
None of the above conditions is satisfied, that brings us to the conclusion that the building in
concern is a “non-business chargeable asset”.

Gain on Disposal of Non-business Chargeable Asset


Incomings Rs. 15,000,000
Less: Outgoings Rs. 6,000,000
Gain on Disposal Rs. 9,000,000
Advance tax to be collected by Malpot Karyalaya (@2.5%) Rs. 225,000
(Since the ownership is more than 5 Years, the rate is 2.5% as per Sec. 95Ka)

Alternative Question:
If the purchase date was 20th Falgun 20X-55 and assuming that he has resided since then,
Ownership and Residence Test is satisfied, which makes us to conclude that the asset in
concern is not a non-business chargeable asset. There would be no taxation as per Income Tax
Act, 2058 in such asset.

12. Mr. C owns a house property in Delhi, inherited from father. Since Mr. C is resident of
Nepal and has no intention of staying in India and sells the property for IRs.1.25 Crores.
Discuss the taxability of this transaction. Will it make a difference if the property is sold
for IRs.60 lakhs only? (Dec 2004, 5 Marks, CA Inter)

© The Institute of Chartered Accountants of Nepal 18


Compilation of Suggested Answers Income Tax

Answer:
A building of a natural person is non-business chargeable asset, when it does not satisfy any
of the following conditions:
• Nature of Asset Test: It is a Business Asset, Trading Stock or Depreciable asset,
• Ownership and Residence Test: It is private building, which the owner has continuously
owned for 10 years and more and has resided for 10 years or more, either continuously. A
building and the area occupied by the building is defined as private building unless the
land area occupied by the building is more than 1 ropani,
• Disposal Value Test: The disposal value of building is less than Rs. 1 Million, and
• Three Generation Test: The building is transferred within three generation without
involving commercial transactions.

Testing the given building against the conditions above:


(a) Nature of Asset Test: The asset is not business asset, trading stock or depreciable asset.
(b) Ownership & Residence Test: As the ownership information is not given, we assume that
it is owned for more less than 10 years and also the residence is for less than 10 years, this
test is also not satisfied.
(c) Disposal Value Test: The building is disposed off for value more than 1 Million, i.e. 20
Million
(d) Three Generation Test: It is not the case of transfer within three generation without
involving commercial transactions.
Conclusion
None of the above conditions is satisfied, that brings us to the conclusion that the building in
concern is a “non-business chargeable asset”. As such, the gain on disposal of such property
is taxable in Nepal as “India Sourced Income”. Even if the property is sold for Rs. 9.6 Million,
the gain on disposal would be taxable in Nepal as “India Sourced Income”.

Concept of Permanent Establishment


13. Bottlers Nepal Ltd. Kathmandu has paid consulting fees Rs. 30 million in the year 20X-
74/X-75 to International Consultancy Group, New York (ICGN). As per the agreement,
the consultant should deploy at least 2 employees at the project site located at Butwal,
Province No. 5 throughout the year. Can we define ICGN as a permanent establishment
in Nepal? What about the tax withholding implications in the following situations?
(June 2018, 5 marks)
Answer:
Definition of Permanent Establishment:
Permanent Establishment (PE) means any place where a person carries on its business, and
the term also includes the following:
a. Any place from where a person carries on business through an agent, who is not a general
agent of independent status (Agency PE),

© The Institute of Chartered Accountants of Nepal 19


CAP II Paper 7 Income Tax and VAT

b. Any place where a person has, is using or is installing main equipment or machinery (Fixed
Base PE),
c. one or more places of a country where a person provides technical, business or consultancy
services through employee or otherwise for more than 90 days in any 12 months period
(Service PE)
d. a place where a person is engaged in a construction, assembly, or installation project for
90 days or more, including a place where a person is conducting supervisory activities in
relation to such a project (Fixed Site PE)
Testing the facts and Conclusion
The consultant deploys at least two employees at the project site located at Butwal, Province
No. 5 throughout the year; or even if it does not deploy staff; the company provides service for
more than 90 days in consecutive twelve-month period.

Therefore, the company is deemed to have permanent establishment in Nepal.

When a company has a permanent establishment in Nepal, it must register itself as Permanent
Establishment (PE), obtain Permanent Account Number, raise invoice from PE in Nepal and
collect the service fee in the bank account of Nepal. The PE should Corporate Tax and
repatriation tax in Nepal, and remit the fund from Nepal after due approval from Nepal Rastra
Bank.
a. If the company is registered in Nepal through its local agent and payment was made
to local agent.
Answer:
If the company is registered in Nepal through its local agent, it would issue invoices in
Nepal. If the invoice was VAT invoice, the withholding tax rate should be 1.5% and if not
so, the withholding tax rate should be 15%.

b. If the company is not registered in Nepal and payment was transferred directly to
their bank maintained in New York.
Answer:
This cannot be the case; the company must register as Permanent Establishment in Nepal.
It is the duty of Bottlers Nepal to advise its vendor regarding possible taxation implications
in Nepal. When it is registered, the implication would be as explained in part(a) of this
question.

14. Discuss on the "Permanent Establishment" in the following cases mentioning the
relevant provisions of Income Tax Act/Rules. (June 2016, 10 Marks)
Definition of Permanent Establishment:
Permanent Establishment (PE) means any place where a person carries on its business, and
the term also includes the following:

© The Institute of Chartered Accountants of Nepal 20


Compilation of Suggested Answers Income Tax

a. Any place from where a person carries on business through an agent, who is not a general
agent of independent status (Agency PE),
b. Any place where a person has, is using or is installing main equipment or machinery (Fixed
Base PE),
c. one or more places of a country where a person provides technical, business or consultancy
services through employee or otherwise for more than 90 days in any 12 months period
(Service PE)
d. a place where a person is engaged in a construction, assembly, or installation project for
90 days or more, including a place where a person is conducting supervisory activities in
relation to such a project (Fixed Site PE)

a. American Sport Inc. has appointed the distributor – Amco Sports Pvt Ltd in Nepal
in order to make a sale of its products. Amco Sport Pvt Ltd is importing the products
from such American Sports Inc. and selling the products determining the price by its
own. There is no ownership of such products, no any restriction or special assistance
from American company on sale price or the prospective customers. The American
company exports its products through Amco Pvt Ltd. Is Amco Pvt Ltd is a
'Permanent Establishment' of American Sport Inc.?
Answer:
In the given case, since Amco Sports Pvt. Ltd. is general agent of independent status,
Permanent Establishment of American Sport Inc. is not formed in Nepal.

b. American Sport Inc. has been awarded as task of making research of potential
opportunities of sports in Nepal. To perform the task, such company sent its
employees on January 1 for 35 days. Again, on July and December of the same year,
the company sent 2 employees for 30 days and 60 days respectively. The employees of
American Sports Inc. worked in two Income Years of Nepal. Is there any consequence
of in Nepal on 'Permanent Establishment" in this case?
Answer:
Total days of service in Nepal during consecutive 12 months period: 35 days in January,
plus 30 days in July and 60 days in December; this is 125 days in any 12-month period.

This makes us to conclude that a service Permanent establishment is formed in Nepal as a


result of service more than 90 days through employee or otherwise in Nepal during any
12-month period.

c. Singapore Construction Company has been awarded 100 man-days for drying the soil
in Melamchi project. Such company has sent 5 employees in order to perform such
task and the employees have completed such task within 90 days. Is there any
consequence in Nepal on 'Permanent Establishment" in this case?

© The Institute of Chartered Accountants of Nepal 21


CAP II Paper 7 Income Tax and VAT

Answer:
Total days of service in Nepal during consecutive 12 months period: less than 90 days in
any 12-month period.
This makes us to conclude that Permanent establishment is not formed in Nepal as service
period is not more than 90 days through employee or otherwise in Nepal during any 12-
month period.

Concept of Tax-Exempt Entity


15. What are the entities considered as tax exempted entities? Whether Sections 87, 88, 88ka
and 89 of Income Tax Act with regard to TDS are attracted to such entities? Explain.
(June 2012, 5 Marks)
Answer
Meaning of Tax-Exempt Entities:
“Exempt organization” means the following entities:
a. Following entities registered with the Department as an exempt organization:
i. Social, religious, educational or charitable organizations of a public character
established without profit motive,
ii. An amateur sporting association formed for the purpose of promoting social or
sporting related amenities not involving the acquisition of gain by its member,
b. Political parties registered with the Election Commission.

Provided that, any entity, giving benefit to any person from the assets of, and amounts derived
by the entity except in pursuit of the entity’s function as per its objectives or as payment for
assets or services rendered to the entity by the person, is not exempt from tax.

Exempt Organization as Withholding Agent:


Tax exempt organization shall have obligation to withhold tax pursuant to Sec. 87, 88, 88Ka
and 89 of the Act as a Withholding Agent.

Exempt Organization as Withholdee:


Exempt Organization derives two types of income, i.e. exempt amounts (u/s 10 (chha)) and
those amounts that are taxable. Exempts amounts of those organizations are relieved from
withholding tax u/s 88 (4) and 89 (4), as such, while any payer makes payments that are exempt
for tax exempt entities, they must not withhold tax. However, the payer must withhold tax on
making payment of any such amounts that are taxable for tax exempt entities, and when such
payments are listed in Sec. 88, 88Ka or 89 of the Act.

16. Write short notes on Exempt organizations under Income Tax Act, 2058.
(Dec 2003, 4 Marks, CA Inter; June 2003, 6 Marks, CA Inter)
“Exempt organization” means the following entities:
a. Following entities registered with the Department as an exempt organization:

© The Institute of Chartered Accountants of Nepal 22


Compilation of Suggested Answers Income Tax

i. Social, religious, educational or charitable organizations of a public character


established without profit motive,
ii. An amateur sporting association formed for the purpose of promoting social or
sporting related amenities not involving the acquisition of gain by its member,
b. Political parties registered with the Election Commission.
Provided that, any entity, giving benefit to any person from the assets of, and amounts derived
by the entity except in pursuit of the entity’s function as per its objectives or as payment for
assets or services rendered to the entity by the person, is not exempt from tax.

© The Institute of Chartered Accountants of Nepal 23


CAP II Paper 7 Income Tax and VAT

Chapter 3: Basis of Taxation


This Chapter includes:

• Concept of Persons and Persons Chargeable to Tax


• Taxable Income
• Adjusted Taxable Income
• Assessable Income
• Sources of Income
17. Define Income, Windfall gain, Assessable Income and Taxable Income.
(2011 June, 5 marks)
Answer:
Income
It means the income earned by any person from employment, business, investment or windfall
gain and the term also includes total amount of such incomes calculated under this Act.

Windfall Gain
“Windfall Gain” means income from lottery, gifts, prize, tips (baksis), award for winning
(Jitauri), and any similar other payment received suddenly.

Assessable Income
Within the meaning under Sec. 6, Subject to Income Tax Act, the assessable income of a person
for an Income year from any employment, business, or investment, or windfall gain shall be as
follows:
a. In respect of a resident person, the person’s income from business, employment,
investment, or windfall gain irrespective of the source of such income, and
b. In respect of a non-resident person, the person’s income from business, employment,
investment or windfall gain from income having source in Nepal
Provided that, the assessable income does not include any income exempt under section 11 or
64 or both.

Taxable Income
The taxable income of a person for an income year is the total of the person's assessable
income for the year from each of the following income heads less any reduction allowed for
the year under Sec. 12, 12Ka, 12Kha, 63 or all four sections:
a. Business,
b. Employment,
c. Investment, and
d. Windfall Gain

18. Discuss and elaborate the following quoting relevant sections of the Act :
a. Classification of taxable Income (Dec 2010, 3 Marks, CA Inter)

© The Institute of Chartered Accountants of Nepal 24


Compilation of Suggested Answers Income Tax

Answer:
The taxable income of a person for an income year is the total of the person's assessable
income for the year from each of the following income heads less any reduction allowed
for the year under Sec. 12, 12Ka, 12Kha, 63 or all four sections:
i. Business,
ii. Employment,
iii. Investment, and
iv. Windfall Gain

b. Assessable Income (Dec 2010, 3 Marks, CA Inter)


Answer:
Within the meaning under Sec. 6, Subject to Income Tax Act, the assessable income of a
person for an Income year from any employment, business, or investment, or windfall gain
shall be as follows:
i. In respect of a resident person, the person’s income from business, employment,
investment, or windfall gain irrespective of the source of such income, and
ii. In respect of a non-resident person, the person’s income from business, employment,
investment or windfall gain from income having source in Nepal

Provided that, the assessable income does not include any income exempt under section 11
or 64 or both.

Concept of Person chargeable to Tax


19. X & Co., a sole trading firm owned by Mr. X, dealing with the business of selling male
garments had a profit of Rs. 1,200,000 in FY 20X-71/X-72. During the same financial
year, X & Co. has opened up another shop at different location for selling exclusively
female garments. The outlet being a new, it has suffered a loss of Rs. 200,000. Mr. X had
registered another proprietorship X & Sons for furniture business. It incurred a loss of
Rs. 300,000. Also, he has a partnership firm with his spouse that too incurred a loss of
Rs. 200,000. For income Tax purpose, Mr. X prepared the financial statement by
clubbing income statements of all firms showing profit of Rs. 500,000. Is the clubbing
done by Mr. X justified? (June 2016, 5 Marks)
Answer:
“Person” means natural person and entity. The sole trading firm (private firm) of a natural
person is also a natural person. The income derived by a sole trading firm is deemed to be
derived by the owner of the firm himself. As such, if a natural person has multiple sole trading
firms, income derived from each of such sole trading firm is consolidated and income tax
liability is computed on such consolidated income.
In the given case, Mr. X has two shops of under the sole trading firm X &Co and another, X&
Sons, a furniture business. These two sole trading firms do not have legal status, but are part

© The Institute of Chartered Accountants of Nepal 25


CAP II Paper 7 Income Tax and VAT

of business of Mr. X and income or loss derived by each of these firms is deemed to be derived
by Mr. X himself.

A partnership is an entity as per the definition of Income Tax Act. Owner of the entity and the
entity are separate for the purpose of taxation (Sec. 52).

When Sec. 11 applies, income derived for each of different businesses is deemed to be derived
by a separate person.

As per Sec. 20 (1), while determining assessable income from business, one can set off the
business loss of any other business of the same person sustained during the same Income Year.

Conclusion:
Based on the facts above, profit from selling male garments of Rs. 1.2 Million can be utilized
to set off loss from selling female garment of Rs. 200,000 and from furniture business of Rs.
300,000; as these are trading business and are not subject to concessions under Sec. 11 and
is the business income or loss of same person.

Since partnership firm is an entity, a separate person from Mr. X, the loss sustained by the
partnership firm cannot be set off against the income of Mr. X.

Concept of Adjusted taxable Income


20. The Profit and Loss Account of XYZ Limited for the year ended Ashad 32, 20X-64 shows
a net profit of Rs. 10,00,000 and other information is given here-under:
a. The expenses include a donation of Rs. 50,000 out of which Rs. 20,000 is provided to
the organization recognized as tax exempted by IRD.
b. During the year the company has incurred Rs. 550,000 on pollution control but the
expenses are not included in the profit.
c. During the year the company has incurred Rs. 300,000 on research and development
but the expenses are not included in the profit.
Calculate the taxable income of XYZ Limited for the income year 20X-63/X-64.
(Dec 2007, 5 marks, CA Inter)
Answer:
Adjusted Taxable income means, taxable income derived by ignoring:
• Reductions u/s 12 (donation), and
• Deductions u/s 14 (2) [interest expense and interest income], u/s 17 (Pollution control
cost) or u/s 18 (Research and Development cost)

© The Institute of Chartered Accountants of Nepal 26


Compilation of Suggested Answers Income Tax

Calculation of Adjusted Taxable Income for the purpose of Sec. 17, 18 and 122:
Particulars For Sec. 17 For Sec. 18 For Sec. 12
Net profit before deduction/reduction u/s 17, 18 1,000,000 1,000,000 1,000,000
& 12
Add: Donation already reduced 50,000 50,000 50,000
Less: Pollution Control Cost (Actual) - 550,000 550,000
Less: Research and Development Cost 300,000 - 300,000
Adjusted Taxable Income (ATI) 750,000 500,000 200,000

Eligible Pollution Control Cost u/s 17


Lower of:
a. Actual Cost 550,000
b. 50% of ATI (50% of 700,000) 375,000

Eligible Research & Development Cost u/s 18:


Lower of:
a. Actual Cost 300,000
b. 50% of ATI (50% of 450,000) 250,000

Eligible Donation u/s 12:


Lower of:
a. Actual donation or gift to Exempt Organization 20,000
b. 5% of ATI (5% of 150,000) 10,000
c. Maximum 100,000

Calculation of Taxable Income:


Taxable Income before deductions/reductions u/s 17, 18 & 12 1,000,000
Less: Deductions u/s 17 375,000
Less: Deduction u/s 18 250,000
Less: Reduction u/s 12 10,000
Taxable Income 365,000

2
There is no consensus in the methodology to compute Adjusted Taxable Income, due to ambiguity in law. As
such, the interpretation by Income Tax Directives issued by Inland Revenue Department has been taken as
reference to compute ATI.

© The Institute of Chartered Accountants of Nepal 27


CAP II Paper 7 Income Tax and VAT

Income Heads Chargeable to Income Tax


21. Distinguish between "Assessable Income" and "Taxable Income" under the provisions
of Sec. 5 and 6 of the Nepal Income tax Act, 2058. (Dec 2005, 2.5 Marks, CA Inter)
Assessable Income
Within the meaning under Sec. 6, Subject to Income Tax Act, the assessable income of a person
for an Income year from any employment, business, or investment, or windfall gain shall be as
follows:
a. In respect of a resident person, the person’s income from business, employment,
investment, or windfall gain irrespective of the source of such income, and
b. In respect of a non-resident person, the person’s income from business, employment,
investment or windfall gain from income having source in Nepal
Provided that, the assessable income does not include any income exempt under section 11 or
64 or both.

Taxable Income
The taxable income of a person for an income year is the total of the person's assessable
income for the year from each of the following income heads less any reduction allowed for
the year under Sec. 12, 12Ka, 12Kha, 63 or all four sections:
a. Business,
b. Employment,
c. Investment, and
d. Windfall Gain

Difference
Taxable income is such residue of assessable income after applying reductions u/s 12, 12Ka,
12Kha and 63.

22. State under what head the following incomes will fall to be assessed?
(Dec 2004, 6 Marks, CA Inter)
a. Interest on Deposits with banks.
b. Interest on Government securities
c. Prize received in purchase of noodles
d. Scholarship received by a student from a foreign government
e. Free foreign tour provided by a distributor of motorcycle to dealer
f. Donation received by a physically handicapped person

Answer:

Particulars Income Head


Interest on Deposit with Banks
• Bank deposit of entity Business
Business

© The Institute of Chartered Accountants of Nepal 28


Compilation of Suggested Answers Income Tax

• Bank deposit of natural person, conducting business Investment


• Bank deposit of a natural person, not conducting business
Interest on Government securities
• Securities held by entity Business
• Securities held by natural person in connection to business Business
• Securities held by natural person, not in connection to Investment
business
Prize received in purchase of noodles Windfall Gain
Scholarship received by a student from a foreign government Income Tax Free
Free foreign tour provided by a distributor of motorcycle to Business
dealer
Donation received by a physically handicapped person Windfall Gain3

23. Mr. B, a resident of Nepal inherits artistic pictures from his father which his father has
drawn during has life time. The sell the same for Rs.50 lakhs to art connoisseurs. Discuss
the taxability of this transaction. (Dec 2004, 5 Marks, CA Inter)
Answer:
The gain on disposal of following assets is chargeable to income tax as per Income Tax Act,
2058:
a. Gain on Disposal of Business Asset: as part of assessable income from business
b. Incomings from trading stock: as part of assessable income from business
c. Gain on Disposal of Depreciable asset of business: as part of assessable income from
business
d. Gain on Disposal of Depreciable asset of investment: as part of assessable income from
investment
e. Gain on Disposal of Non business Chargeable asset: as part of assessable income from
investment
If any asset does not fall within the meaning of business asset, trading stock, depreciable asset
or non-business chargeable asset, the gain on disposal of such asset is not subject to income
tax.
Meaning of assets:
Business Asset: “Business Asset” means an asset used in a business, but excludes trading stock
or a depreciable asset of a business.
Depreciable Asset: “Depreciable Asset” means an asset to the extent to which it is used in the
production of income from a business or investment and that is likely to lose value because of
wear and tear, obsolescence, or the passing of time. Provided that, the term shall not include
any trading stock.

3
The law is ambiguous in this matter. This income may also be treated as “beyond scope from Income Tax”, or tax
exempt.

© The Institute of Chartered Accountants of Nepal 29


CAP II Paper 7 Income Tax and VAT

Trading Stock: “Trading stock” means assets owned by a person that are intended to be sold
in the ordinary course of a business conducted by the person, work in progress on such assets,
and inventories of materials to be incorporated into such assets. The term excludes asset
denominated in foreign currency.
Non-business Chargeable Asset: Non-Business chargeable Asset” means land, building and
interest in any entity, or securities, other than the following assets:
1. Business asset, depreciable asset or trading stock,
2. Private building of a natural person in the following conditions:
a. having owned it for ten or more years continuously, and
b. resided therein for ten or more years continuously or intermittently by such person.

Clarification: For the purpose of this Clause, “Private building” means a building and
lesser of land area occupied by the building or one ropani of such area occupied by
the building.

3. An interest of a beneficiary in a retirement fund,


4. Land, Land and Building and private building of a natural person that has been disposed
of at a price of less than 1 million rupees, or
5. Assets disposed of through transfer by any means within three generations, other than sale
and purchase.

Conclusion
When we go through the definition of these assets, the artistic pictures do not fall under any
category of assets. As such, the amount received from selling the artistic picture to art
connoisseurs is not taxable as per Income Tax Act, 2058.

Identification of Source of Payments


24. Point out the country where the following sources of income are situated according to
Income Tax Act, 2058 (Dec 2004, 4 Marks, CA Inter)
a. Income from an export from Nepal to Bangladesh.
b. Service (otherwise than employment) provided in India for which Government of
Nepal makes payment.
c. Freight received by an air company in Nepal for a ticket issued for Delhi to U.A.E.
d. Income from a property situated in Pakistan
Answer:
a. Nepal, as the business activity is conducted in Nepal [Sec. 67 (6) (Tha) (2)]
b. Nepal, as the payment is made by Government of Nepal [Sec. 67 (6) (Jha)]
c. India, as the cargo embarks the aircraft from India [Sec. 67 (6) (Chha)]
d. Pakistan, as the fixed property is situated in Pakistan [Sec. 67 (6) (Gha)]

© The Institute of Chartered Accountants of Nepal 30


Compilation of Suggested Answers Income Tax

Chapter 4: Calculation of Tax and Tax Rates


This Chapter includes:

• Calculation of Income Tax, not specific to any Income Head


• Fixed Tax: Presumptive Tax
• Tax on Repatriation of Income

Tax on Repatriation of Income


25. Gulf Bank, a Dubai based banking company has an office in Kathmandu for its wholesale
banking service to local banks. During the year it made Rs. 75,00,000 profit from the
operation in Nepal of which Rs. 50,00,000 was repatriated to head office Dubai. Calculate
Tax Liability. (June 2017, 2.5 Marks)
Answer:
Corporate Tax Liability (assuming profit is before tax)
CIT (Corporate Income Tax)- 25% of Rs. 7,500,000 = Rs. 1,875,000

Tax on Repatriation Income


Repatriated Amount: Rs. 5,000,000
Amount before repatriation tax4= Rs. 5,000,000 ÷ 0.95 = Rs. 5,263,158
Repatriation Tax = 5% of Rs. 5,263,158 = Rs. 263,158

26. AMEXO Bank is registered in the USA and operating its liaison office in Kathmandu.
During the financial year 20X-67/X-68, it has following summarized transactions.
Rs.
Income recognized 50,00,000
Expenses 40,00,000 (before Income Tax)
The liaison office has policy to repatriate all the remaining profits to its corporate office.
Compute maximum amount that can be repatriated from Kathmandu liaison office.
(Dec 2011, 5 Marks)
Answer:
As per Sec. 3 (Ka) and (Kha), a foreign permanent establishment (person) is required to pay
corporate income tax in Nepal in its taxable income and also while repatriating the after-tax
profit.
Corporate Tax
In the given case, the taxable income is Rs. 10 Lakhs, corporate tax rate is 25% (Sec. 2 of
Schedule 1) and the tax liability is Rs. 7.5 Lakhs.

Repatriation Tax

4
We have followed interpretation of Income Tax Directives issued by Inland Revenue Department; refer to
immediately succeeding question for further clarity (In Amexo Bank Case, Dec 2011 Question).

© The Institute of Chartered Accountants of Nepal 31


CAP II Paper 7 Income Tax and VAT

AMEXO bank can repatriate Rs. 7.5 lakhs to its head office after paying repatriation tax at the
rate 5% (Sec. 2 (6) of Schedule 1).
Repatriated Amount5
There are two schools of thoughts regarding computation of repatriation tax:

Alternative 1: 5% of Rs. 7.5 Lakhs is repatriation tax, which is Rs. 37,500 and the amount that
can be repatriated is Rs. 712,500. This is backed by interpretation of Inland Revenue
Department through Income Tax Directives.

Alternative 2: 5% of (Rs. 7.5 Lakhs ÷ 1.05), which is Rs. 35,714.29 and the amount that can
be repatriated is Rs. 714,285.71. This is logical in the sense that the taxing person is FPE in
Nepal and the repatriation tax is levied on repatriated amount.

Presumptive Tax
27. Mohit Shrestha, citizen of Nepal provides rental services/hiring services of Mini Bus,
Power Tiller, Auto Rickshaw and other vehicles. Following is the detail of his business.
Vehicles are owned by him in his individual name for fiscal year 20X-69/X-70.
(5 Marks, Dec 2013)
Number of
Rates per No. of services Provided
vehicles
Services (Rs.) during the year
owned
Power Tiller 5,500 220 7
Tractor 5,000 180 11
Auto Rickshaw 5,000 10 1
Micro Bus 8,500 50 4
Car 8,500 310 8
Truck 11,000 200 4

He incurred following expenses during the year.

Particulars Amount (Rs.)


Petrol 1,01,000
Diesel 1,20,000
Oil & Lubricants 21,375
Repair 68,245
Staff's Salary 39,00,000
Required:

5
Students are advised to follow interpretation by Income Tax Manual, that is, Alternative 1 for
Examination purpose. Marks may be awarded in Alternative 2 as well, if sufficiently reasoned.

© The Institute of Chartered Accountants of Nepal 32


Compilation of Suggested Answers Income Tax

a. Calculate the tax to be paid by Mr. Mohit for fiscal year 20X-69/X-70.
b. Mr. Mohit did not submit Annual Income Tax Return under Section 96. What are its
implications?
Answer6:
a. Tax to be paid by Mr. Mohit
Inclusions in income as per Sec. 7 (2) [Note 1 below] 50,000,000
Less: Deductions under Income Tax Act, 2058 (Sec. 13) (4,142,375)
Less: Depreciation u/s 19 -
(Could not be computed in absence of information)
Less: Deduction u/s 16 (Assumed within 7% Limit) (68,245)
Taxable Income 45,857,625
Computation of Tax Liability Assuming Couple
1st Rs. 450,000 0% -
Next Rs. 100,000 10% 10,000
Next Rs. 200,000 20% 40,000
Next Rs. 1,250,000 30% 375,000
Balance Rs. 43,857,625 36% 15,788,745
Total 16,213,745
Note 1:
Rates per No. of services
Number of
Services Provided during Total Earned
vehicles owned
(Rs.) the year
Power Tiller 5,500 220 7 8,470,000
Tractor 5,000 180 11 9,900,000
Auto Rickshaw 5,000 10 1 50,000
Micro Bus 8,500 50 4 1,700,000
Car 8,500 310 8 21,080,000
Truck 11,000 200 4 8,800,000
Total 50,000,000
b. Obligation to File Income Return
Since, the total income of Mr. Mohit exceeds Rs. 40 Lakhs during the year; Mr. Mohit must
file income return. The implication of not filing income return is payment of fees u/s 117,
which is Rs. 100 per month or 0.1% p.a. of Gross Inclusions (i.e. 0.1% p.a. of Rs.
50,000,000). However, CAP II syllabus excludes Sec. 117, so the implication is not
important for CAP II Students.

6
Before Income Year 2075/76, in case a natural person owns vehicle and pays tax u/s 1 (13) of Schedule 1, the
payment of tax would be final, and thus, was required to pay presumptive tax. However, a vehicle owner does not
pay presumptive tax since Income Year 2075/76.

© The Institute of Chartered Accountants of Nepal 33


CAP II Paper 7 Income Tax and VAT

28. Mr. Shrestha, a retired government employee, earns a pension of Rs. 9,500 per month
and 1-month Dashain allowance. He also runs a small business with annual income of Rs.
125,000. Calculate the Taxable Income and the tax liability for income year 20X-66/X-67
assuming Mr. Shrestha adopts single status as per section 50 of Income Tax Act, 2058.
(Dec 2010, 5 Marks, CA Inter)
Answer:
Tax Liability of Mr. Shrestha (assessing Individual)
Assessable Income from Business 125,000
Assessable Income from Employment (9,500 X 13) 123,500
Total Assessable Income 248,500
Balance Taxable Income 248,500
Tax Liability -
(Since the taxable income is within tax-free slab of Rs. 400,000)
Note to students:
Mr. Shrestha cannot pay presumptive tax u/s 4 (4) of the Act, if wishes so, since he has income
also from employment.

Tax Computation- General


29. Super company is a proprietorship industry producing tobaccos. The Industry has total
sales of Rs. 20 million and taxable income of Rs. 3.5 million in Income Year 20X-70/X-71.
The company hasn’t submitted estimated tax return during the year. In addition, it has
not paid any taxes. You are requested to assess the total Tax liability for this industry up
to Mangsir 20, 20X-71. (Dec 2014, 5 Marks)
Answer:
Computation of Tax Liability of a private firm (i.e. natural person) assuming Individual:
Taxable Income/Balance Taxable Income- Rs. 3,500,000
1st Rs. 400,000 0% -
Next Rs. 100,000 10% 10,000
Next Rs. 200,000 20% 40,000
Next Rs. 1,300,000 30% 390,000
Balance Rs. 1,500,000 36% 540,000
Total Tax Liability 980,000

Note:
Since, Sec. 114 and above of Income Tax Act, 2058 is out of syllabus for CAP II, fees and
interest u/s 117, 118 and 119 not computed.

© The Institute of Chartered Accountants of Nepal 34


Compilation of Suggested Answers Income Tax

Chapter 5: Exemptions, Concessions, Reductions, Deductions, Set off


This Chapter includes:

• Exemptions under Section 10


• Concessions, Waivers and Privileges under Section 11
• Deductions to Natural Person from Taxable Income under Section 1 of Schedule 1
• Set off of Losses under Section 20

Exempt Amounts and Extent of Exemptions


30. Mr. ‘Z’ a Canadian citizen is deputed by the Canadian government to work on a
Canadian aided project and he stays in Nepal for 94 days. He receives his salary in
Canada and he is provided a free accommodation and a daily allowance of Rs. 2,000 to
meet his boarding and other expenses. (Dec 2012, 4 Marks)
a. Examine the liability to tax under Income Tax Act, 2058.
b. Will your answer be different if Mr. ‘Z’ comes to Nepal under the services of a private
contractor instead of the Canadian government in the above case?
Answer:
a. As per Sec. 10 (Kha) of Income Tax Act, 2058; any amounts received by an individual as
a result of employment of foreign government is exempt from tax, if both the following
conditions are satisfied:
i. The person is resident of Nepal only because of the employment is non-resident of
Nepal, and
ii. The person receives the amount from the public fund of foreign government.
Conclusion
Since, Mr. Z is deputed by Canadian Government and is receiving income from public fund
of such government and is non-resident in Nepal, his income is exempt from tax in Nepal.
b. If he were hired by a private contractor, his taxable income and tax liability would be as
follows:
Salary and Emoluments Rs. X
Free Accommodation Nil
(Since it is official visit and he is not permanently relocated)
Daily allowance Nil
(Since it is official visit)
Taxable Income would be taxed @ 25%.

© The Institute of Chartered Accountants of Nepal 35


CAP II Paper 7 Income Tax and VAT

31. Nirdhan Samaj Sewa is a not for profit organization registered in Kathmandu. It has
obtained tax exemption certificate from IRD. During the year 20X-67/X-68 the
organization received the following incomes:
Membership Fees Rs. 275,000
Interest Rs. 170,000
Donation Rs. 450,000
Dividend income Rs. 1,200,000
Net profit from running a school Rs. 120,000

Nirdhan Samaj Sewa has claimed that it is a tax exempted organization and so it is not
obliged to pay tax and has filed a refund application for TDS on interest of Rs. 30,000.
Justify its claim on the basis of the provisions of Income Tax Act, 2058.
Also state the legal provisions regarding filing of income tax return and renewal of tax
exemption certificate by Nirdhan Samaj Sewa. (Dec 2011, 5 Marks)
Answer
As per Sec. 10 (chha), the following amount of an exempt organization is exempt from tax:
a. Donation or Gift,
b. Any contribution directly related to the objective of the organization, and where the
contribution does not obtain or expect any consideration against such contribution

Conclusion
a. Membership fee: Condition (b) above satisfied, exempt from tax
b. Interest: both the above conditions are not satisfied, as such the interest amount is taxable
in the hand of Nirdhan Sewa Samaj. If it is received from resident bank, financial
institutions, cooperatives, entity issuing bonds or listed entities, it is final withholding.
c. Donation: Condition (a) above satisfied, exempt from tax
d. Dividend income: both the above conditions are not satisfied, as such the interest amount
is taxable in the hand of Nirdhan Sewa Samaj. If it is paid by resident company, it is final
withholding.
e. Net Profit from running a school: both the above conditions are not satisfied, as such the
interest amount is taxable in the hand of Nirdhan Sewa Samaj.
Therefore, Nirdhan Sewa Samaj may not claim refund of TDS on Interest due to following
reasons:
(a) If it is paid by resident banks, financial institutions, entity issuing bonds, listed entities or
cooperatives, interest income is final withholding.
(b) If it is not paid by persons as above, the interest income form part of assessable income
from business, along with inclusion of profit from running school in assessable income from
business and if dividend is received from non-resident entity, including the same amount as

© The Institute of Chartered Accountants of Nepal 36


Compilation of Suggested Answers Income Tax

well. Then the organization should compute taxable income and tax liability, and adjust the
TDS on interest as advance tax, and pay the remaining due, if any. In case of excess, it may
claim refund of such excess TDS amount.
Filing of Income Return and Renewal of Exemption Certificate
Nirdhan Sewa Samaj shall file income return within three months of the end of Income Year.
If such time is not adequate, it may extend return filing date by three months with permission
of IRD after filing an application in that regard within due date of filing income return.
Tax exemption certificate must be renewed every year. An application to renew tax exemption
certificate must be filed within six months of end of Income year, along with following
documents:
a. Proof of filing income return,
b. Proof of payment of withholding taxes to be withheld by Exempt Organizations, and
c. Proof of filing withholding tax returns.

32. As a tax consultant, you have been enquired of the implication on income tax on Income
of the Securities Board of Nepal. (June 2011, 1 Mark)
Answer:
The income of Securities Board of Nepal is taxable. It must compute assessable income from
business, taxable income and tax liability and pay tax at the rate 25%.

33. Mr. Clinton is working in an embassy representing USA 7 and come to Nepal on 1st
Ashwin, 20X-65. His salary and other emoluments are paid by the country he represents.
He declared the following sources of income during the year 20X-65/X-66.
a. Salary of Rs. 280,000 per month.
b. Dearness allowance Rs. 150,000 per month.
c. Foreign allowance Rs. 80,000 per month.
d. He runs a handicraft business in Nepal and earned a net profit of Rs. 175,000 in the
income year 20X-65/X-66.
e. He was a member of Royal Golf Club of Nepal. He won a prize of Rs. 150,000 from
the game.
However, he claimed the following expenses:
a. Renewal of membership and subscription Rs. 50,000
b. Golf Expenses Rs. 32,000
c. Donation to a school of remote district Rs. 50,000

7
This question is replica of question asked for CA Intermediate, June 2008 question, with minor
change in the facts of the question, but without any difference substantially. Refer Question 36
below

© The Institute of Chartered Accountants of Nepal 37


CAP II Paper 7 Income Tax and VAT

(School is Tax Exempt organization recognized by IRD)


d. Tax deducted at source in Nepal Rs. 2,000

You are required to find Mr. Clinton’s residential status and taxable income for the
income year 20X-65/X-66. (June 2010, 10 Marks)
Answer:
Residential Status of Mr. Clinton
An individual is resident of Nepal, if any of the following three conditions is satisfied:
a. His/her habitual place of abode is in Nepal,
b. S/he stays in Nepal for 183 days or more during any period of consecutive 365 days, or
c. S/he is employee of Government of Nepal, deputed by the employer in any foreign
country during the Income Year.
Since Mr. Clinton has stayed in Nepal for more than 183 days (from Ashoj 20X-65 till Ashad
end 20X-66), he is resident of Nepal, as he satisfies Condition (b) above.

Calculation of Assessable Income, Taxable Income and Tax Liability of Mr. Clinton
For Income Year 20X-65/X-66
Particulars Sec. Ref Amount Working Notes
Inclusions
Salary Income 10 (Kha) - Amounts derived by a natural person
Dearness Allowance 10 (Ga) - from employment in the public service of
Foreign Allowance 10 (Ga) - government of a foreign country,
provided:
a. The person is resident solely by
reason of performing the
employment or is a non-resident
person,
b. The amounts are payable from the
public funds of the country
Assessable Income from Employment -
Income from Business 175,000 Assuming inclusion and deductions are
as per Income Tax Act, 2058
Prize from Playing Golf 88Ka/92 - Windfall Gain
Renewal of Membership & 21 - Expense of Domestic and Personal
Subscription Nature
Golf Expenses 21 - Expense in relation to Final Withholding
Payments
Assessable Income from Business 175,000
Total Assessable Income 175,000

© The Institute of Chartered Accountants of Nepal 38


Compilation of Suggested Answers Income Tax

Less: 8,750 Actual: Rs. 50,000;


Reduction u/s 12- Donation or Gift to 5% of Rs. 175,000; or
Exempt Organization Rs. 100,000: w/e is lower
Taxable Income 166,250
Tax Deducted at Source In Nepal May be refundable, if not related to
prize.

34. What will be the income tax implication of pension received from British Army by Mr.
Ram Gurung, a Nepali citizen? (Dec 2009, 2 Marks, CA Inter)
Answer:
Under Sec. 10 (ja), pension income derived by a Nepali Citizen from public fund of foreign
government as a result of retirement after service the police or army of such country is exempt
from tax.
In the given case, Ram Gurung is a Nepali Citizen. He was retired from the service of British
Army (foreign government). He derives pension income. So far the income is received from
public fund of British Government; the pension income of Mr. Ram is exempt.
In case it is received from any other fund than public fund of British government, the amount
is taxable in Nepal.

35. What are the exempt amounts? Describe the provisions of the Income Tax Act in this
regard. (June 2009, 10 Marks; Dec 2004, 5 Marks, CA Inter)
Answer:
The following amounts shall be exempt from tax:
a. Tax exempt amounts derived by a person entitled to tax exemption facilities pursuant to
bilateral or multilateral treaty or agreement between/among Government of Nepal and any
foreign country or international organization or institution.
b. Amounts derived by a natural person from employment in the public service of government
of a foreign country, provided:
(1) The person is resident solely by reason of performing the employment or is a non-
resident person,
(2) The amounts are payable from the public funds of the country
c. Amounts derived by a natural person who is not a Nepali natural citizen and who is
referred to in clause (b) or his/her immediate family member,
d. Amounts derived by natural person who not a Nepali citizen and appointed in the service
of Government of Nepal with the tax exemption term,
e. All types of allowances in the nature of social security payments made by Government of
Nepal, provincial government or local government
f. Amounts derived by way of gift, bequest, inheritance or scholarship, except as required to
be included in income pursuant to Sec. 7, 8, or 9
g. Following amounts derived by exempt organization:

© The Institute of Chartered Accountants of Nepal 39


CAP II Paper 7 Income Tax and VAT

(1) Donation, gift,


(2) Any other contributions directly related to the activities of the exempt organization
in the nature specified in Sec. 2 (dha), where the contributor does not receive or
expect any consideration
h. Any amount of pension income derived by a Nepali Citizen, who is retired after serving
the army or police of any foreign government, from the public fund of such foreign
government,
i. Any income of government of Nepal, provincial government or local government
j. Amounts generated by Nepal Rastra Bank as per its objective
k. Any income of generated by Drinking Water and Sanitation Consumer Society established
under Water Resources Act, 2049

36. 8Mr. Ford is working in an embassy representing UK and come to Nepal on 1st Ashwin,
20X-63. His salary and other emoluments are paid by the country he represents. He
declared the following sources of income during the year 20X-63/X-64.
(June 2008, 10 Marks, CA Inter)

a. Salary of Rs. 180,000 per month.


b. Dearness allowance Rs. 15,000 per month.
c. Foreign allowance Rs. 80,000 per month.
d. He runs a handicraft business in Nepal and earned a net profit of Rs. 150,000 in the
income year 20X-63/X-64.
e. He was a member of Royal Golf Club of Nepal. He won a prize of Rs. 50,000 from the
game.
However, he claimed the following expenses:
a. Renewal of membership and subscription Rs. 10,000
b. Golf Expenses Rs. 12.000
c. Donation to a school of remote district Rs. 50,000
(School is the organization recognized as tax exempted by IRD)
d. Tax Deducted at source in Nepal
You are required to find Mr. Ford’s residential status and taxable income for the income
year 20X-63/X-64.
Answer:
Residential Status of Mr. Ford
An individual is resident of Nepal, if any of the following three conditions is satisfied:
a. His/her habitual place of abode is in Nepal,

8
This question was repeated by ICAN during June 2010, CAP-II Examination, refer Question 33
above

© The Institute of Chartered Accountants of Nepal 40


Compilation of Suggested Answers Income Tax

b. S/he stays in Nepal for 183 days or more during any period of consecutive 365 days, or
c. S/he is employee of Government of Nepal, deputed by the employer in any foreign
country during the Income Year.
Since Mr. Ford has stayed in Nepal for more than 183 days (from Ashoj 20X-63 till Ashad
end 20X-64) during IY 20X-63/X-64, he is resident of Nepal, as he satisfies Condition (b)
above.

Calculation of Assessable Income, Taxable Income and Tax Liability of Mr. Ford
For Income Year 20X-65/X-66
Particulars Sec. Ref Amount Working Notes
Inclusions
Salary Income 10 (Kha) - Amounts derived by a natural person
Dearness Allowance 10 (Ga) - from employment in the public service of
Foreign Allowance 10 (Ga) - government of a foreign country,
provided:
l. The person is resident solely by
reason of performing the
employment or is a non-resident
person,
m. The amounts are payable from
the public funds of the country
Assessable Income from Employment -
Income from Business 150,000 Assuming inclusion and deductions are
as per Income Tax Act, 2058
Prize from Playing Golf 88Ka/92 - Windfall Gain
Renewal of Membership & 21 - Expense of Domestic and Personal
Subscription Nature
Golf Expenses 21 - Expense in relation to Final Withholding
Payments
Assessable Income from Business 150,000
Total Assessable Income 150,000
Less: 7,500 Actual: Rs. 50,000;
Reduction u/s 12- Donation or Gift to 5% of Rs. 150,000; or
Exempt Organization Rs. 100,000: w/e is lower
Taxable Income 142,500
Tax Deducted at Source In Nepal May be refundable, if not related to
prize.

© The Institute of Chartered Accountants of Nepal 41


CAP II Paper 7 Income Tax and VAT

37. Any kind of Income earned by Nepal Government is exempt from Tax. Is it true?
(June 2008, 1 Mark, CA Inter)
Answer:
Yes, as per Sec. 10 (Jha), any amount earned by Nepal Government, Province Government or
Local Government is exempt from tax.

Income Exempt from Tax under Section 11


38. Kanyam Tea Estate Ltd. is a co-operative society established under the Co-operative
Society Act, 2074. In that society, 40 farmers of Kanyam region are involved in tea
gardening and processing. In the financial year 20X-67/X-68, Kanyam Tea Estate has
earned Rs. 1 Crore. State with reason on the taxability of such income earned. Further,
what will be the impact on income tax if dividends are declared by that society?
(June 2011, 2 Marks)
Answer:
Assuming that Kanyam Tea Estate Ltd. is involved in tea farming and processing, as per Sec.
11 (2), income of Kanyam Tea Estate Ltd. is exempt from tax.

However, if it acts as sales agent between the tea farmers and purchases tea leaves from
farmers and processes it and sales it, the income of Rs. 1 crore is taxable for the Cooperative.

39. Mr. Ram Kumar has opened a bank account with Grameen Biaks Bank in Parbat with
a deposit of Rs. 50,000 with interest at the rate of 8 percent per annum. During the year,
the bank has to pay Mr. Ram Kumar Rs. 4000 as Bank Interest. The bank seeks for your
advice regarding the applicability of withholding tax?
If for same deposit of Rs. 50,000 with the interest rate of 24%, the bank has to pay Rs.
12,0009 for that year, then will your answer differ? (Dec 2010, 5 Marks)
Answer:
Assuming that the Grameen Bikas Bank is based on Rural Municipality in Parbat, the interest
received by Mr. Ram Kumar of Rs. 4,000 is exempt from tax as per Sec. 11 (2Ka). As per Sec.
88 (4), a withholding agent is not required to withhold tax on any amounts exempt from tax.
As such, the bank is not required to withhold tax on payment of interest of Rs. 4,000.
As per Sec. 11 (2Ka), the interest from Grameen Bikas Bank based on Rural Municipality up
to Rs. 25,000 is exempt from tax. Since, the interest income of Mr. Ram Kumar exceeds the
threshold of Rs. 25,000; the bank must withhold tax @ 5% on Rs. 28,000.

40. What are the incomes exempted from tax under the Income Tax Act, 2058?
(June 2009, 5 Marks)
Answer:
The following incomes are exempt under Income Tax Act, 2058:

9
Originally asked for Rs. 12,000; now modified as Rs. 28,000.

© The Institute of Chartered Accountants of Nepal 42


Compilation of Suggested Answers Income Tax

a. Income of non-resident not having source in Nepal (Sec. 6)


b. Incomes that fall under the lists in Sec. 10, which are exempt amounts
c. Agriculture income of an individual subject to provisions of Sec. 11 (1)
d. Income derived by cooperatives subject to provisions of Sec. 11 (2)
e. Dividend income of members of cooperatives subject to provisions of Sec. 11 (2)
f. Interest income of individual subject to provisions of Sec. 11 (2Ka)
g. Income derived by Special Industry subject to provisions of Sec. 11 (3) (Gha) for five-year
time period
h. Income derived by industries established in Special Economic Zones for five years or ten
years, subject to provisions of Sec. 11 (3Ka)
i. Dividend distributed by industries established at Special Economic Zone for five years
from the date of commercial operation
j. Income derived by a person involved in exploration and extraction of minerals, petroleum
or natural gas for five years from commercial operation date, if commences commercial
operation until 2080 Chaitra end
k. Income derived by a person involved in production, transmission or distribution of
electricity for ten years from commercial operation date, if commences commercial
operation until 2080 Chaitra end
l. Income derived by tourism industry or international flight operators for five years subject
to provisions of Sec. 11 (3Ta)
m. Dividend distributed by Special Industry or Tourism industry by capitalizing profit
n. Income derived by micro-industry for up to 7 years subject to provisions of Sec. 11 (3ta)
o. Income of Approved Retirement Fund
p. Compensation received by a resident natural person against physical injury sustained in
personal accident
q. Compensation against death of a natural person

41. Citizen Saving & Credit Co-operative Ltd, established under Co-operative Society Act,
207410, running its business at Lalitpur Sub-Metropolitan City earns Taxable Profit Rs.
500,000 & distributes dividend Rs. 200,000. Calculate dividend tax.
(June 2008, 1 Mark, CA Inter)
Answer:
The dividend is taxed at the rate 5% (Joint reading of Sec. 54, 88 (2) and Sec. 92). Therefore,
the amount of dividend tax is Rs. 10,000.

42. Explain the applicability of tax in following entities: (Dec 2006, 4×2=8 Marks, CA Inter)
a. Community-based saving & credit co-operative society
b. Agriculture income

10
Originally asked referring Cooperatives Act, 2048; which is now amended and as such, question is modified to
that extent

© The Institute of Chartered Accountants of Nepal 43


CAP II Paper 7 Income Tax and VAT

Answer:
(a) Community-based Saving and Credit Cooperative Society: Income of Cooperatives based
on rural municipality is exempt from tax. Such cooperatives’ income is subject to 5%
corporate tax when it is operated in municipality, 7% when in sub-metropolitan city and
10% when it is operated in sub-metropolitan or metropolitan city.
(b) Agriculture Income: Income from agriculture business (i.e. crop farming or leasing the
land for farming of crops) is exempt from tax, except in the following circumstances:
i. Agriculture business is operated by registering firm, partnership or organized
institutions, or
ii. Agriculture business is operated in such land which is held for the purpose of agro-
industrial purpose or industrial purpose u/s 12 (gha) and (Nga) of Land Related Act,
2021.

Concessions in Tax Rates


43. Himali Garments (Pvt.) Ltd. deals with export and local sales of readymade garments.
Based on the following information, determine the taxable income and tax liability from
local sales and export business. (June 2019, 5 Marks)
Particulars Amount (Rs.)
Total Sales 20 Million
Local Sales 5 Million
Opening Stock of Garments 3 Million
Cost of Production 17 Million
Closing Stock 4 Million
Export Expenses 0.5 Million
Promotional Expenses in Nepal 0.9 Million
Depreciation 1.6 Million

Answer:
Calculation of Tax Liability
Particulars Sec. Domestic Export Working Note
Ref Sales Sales
Sales income 7 (2) 5,000,000 15,000,000
Less:
Cost of Trading Stock 15 4,000,000 12,000,000 (Opening stock plus cost of
(3,000,000+17,000,000- production minus closing
4,000,000) in 1:3 ratio stock) in (sales Ratio11)
Export Expense 13 500,000
Promotional Expense 13 900,000 In Nepal, so domestic
expense
Depreciation 19 400,000 1,200,000 In sales ratio, i.e. 1:3

11
Sales ratio is used since there is no other reasonable basis. The best would be output ratio, if given.

© The Institute of Chartered Accountants of Nepal 44


Compilation of Suggested Answers Income Tax

Assessable Income from (300,000) 1,300,000 Loss cannot be set off, as


Business income from export is
deemed to be derived by
separate person [Sec. 11
(4)]
Tax rate 25% 20% Export income of a trading
concern, applying Sec. 11
(3Nga)
Tax Liability 260,000

44. Fine Distillery Pvt. Ltd. in Mustang District started its business of producing apple cider
with 11% alcohol by volume since 20X-69/X-70. For the first time during the income year
20X-72/X-73, company made taxable profit of Rs. 1,150,080 prior to the adjustment of
previous years assessed loss of Rs. 6,55,000. Calculate Tax Liability.
(June 2017, 2.5 Marks)
Answer:
As per Sec. 11 (3Ja), any industry producing brandy, cider and wine based on fruits in highly
undeveloped area are subject to 40% income tax exemption for a period of ten years from the
date of commercial operation.

Mustang is highly undeveloped area as per Industrial Enterprises Act, 2076; therefore, income
from production of Apple Cider by Fine Distillery Pvt. Ltd. in Mustang is subject to 40% tax
exemption for 10 years from 1st Shrawan 20X-69 (assuming commercial production started
that date). As IY 20X-72/X-73 is within such ten year’s period, the exemption is applicable to
taxable income of IY 20X-72/X-73.

Computation of CIT Liability:


Assessable income for the year 1,150,080
Less: Adjustment of Previous Year Loss (can be set-off as it’s not tax holiday) (655,000)
Assessable Income 495,080
(Taxable income as there is no reduction u/s 12, 12Ka or 12Kha)
Tax Liability before Exemption @ 30% 148,524
Less: Exemption @40% of Tax Liability (59,409.60)
Tax Liability for the year 89,114.40

45. Far-western food production Pvt. Ltd is a food manufacturing industry operating since
20X-66 located at Attariya, Kailali district, it is categorized as a special industry. It has
net12 taxable income amounting to Rs. 20 lakhs in Income Year 20X-72/X-73. You are
required to make tax assessment for the Income Year 20X-72/X-73.

12
There is no such term “net taxable income” as per Income Tax Act, the question setter mistook
it. It must be read as taxable income.

© The Institute of Chartered Accountants of Nepal 45


CAP II Paper 7 Income Tax and VAT

(Dec 2016, 2.5 Marks)


Answer:
13
Attariya, Kailali is located at underdeveloped area as per Indsutrial Enterprises Act, 2076.

Under Sec. 11 (3) (Kha), special industries located at under-developed areas are subject to
tax rate at the rate 30% of applicable tax rate for first ten Years from the date of
commencement of commercial operation. As per Sec. 11 (2Kha) (Ga), in case of a special
industry, the rates for further concessions under Sec. 11 shall be taken as the reduced tax rate
obtained after the application of Sec. 11 (2Kha), as such the effective tax rate for special
industry at Kailali is 6% for first ten years from the date of commencement of commercial
operation.

The commercial operation begun from 20X-66, as the question is silent about when the
commercial operation started, we assume that it started on 1st Shrawan 20X-66. The effective
tax rate of 6% is applicable for ten years until 31st Ashad 20X-76.

For IY 20X-73/X-73, the effective tax rate is 6%.


Taxable Income 2,000,000
Tax rate 6%
Tax Liability 120,000

46. Ramhari Rice Mill, a complete manufacturing industry has the following transaction
during the Income Year 20X-69/X-70. The Mill is a special industry under section 11 of
the Income Tax Act, 2058. The Mill is the sole proprietorship firm and the owner of the
Mill has not selected the Section 50 of the income Tax Act, 2058. You are required to
calculate the applicable tax rate and compute the tax liability based on the following
information.
(June 2014, 6 Marks)
Particulars Amount (Rs.)
Local sales 11,250,000
Export 3,750,000
Cost of sales 9,000,000
Indirect cost 500,000

Answer:
Calculation of Balance taxable Income (including Assessable Income & Taxable Income)
Particulars Sec. Domestic Export Working Notes
Ref Sales Sales

13
Questions should have clearly specified the area where the special industry is located. It was
technical error from part of Question-setter, as the question aimed at testing the knowledge of
Industrial Enterprises Act rather than the provisions of Income Tax Act.

© The Institute of Chartered Accountants of Nepal 46


Compilation of Suggested Answers Income Tax

Sales Income 7 (2) 11,250,000 3,750,000


Cost of Sales 15 6,750,000 2,250,000 Assuming as per Sec. 15 and
apportioned in Sales ratio (i.e. 3:1)
Indirect Cost 375,000 125,000 Assuming used for both, and
apportioned in sales ratio
Balance Taxable 4,125,000 1,375,000 No reductions u/s 12, 12Kha & 63
Income and deductions under Sec. 1 of
Schedule 1

Applicable Tax Rate and Tax Liability


Domes Tax on Tax on
Domestic Export Export Export
tic Domestic
Sales Sales Rate Sales
rate Sales
1st Rs. 400,000 300,000 100,000 0% 0% - -
Next Rs. 100,000 75,000 25,000 10% 7.5% 7,500 1,875
Next Rs. 200,000 150,000 50,000 20% 11.25% 30,000 5,625
Next Rs. 1,300,000 975,000 325,000 20% 11.25% 195,000 36,562.50
Balance Rs. 2,625,000 875,000 24% 13.5% 630,000 118,125

Total 1,024,687.50

47. Worthy Jute Industries Limited, a 100% export-oriented entity, is engaged in the
manufacturing of jute products. There are 1501 Nepali citizens working in the company
throughout the year. The accountant of the company computed the taxable income
amounting to Rs. 12,550,000 for F.Y. 20X-70/X-71 and income tax of Rs. 2,008,000. Being
a tax auditor, whether you agree with the tax amount calculated by the accountant of the
Company? Will your answer be different if there are 1,499 Nepali citizens working
throughout the year instead of 1,501? (June 2014, 5 Marks)
Answer
Worthy Jute Industry is Special Industry and its income is from export. The reduced tax rate
for Special industries is 20%.
As per Sec. 11 (3) (Ka), where a special industry provides direct employment to 1,000 or more
Nepali nationals throughout the income year, the effective tax rate of such special industry
shall be 70% of reduced tax rate, i.e. the effective tax rate for IY 20X-70/X-71 is 14% due to
this reason.
Similarly, when Sec. 11 (2kha) (Ga) is jointly read with Sec. 11 (3Nga) (Kha) and (Ga), the
effective tax rate by a manufacturing special industry is 12%.

© The Institute of Chartered Accountants of Nepal 47


CAP II Paper 7 Income Tax and VAT

As per Sec. 11 (5), where multiple exemptions under Sec. 11 are applicable, the exemption as
chosen by the taxpayer is applicable, and a rational taxpayer always chooses lower effective
tax rate.
Assuming that the special industry commenced its commercial operation before 8 years, the
corporate tax liability of Worthy Jute Industries Ltd.:
Taxable Income 12,550,000
Tax Rate 12%
Tax Liability 1,506,000

Conclusion:
The tax amount computed by the company’s accountant is not correct.

Answer to alternative question:


Even if the employee’s number was 1,499; the answer would not be different, as the effective
tax rate would be 12%.

48. Calculate the applicable tax rate on ABC & Co. based on following information:
(June 2012, 5 Marks)
a. ABC & Co. is registered as a special industry as provided in section 11 of Income Tax
Act, 2058.
b. The industry is situated in an underdeveloped area with an employment to 350 people.

© The Institute of Chartered Accountants of Nepal 48


Compilation of Suggested Answers Income Tax

Answer
The reduced tax rate for Special Industry operated by an entity, is 20% (Sec. 11 (2Kha) (kha)-
assuming ABC & Co. is an entity.
A special industry providing direct employment to 300 or more Nepali nationals throughout
the income year is entitled to effective tax rate of 80% of reduced tax rate, i.e. the effective tax
rate, if 300 or more employees out of 350 employees are Nepali national, is 16%.
A special industry located at underdeveloped area is entitled to effective tax rate of 20% of
reduced tax rate for first ten years from the date of commencement of commercial operation.
This means, if the special industry is being operated within 10 Years from the date of
commencement of commercial operation, the effective tax rate is 4%.
As per Sec. 11 (5), where multiple exemptions under Sec. 11 are applicable, the exemption as
chosen by the taxpayer is applicable, and a rational taxpayer always choose lower effective
tax rate.
Conclusion
If the special industry is being operated within first 10 Years of commencement of commercial
operation, ABC & Co., being an entity will pay tax @ 4% on taxable income.
If first ten years have been elapsed:
a. It will pay tax @ 16%, assuming 300 or more employees are Nepali National.
b. But if out of 350 employees, less than 300 employees but 100 or more employees are
Nepali nationals, it will pay tax at the rate 18%.
A special industry providing direct employment to 100 or more Nepali nationals
throughout the income year is entitled to effective tax rate of 90% of reduced tax rate, i.e.
the effective tax rate, if 100 or more employees out of 350 employees are Nepali national,
is 16%
c. If less than 100 employees are Nepali national, it will pay tax @ 20%.
If ABC & Co. was a sole trading firm of a natural person:
The taxable income of the natural person (assuming individual) would be subject to taxation
at following rates-

Within first 10 Income After first ten income years


Years including first >=300 <100
Particular income year of 300<employed
Nepali employed
commencement of Nepali
Nationals as Nepali
commercial operation National>=100
employee Nationals
1st Rs. 400,000 0% 0% 0% 0%
Next Rs. 100,000 2% 8% 9% 10%

© The Institute of Chartered Accountants of Nepal 49


CAP II Paper 7 Income Tax and VAT

Next Rs. 200,000 4% 16% 18% 20%


Next Rs. 4% 16% 18% 20%
1,300,000
Balance 4.8% 20% 22.5% 24%

49. A Company incorporated as a Limited Company and listed in SEBON having involved
in business of tourism sector approached you for getting your advice on tax implication.
Advise the Company giving consideration to the provisions of Income Tax Act, 2058.
(Dec 2011, 3 Marks, CA Inter)
Answer:
As per Sec. 11 (3Chha), an entity involved in tourism service, and listed in recognized stock
exchange is entitled to concessions in tax @ 15% of applicable tax rate. It means, as a result
of listing of tourism sector entity, effective tax rate is 85% of applicable tax rate.

The applicable tax rate for tourism industry is 25%.

Conclusion:
The company should pay tax @ 21.25% on its taxable income.

50. Foreign investors have earned Rs. 50,00,000 as foreign technology and management
service fee from the industry established in the Special Economic Zone.
(June 2011, 1 Mark)
Answer
As per Sec. 11 (3Ka) (Gha), the tax applicable on foreign technology and management service
fee from the industry established in the Special Economic Zone (SEZ) earned by foreign
investors is subject to concessions at the rate 50%.

If the form of income is passive (without active involvement of human resources in Nepal); the
industry at SEZ shall withhold tax at the rate 15% on any income before such concessions.
Amount earned by Foreign Investor 5,000,000
TDS @ 15% 750,000
Less: Concessions u/s 11 (3Ka) (Gha) (375,000)
Effective tax to be withheld 375,000

But, if the involvement is active, it is deemed that permanent establishment of foreign investors
exist in Nepal, and the foreign permanent establishment is subject to 50% concessions on
applicable tax on its income derived as foreign technology and management service fee from
SEZ. The tax exemption is given on look-through approach.

© The Institute of Chartered Accountants of Nepal 50


Compilation of Suggested Answers Income Tax

51. Income earned by the Entities listed in the securities market and engaged in the business
of production, Tourism sector. (June 2011, 1 Mark)
Answer
As per Sec. 11 (3Chha), an entity involved in tourism service, and listed in recognized stock
exchange is entitled to concessions in tax @ 15% of applicable tax rate. It means, as a result
of listing of tourism sector entity, effective tax rate is 85% of applicable tax rate.

52. As a tax consultant, you are required to advice your client with regards to concessions
and exemptions available in case of following: (June 2011, 2.5 Marks, CA Inter)
a. Person engaged in the business of exploration and excavation of Petroleum and
Natural gas.
b. Person receiving royalty income from the export of Intellectual Property and the
income from the sale/ transfer of Intellectual Property.
Answer:
a. Where a person is involved in extraction and exploration of minerals, petroleum or natural
gases commences its commercial operation until B.S. 2080 Chaitra end, there shall be
100% tax exemption for five years from the date of commencement of commercial
operation and 50% tax exemption for next three years [Sec. 11 (3Kha)]

b. Where a person exports intellectual property, such export income is subject to tax
exemption @ 25% [Sec. 11 (3Jha)], and where a person disposes intellectual property by
way of sales, such sales income is subject to tax exemption @ 50% [Sec. 11 (3Yna)].

53. A firm having three Partners, Mr. Ram, his wife and his adult son is engaged in
construction and operation Rope Way in Nepal. State the applicable tax rate.
(June 2008, 1 Mark, CA Inter)
Answer:
As per Sec. 11 (3Cha), in case an entity builds and operates ropeway in Nepal, the entity is
entitled to 20% exemption on tax rate. As such, the partnership firm is required to pay tax @
20% (25%-25% X 20%) on taxable income.

© The Institute of Chartered Accountants of Nepal 51


CAP II Paper 7 Income Tax and VAT

54. Explain the applicability of tax in Special Industry. (Dec 2006, 4 Marks, CA Inter)
Answer:
Sec. 11 (2Kha)
a. In case of a natural person: Instead of tax rate of 30%, the applicable tax rate on such
income shall be 20%.
b. In case of an entity: The income from special industry is subject to tax concession at the
rate 20% on applicable tax rate.
c. The reduced tax rate must be applied while determining other concessions under other
provisions of Sec. 11.
Sec. 11 (3), 11 (3Nga), 11 (3Chha), 11 (3Dha) (Any one out of the following)
Effective
Basis Facilities
Tax rate
Direct employment to 100 or more Nepali Tax to be payable, which is 18%
Nationals throughout the income year 90% of reduced tax rate
Direct employment to 100 or more Nepali Tax to be payable, which is 16.2%
Nationals throughout the income year, where 81% of reduced tax rate
one-third of work force is women, oppressed
and disabled persons
Direct employment to 300 or more Nepali Tax to be payable, which is 16%
Nationals throughout the income year 80% of reduced tax rate
Direct employment to 300 or more Nepali Tax to be payable, which is 14.4%
Nationals throughout the income year, where 72% of reduced tax rate
one-third of work force is women, oppressed
and disabled persons
Direct employment to 500 or more Nepali Tax to be payable, which is 15%
Nationals throughout the income year 75% of reduced tax rate
Direct employment to 100 or more Nepali Tax to be payable, which is 13.5%
Nationals throughout the income year, where 67.5% of reduced tax rate
one-third of work force is women, oppressed
and disabled persons
Direct employment to 1000 or more Nepali Tax to be payable, which is 14%
Nationals throughout the income year 70% of reduced tax rate
Direct employment to 1000 or more Nepali Tax to be payable, which is 12.6%
Nationals throughout the income year, where 63% of reduced tax rate
one-third of work force is women, oppressed
and disabled persons
Operated at highly undeveloped area, and the Tax to be payable, which is 2%
rate is applicable for first ten years from the 10% of reduced tax rate

© The Institute of Chartered Accountants of Nepal 52


Compilation of Suggested Answers Income Tax

date of commencement of commercial


operation
Operated at undeveloped area, and the rate is Tax to be payable, which is 4%
applicable for first ten years from the date of 20% of reduced tax rate
commencement of commercial operation
Operated at highly underdeveloped area, and Tax to be payable, which is 6%
the rate is applicable for first ten years from 30% of reduced tax rate
the date of commencement of commercial
operation
New Industry with capital investment of more Tax exemption for first five 0%
than Rs. 1 Billion and direct employment to years from the date of
more than 500 individuals commercial operation
Concessions @50% for next 10%
three years thereafter
Existing industry with capacity enhancement Tax exemption for first five 0%
by 25%, increment of capital investment to Rs. years from the date of
2 Billion and direct employment to more than commercial operation in the
300 individuals income from enhanced
capacity
Concessions @50% for next 10%
three years thereafter on
income from enhanced
capacity
Export by Special Industry Concession @ 20% 16%
Export by a Manufacturing special industry Additional Concession of 25% 12%
on rates derived after applying
concession @ 20%
Listing of Manufacturing Special Industry in Concessions @ 15% 17%
recognized stock exchange
Special industry involved in production and Concession @ 50% 10%
processing of domestic tea, clothes

If Multiple provisions as explained in the table applies simultaneously for a special industry,
such rate as per its choice should be applied for tax computation.

© The Institute of Chartered Accountants of Nepal 53


CAP II Paper 7 Income Tax and VAT

55. 14Calculate the tax liability of XYZ Co. Ltd. (An Industry situated in an under-developed
are with the employment of 650 people) on the basis of following information:(Dec 2003,
4 Marks, CA Inter)
XYZ Co. is registered as a special industry as provided in Sec. 11 of the Income Tax Act,
2058.
a. The income is Rs. 2,000,000
b. The eligible deduction is Rs. 1,500,000.
c. There is special fee of 3% of profit.
Answer:
Assumption: First ten years from date of commercial operation not elapsed, so the effective
tax rate is 6% (since the effective tax rate derived by applying concessions as a result of
employment to 500 or more Nepali nationals throughout the income year is 15%, which is
higher than 6%)
Computation of Taxable Income
Inclusions u/s 7 2,000,000
Deductions (eligible under Income Tax Act) (1,500,000)
Special Fee Expense (assumed to be expense- 500,000* 3%) (15,000)
(Alternatively, the figure can be taken as 500,000÷103 X 3)
Assessable Income from Business 485,000
(= Taxable income, as there is no reduction u/s 12, 12Ka, or 12Kha)
Tax liability @ 6% 29,100

Tax Holiday and Reduced Rates


56. Comfort Airways Pvt. Ltd. has been established in FY 20X-68/X-69 with total paid up
capital of Rs. 1.5 billion. Currently, as the domestic airliner, it has been operating all
flights by 4 Beach Crafts to different tourist destinations. These Beach Crafts are
acquired directly from the manufacturer on Operating Lease.
Of late, the management of the company is very much certain to earn additional profit
Rs. 7.5 million at the end of F.Y. 20X-73/X-74. To achieve this target, the company plans
to acquire one more 77-seater ATR air craft on finance lease for 15 years period to
expand its wings to India. This is possible subsequent to the intake of additional paid up
capital of Rs. 75 million for which all the shareholders are ready to invest.
As a tax consultant, the management wants to know any tax implication to the company
by adding the aircraft. (Dec 2017, 5 Marks)
Answer:
As per Sec. 11 (3Ta), an airline operating international flights, if commences it commercial
operation with capital investment of more than Rs. 2 Billion, the tax on such income is exempt

14
The question is reconstructed based on suggested answers.

© The Institute of Chartered Accountants of Nepal 54


Compilation of Suggested Answers Income Tax

for five years from the date of commencement of commercial operation and there shall be 50%
exemptions in income tax for three years thereafter.

For existing airlines, there is no such concessions even when they increase capital investment
and capacity of the airlines. As such, the company is not entitled to tax exemption under Sec.
11 (3Ta).

The company should pay tax at normal rate.

57. A Client incorporated as a Limited Liability Company wants to establish industries in


proposed Special Economic Zone (SEZ) in different districts approached you for the
suggestion or guidelines about tax benefits. What do you suggest him about tax benefits
available to him? (June 2012, 3 Marks, CA Inter)
Answer:
As per Sec. 11 (3Ka), Industries established at Special Economic Zone can enjoy following tax
benefits:
a. 100% tax exemption for first ten years from the date of commercial operation and 50% tax
exemption thereafter, if the industry is set up at SEZ located at Himali Region or Hilly
Regions prescribed by Government of Nepal,
b. 100% tax exemption for first ten years from the date of commercial operation and 50% tax
exemption thereafter, if the industry is set up at SEZ located at other areas as described
above
c. 100% tax exemption on distribution of profit for first five years from the date of commercial
operation and 50% tax exemption thereafter for next three years
d. 50% tax exemption on tax applicable on royalty and foreign investment or management
fee derived by foreign investor investing in industries located at SEZ
The above-mentioned concessions/exemptions are not available to any industry that is set up
using the machineries/equipment of another similar or any other industry [Sec. 11 (7)].

Set off of Losses


58. 15Mr. Sonu, a sole shareholder of M/s Sonu Industries Pvt. Ltd., was worried about the
performance of the Company as it incurred losses of Rs. 5 crores during the last 4
financial years ending F.Y. 20X-73/X-74. Mr. Rakesh, an expert acquired 60% stake in
the Company on Ashad 31, 20X-74. The company earned Rs. 1.5 crore as profit in F.Y.
20X-74/X-75. The Company has submitted the income tax return by assessing a taxable
loss of Rs. 3.5 crore for F.Y. 20X-74/X-75 under self-assessment by adjusting the carry
forward losses of Rs. 5 crores up to F.Y. 20X-73/X-74 u/s 20 of Income Tax Act, 2058.
The Chief Tax Officer issued an order to pay income tax on Rs. 1.5 crore along with

15
The question is replica of question asked in June 2014 Examinations of CAP-II Level, please refer Question 59
below.

© The Institute of Chartered Accountants of Nepal 55


CAP II Paper 7 Income Tax and VAT

interest thereon. The management of the Company seeks your advice on the said order
of Inland Revenue Office. (June 2019, 5 Marks)
Answer:
As per Sec. 20 (1), where a person has assessable income from any business or investment
during any Income Year, such income may be used to set off losses sustained by the same
person from any other business during the same income year or any unrelieved portion of such
losses sustained by the same person in the same or any other business in last seven income
year(s).

As per Sec. 57 (2) (Kha), where there is change in ownership by 50% of any entity while
compared with the ownership three years previously, the income generated by the entity after
the change of control cannot be used to set off any unrelieved loss sustained during the period
before such change in control.

As per Sec. 57 (1Ka), where the changes in ownership of such shareholders holding 1% or
more shares shall be counted to determine the threshold of 50%, and in case of persons holding
less than 1% shares, changes in respect of such holders who are associated to holders holding
1% or more of ownership shall only be counted.

Conclusion:
In the given case, there is change in 60% ownership of the entity, which means, the income
generated by the entity after the change of control cannot be used to set off any unrelieved loss
sustained during the period before such change in ownership.

As such, the company’s act of setting of losses of the period before such change from the
income derived after such change is not justifiable as per Sec. 57 (2) (Kha). The tax officer’s
contention is correct in this regard.

59. 16Mr. Shyam, a sole shareholder of M/s Ganpati Industries Pvt. Ltd., was worried about
the performance of the Company as it incurred losses to the tune of Rs. 5 crores during
last 4 financial years ending the F.Y. 20X-68/X-69. Mr. Mohan, an expert acquired 60%
stake in the Company on Ashad 31, 20X-69. Miracally, the company has managed to earn
Rs. 1.5 crore as profit in F.Y. 20X-69/X-70. The Company has submitted the Income Tax
Return by assessing a taxable loss of Rs. 3.5 crore for F/Y 2069/70 under self-assessment
by adjusting the carry forward losses of Rs. 5 crores up to F/Y 2068/69 u/s 20 of Income
Tax Act, 2058. The Chief Tax Officer issued an order to pay income tax on Rs. 1.5 crore
along with interest thereon. The management of the Company seeks your advice on the
said order of Inland Revenue Office. (June 2014, 4 Marks)

16
Similar question was repeated in June 2019, CAP-II Examinations. Refer Question 58 for the
same.

© The Institute of Chartered Accountants of Nepal 56


Compilation of Suggested Answers Income Tax

Answer:
As per Sec. 20 (1), where a person has assessable income from any business or investment
during any Income Year, such income may be used to set off losses sustained by the same
person from any other business during the same income year or any unrelieved portion of such
losses sustained by the same person in the same or any other business in last seven income
year(s).

As per Sec. 57 (2) (Kha), where there is change in ownership by 50% of any entity while
compared with the ownership three years previously, the income generated by the entity after
the change of control cannot be used to set off any unrelieved loss sustained during the period
before such change in control.

As per Sec. 57 (1Ka), where the changes in ownership of such shareholders holding 1% or
more shares shall be counted to determine the threshold of 50%, and in case of persons holding
less than 1% shares, changes in respect of such holders who are associated to holders holding
1% or more of ownership shall only be counted.

Conclusion:
In the given case, there is change in 60% ownership of the entity, which means, the income
generated by the entity after the change of control cannot be used to set off any unrelieved loss
sustained during the period before such change in ownership.
As such, the company’s act of setting of losses of the period before such change from the
income derived after such change is not justifiable as per Sec. 57 (2) (Kha). The tax officer’s
contention is correct in this regard.

60. Singha Road Ltd. located at Hetauda is a public infrastructure project. The project will
be completed on Chaitra end 20X-75 as per estimation, and then it will be handover to
Government of Nepal. The company has incurred loss continuously from Income Year
20X-61/X-62 to 20X-68/X-69 as follows:
Income Year Loss amount Rs.
20X-61/X-62 1,000,000
20X-62/X-63 500,000
20X-63/X-64 600,000
20X-64/X-65 200,000
20X-65/X-66 150,000
20X-66/X-67 100,000
20X-67/X-68 75,000
20X-68/X-69 10,000
During income year 20X-69/X-70, it has incurred the profit of Rs. 1,500,000. Compute
the taxable income for the income year 20X-69/X-70 and carry forward losses for the

© The Institute of Chartered Accountants of Nepal 57


CAP II Paper 7 Income Tax and VAT

income year 20X-70/X-71 for set off with references to the provisions of Income Tax Act,
2058 relevant to this project. (June 2014, 5 Marks)

Answer:
As per Sec. 20 (1), where a person has assessable income from any business or investment
during any Income Year, such income may be used to set off losses sustained by the same
person from any other business during the same income year or any unrelieved portion of such
losses sustained by the same person in the same or any other business in last seven income
year(s).
As per the proviso of Sec. 20 (1), losses sustained by public infrastructure projects that are
ultimately handed over to Government of Nepal can be carried forward until 12 Years, if built
and operated by the same person..

Testing the case with the provisions of law


The question is not clear whether Singha Road Ltd. is a construction company who works as
contractor or it is undertaking the project under Build and Operate modality. We assume the
latter is the case.

In such scenario, the income of income year 20X-69/X-70 can be used to set off any unrelieved
losses from the same projects since last 12 income years.

Conclusion
Assessable income of IY 20X-69/X-70 before set off of losses from Previous Years 1,500,000
Less: Losses of the project of since IY 20X-57/X-58 (2,635,000)
Losses for the year (1,135,000)

This loss is in relation to Income Year 20X-63/X-64 and onwards. The carried forward loss
eligible for set off from income of IY 20X-70/X-71 is Rs. 1,135,000.

Alternatively:
If we assume that the company is a construction company and is not undertaking the project
under build and operate modality, losses of last seven income years can only be set off from
income of income year 20X-69/X-70, which means losses since IY 20X-62/X-63 can only be set
off. Total losses since IY 20X-62/X-63 is Rs. 1,635,000; and available income is Rs. 1,500,000,
this can only set off all losses of IY 20X-62/X-63 to IY 20X-65/X-66 and Rs. 50,000 of IY 20X-
66/X-67.
The carried forward loss eligible for set off from income of IY 20X-70/X-71 is Rs. 1,135,000,
belonging to IY 20X-66/X-67 and onwards.

© The Institute of Chartered Accountants of Nepal 58


Compilation of Suggested Answers Income Tax

61. Answer the followings given facts based on the provisions of the Income Tax Act, 2058.
The answer must also consist of implication as per the Act and opinion/ recommendation.
a. The BB Company has made a loss of Rs. one million from the sale of investment made
in shares of another resident company and claims such loss as deductible expenses
under section 13 of the Act as normal business expenses.
(June 2007, 1.67 marks, CA Inter)
Answer
Such loss is not deductible expense under Sec. 13, since it is loss on disposal of business
asset. As per Sec. 36, such loss can be set off from gain on disposal of any other business
asset or business liability of M/s BB Company.

b. Mr. A. Sharma purchased equity shares in Dreamland Hotel Ltd. on Ashad 15, 20X-
61. He sold the share on Ashad 21, 20X-62 at a loss of Rs. 65,000. He wants to set off
the loss against his business income during the year. Examine whether such set off is
permissible. (June 2006, 5 marks, CA Inter)
Answer
Such loss is not deductible from business income. Only the business loss can be set off from
business income [Sec. 20 (1) jointly read with sec. 20 (2)].

Similarly, investment losses can be set off from investment income, with maximum carry
forward to next seven income years [Sec. 20 (2)]
Equity shares of Dreamland Hotel Ltd. for Mr. Sharma is “non business chargeable asset”.
Loss on disposal of NBCA can be set off from gain on disposal of NBCA [Sec. 36 (2)]
Conclusion
Based on the above facts, Mr. Sharma cannot set off loss on sales of shares with business
income or investment income, but can only set it off from gain on disposal of any other
Non-Business Chargeable asset.

62. Ms. S. Sharma had placed to fixed deposit of Rs. 15 lakhs in her personal capacity in a
bank on which she received interest of Rs. 90,000. She had also borrowed Rs. 5 lakhs
from the same bank on the security of fixed deposit to conduct her business and was liable
to pay Rs. 45,000 by way of interest to the bank. She therefore, offered the difference
between two amounts of Rs. 45,000 as income from investment. Is this correct?
(June 2006, 5 marks, CA Inter)
Answer
The interest income on fixed deposit (not on business capacity) in bank (assuming resident) is
final withholding. The tax withheld for final withholding payments cannot be treated as
advance tax [Sec. 93 (2) jointly read with Sec. 92 (2)]

© The Institute of Chartered Accountants of Nepal 59


CAP II Paper 7 Income Tax and VAT

The interest expense paid by her is business expense. As per Sec. 14, in case an interest income
is incurred on loan used for business purpose, the expense is deductible while calculating
assessable income from business.

Therefore, Ms. Sharma’s intent to set off interest expense from interest income is not correct.

63. Explain the provision and procedure of carry back of losses.


(Dec 2004, 5 Marks, CA Inter)
Answer:
As per Sec. 20 (4), if all the following conditions are satisfied, any loss sustained during latter
income year can be set off from the income of previous income years:
a. The loss shall be related to “Long term Contract”,
b. The long-term contract shall be awarded to the contractor on global bidding,
c. The loss shall be sustained by the contractor at the time of completion of contract, or at
such time when the contract is otherwise disposed,
d. There shall be sufficient taxable income from the same contract in previous income year;
i.e. set off is allowed from previous years’ taxable income of same contract;
e. There is written permission from Inland Revenue Department to set off such losses.

Others
64. Discuss the taxability or otherwise of the following receipts by quoting the relevant
sections of the Income Tax Act 2058. (June 2005, 10 Marks, CA Inter)
a. Sri Ram Bahadur Khadga receives a pension of Rs. 5,000 per month from Kolkata
Police department.
Answer:
The amount is exempt under Sec. 10, if the pension is paid from public fund of Indian
Government and Mr. Khadga is Nepali citizen. Otherwise, it is taxable under head
employment.

b. Sri Dinesh Basnyat receives an award of Rs. 100,000 for exemplary services during
his term in Lebanon from United Nations through the Nepal Army Headquarters.
Answer:
The amount is taxable as windfall gain @ 25%.

c. Sri Man Bahadur was a student with Mr. Bhattarai, who is a teacher by profession.
He gets admitted to Harvard University. After successfully completing his Ph. D. he
returns to Nepal and in token of his respect to the teacher for his blessings and
teachings before proceeding to USA, he pays Rs. 100,000 to Mr. Bhattarai on his
return from USA.
Answer
The amount is taxable as windfall gain.

© The Institute of Chartered Accountants of Nepal 60


Compilation of Suggested Answers Income Tax

d. Mr. Rudra received a dividend of Rs. 50,000 from the dividend distributed by B co.
Ltd. B co. Ltd. distributed a dividend of Rs.600,000 out of Rs.950,000 received as
dividend form A. co. Ltd. who declared a dividend of Rs.2,500,000 and distributed
the dividend after deducting tax at source.
Answer:
As per Sec. 54 (3), dividend tax is not applicable on any distribution made out of such
profits where distribution tax u/s 54 (1) is already levied. Therefore, the amount received
by Mr. Rudra is not taxable.

e. Mr. S. Shrestha is the director of various companies. He receives fee for attending the
Board Meeting. The total fee earned by him for attending Board Meetings during the
income year was Rs.90, 000.
Answer
The amount is taxable, and it is a final withholding payment (assuming per meeting fee is
less than Rs. 20,000). As per Sec. 88, the TDS rate is 15%.

© The Institute of Chartered Accountants of Nepal 61


CAP II Paper 7 Income Tax and VAT

Chapter 6- Characterization, Allocation and Quantification


This Chapter includes:

• Concept and definition of Characterization, allocation and Quantification


• Quantification provisions under Section 27
• Quantifications of Annuities, Leases and Installment Sales under Section 32
• Other Re-characterization including Foreign Exchange Transactions, Joint investment of
individuals, Compensations, indirect payments, transfer pricing arrangements, income
splitting and General Anti-avoidance rules

Quantification of Perquisites (Section 27)


65. Quantify the value of consideration as per Section 27 of Income Tax Act, 2058 under the
following circumstances: (Dec 2018, 5×1=5)
a. Mr. Ram gave his smart TV for settlement of payables to a shopkeeper. Mr. Ram had
purchased the TV for Rs. 45,000 three years back, and the market value at the time
of handing over to the shop was Rs. 25,000.
Answer
As per Sec. 27 (1) (Ka), where consideration is payable in kind, the market value of such
kind is taken as the value of consideration.
As such, the value of consideration paid by Mr. Ram is Rs. 25,000.

b. XYZ limited provides free motor vehicle costing Rs. 15,000,000 to Mr. Ganpath, a
government official.
Answer
As per Sec. 27 (1) (Kha) (1) jointly read with Rule 13 (1) (Kha), the amount be quantified
is 1% p.a. of market value of asset.
In this case, market value of motor vehicle is not given, therefore, quantification cannot be
done.

c. PQR Limited provides two security personnel free of cost to Mr. Mohan, a marketing
manager, for which the company pays Rs. 240,000 for a year.
Answer
As per Sec. 27 (1) Nga), the amount to be quantified is the value of benefit received by the
person. As such, Rs. 240,000 should form part of income of Mr. Mohan.

d. K&K Ltd. provides free accommodation to a public director of the Company, for
which the Company pays annual rent Rs. 500,000.
Answer
Sec. 53 jointly read with Sec. 56 (3) and Rule 18 relieves a company from quantifying any
facilities in the form of availability of its assets to its beneficiaries. As such, quantification

© The Institute of Chartered Accountants of Nepal 62


Compilation of Suggested Answers Income Tax

is not required. The company cannot claim any expense in connection to providing such
facilities for tax purpose.

e. K&K Ltd. provides its own bungalow costing Rs. 20,000,000 as free accommodation
to a local leader, normal rent for the bungalow is Rs. 100,000 per month.
Answer
As per Sec. 27 (1) (Kha) (2) read together with Rule 13 (2) (Kha), the amount to be
quantified in such case is 25% of actual rent paid. As such, the local leader is deemed to
receive income of Rs. 25,000 per month against such facility.

66. Quantify the value of perquisite as per the provision of Income Tax Act, 2058 provided
by PQR Ltd. to the employee and others as under: (Dec 2017, 5 Marks)
a. The company pays Rs. 35,000/month for a furnished accommodation facility to the
Executive Officer (Mr. Lama). The value of the furniture and equipment is Rs.
1,50,000. Mr. Lama’s basic salary and grade was Rs. 1,00,000 p.m. during the
financial year 20X-73/X-74.
Answer
As per Sec. 27 (1) (Kha) (2) read together with Rule 13 (2) (Ka), the amount to be quantified
in such case 2% of Khai paai aaeko talab of the employee.
Khai paai aaeko talab has been interpreted by Income Tax Directive issued by Inland
Revenue Department as sum of basic salary and grade drawn by the individual for the
income year from the employer providing such facility.
As such the amount to be quantified as income of Mr. Lama is 2% of Rs. 1,200,000
(assuming he drew salary throughout the year), i.e. Rs. 24,000.

b. The company pays Rs. 15,000/month for a flat provided to Program Officer, Mr.
Tamang. Mr. Tamang’s basic salary and grade was Rs. 50,000 per month during the
financial year 20X-73/X-74.
Answer
As per Sec. 27 (1) (Kha) (2) read together with Rule 13 (2) (Ka), the amount to be quantified
in such case 2% of Khai paai aaeko talab of the employee.
Khai paai aaeko talab has been interpreted by Income Tax Directive issued by Inland
Revenue Department as sum of basic salary and grade drawn by the individual for the
income year from the employer providing such facility.
As such the amount to be quantified as income of Mr. Tamang is 2% of Rs. 600,000
(assuming he drew salary throughout the year), i.e. Rs. 12,000.

© The Institute of Chartered Accountants of Nepal 63


CAP II Paper 7 Income Tax and VAT

c. The company pays Rs. 1,00,000/month for a flat provided to Mr. Thapa, a board
director. The value of the furniture and equipment provided by the company in the
rented flat was Rs. 5,00,000.
Answer
Sec. 53 jointly read with Sec. 56 (3) and Rule 18 relieves a company from quantifying any
facilities in the form of availability of its assets to its beneficiaries. As such, quantification
is not required. The company cannot claim any expense in connection to providing such
facilities for tax purpose.

d. Free four-wheeler facility is provided to the Executive Officer (Mr. Lama). The
company meets the cost of fuel and maintenance charge of the vehicle provided to Mr.
Lama. The cost of fuel and maintenance charges paid by the company was Rs.
2,50,000 and the value of four-wheeler was Rs. 75,00,000. Mr. Lama’s basic salary
and grade was Rs. 100,000 per month during the financial year 20X-73/X-74.
Answer
As per Sec. 27 (1) (Kha) (1) read together with Rule 13 (1) (Ka), the amount to be quantified
in such case 0.5% of Khai paai aaeko talab of the employee.
Khai paai aaeko talab has been interpreted by Income Tax Directive issued by Inland
Revenue Department as sum of basic salary and grade drawn by the individual for the
income year from the employer providing such facility.
As such the amount to be quantified as income of Mr. Lama is 0.5% of Rs. 1,200,000
(assuming he drew salary throughout the year), i.e. Rs. 6,000

e. The company meets the cost of fuel and maintenance charge of vehicle provided to
Mr. Thapa, the board director. The cost of fuel and maintenance charges paid by the
company was Rs. 3,00,000 and the value of four-wheeler was Rs. 1,00,00,000.
Answer
Sec. 53 jointly read with Sec. 56 (3) and Rule 18 relieves a company from quantifying any
facilities in the form of availability of its assets to its beneficiaries. As such, quantification
is not required. The company cannot claim any expense in connection to providing such
facilities for tax purpose.

67. Mr. A has been working in the capacity of senior manager for XYZ Co. Ltd. The
company provided him a Volkswagon car for official as well as personal use. The current
market price of the car is Rs. 5,700,000. Details of Mr. A’s remuneration during the
Income year 20X-66/X-67 are as follows:
Salary Rs. 1,50,000 per month
Grade Rs. 75,000 per annum
Allowance Rs. 7,000 per month
Other Allowances Rs. 9,000 per month

© The Institute of Chartered Accountants of Nepal 64


Compilation of Suggested Answers Income Tax

Quantify the amount for the vehicle facility provided for computing Mr. A’s taxable
income from remuneration for the F.Y. 20X-66/X-67.
Will the answer be different in case XYZ Co. Ltd. hired Mr. A for four months in the
capacity of consultant for a fee of Rs. 700,000 in total? (Dec 2010, 5 Marks)
Answer
As per Sec. 27 (1) (Kha) (1) read together with Rule 13 (1) (Ka), the amount to be quantified
in such case is 0.5% of Khai paai aaeko talab of the employee.
Khai paai aaeko talab has been interpreted by Income Tax Directive issued by Inland Revenue
Department as sum of basic salary and grade drawn by the individual for the income year from
the employer providing such facility.
As such the amount to be quantified as income of Mr. Lama is 0.5% of Rs. 1,875,000 (assuming
he drew salary throughout the year), i.e. Rs. 9,375.
If he were a consultant:
As per Sec. 27 (1) (Kha) (1) read together with Rule 13 (1) (Kha), the amount to be quantified
in such case is 1% p.a. of market value of motor vehicle.
Therefore, amount to be treated as part of his service fee is:
Market Value 5,700,000
1% of above 57,000
To be quantified (for four months) [57,000÷12 X4] 19,000

Exchange Gain Loss


68. Oppo International Pvt. Ltd., Kathmandu is a dealer of Oppo Electronics. The Oppo
Electronics are manufactured in China by Oppo International. The company imported
the electronics goods worth of US $ 50,000 on 23 Baishakh, 20X-74, the payment was
done on 23 Shrawan, 20X-74. The exchange rate of the transaction was as follows:
Baishakh 23: Rs. 104 per US $
Closing of Ashad: Rs. 107 per US $
Shrawan 23: Rs. 106 per US $
You are required to compute the amount for inclusions or deductions for the tax
assessment of this transaction mentioning the relevant provisions of Income Tax Act,
2058. (Dec 2017, 5 Marks)
Answer
As per Sec. 28 of Income Tax Act, 2058; the accounting of foreign currency transaction shall
be done on spot rate, but when allowed by IRD, average rate prescribed by IRD may be used
to translate any foreign currency transaction into Nepalese currency.
In the given case, the company would pass the following entry as on Baisakh 23, 20X-74 to
record the purchase transaction for tax purpose:
Purchase A/c Dr. [ $50,000 X Rs. 104/$) 5,200,000
To Payable A/c [Payable in dollar in shrawan 23] 5,200,000

© The Institute of Chartered Accountants of Nepal 65


CAP II Paper 7 Income Tax and VAT

There will be no entries for tax purpose on Ashad end, as the foreign currency is translated
using spot rate.

The following entry will be passed on Shrawan 23, 20X-74 to record the payment to suppliers:
Payable A/c Dr. 5,200,000
Loss on Disposal of Business Liability Dr. 100,000
(Difference between actually paid and payable)
To Bank A/c [$50,000 X 106 Rs./$] 5,300,000

Conclusion:
The resulting gain/loss on settlement of foreign currency asset or liability is claimed as
deduction (can only be set off against any gain on disposal of any other business asset or
business liability) during the year of settlement.

Joint Investment
69. 17Saina and Safina decided to jointly invest Rs. 2,000,000 and Rs. 3,000,000 respectively.
Out of the collected amount, they bought land for Rs. 4,500,000 with additional
registration and other expenses Rs. 500,000. They sold the land for Rs. 8,600,000, out of
which, they spent Rs. 600,000 for commission and other expenses. Calculate their taxable
income as per Income Tax Act, 2058?
Compute the amount to be included as income in the hand of Mr. A and Mr. B.
(Dec 2018, 5 Marks)
Answer
As per Sec. 30 of Income Tax Act, 2058; where two or more natural persons jointly invest in
owning an investment asset, the incomings and outgoings in relation to such an asset shall be
apportioned between the co-investors in their investment ratio.

Investment ratio in the given case is 2:3.


Allocation of Incomings and Outgoings and Gain accrued to each individual
Particulars Total Saina Safina
Incomings from Sales of Land 8,600,000 3,440,000 5,160,000
Outgoings of Land
Purchase cost 4,500,000
Add: Registration and Other Expenses 500,000
Add: Commission and Other Exp. on Sales 600,000
5,600,000 2,240,000 3,360,000
Gain on Disposal of Land (to be included 3,000,000 1,200,000 1,800,000
as income in each other’s hand)

17
Similar question was asked in December 2010 CAP II Examination.

© The Institute of Chartered Accountants of Nepal 66


Compilation of Suggested Answers Income Tax

70. 18Mr. A and Mr. B contributed Rs. 4 million and Rs. 6 million respectively and deposited
into a common bank account to be used to purchase a plot of land and to resale the same
in the future. After 6 months of depositing the money into the bank account, they
purchased a plot of land for Rs. 9 million and got the land registered in their joint name.
Expenses on registration and commission at the time of purchasing the land are
amounted to Rs. 1 million. Later, they sold the plot of land for Rs. 17.2 million. Also, they
paid Rs. 1.2 million as sales commission and other incidental expenses pertinent to the
disposal. The bank has given interest @ 12% per annum on such deposit.(June 2018, 5
Marks)
Answer
As per Sec. 30 of Income Tax Act, 2058; where two or more natural persons jointly invest in
owning an investment asset, the incomings and outgoings in relation to such an asset shall be
apportioned between the co-investors in their investment ratio.

Investment ratio in the given case is 2:3.


Allocation of Incomings and Outgoings and Gain accrued to each individual
Particulars Total Mr. A Mr. B
Incomings from Sales of Land 17,200,000 6,880,000 10,320,000
Outgoings of Land
Purchase cost 9,000,000
Add: Registration and Other Expenses 1,000,000
Add: Commission and Other Exp. on Sales 1,200,000
11,200,000 4,480,000 6,720,000
Gain on Disposal of Land (to be included 6,000,000 2,400,000 3,600,000
as income in each other’s hand)

Treatment of Interest income from Bank


Applying the standard set forth by Sec. 30 of Income Tax Act, 2058; the interest income from
saving account of bank is treated as received by Mr. A and Mr. B from jointly held property in
their investment ratio. As such, the interest income is final withholding in the hand of Mr. A
and Mr. B.

71. Ram and Krishna had contributed Rs. 20 lakhs & Rs 30 lakhs respectively and deposited
the amount into a common bank account to be used to purchase a plot of land to resale
the same later on. After 6 months of depositing the money into the bank account, they
purchased a plot of land for Rs. 45 lakhs and got the land registered in their joint name
(i.e. in the name of both Ram & Krishna jointly). Expenses on registration and
commission at the time of purchase of land amounted to Rs. 500,000. They later on sold
that plot of land at Rs. 86 lakhs and they have paid Rs. 6 lakhs as sales commission and

18
Similar question was asked in June 2012 CAP II Examination.

© The Institute of Chartered Accountants of Nepal 67


CAP II Paper 7 Income Tax and VAT

other incidental expenses related with sales. The bank has given interest @18% per
annum on such bank deposit.
Calculate the amount to be incorporated as “income” in the hand of Ram and Krishna
respectively from above mentioned transaction. How much would be the interest income
and how it is treated in the hand of Ram & Krishna? (June 2012, 5 Marks)
Answer
As per Sec. 30 of Income Tax Act, 2058; where two or more natural persons jointly invest in
owning an investment asset, the incomings and outgoings in relation to such an asset shall be
apportioned between the co-investors in their investment ratio.

Investment ratio in the given case is 2:3.


Allocation of Incomings and Outgoings and Gain accrued to each individual
Particulars Total Ram Krishna
Incomings from Sales of Land 8,600,000 3,440,000 5,160,000
Outgoings of Land
Purchase cost 4,500,000
Add: Registration and Other Expenses 500,000
Add: Commission and Other Exp. on Sales 600,000
5,600,000 2,240,000 3,360,000
Gain on Disposal of Land (to be included 3,000,000 1,200,000 1,800,000
as income in each other’s hand)

Treatment of Interest income


Applying the standard set forth by Sec. 30 of Income Tax Act, 2058; the interest income from
saving account of bank is treated as received by Mr. Ram and Mr. Krishna from jointly held
property in their investment ratio. As such, the interest income is final withholding in the hand
of Ram and Krishna.

72. Mr. Ram & Mr. Shyam jointly owned a house property costing Rs. 50,000,000. At the
time of acquisition of the said house property, Mr. Ram had invested Rs. 1 crore and rest
of the amount invested by Mr. Shyam. This house property has been sold out for Rs.
64,000,000, and incurred Rs. 700,000 on account of brokerage commission plus other
incidental charges.
You are required to appropriate between Mr. Ram & Mr. Shyam- as per sec 30 of Income
Tax Act 2058- for the net income that originated from the jointly owned investment.
(Dec 2010, 5 Marks)

© The Institute of Chartered Accountants of Nepal 68


Compilation of Suggested Answers Income Tax

Answer
As per Sec. 30 of Income Tax Act, 2058; where two or more natural persons jointly invest in
owning an investment asset, the incomings and outgoings in relation to such an asset shall be
apportioned between the co-investors in their investment ratio.

Investment ratio in the given case is 1:4 for Ram and Shyam.
Allocation of Incomings and Outgoings and Gain accrued to each individual
Particulars Total Ram Shyam
Incomings from Sales of house property 64,000,000 12,800,000 51,200,000
Outgoings of House Property
Purchase cost 50,000,000
Add: Commission and Other Exp. on 700,000
Sales 50,700,000 10,140,000 40,560,000
Gain on Disposal of Land (to be included 13,300,000 2,660,000 10,640,000
as income in each other’s hand)

Transfer Pricing, Income Splitting and GAAR


73. AB (Pvt.) Ltd has made an agreement with ABC (Pvt.) Ltd to arrange the cash flow for
its transactions. Chairman of AB (Pvt.) Ltd. is also a representative board member of
ABC (Pvt.) Ltd. In Shrawan 1, 20X-72, loan amounting Rs. 50 million had been provided
by the ABC to the AB. Interest rate mutually agreed was 10 percent. Both the companies
are located at Kathmandu. Tax Officer denied the interest expenses of Rs. 5 million claimed by
AB (Pvt.) Ltd. on the ground that this transaction is a related party's transaction. Answer whether
the Tax Officer's assessment is correct. (June 2018, 5 Marks)
Answer
As per Sec. 33 of Income Tax Act, 2058; in any arrangement between persons who are
associates, the Department may, by notice in writing, distribute, apportion, or allocate
amounts to be included or deducted in calculating income and foreign income tax paid between
the persons as is necessary to reflect the taxable income or tax payable that would have arisen
for them if the arrangement had been conducted at arm’s length.

This means, where the transactions are conducted in arm’s length, IRD cannot use the power
under Sec. 33 to allocate, apportion or distribute any amounts to be included or deducted;
even if the transaction is between related parties or associated persons.

Deduction of interest expense is dealt by Sec. 14 of Income Tax Act, 2058; and so far the loan
on which interest expense is incurred satisfies conditions under Sec. 14 (1); the total interest
expense is deductible subject to Sec. 14 (2).

Sec. 14 (2) limits the amount of interest expenses to be deductible where a resident entity
controlled by exempt organization pays interest to its holder of shares and that are treated as

© The Institute of Chartered Accountants of Nepal 69


CAP II Paper 7 Income Tax and VAT

exempt entities under the clarification clause of Sec. 14. Provisions of Sec. 14 (2) are in
deferral in nature, not in restriction of allowances of interest expenses.

As such, if conditions of Sec. 14 (1) are satisfied and subject to provisions of Sec. 14 (2), IRD
cannot deny the deduction of interest expenses in the name of “related party transaction”.

74. What do you mean by Income Splitting? Mention the right available to Inland Revenue
Department to prevent any reduction in tax payable as a result of the splitting of income.
(June 2010, 5 marks)
Answer
Meaning of Income Splitting:
As per Sec. 34 (2); a person having attempted to split income include a reference to a transfer
of the following amounts, either directly or indirectly through one or more interposed entities,
between the person and the associate so as to lower the total tax payable by the person or an
associate:
a. Amounts to be received or costs to be incurred, or
b. An amount received or enjoyed by the transferee of an asset that is derived from the asset;
or an amount paid or expenses incurred in owning the asset.
In determining whether a person is seeking to split income, the Department shall consider the
market value of any payment made for the transfer
Power of Inland Revenue Department
Where a person attempts to split income with another person, and in case that will cause a
reduction in the amount of tax payable, the Department may, by notice in writing, adjust
amounts to be included or deducted in calculating the income of each person to prevent any
such reduction in the amount of tax payable.

75. What is the Tax Avoidance Scheme? Mention the power available to Inland Revenue
Department under General Anti Avoidance Rule for determining tax liabilities.
(Dec 2009, 5 Marks)
Answer
Meaning of Tax Avoidance Scheme
For the purpose Section 35, “tax avoidance scheme” means any arrangement, one of the main
purposes of which is the avoidance or reduction of liability to tax of any person.

Power of Inland Revenue Department


For the purposes of determining tax liability under this Act, the Department may:
a. Re-characterize an arrangement or a part of an arrangement that is entered into or sought
to be entered into as part of a tax avoidance scheme,
b. Disregard an arrangement or a part of an arrangement that does not have substantial
economic effect, or

© The Institute of Chartered Accountants of Nepal 70


Compilation of Suggested Answers Income Tax

c. Re-characterize an arrangement or a part of an arrangement that does not reflect its


substance

© The Institute of Chartered Accountants of Nepal 71


CAP II Paper 7 Income Tax and VAT

Computation of Taxable Income

Mix of all Income Heads


76. Mr. Nayak Shakya, who is going to retire from 1st Shrawan 20X-67, submits the following
details with respect to his employment for the Income Year 20X-66/X-67.
19Net Salary received (after TDS) Rs. 4,57,000
TDS Paid in respect of Salary Rs. 37,000
Contribution by employer to retirement fund (approved) Rs. 1,00,000
His Contribution to approved retirement fund Rs. 1,50,000
Compensation received as per Employee Rule Rs. 70,000
Investment Insurance Premium paid by Employer on his behalf Rs. 19,000
(insured sum of Rs. 2,00,000)
Encashment of un-availed leave upto Chaitra 18, 2058 Rs. 50,000
Encashment of un-availed leave after Chaitra 18, 2058 Rs. 1,70,000
Emergency Medical Treatment paid by Employer Rs. 10,000
(He didn’t claim Medical Tax Credit)
Reimbursement of Tour Expenses during Official visit to Rs. 2,40,000
Hongkong
Gratuity Received (lumpsum payment)
Upto 18 Chaitra, 2058 Rs. 4,00,000
After 18 Chaitra, 2058 (approved) Rs. 7,00,000
Provident Fund Received upto Chaitra 18, 2058 (Lumpsum Rs. 4,00,000
Payment)
(unapproved)
Provident Fund Received After Chaitra 18, 2058 (approved) Rs. 6,00,000

Mr. Nayak also conducts a small trading business M/S Nayak Concerns of his own.
Information relating to which are as follows:
Particulars Rs. Particulars Rs.
To Opening Stock 80,000 By Sales 5,00,000
To Purchase 3,90,000 By Closing Stock 1,20,000
To Gross Profit 1,50,000
Total 6,20,000 Total 6,20,000
To Personal Drawings 60,000 By Gross Profit 1,50,000
To Office Expenses 18,000 By Office Furniture 35,000
Sales

19
Question modified to make it more understandable to student

© The Institute of Chartered Accountants of Nepal 72


Compilation of Suggested Answers Income Tax

To Printer Purchased 7,000 By Bad Debt 1,00,000


Recovered
To Net Profit 2,50,000 By Dividend 50,000

Ms. Nayak works in a private institution & earns a total taxable salary of Rs. 2,00,000.
The couple has opted to be assessed separately for Income Tax Purpose.
Additional Information:
a. M/S Nayak Concern has the following WDV of Fixed Assets as on 01.04.20X-66.
Furniture Rs. 1,00,000
Computer Rs. 45,000
Motorcycle Rs. 80,000
One Spare Table is sold on Rs. 35,000
b. Above Office Expenses includes Rs. 2,000 for the purpose of rice cooker gifted to a
customer on his wedding party.
c. Printer was purchased on 15th Poush, 20X-66.
d. Bad debt Recovered Rs. 1,00,000 was not allowed as expenses in F.Y 20X-64/X-65.
e. Dividend Rs. 50,000 is from NIC Bank Ltd.
He also provides following further information not related to his employment, and
business.
d. Gain Rs. 1,00,000 on Sale of shares.
e. Donation to a political Party Rs. 50,000.
f. Interest of Rs. 20,000 from personal deposit account of NATURAL BANK LTD.
Required: (Dec 2009, 20 Marks)
a. Assessable Income from Employment
b. Assessable Income from Business
c. Total Assessable Income
d. Tax Liability
Answer:
Computation of Assessable Income from Employment
Particulars Sec. Amount Working Notes
Ref
Net Salary (After TDS) 8 (2) 457,000
TDS paid in respect of salary 8 (2) 37,000 Reimbursement of Personal
Expense
Employers’ Contribution to 8 (2) 100,000
Retirement Fund
Compensation received as per 8 (2) 70,000 Employment related compensation
Employee Rule & 31 – assumed not during employment

© The Institute of Chartered Accountants of Nepal 73


CAP II Paper 7 Income Tax and VAT

Investment insurance premium paid 8 (2) 19,000 Assumed facility from employer as
by employer per Employment Policy
Leave Encashment 8 (3) - Retirement payment, Final
/92 withholding
Emergency medical treatment paid by 8 (2) 10,000 Reimbursement of personal
employer expense and it exceeds Rs. 500
Reimbursement of Tour Expense 8 (3) - Expense serving proper business
during official visit purpose of employer
Gratuity Received 8 (3) - Retirement Payment, Final
Withholding
Provident Fund Received 8 (3) - Retirement Payment, Final
Withholding
Assessable Income from Employment 693,000

Computation of Assessable Income from Business


Particulars Sec. Amount Working Notes
Ref
Sales 7 (2) 500,000
Office Furniture Sold Inclusion in Depreciation calc.
Bad Debt Recovered 7 (2) - Recovery to the extent deducted
/ 25 previously are included, and all
bad debt was not deductible
previously in the given case
Dividend 7 (3) - Final Withholding, assuming NIC
/ 92 Bank Ltd. is resident bank
Total Inclusions (A) 500,000
Less: Deductions
Interest Expense u/s 14 (1) 14 -
Cost of Trading Stock 15 350,000 Opening Stock value + Purchases –
Closing Stock Value
Depreciation 19 45,250 Working Note (1) below
Repair and Improvement cost 16 -
General Deductions 13 16,000 Rice Cooker gifted to a customer in
event of wedding is personal
expense.
Interest Expense u/s 14 (2) 14 -
Pollution Control Cost 17 -
Research and Development Cost 18 -
Total Deductions (B) 411,250
Assessable Income from Business 88,750
(A- B)

Computation of Assessable Income from Investment

© The Institute of Chartered Accountants of Nepal 74


Compilation of Suggested Answers Income Tax

Particulars Sec. Amount Working Notes


Ref
Gain on Disposal of Shares 9 (2) 100,000 Assumed from Listed shares
Interest from personal account 9(3) - Final withholding, assuming
& 92 Natural Bank is resident
Assessable Income from Investment 100,000

Computation of Total Assessable Income, Taxable Income and Balance Taxable Income
Particulars Sec. Amount Working Notes
Ref
Assessable Income from Employment 693,000
Assessable Income from Business 88,750
Assessable Income from Investment 100,000
Total Assessable Income 881,750
Less: Reductions u/s 12, 12Kha & 63,
if any
Reduction u/s 12 - Assuming political party is not
registered with Election
Commission
Reduction u/s 63 250,000 Lower of Rs. 300,000 or 1/3rd of
Assessable income (293,917) or
actual contribution (Rs. 250,000)
Taxable Income 631,750
Less: Investment Insurance Premium 19,000 Lower of Actual (Rs. 19,000) or Rs.
paid 25,000
Balance Taxable Income 612,750

Statement of Tax Liability (Assessed as Individual- Working note 2 for the reason)
Balance Taxable Income Less Gain on Disposal of NBCA: 512,750
Apply normal rates to this amount of Rs. 512,750; assuming no contribution to contribution
based SSF; and rates of NBCA on Gain on disposal of NBCA:
1st Rs. 400,000 1% (W.N. 3) 4,000
Next Rs. 100,000 10% 10,000
Next Rs. 12,750 20% 2,250
Balance Rs. 100,000 (NBCA) 5% 5,000
Total tax Liability (apart from final withholding taxes) 21,250

Working Notes:
1. Calculation of Depreciation
Particulars Pool B Pool C

© The Institute of Chartered Accountants of Nepal 75


CAP II Paper 7 Income Tax and VAT

Opening Depreciation Base (I)


Furniture 100,000
Computer 45,000
Motorcycle 80,000
Add: Absorbed Additions (II) 7,000
(100% of printer purchased as added in Poush)
Less: Disposal Value (III) (35,000)
Depreciable basis (or Depreciation Base) 117,000 80,000
(IV= I + II – III)
Rate of Depreciation (V) 25% 20%
Depreciation Amount 29,250 16,000

2. Since, his wife is also earning income; he shall not opt for couple assessment. The spouse
will pay tax separately on her income, which is max. Rs. 2,000.
3. Computation of taxes in respect of final withholding payments [Voluntary (for
understanding purpose of students)]
Income Head Tax Reason
Amount
Leave Encashment (accrued until Not taxable- Rule 20 (6), Income
2058/12/18) Tax Regulation
Leave Encashment (accrued after 25,500 15% of Rs. 170,000
2058/12/18)
Gratuity (accrued until 2058/12/18) Not taxable- Rule 20 (6), Income
Tax Regulation
Gratuity (accrued after 2058/12/18) 105,000 15% of Rs. 700,000
Provident Fund (accrued until Not taxable- Rule 20 (6), Income
2058/12/18) Tax Regulation
Provident Fund (accrued after 5,000 5% of Gain (Rs. 100,000)- Gain
2058/12/18) from approved RF is:
Accrued amount less
higher of (Rs. 500,000 or 50% of
accrued amount)]
Total taxes in respect of final withholding 135,500
payments

© The Institute of Chartered Accountants of Nepal 76


Compilation of Suggested Answers Income Tax

77. Answer how much income retained after tax for Income Year 20X-66/X-67:
(Dec 2009, 12 Marks, CA Inter)
a. Mrs. B is administrator of incapacitated husband Mr. B. A firm was registered in the
name of Mr. B before he becomes incapacitated 20and the firm earns taxable income
of Rs. 500,000 from export of merchandise. Mrs. B has not any income.
Answer
As per Sec. 2 (5) of Schedule 1 of Income Tax Act, 2058; a trust taking care of property of
incapacitated natural person shall be taxable as resident individual.
Total taxable income 500,000
Less: Deduction availed to disabled person
(Not available, since it’s an entity)
The tax liability is as follows:
1st Rs. 400,000 0% -
Balance Rs. 100,000 10% 10,000
Total Tax Liability 10,000

b. Mrs. C has taxable income from investments of Rs. 161,000.


Answer:
The tax liability is Zero, as it falls within the Basic Exemption Limit.

c. Mrs. Bantawa have following incomes: Local Himali Herbs Industry Rs. 100,000,
trading of shoes Rs. 100,000, interest from local cooperative Rs. 8,000 and salary
from same cooperative as market executives Rs. 100,000. She opted as couple.
Answer
Interest from local cooperative is final withholding (Sec. 92).

The total Taxable Income is Rs. 300,000; which is within the Basic Exemption Limit. Since,
her employment income is not from pension and assuming she does not contribute to Social
Security Fund, she has to pay 1% tax on Employment Income up to Rs. 450,000.

Therefore, her tax liability:


1st Rs. 100,000 1% 1,000

Female tax credit is not available, since has income from other sources also.

20
Grammatical mistake corrected

© The Institute of Chartered Accountants of Nepal 77


CAP II Paper 7 Income Tax and VAT

d. Mr. Majgain from Nagarkot has employment income of Rs. 100,000 per month
before deducting the contribution to approved retirement fund 40%. He is widower
with a child.
Answer
Total Inclusions in Employment Income 1,200,000
Total Assessable Income 1,200,000
Less:
Contribution to Approved Retirement Fund- lower of: 300,000
i. Actual (40% of salary) 480,000
rd
ii. 1/3 of Assessable Income 400,000
iii. Maximum 300,000
Taxable Income 900,000

Calculation of Tax Liability- Deemed Couple


1st Rs. 450,000 1% 4,500
Next Rs. 100,000 10% 10,000
Next Rs. 200,000 20% 40,000
Balance Rs. 200,000 30% 60,000
Total Tax Liability 114,500
Assuming that there is no contribution in Contribution Based Social Security Fund.

78. Mr. A, who is a resident of Nepal and a married person, submits following particulars of
his income for the financial year ending 15.07.20X5. Compute his total income and tax
payable.
(Dec 2006, 8 Marks, CA Inter)
Rs.
Salary 1,00,000
Dividends from Indian companies 2,600
Interest on fixed deposit with bank 7,400
Income from the units of the Unit Trust of India 3,900
Interest accrued on National Savings Certificates VIII 2,000
Issue
Honorarium received as examiner not being casual 3,500
income
Gift from friend received on 26.1.20X5 30,000
Employer's contribution to Unrecognized Provident (Interest included is
Fund and interest received thereon 26,000 Rs. 2,000)
Assessee's own contribution to Unrecognized Provident 21,000
Fund

© The Institute of Chartered Accountants of Nepal 78


Compilation of Suggested Answers Income Tax

Interest on his own contribution to Unrecognized


Provident Fund 5,000
Interest on deposit with private concerns 4,000
Interest on Debentures from listed companies 2,600
Winning from TV Game Show (Gross) 1,50,000
T.D.S. on Winnings from TV Games Show 37,500
Deposit in recognized retirement fund with CIT A/c 58,000
Contribution to Life Insurance Policy (Policy amount Rs.
2,00,000) 10,000
Medi-claim Insurance premium for self and wife 5,000

Answer:
Statement of Assessable Income from Employment
Sec.
Particulars Amount Working Notes
Ref
Salary 8 (2) 100,000
Assessable Income from
100,000
Employment

Statement of Assessable Income from Investment


Sec.
Particulars Amount Working Notes
Ref
9 (2)
Assuming Indian Company is not
Dividends from Indian Company & 54 2,600
resident of Nepal
(2)
Interest on Fixed Deposit with Final withholding, assuming not related
9 (3) -
Bank to his business
9 (2)
Income from the units of the Unit Assuming Indian Company is not
& 54 3,900
Trust of India resident of Nepal
(2)
Interest accrued on National
9 (2) 2,000
Savings Certificates VIII Issue
Honorarium received as examiner
92 - Final withholding
not being casual income
Gift from friend received on not taxable, income does not fall under
5 -
26.1.20X5 any income head
It is taxable only when the retirement
Interest on his own contribution to
- fund makes payment at the time of
Unrecognized RF
retirement
Interest on deposit with private
9 (2) 4,000
concerns
Interest on Debentures from listed
92 - Final withholding
companies

© The Institute of Chartered Accountants of Nepal 79


CAP II Paper 7 Income Tax and VAT

Winning from TV Game Show


92 - Final withholding
(Gross)
Assessable Income from
12,500
Investment

Statement of Total Assessable Income, Taxable Income and Balance Taxable Income
Sec.
Particulars Amount Working Notes
Ref
Assessable Income from
100,000 From above Calculation
Employment
Assessable Income from
12,500 From above Calculation
Investment
Total Assessable Income 112,500
One-third of Total Assessable Income,
Less: Contribution to Approved
63 37,500 or Actual deposit or Rs. 300,000;
Retirement Fund
whichever is lower
Taxable Income 75,000
Sch.
Less: Deposit of Life Insurance Actual premium paid or Rs. 25,000;
1, 10,000
Premium whichever is lower
1(12)
Less: Payment of Medical Sch. Actual premium paid or Rs. 20,000;
Insurance Premium to Resident 1, 5,000 whichever is lower and assuming
Insurance Co. 1(17) Couple Assessment
Balance Taxable Income 60,000

Statement of Tax Liability


Higher taxation in respect of Social
1st Rs. 60,000 1% 600
Security Tax interpreted

Taxes paid in relation to Final


Withholding Payments
Interest on Fixed Deposit with
5% 370
bank
Honorarium received as Examiner 15% 525
Interest on Debentures from Listed
5% 130
Companies
Winning of TV Game Show 25% 37,500
Total 38,525

© The Institute of Chartered Accountants of Nepal 80


Compilation of Suggested Answers Income Tax

Chapter 7: Income from Business

Inclusions
79. By virtue of an agreement entered on 20X-61/7/11 between A Ltd. and B Ltd., A Ltd. agree not to carry on any business
relating to washing machine within Kathmandu valley for next 5 years, for which B Ltd. agrees to pay a sum of Rs. 15,00,000
to A Ltd. The said amount was paid on 20X-62/2/27. Indicate treatment of such receipt in the hands of A Ltd. for the income
year 20X-61/X-62. (June 2006, 5 marks, CA Inter)
Answer
As per Sec. 7 (2) (Cha), any amount received in connection to acceptance of business restriction forms part of business income. As
such, the amount of Rs. 1,500,000 received by A Ltd. is business income of A Ltd.
As per Sec. 22, a company must follow accrual basis of accounting for income tax purpose. Therefore, the amount that is received
for five years shall be treated as income of five years.

Conclusion
The amount shall be treated as income for following income years as follows:
20X-61/X-62 [1,500,000/5 (for 8 months 19 days (days can also be taken)] Rs. 215,833
20X-62/X-63 300,000
20X-63/X-64 300,000
20X-64/X-65 300,000
20X-65/X-66 300,000
20X-66/X-67 84,167

Determination of Assessable Income


80. Juntara Trades is a partnership firm with 4 partners. It is an authorized distributer of SUPERB brand of household goods.
The firm has drawn income statement relating to Income Year (IY) 20X-74/X-75 as follows:
Particulars Amount (Rs.)
Sales 55,194,100
Less: Cost of Goods Sold 44,155,280

© The Institute of Chartered Accountants of Nepal 81


CAP II Paper 7 Income Tax and VAT

Opening Stock 3,219,660


Materials Purchased 46,455,030
Less: Closing Stock (5,519,410)
Gross Profit 11,038,820
Less: Administrative & Selling Expenses 4,723,196
Salary & Wages 3,311,646
Business Promotion Expenses 500,000
Repairs & Maintenance 375,000
Office Expenses 50,000
Lawyers Fee 45,000
Marketing Expenses 441,550
Net Profit 6,315,624
The accountant provides the following additional information in relation to the above transactions:

a. Salary and Wages includes Rs. 50,000 paid for installing plant Rs. 10 lakhs purchased on Chaitra 2074.
b. The firm didn't deduct tax on salary Rs. 40,000 at the time of payment but it was paid by the firm during Ashad end. The
deposited amount is debited to Salary and Wages.
c. The firm was awarded a motorcycle with market value of Rs. 220,000 as it met Sales target. The motorcycle was
immediately sold to one of the employees at the market value. The transaction has not been reflected in the income
statement above.
d. The details of depreciable assets are as follows:

Block Opening Depreciation Base (Rs.)


A 11,123,750
B 556,190
C 2,500,000
D 4,449,500

© The Institute of Chartered Accountants of Nepal 82


Compilation of Suggested Answers Income Tax

e. The sole assets in block C is a delivery van which is 100% financed by AB Bank Ltd. The delivery van is registered in the
name of the bank. The related interest expense of Rs. 250,000 has not been paid by the firm till the end of the year and is
not considered as expenses in preparing the income statement above.
f. 20% of repairs & maintenance is related to Block B and remaining is related to Block D.
g. The company has a policy to provide commission to persons referring customers. The company paid Rs. 40,000 this year
as referral commission to persons for referring customers in the previous year and this has been reduced from Sales
above. Similarly, the company paid Rs. 30,000 after closure of the financial year as referral commission to persons for
referring customers in IY 20X-74/X-75 which is not shown as expenses in the income statement above.
h. Lawyer’s fee is paid to a lawyer for defending assessment of the firm made by income tax officer in relation to IY 20X-
73/X-74. The tax officer issued an assessment order demanding additional income tax Rs. 200,000. The related liability is
not provided by the firm in the above income statement.
i. Office expenses include monthly tuition fees of children of marketing director of Rs. 12,000 as per terms of employment.

Required: (June 2019, 20 Marks)

a. Compute tax payable by the firm for IY 20X-74/X-75.


b. Would the tax liability change if the firm had engaged more than 300 Nepalese employees throughout the year?

Answer
Statement of Assessable income from Business
Sec.
Particulars Amount Working Notes
Ref
Inclusions:
Sales 7 (2) 55,234,100 Refer W.N. 1
Gift (Motorcycle) (Additional note "c") 7 (2) 220,000
Total Inclusions 55,454,100
Deductions
Interest Expense 14 250,000 Interest expense of delivery van, additional note "e".
Cost of Trading Stock 15 44,155,280 Refer W.N. 2

© The Institute of Chartered Accountants of Nepal 83


CAP II Paper 7 Income Tax and VAT

Depreciation 19 1,967,660 Refer W.N. 3


Repair and Improvement Cost 16 338,933 Refer W.N. 3
Other Costs deductible u/s 13 13 4,288,196 Refer W.N. 4
Total Deductions 51,000,069

Assessable Income from Business 4,454,031

Less: Reductions u/s 12, 12A, 12B -


Taxable Income 4,454,031
Tax Liability @25% 1,113,508

Answer Part (b)


The concessions in taxation as a result of employing 300 Nepali nationals throughout the Income Year are entitled to Special
Industry. As per Clarification Clause to Sec. 11, Special Industry refers to Production based industries (except through producing
alcohols, beers or products using tobacco as basic raw materials, agro and forest-based industries and mineral based industries.
Since, Juntara Trades is a trading firm, the tax rate remains 25% irrespective of it being able to employ 300 Nepali nationals.

Working Notes
1 Adjusted Value of Sales
Given 55,194,100
Add: Commission of last year reduced from
40,000
sales
Sales to be included 55,234,100

2 Cost of Trading Stock


Particulars Sec. Ref Amount
Opening Stock 3,219,660
Add: Purchases/Cost of Conversion 46,455,030

© The Institute of Chartered Accountants of Nepal 84


Compilation of Suggested Answers Income Tax

Less: Closing Stock (5,519,410)


Cost of Trading Stock 44,155,280

3 Calculation of Depreciation and Repair and Improvement Cost


Particulars Block A Block B Block Block D Total
Opening Depreciation Base 11,123,750 556,190 2,500,000 4,449,500
Add: Absorbed additions
Motorcycle obtained as gift
220,000
(assumed until Poush)
Cost of Plant and Plant
Installation Cost (2/3) 700,000
(2/3rd of Rs. 1,050,000)
Less: Disposal Proceeds of
Assets
(220,000)
(Sales of Motorcycle to
employee)
Depreciable Basis 11,123,750 556,190 2,500,000 5,149,500
Depreciation Rate 5% 25% 20% 15%
Depreciation Amount 556,188 139,048 500,000 772,425 1,967,660

4 Expense u/s 13 Reason


Salary and Wages 3,311,646
Less: Incurred for installation of Plant (50,000) to be capitalized as plant in Block D
Less: TDS on Salary and Wages (40,000) 3,221,646 to be booked as receivable from employee
Business Promotion Expense 500,000
Add: Referral Commission for the Year 30,000 530,000 Commission for X-74/75 to be claimed during the year
Monthly tuition fee of employees' children is deductible, as
Office Expenses 50,000
such amount forms part of income of employee
Lawyers Fee 45,000 Expenses related to lawsuit is deductible

© The Institute of Chartered Accountants of Nepal 85


CAP II Paper 7 Income Tax and VAT

Marketing Expense 441,550


Total 4,288,196

81. 21Mega Vision Pvt. Ltd., Pokhara, a manufacturer and exporter of garments; has the following incomes and expenses for
Income Year (IY) 20X-74/X-75: (Dec 2018, 20 Marks)
Particulars Amount (Rs.)
Export sales 50,000,000
Misc. income 2,850,000
Dividend income 950,000
Opening stock 15,000,000
Raw material import 14,000,000
Freight for raw material 900,000
Custom duty paid for raw material 1,500,000
Custom agent fee 150,000
VAT paid for import 1,500,000
Wages for production 3,000,000
Overhead cost (production) 2,000,000
Administrative expenses 1,500,000
Selling and distribution expenses 1,000,000
Taxes paid to state and local government 1,500,000
Interest paid to financial institution 300,000
Penalties paid to metropolitan office 200,000
Donation 1,100,000
Depreciation 1,200,000

21
Similar question was asked in December 2017 Examinations

© The Institute of Chartered Accountants of Nepal 86


Compilation of Suggested Answers Income Tax

The following information is relevant for income tax calculation:


a. Sales include VAT refund of Rs. 1,500,000 which was paid at the customs point for import of raw materials.
b. Miscellaneous income Rs. 2,000,000 is the incentive given by the Government of Nepal (GON) for export of previous year
and is received during this year through Nepal Rastra Bank as the right to such incentive was established during the
current income year. Remaining miscellaneous income Rs. 850,000 is a net of tax amount received from the bank deposit
incidental to the business.
c. Dividend income is also a net of tax amount received from a subsidiary company.
d. The opening stock and closing stock have been recorded 1,000 pieces and 1,200 pieces respectively. The opening stock
includes fixed factory overhead Rs. 10 per piece and machineries repair and maintenance Rs. 15 per piece. During the
year, the Company produced 2,000 pieces of garments and the overhead cost includes equal rate of previous years' fixed
overhead and repair and maintenance cost.
e. Administrative expenses include employees' travelling expenses Rs. 100,000 during the import of raw materials and also
include entertainment expenses Rs. 50,000 provided to the Custom's employees.
f. The Company has made employees' bonus provision Rs. 2,200,000 as per the Bonus Act. During the year, it has paid Rs.
400,000 as full settlement of previous year's bonus. The administrative expense presented in the above table includes the
previous year's bonus but excludes the current year's bonus.
g. The Company has made provision for Corporate Social Responsibility (CSR) for Rs. 187,000 as 1 percent of profit after
tax as per Section 48 of Industrial Enterprises Act, 2073. The amount is included in the administrative expenses.
h. Selling and distribution expenses were paid to the cargo agent for export, invoices amounting Rs. 300,000 from the cargo
agent were received, and the remaining amount pertains to goods transportation from the factory to depot point as per
details given by the agent.
i. Interest expense is charged against the loan taken to purchase the factory machinery costing Rs. 2,000,000. The
machinery was purchased on 1st Baishakh, 20X-75 and has been put to use from 20th Bhadra, 20X-75.
j. Donation includes Rs. 100,000 directly paid to the flood victims; the remaining amount was deposited into Reconstruction
Fund established by GON for earthquake victims.
k. Opening balance of depreciable assets as on 20X-74/4/1 as per tax return records are as follows:
Building Rs. 6,000,000

Cars Rs. 3,000,000

© The Institute of Chartered Accountants of Nepal 87


CAP II Paper 7 Income Tax and VAT

Plant & Machinery Rs. 8,000,000

l. The company disposed a Santro car for Rs. 800,000 during the year and the written down value of the car at the disposal
time was Rs. 1,000,000.
You are requested to ascertain:
a. Taxable income
b. Tax liability
c. Dividend tax that must be paid by the Company, if the Company has proposed to declare dividend of Rs. 3,000,000.
Answer
Statement of Assessable income from Business
Export Other
Particulars Sec. Ref Working Notes
Income Income
Inclusions:
Export Sales 7 (2) 48,500,000 Refer W.N. 1
Miscellaneous Income 7 (2)
As right to receive the payment is established during the
Export Incentive 2,000,000
Income Year
Interest Income 1,000,000 Grossing up net interest applying TDS rate of 15%
Assuming subsidiary company is resident, the income is
Dividend Income 92 -
final withholding
Total Inclusions 50,500,000 1,000,000
Deductions
Asset purchased under the loan for which interest is
Interest Expenses 14 - accrued is put to use only during next Income year; as such
interest expense is not deductible
Cost of Trading Stock 15 23,593,000 Refer W.N. 2
Depreciation for the Year 19 2,586,667 Refer W.N. 3
Repair and Improvement Cost 16 30,000 Refer W.N. 3

© The Institute of Chartered Accountants of Nepal 88


Compilation of Suggested Answers Income Tax

a. Travel expense of employee is administrative expense


b. Entertainment expense is in nature of bribe, so not
Administrative Expenses 13 1,050,000 deductible u/s 21 (1) (Cha)
c. Previous year bonus does not satisfy "during the year"
criteria for expense deduction u/s 13
Selling and Distribution
13 300,000 Expense without proper invoice is not deductible
Expenses
Taxes paid to State and Local
13 1,500,000 Specifically deductible u/s 21
Government
Penalties paid to Metropolitan
21 - Penalties against legal non compliances
Office
Corporate Social Responsibility - Expense is not incurred, and this is not a fixed obligation

Practice differs in allocating amount, used 10% of


Employee Bonus Provision for
2,208,700 100,000 assessable profit before taxes to determine the amount in
the Year
this case
Total Deductions 31,203,700 100,000
Assessable Income from
19,296,300 900,000
Business

Statement of Taxable income and tax liability


Export Other
Particulars Notes
Income Income
Assessable Income from Business 19,296,300 900,000
Total Assessable Income 19,296,300 900,000
Less:
Reductions u/s 12 -
Reductions u/s 12Ka -
Reductions u/s 12Kha- National Paid directly to flood victim is not a qualified
955,437 44,563
Reconstruction Fund of GON donation for reduction

© The Institute of Chartered Accountants of Nepal 89


CAP II Paper 7 Income Tax and VAT

Taxable Income 18,340,863 855,437


Tax Rate 12% 25%
Tax Liability 2,200,904 213,859

Working Notes
1 Adjusted Sales
Given 50,000,000
Less: VAT Refund (1,500,000)
Sales to be included 48,500,000

Treatment of VAT Refund


VAT is not the expense of any business person. When a person makes payment of VAT, a VAT Receivable Account is debited that is
an account representing Receivables from Government of Nepal. When Government refunds VAT amount, the same asset account
is credited with corresponding effect being given to bank or cash account. As such, VAT refund does not have any impact on
profitability of any person.

2 Cost of Trading Stock for the Year


Particulars Details Amount Note
Value of Opening Stock
Given 15,000,000
Less: Repair of Machineries in the value (15,000) 14,985,000 Repair cost does not form part of cost of stock
Purchases/Cost of Conversion
Raw Material Import 14,000,000
Freight for Raw Material 900,000
Customs duty paid for Raw Material 1,500,000
Customs Agent Fee 150,000
VAT Paid for Import - Receivable amount, not an expense
Wages for Production 3,000,000

© The Institute of Chartered Accountants of Nepal 90


Compilation of Suggested Answers Income Tax

Production Overhead 2,000,000


Less: Repair and Maintenance cost on OH (30,000) 21,520,000
Per unit cost of Production 10,760 Cost of conversion / Units produced
Per unit cost of production * No. of closing stock,
Value of Closing Stock (12,912,000)
assuming FIFO
Cost of Trading Stock 23,593,000

3 Computation of Depreciation Allowances

Particulars Block A Block B Block C Block D Total


Opening Depreciation Base 6,000,000 - 3,000,000 8,000,000
Add: Absorbed additions
Less: Disposal Proceeds of Assets
(Sales of Santro car) (800,000)
Depreciable Basis 6,000,000 - 2,200,000 8,000,000
Depreciation Rate 5% 25% 20% 15%
Depreciation Amount (Normal) 300,000 - 440,000 1,200,000 1,940,000
One-third additional (as it is special industry) 100,000 - 146,667 400,000 646,667
Total Depreciation for the year 400,000 - 586,667 1,600,000 2,586,667

7% of Depreciable Basis 420,000 - 154,000 560,000


Actual Repair and Improvement Expense - - 30,000
Allowable Repair and Improvement Cost
- - - 30,000 30,000
(Minimum of 7% of Depreciation Base or Actual)
Comment on Machinery Additions:
There is no addition, as the pooling date is later of use date or purchase date, and the machinery purchased using loan is
used during next I.Y.
4 Determination of Tax Rate

© The Institute of Chartered Accountants of Nepal 91


CAP II Paper 7 Income Tax and VAT

Income from export by manufacturing business is treated as received by separate person, as Sec. 11 is applicable and earning
interest income is treated as earned by separate person.
Generally, there is no administrative or other expense in generating interest income when there is no treasury department,
therefore, there is no deduction (it is a presumption and students may presume otherwise with valid logics)
The tax rate for interest income is 25%.
The tax rate Export by Manufacturing business
Applicable Tax Rate 25% (Schedule 1)
Reduction in Tax rate to Special Industry 20% Sec. 11 (2Kha) (Kha)
Reduced Tax Rate 20%
Concessions availed to Export business 20% (Sec. 11 (3Nga) (Kha)
Rate after concession 16%
Additional Concessions to Manufacturing industry
25% (Sec. 11 (3Nga) (Ga)
that exports goods
Effective Rate 12%

82. 22Mustang Cashmere Manufacturing Co. Pvt. Limited is engaged in manufacturing and sale of high-grade cashmere clothes.
It provides employment opportunity to 650 Nepalese people throughout the year. Following are the extracts of the Income
Statement of the company for the year ended on Ashad 31, 20X-74.(June 2018, 20 Marks)
Particulars Amount Rs.
Income:
Export Sales 6,000,000.00
Domestic Sales 4,000,000.00
Dividend Received (Net of Tax) 150,000.00
Rent Income (Related with Business) 50,000.00
Total Income 10,200,000.00
Expenditure:

22
Similar question was asked in June 2017 CAP II Examinations

© The Institute of Chartered Accountants of Nepal 92


Compilation of Suggested Answers Income Tax

Cost of Materials Consumed 3,000,000.00


Manufacturing Expenses 500,000.00
Employee Cost 1,000,000.00
Selling and Administrative Expenses 1,500,000.00
Interest and Bank Charges 500,000.00
Exchange Loss 250,000.00
Loss on Sale of Depreciable Assets 300,000.00
Total Expenditure 7,050,000.00
Operating Profit 3,150,000.00
Less: Provision for Bonus 300,000.00
Profit Before Tax 2,850,000.00

Additional information:
a. Cost of materials is consumed in the ratio of sales.
b. Exchange loss includes Rs 100,000.00 against revaluation of creditors at the year-end.
c. Selling and Administrative Expenses include Rs 70,000.00 donation given to Prime Minister Disaster Relief Fund and
Rs 300,000.00 given for construction of school.
d. Out of total provision for bonus, Rs 200,000.00 was distributed to the employees till the time of filing income tax return.
It has been decided by the management not to pay the undistributed portion.
e. You are given the following information in regard to some of the expenses:
i) Rs 50,000.00 included in employee cost as staff welfare is personal expenses of directors.
ii) Manufacturing expenses include Rs 30,000.00 for electricity bill of previous year.
iii) Selling and Administrative Expenses include Rs 10,000.00 for business promotion which is not related to business.

Based on the above information, Compute tax liability of the Company


Answer
Statement of Assessable Income from Business

© The Institute of Chartered Accountants of Nepal 93


CAP II Paper 7 Income Tax and VAT

Sec. Export Other


Particulars Working Notes
Ref Income Income
Inclusions:
Export Sales 7 (2) 6,000,000
Domestic Sales 7 (2) 4,000,000
Rent Income - 50,000
Assuming received from resident company, the
Dividend Income 92 -
income is final withholding
Total Inclusions 6,000,000 4,050,000
Deductions
In proportion to Export Sales and Domestic Sales
Interest Expenses 14 300,000 200,000
(i.e. 3:2)
Cost of Trading Stock 15 2,082,000 1,388,000 Refer W.N. 1
There is loss on sale of depreciable asset, it is
Depreciation for the Year 19 180,000 120,000 assumed that the pool was disposed, and allocated
in proportion of sales
Exchange Loss - Revaluation loss is not deductible u/s 28
Rs. 50,000 is not deductible as it is personal
expense.
Employee Cost 13 570,000 380,000
Remaining amount is distributed in proportion to
sales
Selling and Administrative Expenses 13 672,000 448,000 Refer W.N. 2
Bonus provision for the year is deductible as it is
fixed obligation
Employee Bonus Provision for the Year 180,000 120,000
Any amount undistributed is added back to income
in the next income year
Total Deductions 3,984,000 2,656,000
Assessable Income from Business 2,016,000 1,394,000
Statement of Taxable income and tax liability

© The Institute of Chartered Accountants of Nepal 94


Compilation of Suggested Answers Income Tax

Assessable Income from Business 2,016,000 1,394,000


Total Assessable Income 2,016,000 1,394,000
Less:
Reductions u/s 12 -
Reductions u/s 12Ka -
Reductions u/s 12Kha- National Reconstruction
42,000 28,000 In sales ratio, as the question suggests
Fund of GON
Taxable Income 1,974,000 1,366,000
Tax Rate 12% 15%
Tax Liability 236,880 204,900

Working Notes
1 Cost of Trading Stock for the Year
Cost of Materials consumed 3,000,000
Manufacturing Expenses 500,000
Less: Prior period expenses (30,000)
Total Cost 3,470,000
Towards Export Sales 2,082,000
(in sales ratio)
Towards Domestic Sales 1,388,000

2 Selling and Administrative Expense


Given 1,500,000
Less:
Donation to PM Relief Fund (70,000)
Donation for Construction of School (300,000)
Business promotion, not related to business (10,000) 1,120,000
Allocated to Export sales 672,000
(in sales ratio)
Allocated to Domestic Sales 448,000

© The Institute of Chartered Accountants of Nepal 95


CAP II Paper 7 Income Tax and VAT

3 Identification of Tax Rate


Export Domestic
Applicable Tax Rate 25% 25%
Reduction to special industry 20% Sec. 11 (2Kha) (Kha)
Reduced Tax Rate 20% 20%
Additional Reduction due to employment 25% Sec. 11 (3) (ka)
Effective Rate due to effect of employment 15% 15%
Reduction due to export 20% 16% Sec. 11 (3Nga) (Kha)
Additional Reduction to manufacturing
25% 12% Sec. 11 (3Nga) (Ga)
exporter
Lower Rate 12% 15%

83. I & M Pvt. Ltd. Kathmandu, a manufacturing company, deals in production and sales of garments. Based upon the following
information, you are requested to ascertain taxable income and tax liability for Income Year 20X-73/X-74.
(Dec 2017, 20 Marks)
Particulars Amount (Rs.)
Export Sales 3,00,00,000
Misc. Income 25,00,000
Dividend Income 10,00,000
Opening stock 75,00,000
Raw material import 70,00,000
Freight for raw material 9,00,000
Custom duty paid for raw material 15,00,000
Custom agent fee 1,50,000
VAT paid for import 15,00,000
Wages for production 30,00,000
Overhead cost 20,00,000

© The Institute of Chartered Accountants of Nepal 96


Compilation of Suggested Answers Income Tax

Administrative expenses 15,00,000


Selling and distribution expenses 10,00,000
Interest paid to Financial Institution 3,00,000
Penalties paid to Metropolitan Office 2,00,000
Donation 2,00,000
Depreciation 12,00,000
Additional information:
a) Sales include VAT refund of Rs. 10,00,000 paid at custom point for import of raw material. Miscellaneous income Rs.
20,00,000 is the incentive given by the Nepal government for export of previous year and is received during this year
through Nepal Rastra Bank. Remaining Miscellaneous income Rs. 5,00,000 has been generated from the bank deposit.
b) Dividend income has been received from Joint Venture.
c) The opening stock and closing stock have been recorded 1,000 pieces and 1,200 pieces respectively. The opening stock
includes factory fixed overhead Rs. 10 per piece and repair and maintenance Rs. 15 per piece. During the year, the
company produced 2,000 pieces of garments and the overhead cost includes equal rate of previous years fixed overhead
and repair and maintenance cost.
d) Administrative expenses, includes travelling cost Rs. 1,00,000 of employees during the import of raw material,
entertainment expenses Rs. 50,000 provided to the Custom's employee and custom agent during the import.
e) The company has the policy for employee bonus; annually it makes 10 % provision from the profit as per the Bonus Act.
During this year, it has paid Rs. 2,00,000 and booked under the administrative expenses.
f) Selling and distribution expenses were paid to the cargo agent for export, invoices amounting Rs. 2,00,000 from the cargo
agent were received, and the remaining amount pertaining to the transportation of the goods from the factory to the
depot point as per details given by the agent.
g) Interest expense has been charged against the loan taken to purchase the factory machinery costing Rs. 20,00,000. The
machinery was purchased on 1st Ashad, 20X-74 and has been used from 29th Shrawan, 20X-74.
h) The donation includes Rs. 1,00,000 directly paid to the flood victims; the remaining amount was deposited into
Reconstruction Fund established by Nepal government for earthquake victims.
i) Opening balance (WDV) of depreciable assets as on 20X-73/4/1 as per books of account are as follows:
a. Land Rs. 20,00,000
b. Building Rs. 50,00,000

© The Institute of Chartered Accountants of Nepal 97


CAP II Paper 7 Income Tax and VAT

c. Cars Rs. 26,00,000


d. Plant & Machinery Rs. 80,00,000
j) The company disposed a Santro car having written down value of Rs. 10,00,000 for Rs. 8,00,000 during the year.
Answer
Assessable Income from Business
Sec. Export Other
Particulars Working Notes
Ref Income Income
Inclusions:
Export Sales 7 (2) 29,000,000 Refer W.N. 1
Miscellaneous Income 7 (2)
As right to receive the payment is established during
Export Incentive 2,000,000
the Income Year
Interest Income 500,000
Assuming joint venture is resident, the income is final
Dividend Income 92 -
withholding
Total Inclusions 31,000,000 500,000
Deductions
Asset purchased under the loan for which interest is
Interest Expenses 14 - accrued is put to use only during next Income year; as
such interest expense is not deductible
Cost of Trading Stock 15 9,993,000 Refer W.N. 2
Depreciation for the Year 19 2,413,333 Refer W.N. 3
Repair and Improvement Cost 16 30,000 Refer W.N. 3
Net off fixed factory overhead charged to cost of
Overhead Cost 1,980,000
trading stock

© The Institute of Chartered Accountants of Nepal 98


Compilation of Suggested Answers Income Tax

a. Travel expense of employee is administrative


expense
Administrative Expenses 13 1,450,000 b. Entertainment expense is in nature of bribe, so not
deductible u/s 21 (1) (Cha)
c. Current year bonus is eligible expense
Any expense in excess of Rs. 1,000 require invoice
Selling and Distribution Expenses 13 200,000
disclosing Permanent Account Number of vendor
Penalties paid to Metropolitan Office 21 - Penalties against legal non compliances
Total Deductions 16,066,333 -
Assessable Income from Business 14,933,667 500,000

Statement of Taxable income and tax liability


Assessable Income from Business 16,066,333 -
Total Assessable Income 14,933,667 500,000
Less:
Reductions u/s 12 -
Reductions u/s 12Ka -
Paid directly to flood victim is not a qualified
Reductions u/s 12Kha- National 96,760 3,240 donation for reduction, remaining amount allocated
Reconstruction Fund of GON
on the basis of assessable income
Taxable Income 14,836,906 496,760
Tax Rate 12% 25%
Tax Liability 1,780,429 124,190

Working Notes
1 Adjusted Sales
Given 30,000,000
Less: VAT Refund (1,000,000)

© The Institute of Chartered Accountants of Nepal 99


CAP II Paper 7 Income Tax and VAT

Sales to be included 29,000,000

Treatment of VAT Refund


VAT is not the expense of any business person. When a person makes payment of VAT, a VAT Receivable Account is debited
that is an account representing Receivables from Government of Nepal. When Government refunds VAT amount, the same
asset account is credited with corresponding effect being given to bank or cash account. As such, VAT refund does not have
any impact on profitability of any person.
2 Cost of Trading Stock for the Year
Particulars Details Amount Notes
Value of Opening Stock
Given 7,500,000
Less: Repair of Machineries in the Repair cost does not form part of cost of stock
(15,000) 7,485,000
value 1000 pieces @ Rs. 15 per piece
Purchases/Cost of Conversion
Raw Material Import 700,000
Freight for Raw Material 900,000
Customs duty paid for Raw
1,500,000
Material
Customs Agent Fee 150,000
VAT Paid for Import - Receivable amount, not an expense
Wages for Production 3,000,000
Fixed factory overhead charged to cost of conversion
Alternatively, overhead cost can be treated as
Production Overhead 20,000
production overhead with another effect on
deduction of repair cost of Rs. 30,000
Less: Repair and Maintenance cost
6,270,000 As it has not been allocated to cost of production
on OH
Per unit cost of Production 3,135 Cost of conversion / Units produced

© The Institute of Chartered Accountants of Nepal 100


Compilation of Suggested Answers Income Tax

Per unit cost of production * No. of closing stock,


Value of Closing Stock (3,762,000)
assuming FIFO
Cost of Trading Stock 9,993,000

3 Computation of Depreciation Allowances


Particulars Block A Block B Block C Block D Total
Opening Depreciation Base 5,000,000 - 2,600,000 8,000,000
Add: Absorbed additions
Less: Disposal Proceeds of Assets
(Sales of Santro car) (800,000)
Depreciable Basis 5,000,000 - 1,800,000 8,000,000
Depreciation Rate 5% 25% 20% 15%
Depreciation Amount (Normal) 250,000 - 360,000 1,200,000 1,810,000

One-third additional (as it is special


industry) 83,333 - 120,000 400,000 603,333

Total Depreciation for the year 333,333 - 480,000 1,600,000 2,413,333

7% of Depreciable Basis 350,000 - 126,000 560,000


Actual Repair and Improvement
Expense
(2000 units @ 15 per unit) - - 30,000
Allowable Repair and Improvement
Cost
- - - 30,000 30,000
(Minimum of 7% of Depreciation Base
or Actual)
Comment on Machinery Additions:

© The Institute of Chartered Accountants of Nepal 101


CAP II Paper 7 Income Tax and VAT

There is no addition, as the pooling date is later of use date or purchase date, and the machinery purchased using loan is used
during next I.Y.
Land is not a depreciable asset

4 Determination of Tax Rate


Income from export by manufacturing business is treated as received by separate person, as Sec. 11 is applicable and earning
interest income is treated as earned by separate person.
Generally, there is no administrative or other expense in generating interest income when there is no treasury department,
therefore, there is no deduction (it is a presumption and students may presume otherwise with valid logics)
The tax rate for interest income is 25%.
The tax rate Export by Manufacturing business
Applicable Tax Rate 25% (Schedule 1)
Reduction in Tax rate to Special Industry 20% Sec. 11 (2Kha) (Kha)
Reduced Tax Rate 20%
Concessions availed to Export business 20% (Sec. 11 (3Nga) (Kha)
Rate after concession 16%
Additional Concessions to Manufacturing industry
25% (Sec. 11 (3Nga) (Ga)
that exports goods
Effective Rate 12%

84. Citizen Cement Ltd. is a company situated in Biratnagar and listed in Nepal Stock Exchange and engaged in manufacturing
and sale of premium grade cement for Nepal and Export. Following is the provisional Income Statement of Citizen Cement
Ltd., for the year ended Ashad 31, 20X-73.
Particulars Amount (Rs.)
Income
Export Sales 60,00,000

© The Institute of Chartered Accountants of Nepal 102


Compilation of Suggested Answers Income Tax

Domestic Sales 40,00,000


Dividend Received (Net of Tax) 1,50,000
Rent Income (Related with Business) 50,000
Total Income 1,02,00,000
Expenditure
Cost of Materials Consumed 30,00,000
Manufacturing Expenses 5,00,000
Employee Cost 10,00,000
Selling and Administrative Expenses 15,00,000
Interest and Bank Charges 5,00,000
Exchange Loss 2,50,000
Depreciation 5,00,000
Bad Debt Written Off 70,000
Loss on Sale of Assets 3,00,000
Total Expenditure 76,20,000

Operating Profit 25,80,000


Add: Provision for Doubtful Receivables Written Back 7,20,000
Less: Provision for Bonus 3,00,000
Profit Before Tax 30,00,000

Additional information:
a) Cost of material consumed
For export sales : Rs. 18,00,000
For local sales : Rs. 12,00,000
b) Exchange loss includes Rs. 1,00,000 against revaluation of creditors at the year-end date.
c) Asset detail for income tax purpose is as below –

© The Institute of Chartered Accountants of Nepal 103


CAP II Paper 7 Income Tax and VAT

Plant and Computer, Office


Building Vehicles
Machinery Equipment and Furniture
Opening Depreciation Base 11,00,000 1,00,00,000 15,00,000 29,50,000
Addition upto Poush End - - 2,00,000 -
Addition Magh to Chaitra end 6,00,000 - 1,50,000 -
Addition Baishak to Ashad End - - - 9,00,000
Sales Proceed - 15,00,000

d) Manufacturing expenses includes repair and maintenance expenses as below:


Building Repair and Maintenance : Rs. 1,50,000
Office Equipment Repair and Maintenance : Rs. 50,000
Vehicle Repair and Maintenance : Rs. 20,000
e) Selling and Distribution Expenses includes Rs. 7,00,000 donations given to Prime Minister Disaster Relief Fund and Rs.
3,00,000 given for construction of school.
f) Bonus of Rs. 2,00,000 only distributed to the employees till the time of filing of income tax return. It has been decided by
the management not to pay the undistributed portion.
g) Following expenses are of below nature:
i) Rs. 50,000 included in employee cost for staff welfare is for personal use of directors.
ii) Manufacturing expenses includes Rs. 30,000 for electricity bill of previous years.
iii) Selling and administrative expenses includes Rs. 10,000 for business promotion which is not related to business.
h) Company employed 1,300 Nepali employees during the whole year out of which 50 employees are foreign employee.
Based on the above information, please advise Citizen Cement Ltd. regarding. (June 2017, 5+15=20)
a. Various tax exemptions available to them as per Income Tax Act, 2058, and which tax exemptions Citizen Cement Ltd.
should opt for?
b. Compute taxable income and tax liability of the company segregating income on the basis of cost of material consumed
for common cost and income

© The Institute of Chartered Accountants of Nepal 104


Compilation of Suggested Answers Income Tax

Answer
Statement of Assessable income from Business
Export Other
Particulars Sec. Ref Working Notes
Income Income
Inclusions:
Export Sales 7 (2) 6,000,000
Domestic Sales 7 (2) 4,000,000
Rent Income - 50,000
Assuming subsidiary company is resident, the income
Dividend Income 92 -
is final withholding
Total Inclusions 6,000,000 4,050,000
Deductions
In proportion to Export Sales and Domestic Sales
Interest Expenses 14 300,000 200,000
(i.e. 3:2)
Cost of Trading Stock 15 1,950,000 1,300,000 Refer W.N. 1
Depreciation for the Year 19 1,900,000 1,266,667 Refer W.N. 3
Repair and Improvement Cost 16 105,000 70,000 Refer W.N. 3
Exchange Loss - Revaluation loss is not deductible u/s 28
Rs. 50,000 is not deductible as it is personal expense.
Employee Cost 13 570,000 380,000 Remaining amount is distributed in proportion to
sales
Selling and Administrative Expenses 13 294,000 196,000 Refer W.N. 2
Bonus provision for the year is deductible as it is
Employee Bonus Provision for the fixed obligation
180,000 120,000
Year Any amount undistributed is added back to income in
the next income year
Bad Debt written off Assumed that conditions of Sec. 40 (3) (ga) not met
Is considered while computing depreciation
Loss on Sale of Assets
allowances

© The Institute of Chartered Accountants of Nepal 105


CAP II Paper 7 Income Tax and VAT

Total Deductions 5,299,000 3,532,667


Assessable Income from Business 701,000 517,333

Statement of Taxable income and tax liability


Assessable Income from Business 5,299,000 3,532,667
Total Assessable Income 701,000 517,333
Less:
Reductions u/s 12 -
Reductions u/s 12Ka -
Reductions u/s 12Kha- National
420,000 280,000 In sales ratio, as the question suggests
Reconstruction Fund of GON
Taxable Income 281,000 237,333
Tax Rate 12% 14%
Tax Liability 33,720 33,227

Working Notes
1 Cost of Trading Stock for the Year
Cost of Materials consumed 3,000,000
Manufacturing Expenses 500,000
Less: Repair and Maintenance expenses (220,000)
Less: Prior period expenses (30,000)
Total Cost 3,250,000
Towards Export Sales 1,950,000 (in sales ratio)
Towards Domestic Sales 1,300,000

2 Selling and Administrative Expense


Given 1,500,000
Less:

© The Institute of Chartered Accountants of Nepal 106


Compilation of Suggested Answers Income Tax

Donation to PM Relief Fund (700,000)


Donation for Construction of School (300,000)
Business promotion, not related to business (10,000) 490,000
Allocated to Export sales 294,000 (in sales ratio)
Allocated to Domestic Sales 196,000
Computation of Depreciation Allowances
3
Calculation of Depreciation and Repair and Improvement Cost
Particulars Block A Block B Block C Block D Total
Opening Depreciation Base 1,100,000 1,500,000 2,950,000 10,000,000
Add: Absorbed additions
Up to Poush (100% of cost) 200,000
From Magh to Chaitra (2/3rd of Cost) 400,000 100,000
From Baisakh to Ashad (1/3rd of Cost) 300,000
Less: Disposal Proceeds of Assets
(Sales of Santro car) (1,500,000)
Depreciable Basis 1,500,000 1,800,000 1,750,000 10,000,000
Depreciation Rate 5% 25% 20% 15%
Depreciation Amount (Normal) 75,000 450,000 350,000 1,500,000 2,375,000
One-third additional (as it is special industry) 25,000 150,000 116,667 500,000 791,667
Total Depreciation for the year 100,000 600,000 466,667 2,000,000 3,166,667

7% of Depreciable Basis 105,000 126,000 122,500 700,000


Actual Repair and Improvement Expense 150,000 50,000 20,000 -
Allowable Repair and Improvement Cost
(Minimum of 7% of Depreciation Base or 105,000 50,000 20,000 - 175,000
Actual)

4 Identification of Tax Rate

© The Institute of Chartered Accountants of Nepal 107


CAP II Paper 7 Income Tax and VAT

Particulars Details Export Domestic


Applicable Tax Rate 25% 25%
Reduction to special industry 20% Sec. 11 (2Kha) (Kha)
Reduced Tax Rate 20% 20%
Additional Reduction due to employment 30% Sec. 11 (3) (ka)
Effective Rate due to effect of employment 14% 14%
Reduction due to export 20% 16% Sec. 11 (3Nga) (Kha)
Additional Reduction to manufacturing
25% 12% Sec. 11 (3Nga) (Ga)
exporter
Lower Rate 12% 14%

As per Sec. 11 (3) (ka), where a special industry provides employment to 1,000 or more Nepali nationals, the tax rate is 70%
of reduced tax rate, i.e. 14%
Application of Sec. 11 (3Nga) jointly read with Sec. 11 (3Kha) will give tax rate of 12% in case of export oriented
manufacturing special industry.
As such, tax rate of 14% is applicable in case of domestic sales income and that of 12% is applicable in case of export income,
as a rational taxpayer opts for beneficial options.

85. Heritage Fashion Pvt. Ltd. is a garment industry; which exports all its garments to USA. The company maintains its books
of account in cash basis. The company records in a loose sheet of the transactions of sales and purchases before accounting
in the books. The receipts and payments based on the accounting of the company during the Income Year 20X-72/X-73 are
as follows:
Particulars Payments (Rs.) Receipts (Rs.)

4,200,000
Loan 2,500,000
Raw material 6,000,000
Wages for labour 500,000
Variable overhead for production 250,000

© The Institute of Chartered Accountants of Nepal 108


Compilation of Suggested Answers Income Tax

Factory rent 1,200,000


Production supervisory salary 250,000
Insurance for factory 117,000
Administrative salary 650,000
Office expenses 400,000
Plant & Machinery 4,200,000
Generator 800,000
Sale of old computer 10,000
Export 12,000,000
Advance Income tax 200,000
17,067,000 16,210,000

You are asked by the Company to assess the tax payable for the Income year based on the above transactions and following
additional information: (December 2016, 20 Marks)
a) Closing balance of Income Year 20X-71/X-72 related to the above transactions were assessed as follows:
Particulars Amount (Rs.)
Loan 2,500,000
Garment stocks (1000 units) 2,400,000
Plant & Machinery 800,000
Computers 170,000
Cars 1,600,000
b) The company has borrowed the loan for the purchase of plant & Machinery on 1st Magh, 20X-72 from a commercial
bank. The opening loan was paid on 30th kartik, 20X-72. The agreed rate of interest was 12 % for this loan including
opening.
c) Normal production capacity of the company is 5000 units per year, but actual production was 4500 units during the
income year. The company has 1,500 units in stocks as per FIFO method at the end of the year.
d) The company has purchased a generator on 15 Chaitra 20X-72.
e) Payment of office expense includes advance amounting of Rs. 200,000 to Fulbari printing Pvt. Ltd.
f) No need to consider the VAT amount on above transactions.

© The Institute of Chartered Accountants of Nepal 109


CAP II Paper 7 Income Tax and VAT

Answer
Statement of Assessable income from Business
Sec. Export
Particulars Working Notes
Ref Income
Inclusions:
Export Sales 7 (2) 12,000,000
Total Inclusions 12,000,000
Deductions
Assuming the asset purchased utilizing loan has been in use
during the year, 12% p.a. of Rs. 4,200,000 for six months
Interest Expenses 14 252,000
Further assumed that the opening balance of loan was repaid at
the beginning of year
Cost of Trading Stock 15 7,944,667 Refer W.N. 1
Depreciation for the Year 19 1,306,667 Refer W.N. 2
Administrative Salary 13 650,000 Assumed that it is accrued and paid
Office Expenses 13 200,000 Prepaid expense is not deductible
Total Deductions 10,353,333
Assessable Income from Business 1,646,667

Statement of Taxable income and tax liability

Assessable Income from Business 1,646,667


Total Assessable Income 1,646,667
Less:
Reductions u/s 12 -
Reductions u/s 12Ka -
Reductions u/s 12Kha- National
-
Reconstruction Fund of GON
Taxable Income 1,646,667

© The Institute of Chartered Accountants of Nepal 110


Compilation of Suggested Answers Income Tax

Tax Rate 12%


Tax Liability 197,600
Working Notes
1 Cost of Trading Stock for the Year
Value of Opening Stock 2,400,000
Cost of Conversion
Purchase of Raw Material 6,000,000
Wages for Labour 500,000
Variable Factory Overhead 250,000
Factory Rent (Fixed Factory OH) 1,200,000
Production Supervisors Salary 250,000
Insurance for Factory 117,000 8,317,000
Per unit Cost of Production 1,848 Cost of conversion divided by actual production units
Per unit cost of Production multiplied by no. of closing
Value of Closing Stock (2,772,333)
stocks units
Cost of Trading Stock for the Year 7,944,667

2 Computation of Depreciation Allowances


Calculation of Depreciation and Repair and Improvement Cost
Particulars Block A Block B Block C Block D Total
Opening Depreciation Base - 170,000 1,600,000 800,000
Add: Absorbed additions
Up to Poush (100% of cost)
From Magh to Chaitra (2/3rd of Cost)
(Purchase of generator and Plant & 3,333,333
machinery- Refer below)
From Baisakh to Ashad (1/3rd of Cost)
Less: Disposal Proceeds of Assets (10,000)
(Sales of old Computer)

© The Institute of Chartered Accountants of Nepal 111


CAP II Paper 7 Income Tax and VAT

Depreciable Basis - 160,000 1,600,000 4,133,333


Depreciation Rate 6.67% 33.33% 26.67% 20.00%
Depreciation Amount - 53,333 426,667 826,667 1,306,667
Note: It is assumed that the plant and machinery is used since the date of purchase

3 Identification of Tax Rate


Export
Applicable Tax Rate 25%
Reduction to special industry 20% Sec. 11 (2Kha) (Kha)
Reduced Tax Rate 20%
Reduction due to export 20% 16% Sec. 11 (3Nga) (Kha)
Additional Reduction to manufacturing exporter 25% 12% Sec. 11 (3Nga) (Ga)
Lower Rate 12%

86. Mr. Hari Shrestha has a business of selling Computers. During the FY 20X-72/X-73, he has made a profit of Rs. 15,00,000
from sale of Computer business. Further, during the year, he has sold his entire office equipment and furniture related with
computer business for Rs. 10,00,000. He has purchased those office equipment and furniture for Rs. 15,00,000, and opening
WDV as per last year’s tax return of the said office equipment and furniture was Rs. 7,00,000. He has purchased no any
other office equipment and furniture during the year, and he has no any other office equipment and furniture used in other
business as well.
Mr. Hari Shrestha has another business of developing Compute Software. The Computer software development business
was run on rented office space. During the FY 20X-72/X-73, he has incurred net loss of Rs. 6,00,000 in the said business of
developing Compute Software. His business of developing Compute Software has been in loss for past several years, and has
carried forwarded eligible business loss of Rs 10,00,000 as per his last years tax return.
Mr. Hari Shrestha has incurred loss of Rs 5,00,000 in sale of Shares of a Bank. He has made profit of Rs. 3,00,000 in sale of
shares of an Insurance Company.
Calculate net taxable income of Mr. Hari Shrestha for the FY 20X-72/X-73. (Dec 2016, 5 Marks)
Answer

© The Institute of Chartered Accountants of Nepal 112


Compilation of Suggested Answers Income Tax

Assessable Income from Computer Business - Rs. 1,800,000


(Rs. 1,500,000+Gain on Disposal of Block B of Depreciable Asset)
Assessable Loss from Software Business- Rs. 600,000
Carried forward business loss on Computer software business- Rs. 1,000,000
(assumed to be since last 7 Income Years)
Loss on Sale of Shares of Bank Rs. 500,000
Profit on Sale of shares of insurance companies 300,000

As per 20 (1), business loss of any Income Year can be set off against another business income of the same person, or investment
income of the same person. Here, Computer selling business and software development business is not treated as conducted by
separate person u/s 11 of the Act, therefore, loss on Computer software business can be set off against computer trading business.
Furthermore, the loss of previous seven income years can be set off against business or investment income of current Income Year.

As per Sec. 20 (2), investment loss of same year or of previous seven income years can only be set off against investment income.

As per Sec. 20 (6), the taxpayer has the choice to make to set off losses, and a rational taxpayer always applies the choices such that
there is less tax obligation.

Therefore, Mr. Hari Shrestha will set off investment loss of Rs. 5 Lakhs against investment income of Rs. 300,000 and will carry
forward Rs. 2 Lakhs loss for set off in subsequent income year(s).

Mr. Hari Shrestha will apply business income of Rs. 18 Lakhs to set off current year loss of Rs. 6 Lakhs and previous year loss of
Rs. 10 Lakhs, and will have assessable income from business of Rs. 2 Lakhs.

87. Shalimar trading & manufacturing Co. Ltd., located at Birjung in which Garima Charitable Trust an exempt entity holds
35% of share capital of Shalimar trading & manufacturing Co. Ltd. provide you manufacturing and profit and loss account
for the financial year 20X-71/X-72 are as follows:
Particulars Amount (Rs.) Particulars Amount (Rs.)
To, Opening Stock RM 550,000 By, Sales 3,000,000

© The Institute of Chartered Accountants of Nepal 113


CAP II Paper 7 Income Tax and VAT

To, Purchases 1,150,000


To, Direct mfg. costs 300,000
To, Freight 100,000
To, Gross Profit 1,100,000 By, Closing Stock 200,000
Total 3,200,000 Total 3,200,000
To, Office Expenses 75,000 By, Gross Profit 1,100,000
To, Depreciation calculated as per IT 125,000 By, Interest-Business (net) 85,000
Act
To, Interest expenses 225,000
To, Repair & Maintenance 85,000 By, Bad debts recovered 185,000
To, Salaries 625,000 By, Dividend (net) from Resident. Co. 340,000
To, Fine & Penalties 5,000
To, Income Tax Provision 18,000
To, Net Profit 552,000
Total 1,710,000 Total 1,710,000

Additional information:
• Interest expenses include Rs. 185,000 paid to Garima Charitable Trust.
• Bad debts recovered during the year were not allowed to deduct during last year.
• Opening stock is undervalued by 20% and closing stock is overvalued by 25%.
• Depreciation is not allowed related to assets under Block-C category by Rs. 25,000, which is included in calculated figure
Rs. 125,000.
• Repair & maintenance include Rs. 45,500 related to assets under Block-B.
• For the purpose of repair and maintenance, opening depreciation base of Block-B is Rs. 600,000 and addition of Block-
B assets as on 25th Jestha 20X-72 amounted to Rs. 150,000. ( Ignore additional depreciation, if applicable, for repair and
maintenance limit calculation).
• Donation provided to tax exempt entity amounted to Rs. 5,500.

Required: (June 2016, 15+5=20)

© The Institute of Chartered Accountants of Nepal 114


Compilation of Suggested Answers Income Tax

a. Compute the taxable business income & tax liability of Shalimar trading & manufacturing Co. Ltd. with working notes
for the financial year 20X-71/X-72 as per Income Tax Act, 2058?
b. Explain “a resident entity controlled by an organization entitled to tax exemption" as per explanation provided for the
purpose of section 14(2) of Income Tax Act, 2058?

Answer

Assessable Income of Shalimar Trading & Manufacturing


Export
Particulars Sec. Ref Working Notes
Income
Inclusions:
Sales 7 (2) 3,000,000
Interest Income 7 (2) 85,000 Separate effect not given
as it is reduction on tax base of debt, and not an
Bad debt recovered 25 -
income for tax purpose
Dividend from resident company 92 - Final withholding

Total Inclusions 3,085,000


Deductions
Cost of Trading Stock 15 1,980,833 Refer W.N. 1
Depreciation for the Year 19 100,000 Given less not allowed amount of Rs. 25000
Repair and Improvement Cost 16 85,000 Refer W.N. 2
Salaries 13 625,000
Office Expenses 13 75,000
Not deductible, as it is assumed to be paid for
Fine and Penalties 21 -
non-compliance of laws and regulations
Income Tax Provision 21 - Tax paid under Income Tax Act

Total deductions before adjustment to determine ATI 2,865,833

© The Institute of Chartered Accountants of Nepal 115


CAP II Paper 7 Income Tax and VAT

Paid to other than entity with controlling


Interest expense where Sec. 14 (2) is not applicable 40,000
interest
Interest expense where Sec. 14 (2) is applicable 152,083 Refer W.N. 3
Total Deductions 3,057,917
Assessable Income from Business 27,083

Statement of Taxable income and tax liability


Assessable Income from Business 27,083
Total Assessable Income 27,083
Less:
Reductions u/s 12 - Refer W.N. 4
Reductions u/s 12Ka -
Reductions u/s 12Kha -
Taxable Income 27,083
Tax Rate 20%
Tax Liability 5,417

Working Notes
1. Cost of Trading Stock for the Year
Value of Opening Stock
Given 550,000
Add: Undervalued amount 137,500 687,500
Cost of Conversion
Purchase of Raw Material 1,150,000
Direct Manufacturing cost 300,000
Freight 10,000 1,460,000
Per unit cost of Production multiplied by no. of
Value of Closing Stock
closing stocks units
Given 200,000

© The Institute of Chartered Accountants of Nepal 116


Compilation of Suggested Answers Income Tax

Less: Overvalued amount (33,333) (166,667)

Cost of Trading Stock for the Year 1,980,833

2. Computation of Repair and Improvement Cost


The question specifies that repair cost of Rs. 45,500 is related to Block B, and silent about remaining repair cost. It is assumed that
the remaining repair cost is eligible under Sec. 16
Calculation of Repair and Improvement Cost
Particulars Block B Total
Opening Depreciation Base 600,000
Add: Absorbed additions
Up to Poush (100% of cost)
From Magh to Chaitra (2/3rd of Cost)
From Baisakh to Ashad (1/3rd of Cost) 100,000
Less: Disposal Proceeds of Assets
Depreciable Basis 700,000

7% of Depreciable Basis 49,000


Actual Repair and Improvement Expense 45,500
Allowable Repair and Improvement Cost
45,500
(Minimum of 7% of Depreciation Base or Actual) 45,500
Add: Remaining Repair and Improvement Cost
39,500
Total Eligible cost
85,000

3. Adjusted Taxable income and the eligible expenses for Sec. 14(2)
Inclusions before Interest Income 3,000,000

© The Institute of Chartered Accountants of Nepal 117


CAP II Paper 7 Income Tax and VAT

Deductions before Interest expenses, PCC, and R&D


(2,865,833)
Cost
Deductions of PCC and R&D Cost (Actual) -
Reductions u/s 12, 12A -
Adjusted Taxable Income 134,167
50% of Adjusted Taxable Income (i) 67,083
Add: Interest Income (ii) 85,000
Maximum Eligible amount (A= i+ii) 152,083
Actual accrued amount (B) 185,000
Eligible amount (Min of "A" or "B") 152,083

4. Adjusted Taxable income and the eligible expenses for Sec. 12


Eligible Donation u/s 12
Inclusions before Interest Income 3,000,000
Deductions before Interest expenses, PCC, and R&D
(2,865,833)
Cost
Deductions of PCC and R&D Cost (Actual) -
Deduction of Interest Expense- Sec. 14(2) not
(40,000)
applied- eligible
Deduction of Interest Expense- Sec. 14(2) applied-
(185,000)
actual
Reductions u/s 12, 12A -
Adjusted Taxable Income (90,833)
Since ATI is negative, there is no reduction of
donation

Identification of Tax Rate


Special
Industry
Applicable Tax Rate 25%

© The Institute of Chartered Accountants of Nepal 118


Compilation of Suggested Answers Income Tax

Reduction to special industry 20% Sec. 11 (2Kha) (Kha)


Reduced Tax Rate 20%

88. An accounting firm owned by Amber Poudel maintains its books of account on cash basis. Mr. Poudel submitted his financial
information for the financial year 20X-71/X-72 as follows:
Receipts Amount in (Rs.) Payments Amount in (Rs.)
To, Opening cash/bank balance. 60,000 By, office expenses 75,000
To, Consultancy fees-gross 648,000 By, salary to staff 425,000
To, Book writing fee 1,700,000 By, Life Insurance policy
26,000
To, Bank Loan @ 10 % 600,000 By, Purchase of furniture 90,000
To, Sale of old journals 12,000 By, Office Rent 500,000
To, Lottery prize 150,000 By, Interest Exp. 60,000
To, Paper checking from T.U. 200,000 By, Water & Electricity Expenses 125,000
To. Part time lecture 135,000 By, Purchase of journals 4,000
By, Transportation Expenses 100,000
By, Closing cash/bank bal. 2100,000
3,505,000 3,505,000

Additional information:

a. Salary expenses include salary paid to a staff for last three years amounted to Rs. 125,000 and advance salary given to
another staff amounted to Rs. 25,000.
b. Opening WDV of furniture was Rs. 170,000 and during the year additional furniture was purchase during the end of
Jestha, 20X-72.
c. Consultancy fee received includes advance from Paban Limited Rs. 125,000.
d. Office Rent paid includes payments for financial 207X-2/X-73, 20X-73/X-74 & 20X-74/X-75
e. Fifty percent of loan used for personal purpose by Mr. Poudel.
Required: (Dec 2015, 5+5=10)

© The Institute of Chartered Accountants of Nepal 119


CAP II Paper 7 Income Tax and VAT

Compute the taxable income and tax liability of accounting firm owned Mr. Poudel for the financial year 20X-71/X-72.
Answer
Since, a private firm can follow cash basis of accounting, hence, the income is calculated under cash basis.
Assessable Income of the accounting firm
Sec.
Particulars Amounts Working Notes
Ref
Inclusions:
Consultancy Fees 7 (2) 648,000
Book writing Fee 7 (2) 1,700,000
Sale of Old Journals 7 (2) 12,000
Lottery Prize 92 - Final withholding
Paper checking from T.U. 92 - Final withholding
Part time lecture 92 - Assumed occasional, and hence, final withholding

Total Inclusions 2,360,000


Deductions
50% loan used for personal purpose, interest on
Interest expense 14 30,000
which is not deductible
Depreciation for the Year 19 50,000 Refer W.N. 1
Salary to Staff 13 425,000 Cash basis, hence all deductible
Office Rent 13 500,000 Cash basis, hence all deductible
Water & Electricity Expense 13 125,000
Purchase of Journals 13 4,000
Transportation Expenses 13 100,000
Total Deductions 1,234,000

Assessable Income from Business 1,126,000

© The Institute of Chartered Accountants of Nepal 120


Compilation of Suggested Answers Income Tax

Statement of Taxable Income and Balance Taxable


Income
Assessable Income from Business 1,126,000
Assessable Income from Employment -
Assessable Income from Investment -
Total Assessable Income 1,126,000
Less: Reductions u/s 12, 12Kha & 63 -
Taxable Income 1,126,000
Less: Deductions under Sec. 1 of Sch. 1
Life Insurance Premium Paid
(Rs. 25000 or actual, whichever is lower) 25,000
Balance Taxable Income 1,101,000

Statement of Tax Liability (Couple)


1st Rs. 450,000 0% -
Next Rs. 100,000 10% 10,000
Next Rs. 200,000 20% 40,000
Balance Rs. 351,000 30% 105,300
Total Tax Liability 155,300

Working Note
1. Depreciation
Block B
Opening Depreciation Base 170,000
Add: Absorbed Additions 30,000
(1/3rd of 90,000)
Less: Disposal Proceeds -
Depreciable Basis 200,000
Depreciation @ 25% 50,000

© The Institute of Chartered Accountants of Nepal 121


CAP II Paper 7 Income Tax and VAT

89. Gorkhali Cement (P.) Ltd., Rupendehi, has the following transactions during the year 20X-71/X-72:
Profit & loss Account

Particulars Amount (Rs.)


Sales 31,000,000
Cost of Sales 17,600,000
Gross profit 12,400,000
Other Income 500,000
Administrative expenses 2,000,000
Selling & Distribution Expenses 600,000
Depreciation 6,800,000
Interest expenses 2,000,000
Profit before tax 2,500,000

Additional Information:
a. Other income was the interest from Bank of Kathmandu. It has presented net of Tax.
b. Cost of sales includes the following:

Particulars Amount (Rs.) Amount (Rs.)


Opening stocks 700,000
Clinker and Other raw material purchase 18,000,000
Manufacturing Expenses
Salary and wages for factory 1,200,000
Repair and maintenance expenses 400,000
Other overhead expenses 300,000
Total manufacturing expenses 1,900,000
Closing stocks 3,000,000
Cost of sales 11,900,000

© The Institute of Chartered Accountants of Nepal 122


Compilation of Suggested Answers Income Tax

c. Cost of stocks includes the repair and maintenance expenses, which is approximately 2 % of the valuation.
d. Depreciation includes Rs. 2,000,000 for the construction of Factory which was completed on 20 Mangsir, 20X-71 with
total cost of Rs. 40,000,000 without charging the interest. Depreciation for others are as follows:
Block Depreciation base (Rs.) Depreciation (Rs.)
B 1,200,000 300,000
D 30,000,000 4,500,000
e. Interest expense was charged for the loan borrowed for factory construction. The loan Rs. 20,000,000 with 10 percent
interest rate was disbursed on 1st Ashad 20X-71 from concerned bank and remains even during the year.

Based on the above information, you are required to calculate Taxable income and Tax liability of the Pvt. Ltd. for Income
Year 20X-71/X-72. (July 2015, 10 Marks)

Answer

Statement of Assessable income from Business (Gorkhali Cements)


Particulars Sec. Ref Amount Working Notes
Inclusions:
Sales 7 (2) 31,000,000
Separate treatment not done, and grossed up to consider
Other Income (500,000 divided by 0.85) 7 (2) 588,235
TDS
Total Inclusions 31,588,235
Deductions
Factory is used during the Income Year, as such, expense is
Interest Expenses 14 2,000,000
deductible
Cost of Trading Stock 15 17,186,000 Refer W.N. 1
Depreciation for the Year 19 9,088,889 Refer W.N. 2
Repair and Improvement Cost 16 240,000 Refer W.N. 2, assuming the repair costs belong to Block D
Administrative Expense 13 2,000,000
Selling and Distribution Expense 13 600,000
Total Deductions 31,274,889

© The Institute of Chartered Accountants of Nepal 123


CAP II Paper 7 Income Tax and VAT

Assessable Income from Business 313,346

Statement of Taxable income and tax


liability
Assessable Income from Business 313,346
Total Assessable Income 313,346
Less:
Reductions u/s 12 -
Reductions u/s 12Ka -
Reductions u/s 12Kha- National
-
Reconstruction Fund of GON
Taxable Income 313,346
Tax Rate 20%
Tax Liability 62,669

Working Notes
1. Cost of Trading Stock for the Year
Value of Opening Stock
Given
700,000
Less: Repair and Maintenance of
686,000 Assumed the repair expense is of depreciable asset
Depreciable asset (14,000)

Cost of Conversion
Purchase of Raw Material (Clinker and
Others) 18,000,000
Salary and Wages for Factory
1,200,000
Repair & Maintenance Expenses - Assumed to be of Depreciable asset

© The Institute of Chartered Accountants of Nepal 124


Compilation of Suggested Answers Income Tax

Other Overhead Expenses


300,000
19,500,000
Assuming FIFO and repair cost already excluded while
Value of Closing Stock (3,000,000)
calculating cost of conversion
Cost of Trading Stock for the Year 17,186,000

2. Computation of Depreciation Allowances


Calculation of Depreciation and Repair and Improvement Cost
Particulars Block A Block B Block D Total
Opening Depreciation Base 1,200,000 30,000,000
Add: Absorbed additions
Up to Poush (100% of cost)
(Factory construction, interest cost of Ashad 20X-71 40,333,333
capitalized)
From Magh to Chaitra (2/3rd of Cost) -
From Baisakh to Ashad (1/3rd of Cost)
Less: Disposal Proceeds of Assets
Depreciable Basis 40,333,333 1,200,000 30,000,000
Depreciation Rate 6.67% 33.33% 20.00%
Depreciation Amount 2,688,889 400,000 6,000,000 9,088,889
7% of Depreciable Basis 2,823,333 84,000 2,100,000
Actual Repair and Improvement Expense - - 400,000
Allowable Repair and Improvement Cost
- - 400,000 400,000
(Minimum of 7% of Depreciation Base or Actual)

3. Identification of Tax Rate


Export

© The Institute of Chartered Accountants of Nepal 125


CAP II Paper 7 Income Tax and VAT

Applicable Tax Rate 25%


Reduction to special industry 20% Sec. 11 (2Kha) (Kha)
Reduced Tax Rate 20%

90. Income Statement of a company registered under Company Act, 2063 for the income year 20X-67/X-68 is given below:
Expense Rs. Income Rs.
Cost of Goods sold 7,000,000 Sales 10,000,000
Interest expense 500,000 Interest income 100,000
Personnel expense 1,500,000 Other income 200,000
Administrative expense 900,000
Net profit 400,000
10,300,000 10,300,000
Further information is as follows:
a. Cost of goods sold includes factory depreciation Rs. 300,000, the allowable depreciation as per Income Tax Act is Rs.
250,000 only. Similarly, administrative expense includes depreciation of Rs. 100,000 and the allowable depreciation is Rs.
250,000.
b. The shareholders of the company are foreigners and 50% of interest expense is the amount paid to them, remaining
amount is paid for foreign bank loan.
c. Donation of Rs. 20,000 given to an exempt entity is included in administrative expense; staff themselves donated to the
same entity Rs. 70,000.
From the above information, find current tax payable by the Company. (Dec 2011, CA Inter, 15)
Answer
Statement of Assessable income from Business
Particulars Sec. Ref Amount Working Notes
Inclusions:
Sales 7 (2) 10,000,000
Interest Income 7 (2) 100,000

© The Institute of Chartered Accountants of Nepal 126


Compilation of Suggested Answers Income Tax

Separate treatment not done, and grossed up to


Other Income 7 (2) 200,000
consider TDS
Total Inclusions 10,300,000
Deductions
Cost of Trading Stock 15 6,700,000 Refer W.N. 1
Refer additional information (a) and considering
Depreciation for the Year 19 250,000 that total allowable depreciation is Rs. 250,000.
Language is ambiguous though
Deduction of depreciation and donation included in
Administrative Expense 13 780,000
this expense
Personnel Expenses 13 1,500,000
Deductions before considering Interest Expense,
9,230,000
PCC and R&D Cost
Interest expense, where Sec. 14 (2) is not
250,000 Paid to foreign bank
applicable
Refer W.N. 2, paid to person having controlling
Interest expense, where Sec. 14 (2) is applicable 250,000
interest
Total Deductions 9,730,000
Assessable Income from Business 570,000

Statement of Taxable income and tax liability


Assessable Income from Business 570,000
Total Assessable Income 570,000
Less:
Reductions u/s 12 20,000 Refer W.N. 3
Reductions u/s 12Ka -
Reductions u/s 12Kha- National Reconstruction
-
Fund of GON
Taxable Income 550,000
Tax Rate 25% Since it is a trading company

© The Institute of Chartered Accountants of Nepal 127


CAP II Paper 7 Income Tax and VAT

Tax Liability 137,500

Working Notes
1. Cost of Trading Stock for the Year
Cost of Goods Sold
Given 7,000,000
Less: Depreciation of factory included in COGS (300,000) 6,700,000
Cost of Trading Stock for the Year 6,700,000

2. Calculation of Allowable Interest expense

Inclusions without considering Interest Income 10,200,000


Deductions without considering interest expense,
9,230,000
PCC and R&D Cost
Less:
Actual PCC incurred during the Year -
Actual R&D Cost incurred during the Year -
Expenditure u/s 12A -
Contribution u/s 12B - -
Adjusted Taxable Income for the purpose of Sec.
970,000
14 (2)

Allowable:
Interest income plus 50% ATI Calculated Above
585,000
(A)
Actual interest expenses in respect of controlling
250,000
person (B)
Eligible ("A" or "B", whichever is lower) 250,000

© The Institute of Chartered Accountants of Nepal 128


Compilation of Suggested Answers Income Tax

3. Calculation of Allowable Donation

Inclusions 10,300,000
Deductions without considering interest expense,
9,230,000
PCC and R&D Cost
Less:
Eligible Interest expense, where Sec. 14 (2) is not
250,000
applicable
Actual Interest Expense, where Sec. 14 (2) is
250,000
applicable
Actual PCC incurred during the Year -
Actual R&D Cost incurred during the Year -
Expenditure u/s 12A -
Contribution u/s 12B - 500,000
Adjusted Taxable Income for the purpose of Sec.
570,000
12

Allowable:
5% of ATI Calculated above (A) 28,500
Maximum (B) 100,000
Actual (C) 20,000 Donated by staff is not the amount paid by company
Eligible (Lower of "A", "B" or "C") 20,000

© The Institute of Chartered Accountants of Nepal 129


CAP II Paper 7 Income Tax and VAT

91. From the following Income Statement of M/S ABC Pvt. Ltd., prepare the taxable income for the income year 20X-66/X-67.
(Dec 2010, 10 Marks)
Rs. In ‘000
Sales 8,000
Cost of sales 5,000
Gross profit 3,000
Administrative expenses 500
Donation 100
Interest expenses 300
Pollution control expenses 1,500
Research & Development expenses 1,000
Depreciation 50
Balance (450)
Other Income 50
Net Profit/ (loss) (400)

Answer
Statement of Assessable income from Business (ABC Pvt. Ltd.)
Particulars Sec. Ref Amount Working Notes
Inclusions:
Sales 7 (2) 8,000,000
Other Income 7 (2) 50,000

Total Inclusions 8,050,000


Deductions

Cost of Trading Stock 15 5,000,000 There is no adjustment, therefore, assumed to be as per Tax

© The Institute of Chartered Accountants of Nepal 130


Compilation of Suggested Answers Income Tax

Depreciation for the Year 19 50,000 There is no adjustment, therefore, assumed to be as per Tax

Administrative Expense 13 500,000 There is no adjustment, therefore, assumed to be as per Tax


Deductions before considering
Interest Expense, PCC and R&D 5,550,000
Cost

Interest expense, where Sec. 14 (2) Since there is no further information, it is assumed that Sec.
300,000
is not applicable 14 (2) is not applicable

Interest expense, where Sec. 14 (2)


-
is applicable
Pollution Control Cost 600,000 Refer W. N. 1
Research and Development Cost 350,000 Refer W. N. 1

Total Deductions 6,800,000


Assessable Income from Business 1,250,000

Statement of Taxable income and


tax liability
Assessable Income from Business 1,250,000
Total Assessable Income 1,250,000
Less:
Reductions u/s 12 - Refer W.N. 2
Reductions u/s 12Ka -
Reductions u/s 12Kha- National
-
Reconstruction Fund of GON
Taxable Income 1,250,000

Working Notes

© The Institute of Chartered Accountants of Nepal 131


CAP II Paper 7 Income Tax and VAT

1. Calculation of Pollution Control cost and R&D Cost


For PCC For R &D Cost
Inclusions as per Income Tax Act 8,050,000 8,050,000
Deductions without considering interest expense, PCC and
5,550,000 5,550,000
R&D Cost
Less:
Eligible interest expense, where Sec. 14 (2) is not applicable 300,000 300,000
Actual interest cost, where Sec. 14 (2) is not applicable - -
Actual PCC incurred during the Year - 1,500,000
Actual R&D Cost incurred during the Year 1,000,000
Expenditure u/s 12A -
Contribution u/s 12B - 1,300,000 1,800,000
Adjusted Taxable Income for the purpose of Sec. 14 (2) 1,200,000 700,000

Allowable:
50% ATI Calculated Above (A) 600,000 350,000
Cost incurred during the year (B) 1,500,000 1,000,000
Eligible ("A" or "B", whichever is lower) 600,000 350,000

2. Calculation of Allowable Donation

Inclusions 8,050,000
Deductions without considering interest expense, PCC and
5,550,000
R&D Cost
Less:
Eligible Interest expense, where Sec. 14 (2) is not
300,000
applicable
Actual Interest Expense, where Sec. 14 (2) is applicable -
Actual PCC incurred during the Year 1,500,000

© The Institute of Chartered Accountants of Nepal 132


Compilation of Suggested Answers Income Tax

Actual R&D Cost incurred during the Year 1,000,000


Expenditure u/s 12A -
Contribution u/s 12B - 2,800,000
Adjusted Taxable Income for the purpose of Sec. 12 (300,000)

Allowable:
5% of ATI Calculated above (A) (15,000)
Maximum (B) 100,000
assumed to be donated to exempt
100,000
Actual (C) entity
Eligible (Lower of "A", "B" or "C") - Since, ATI is negative

92. Kailali (P.) Ltd. engaged in producing edible oils, could not prepare its accounts on accrual basis of accounting and prepared
only receipt and payment accounts. You just joined in the company as Chief Accountant. With the help of existing assistants,
you could gather following information. Find the tax payable for the income year 20X-66/X-67.
Particulars Rs. Particulars Rs.
Opening Bank 100,000 Loan repaid 300,000
Payment for purchase 7,000,000 Sales realized 10,000,000
Interest paid 500,000 Interest income 100,000
Personnel 1,500,000 Other income 200,000
Administrative Expense
paid
Wage expense paid 900,000 Bank balance 200,000
Loan received 800,000
10,800,000 10,800,000

Further information gathered by the assistants are as follows:

© The Institute of Chartered Accountants of Nepal 133


CAP II Paper 7 Income Tax and VAT

Particulars Opening Closing


Rs. Rs.
Debtors 100,000 1,300,000
Creditors 300,000 100,000
Paid up capital 1,000,000 1,500,000
Machineries 1,500,000 2,100,000
Interest payable 50,000 150,000
Wage payables 50,000 100,000
3,000,000 5,250,000
b) Depreciation has not been charged in the books. Additional investments in machineries have been done on Ashad 20X-
67. Trading stock of Rs. 100,000 has not been shown in the above information.
c) The shareholders of the company are foreigners and interest are paid to one of the foreign banks.
d) Donation of Rs. 20,000 has been given to an exempt entity which is included in administrative expense. Staff also donated
Rs. 7,000 to the same entity.
e) Personnel expense excludes a gratuity provision of Rs. 200,000. During this year Rs. 30,000 was paid as retirement
payment from this provision account.
From the above information, find current tax expense for income year 20X-66/X-67. (Dec 2010, 20 marks, CA Inter)
Answer
Working Notes
1. Value of Sales
Closing Debtors Values 1,300,000
Add: Sales Realization 10,000,000
Less: Opening Debtors (100,000)
Sales 11,200,000

2. Value of Purchases
Closing Creditors 100,000
Add: Payment for Purchases 7,000,000

© The Institute of Chartered Accountants of Nepal 134


Compilation of Suggested Answers Income Tax

Less: Opening Creditor (300,000)


Purchases during the year 6,800,000

3. Interest Expenses
Closing Interest Payable 150,000
Add: Interest paid during the Year 500,000
Less: Opening Interest Payable (50,000)
Interest Expenses 600,000

4. Accrued Wages
Closing Wages Payable 100,000
Add: Wages paid during the Year 900,000
Less: Opening Wages Payable (50,000)
Wage Expenses 950,000

5. It is assumed that:
a. Interest is paid to the extent of accrued.
b. Personnel expense is paid to the extent of accrued.
c. Interest income and other income is received to the extent of accrued.

Statement of Assessable Income from Business


Particulars Sec. Ref Amount Notes
Inclusions
Sales income 7 (2) 11,200,000
Interest Income 7 (2) 100,000
Other Income 7 (2) 200,000

Total Inclusions 11,500,000


Deductions

© The Institute of Chartered Accountants of Nepal 135


CAP II Paper 7 Income Tax and VAT

Cost of purchases minus value of


Cost of Trading Stock 15 6,700,000 closing stock as in additional
information (a)
Depreciation 19 340,000 W.N. 1 below
Administrative Expense 13 1,510,000 W.N. 2 below
Wage Expense 13 950,000
Deductions before deduction of Interest Expense, PCC and R&D
9,500,000
Cost
Interest expense, where Sec. 14(2) is not applicable -
Interest expense, where Sec. 14(2) is applicable 600,000 Refer W.N. 3 below
Total Deductions 10,100,000
Assessable Income from Business 1,400,000

Statement of Taxable Income


Assessable Income from Business 1,400,000
Assessable Income from Investment -
Total Assessable Income 1,400,000
Less:
Reduction u/s 12 20,000 Refer W.N. 4 below
Reduction u/s 12Ka
Reduction u/s 12Kha
Taxable Income 1,380,000

Calculation of Current Tax Expenses


Taxable Income 1,380,000
Corporate Tax @ 20% 276,000 Special Industry

1. Calculation of Depreciation

© The Institute of Chartered Accountants of Nepal 136


Compilation of Suggested Answers Income Tax

Block D
Opening Depreciation Base 1,500,000 Assumed this is tax base
Add: Absorbed Additions 200,000
(1/3rd of Rs. 600,000) as machinery is used since Ashad
Less: Disposal Value -
Depreciable Basis 1,700,000
Depreciation @ 20% (Special industry) 340,000

2. Administrative Expense
Given 1,500,000
Less: Donation included in it (20,000)
As gratuity is probable obligation, the
expense in such case is deductible
Add: Gratuity actually Paid 30,000
when there is actual outflow of
resource
1,510,000

3. Deductible interest expense, where Sec. 14 (2) is applicable


Particulars Details Amount Note
Inclusions without interest income 11,400,000
Less: Deductions without Interest expense, PCC and R&D Cost 9,500,000
Less: Actual PCC -
Less: Actual R& D Cost -
Less: Reductions u/s 12Ka -
Less: Reductions u/s 12Kha -
Adjusted Taxable income for the purpose of Sec. 14 (2) 1,900,000
50% of Adjusted Taxable Income (A) 950,000
Interest Income (B) 100,000
Maximum Limit (A+B) 1,050,000

© The Institute of Chartered Accountants of Nepal 137


CAP II Paper 7 Income Tax and VAT

Assuming all paid to the persons


Eligible (Actual not exceeding maximum limit) 600,000
having controlling interest

4. Eligible Donation u/s 12


Inclusions as per Income Tax Act 11,500,000
Less: Deductions without Interest expense, PCC and R&D Cost 9,500,000
Less: Eligible Interest expense, where Sec. 14 (2) is not
-
applicable
Less: Actual Interest cost, where Sec. 14 (2) is applicable 600,000
Less: Actual PCC -
Less: Actual R&D Cost -
Less: Reductions u/s 12Ka -
Less: Reductions u/s 12Kha -
Adjusted Taxable income for the purpose of Sec. 12 1,400,000
5% of Adjusted Taxable Income (A) 70,000
Actual Donation to Exempt Organization (B) 20,000
Maximum Limit [C] 100,000
Eligible (Lower of "A", "B" or "C") 20,000

93. Income Statements for 20X-66/X-67 for a company registered under Company Act, 2063 is abstracted as follows:
Rs. Income Rs.
Expense
Cost of Goods sold 7,000,000 Sales 10,000,000
Interest expense 500,000 Interest income 100,000
Personnel Expense 1,500,000 Other income 200,000
Administrative expense 900,000
Net profit 400,000
10,300,000 10,300,000
Further information is as follows:

© The Institute of Chartered Accountants of Nepal 138


Compilation of Suggested Answers Income Tax

a. Cost of goods sold includes factory depreciation Rs. 300,000, while income tax allowed Rs. 250,000 only. Similarly
administrative expense includes depreciation of Rs. 100,000, income tax allows for Rs. 250,000.
b. The shareholders of company are foreigners and 50% of interest is paid for them, remaining paid to one of foreign
bank loan.
c. Donation Rs. 20,000 has given to an exempt entity included in administrative expense, staff donated to the same entity
Rs. 70,000.
From the above information, find current tax expense for 20X-66/X-67. (June 2010, 15 Marks, CA Inter)
Answer: Same as question 90.

94. Mr. Sitaram Keshari is a consultant appointed for an assignment by a reputed banking company for Rs. 50,000. For
execution of the assignment the bank has provided Mr. Sitaram Keshari a vehicle for 3 months which he can use for both
official as well as personal purposes. Current market price of the vehicle is Rs. 20,00,000. Compute his income from business
with respect to this transaction. Give your answer by quoting relevant provisions of the Income Tax Act/Rules.
(June 2010, 5 Marks, CA Inter)
Answer
When a person provides vehicle facility to any other person who is not employee of the first person, 1% per annum of market value
of such vehicle forms part of income of the person enjoying the facility pursuant to Sec. 27 of the Act; therefore, the amount to be
quantified as vehicle facility is:
Rs. 20,00,000 * 1% * 3/12= Rs. 5,000
The amount to be included with respect to the transaction is Rs. 5,000.

95. Given below is the Trading and Profit and Loss Account of X, a Trader, for the previous year.(Dec 2008, 6+4=10, CA
Inter)
Particulars Amount Particulars Amount
(Rs.) (Rs.)
To Opening Stock 50,000 By Sales 1,400,000
To Purchases 750,000 By Service Charge 48,600

© The Institute of Chartered Accountants of Nepal 139


CAP II Paper 7 Income Tax and VAT

To Salaries & Wages 175,000 By Miscellaneous Income 60,000


To Commission & Discount 30,000 By Agriculture Income 50,000
To Donation to VDC Rural 25,500 By Closing Stock 75,000
Municipality
To Administrative Expenses 32,000
To Insurance Premium 20,500
To Carriage Outward 18,600
To Interest on Loan 110,000
To Legal Expenses 30,000
To Loss of Stock 25,000
To Miscellaneous Expenses 15,000
To Rent 96,000
To General Expenses 42,000
To Provision for Dividend 20,000
To Compensation to Employee 7,000
To Net Profit 187,000
1,633,600 1,633,600

Additional Information:
a. Salaries & wages include Rs. 20,000 paid to Mr. Bikash for the month of Asadh, manager without deducting TDS.
b. Compensation of Rs. 20,000 which was received from insurance company due to loss of stock of Rs. 25,000 (debited in
the profit and loss account).
c. Out of the total insurance premium Rs.5,000 is related to the furniture given on hire.
d. Rent Rs. 1,05,000 was paid in a cash payment even though bank facility is available. This amount has not been included
in above Profit and Loss Account.
e. Purchase include Rs. 1,00,000 purchase cost of computer which was purchased on 10th Magh of the previous income
year and Rs. 50,000 being the cost of custom duty and transport freight in connection with the purchase of computer
that was not shown in the above account. But those computers are booked as assets in company's account.
f. Firm claims to have brought forward previous year cumulative loss Rs. 50,000.

© The Institute of Chartered Accountants of Nepal 140


Compilation of Suggested Answers Income Tax

Required:
a. Assessable income from business.
b. Statement of total taxable income.
Answer
Statement of Assessable Income from Business (of Mr. X)
Particulars Sec. Ref. Amount Notes
Inclusions
Sales 7 (2) 1,400,000
Service Charge 7 (2) 48,600
Assumed that compensation under additional information (b),
Miscellaneous Income 7 (2) 60,000
is included in miscellaneous income
It is assumed that agriculture income is exempt assuming the
Agriculture Income 11 - income is not generated through registration of private firm
and is from the land within the threshold prescribed by GoN.
Total Inclusions 1,508,600
Deductions
Interest Expense 14 110,000
Cost of Trading Stock 15 650,000 Refer W. N. 1
Depreciation 19 25,000 Refer W. N. 2
Failure to withhold tax triggers interest and fees under Sec.
117 and 119 of the Act, and there is no any provision in
Salaries & Wages 13 175,000
Income Tax Act which makes the expense not deductible
merely due to the failure to withhold tax
Commission and Discount 13 30,000
Administrative Expense 13 32,000
Insurance premium paid for furniture given on hire is also
Insurance Premium 13 20,500 business expense, as furniture hire income forms part of
business income

© The Institute of Chartered Accountants of Nepal 141


CAP II Paper 7 Income Tax and VAT

Alternatively, furniture hire income of a natural person may be


treated as invesment income and accordingly, this expense will
be treated as investment expense
Carriage Outward 13 18,600
Legal Expense 13 30,000
Miscellaneous Expense 13 15,000
96,000+1,05,000
Though rent is paid in cash, as the turnover of the person is
Rent 13 201,000
1,508,600 that is less than Rs. 20 Lakhs, the amount cannot be
made not deductible merely due to the reason of cash payment
General Expense 13 42,000
Provision for dividend 21 - Not deductible
Compensation to Employee 13 7,000
Total Deductions 1,356,100
Deductible carried forward losses 20 50,000 Assuming the loss was since 7 years ago
Total deductions including losses 1,406,100
Assessable Income from Business 102,500

Statement of Taxable Income


Assessable Income from Business 102,500
Assessable Income from
-
Employment
Assessable Income from
-
Investment
Total Assessable Income 102,500
Less: Reductions
Under Sec. 12 - Rural Municipality is not an exempt entity
Under Sec. 12Kha -
Under Sec. 63 -
Taxable Income 102,500

© The Institute of Chartered Accountants of Nepal 142


Compilation of Suggested Answers Income Tax

Working Notes
1. Cost of Trading Stock
Value of Opening Stock 50,000
Add: Purchases 750,000
(100,000
Less: Accounting errors 650,000 Purchase of computer included in purchase, now rectified
)
Loss of stock is cost of disposal of stock, and it is part of cost
Add: Loss of Stock 25,000
of stock
Value of Closing Stock (75,000)
Cost of Trading Stock 650,000

2. Depreciation
Block B
Opening Depreciation Base -
Add: Absorbed Additions 100,000
(2/3rd of Rs. 150,000) Added in Magh as per additional information point (e)
Less: Disposal Proceeds -
Depreciable Basis 100,000
Depreciation @ 25% 25,000

96. Hard Steel Udyog, proprietorship firm owned by Mr. Jeevan unmarried individual handicapped has following Trading &
Profit and Loss Account for F.Y. 20X-63/X-64 on accrual basis.

Particulars Rs. Particulars Rs.


To Opening Stock By Sales 1,500,000
Raw Material - By Closing Stock
Finished Goods - - Raw Material -

© The Institute of Chartered Accountants of Nepal 143


CAP II Paper 7 Income Tax and VAT

Raw Material Purchase 500,000 - Finished Goods 560,000


Production Expenses
To Labour Wages 150,000
To Electricity 200,000
To Production Salary 100,000
To Depreciation (Block D) 200,000
To Plant Repair & Improvement 80,000
To Insurance Premium 10,000
(Machinery)
Gross Profit 820,000
Total 2,060,000 Total 2,060,000

By Gross Profit C/D 820,000


To Selling & Distribution 150,000 By Dividend (Gross) 200,000
To Insurance Premium Expenses 60,000
(Office Equipment)
To Other Expenses 4,000
To Net Profit 806,000
Total 1,020,000 1,020,000

Additional Information
a. It purchases raw material 50,000 kg @ Rs. 10/kg from local market on 20X-63/4/1.
b. Depreciation Base of Block ‘D’ is Rs. 1,000,000.
c. Selling & Distribution Expenses includes payment against truck freight Rs. 75,000 in cash on 20X-63/7/10 to Mr. Rajan,
truck driver.

© The Institute of Chartered Accountants of Nepal 144


Compilation of Suggested Answers Income Tax

d. Insurance Premium Expenses (Office Equipment) is related to 20X-62/10/01 to 20X-64/03/32 and Insurance Bill is
attached with Voucher.
e. Data for consumption and production were as follows:
Particulars Opening Purchase/ Total Consumption/ Closing
Production Sales

Raw Material (kg) - 50,000 50,000 50,000 -

Finished Goods (kg) - 48,000 48,000 32,000 16,000

2,000 kg (50,000 – 48,000) is normal production loss.

f. He has received Salary Rs. 110,000 of F.Y. 20X-62/X-63 and Rs. 50,000 for F.Y. 20X-63/X-64 in cash from part time
employment in other companies.
g. He received dividend from Moon Rise Bank Ltd., registered as per Company Act, 2063.
h. He contributed Rs. 300,000 to Citizen Investment Trust (CIT).
i. Closing Stock of Finished Goods has been valued at Closing Market Rate i.e. Rs. 35/Kg.
Required: (June 2008, 12 Marks, CA Inter)
a. Calculate Taxable Income
b. Calculate Tax Liability

Answer
Statement of Assessable Income from Business (Mr. Jeevan)
Particulars Sec. Ref. Amount Note
Inclusion
Sales 7 (2) 1,500,000
Dividend Income 92 - Final withholding,as it is received from resident company
Total Inclusions 1,500,000

© The Institute of Chartered Accountants of Nepal 145


CAP II Paper 7 Income Tax and VAT

Deductions
Cost of Trading Stock 15 640,000 Refer W.N. 1
Depreciation 19 150,000 Refer W. N. 2
Repair and Improvement Cost 16 70,000 Refer W. N. 2
Cash payment exceeding Rs. 50,000 as truck freight is not
not-deductible merely due to the reason of payment in cash,
Selling and Distribution Expense 13 150,000
as the turnover (15 lakhs plus 1.6 lakhs) is less than Rs. 20
Lakhs
Insurance Premium 13 40,000 60,000 divided by 18 multiplied by 12
Other Expenses 13 40,000
Total Deductions 1,090,000
Assessable Income from Business 410,000

Statement of Taxable Income and Balance Taxable Income


Assessable Income from Business 410,000
Cash basis and assumed that related to 20X-62/X-63 is
Assessable Income from Employment 160,000
received in cash during 20X-63/X-64
Assessable Income from Investment -
Total Assessable Income 570,000
Less: Reductions-
under Sec. 12
under Sec. 12Kha
Lower of:
a. 1/3rd of Total Assessable Income (Rs. 190,000)
under Sec. 63 190,000
b. Actual Contribution to ARF (Rs. 300,000)
c. Max. Rs. 300,000
Taxable Income 380,000
Less:

© The Institute of Chartered Accountants of Nepal 146


Compilation of Suggested Answers Income Tax

Deductions availed under Sch. 1 to handicapped


200,000 50% of basic exemption limit
person
Balance Taxable Income 180,000

Statement of Tax Liability


1st Rs. 170,000 1% 1,700 SST on employment Income
Balance Rs. 10,000 0% - Falls within basic exemption limit of Rs. 400,000
Total Tax Liability 1,700

Working Note
1. Cost of Trading Stock
Value of Opening Stock -
Cost of Conversion
Raw Material Purchase 500,000
Labour Wages 150,000
Electricity 200,000
Production Salary 100,000
Insurance Premium 10,000 960,000
Per unit Cost of Production 20 Cost of conversion divided by units produced
Cost or Market Value, whichever is lower
Value of Closing Stock (320,000) Cost= 16,000*20= 320,000
Market Value= Given = 560,000
Cost of Trading Stock 640,000

2. Depreciation and Repair Cost


Block D
In absence of information, depreciation of Block B
Opening Depreciation Base 1,000,000
cannot be calculated
Add: Absorbed Additions -

© The Institute of Chartered Accountants of Nepal 147


CAP II Paper 7 Income Tax and VAT

Less: Disposal Proceeds -


Depreciable Basis 1,000,000
Depreciation @ 15% 150,000

7% of Depreciable Basis (A) 70,000


Actual Repair Cost (B) 80,000
Eligible Cost ("A" or "B", w/e is lower) 70,000

97. A proprietorship firm trading on beer earns taxable profit of Rs. 300,000 Calculate Tax Liability.
(June 2008, 1 Mark, CA Inter)
Answer:
Since the amount falls within basic exemption limit, there is no tax liability of the person.

98. X Ltd. of Delhi has got 70% shares in Yoyo Private Ltd. of Nepal. The Profit and Loss Account and Balance Sheet of the
company for the first year of operation ending on 31.3.2062 was as follows: (Dec 2006, 4 Marks, CA Inter)
Balance Sheet
Rs. in lakhs
Share Capital 270 Plant & Machinery 400
Secured Loan for Building 200 Department Building 350
Unsecured Loan 150 Current Assets 183
Current Liabilities 80 Profit & Loss Account 67
700 700
Profit & Loss Account
Rs. in lakhs
Cost of Sales 450 Sales 500
Administration Expenses 75 Interest Received 2
Interest on Loan 42 Loss for the year 65
567 567

© The Institute of Chartered Accountants of Nepal 148


Compilation of Suggested Answers Income Tax

Interest on Bank Loan is 10% and the interest on unsecured loan is15%. Compute the taxable income.
Answer
Note:
It is assumed that there is no loan from X Ltd. of Delhi

Statement of Assessable income from Business (Yoyo Pvt. Ltd.)


Particulars Sec. Ref. Amount Note
Inclusions
Sales 7 (2) 50,000,000
Interest Received 7 (2) 200,000
Total Inclusions 50,200,000
Deductions
Interest Expense 14 4,250,000 Assuming loan is used evenly throughout the year
Cost of Sales 15 45,000,000
Depreciation 19 7,750,000 Assuming assets are from beginning of year
Administrative Expenses 13 7,500,000
Total Deductions 64,500,000
Assessable Income from
Business (14,300,000) Since there is loss, there is no taxable income

Working Notes
Depreciation
Block A Block D
Opening Depreciation Base 35,000,000 40,000,000
Add: Absorbed Additions - -
Less: Disposal Proceeds - -
Depreciable Basis 35,000,000 40,000,000

© The Institute of Chartered Accountants of Nepal 149


CAP II Paper 7 Income Tax and VAT

Depreciation Rate 5% 15%


Depreciation 1,750,000 6,000,000

99. Shri Hari Upadhyaya is a Chartered Accountant practicing at Kathmandu. The following is the analysis of his receipts and
payments account for the year 20X-61/X-62. (Dec 2005, 6 Marks, CA Inter)
Receipts Rs. Payments Rs.
Balance b/d 9,500 Salaries 1,64,000
Professional income 3,90,000 Rent 12,000
House rent for 8 months 40,800 Telephone expenses 5,500
Share of Income from 6,250 Professional expenses 3,000
firm
Interest on Bank Deposit 11,280 Motor car expenses 7,500
Insurance policy matured 77,750 Misc. Office expenses 1,500
with bonus
Purchase of car 80,000
Advance Income-tax 12,500
Personal expenses 42,400
Entertainment expenses 17,000
House property expenses :
Municipal Taxes 6,000
- Repairs 2,500
- Insurance 2,000
- Collection charges 600
Balance c/d 1,79,080
Total Rs. 5,35,580 Total Rs. 5,35,580

Compute Shri Hari Upadhyaya's total income after taking into account, the following:

© The Institute of Chartered Accountants of Nepal 150


Compilation of Suggested Answers Income Tax

a. Value of benefits received from clients during the course of profession is Rs.5,000/-;
b. Allowable rate of depreciation on motor car is 20%;
c. Municipal value of the house property is Rs.60,000/-. This house was self occupied for residence for 4 months during
the year.
Answer
Statement of Assessable Income from Business
Sec.
Particulars Amount Notes
Ref.
Inclusions
Professional Income 7 (2) 390,000
House rent income of investment nature of a natural person is not
House Rent Income -
subject to income tax. Refer Definition of Rent
Share Income from Firm 92 - Assumed to be dividend, which is final withholding
Assumed to be investment nature and bank is assumed to be resident
Interest on Bank Deposit 92 -
bank
Insurance Policy Matured 92 - Assumed to be received from resident insurance company
Benefit received from
7 (2) 50,000
client
Total Inclusions 440,000
Deductions
Salaries 13 164,000
Rent 13 12,000
Telephone expenses 13 5,500
Professional Expenses 13 3,000
Motor Car expenses 13 7,500
Miscellaneous office expense 13 1,500
Depreciation 19 16,000 Assumed to be put to use until Poush
Advance Income Tax 21 - Not deductible
Personal Expense 21 - Not deductible

© The Institute of Chartered Accountants of Nepal 151


CAP II Paper 7 Income Tax and VAT

House Property Expense 13 - Not deductible, as the income is also not taxable
Total Deductions 209,500
Assessable Income from Business 230,500

100. M/s Pashupatinath Industries, a partnership firm, submits the following profits and loss account to you for computation
of taxable business income for the 23income year 20X-60/X-61 :(June 2005, 15 Marks, CA Inter)
Profit and Loss Account for the year ending 31/3/20X-61

Amount Amount
(Rs.) (Rs.)
Cost of Raw materials used 1,500,000 Sales 2,600,000
Production Expenses 500,000 Misc. Income 50,000
Gross Profit 650,000
2,650,000 2,650,000
To Salaries 348,000 By Gross Profit 650,000
To Rent 24,000 By Sundry 7,000
To Printing & Stationery 4,700 creditors w/back 13,500
To Telephone 2,800 By Dividend from
To Conveyance 19,500 Nabil bank
To Traveling 16,000
To Interest 68,000
To Depreciation 20,000
To Legal fees 12,000
To Auditor’s fees 12,000
To Provident Fund contribution 18,000
To Net Profit 125,500
Total Rs. 670,500 670,500

23
“assessment” replaced by “Income”

© The Institute of Chartered Accountants of Nepal 152


Compilation of Suggested Answers Income Tax

Additional Information:
a. Salaries include Rs.120, 000 paid to working partner X and Rs.80,000 to wording partner Y.
b. Rent of Rs. 24, 000 is paid to the premises belonging to partner Y who has let it out to the firm.
c. Interest paid includes Rs.60, 000, being interest paid on loan given by partner Y at the rate of 15% simple interest.
d. Out of Provident Fund Contribution debited to P & L. Account Rs.7, 000 was outstanding unpaid under the Income
Tax Act.
e. The firm normally purchases goods issuing crossed cheques and Banks drafts only except in the case of one bill for
Rs.75, 000 for which payment has been made by cash.
f. Depreciation debited in the accounts is as per the Income Tax Act.
g. Legal Fees include Rs.10, 000 fees paid in respect of appeal against the income tax assessment for the earlier year.
Answer
Statement of Assessable Income from Business
Sec.
Particulars Amount Notes
Ref.
Inclusions
Sales 7 (2) 2,600,000
Miscellaneous Income 7 (2) 50,000
Net Gain on Disposal of Liability 7 (2) 7,000 Sundry creditors written back
Dividend from Nabil Bank 92 - Final withholding assuming Nabil Bank is Resident company

Total Inclusions 2,657,000


Deductions
There is no provision in Income Tax Act that makes interest paid to
owner against loan used for business purpose not-deductible. IRD
Interest Expenses 14 68,000
may apply GAAR in case when it can prove any tax avoidance
practices. As such, interest to partner is deductible
Cost of Trading Stock 15 2,000,000 Sum of Cost of raw materials used and production expenses
Depreciation 19 20,000 Refer additional information (f)

© The Institute of Chartered Accountants of Nepal 153


CAP II Paper 7 Income Tax and VAT

Salaries paid to working partner is deductible. No provision in the


Salaries 13 348,000
Income Tax Act makes it not-deductible
There is no provision in Income Tax Act that makes rent paid to
owner against not-deductible. IRD may apply GAAR in case when it
Rent 13 24,000
can prove any tax avoidance practices. As such, rent to partner is
deductible
Printing and Stationery 13 4,700
Telephone 13 2,800
Conveyance 13 19,500
Travelling 13 16,000
Appeal against authority's decision is legal right, as such, expense
Legal Fees 13 12,000
cannot be made not deductible.
Audit Fee 13 12,000
There is no provision in Income Tax Law that makes PF
PF Contribution 13 18,000
contribution deductible only when it is paid.

Total Deductions 2,545,000


Assessable Income from Business 112,000

101. Mansubha Ltd is a business house dealing in readymade garments. During the year 20X3/X4, it had the following
transactions. Find its income from exports and the taxable income and tax liability: (Dec 2004, 10 Marks, CA Inter)

Total Sales 2,000 Lakhs


Export Sales 1,500 Lakhs
Cost of purchase of garments 2,000 Lakhs
Export expenses 100 Lakhs
Administrative expenses 75 Lakhs
Depreciation 9 Lakhs
Closing stock of garments 200 Lakhs
Answer

© The Institute of Chartered Accountants of Nepal 154


Compilation of Suggested Answers Income Tax

Statement of Assessable Income


Sec. Domestic
Particulars Export Sales Notes
Ref Sales
Inclusions
Export Sales 7 (2) 150,000,000
Domestic Sales 7 (2) 50,000,000
Total Inclusions 150,000,000 50,000,000
Deductions
Equals to cost of purchase minus closing stock
Cost of Trading Stock 15 135,000,000 45,000,000
Apportioned in Sales Ratio
Export Expenses 13 10,000,000 Apportionment not required as per matching principle
Administrative Expenses 13 5,625,000 1,875,000 Apportioned in Sales ratio
Assuming that the given amount is as per Income Tax
Depreciation 19 675,000 225,000
Act, Apportioned in Sales ratio
Total Deductions 151,300,000 47,100,000
Assessable Income from
(1,300,000) 2,900,000
Business
Tax Rate 20% 25% Normal business
Tax Liability - 725,000

Where a person conducts multiple transactions, where exemptions under Sec. 11 are applicable; each transaction is treated
as being conducted by separate person. As such, there cannot be offsetting of loss between export income and domestic sales
income.

102. The Profit and Loss account of PQR & Co., a manufacturing company, shows a Net Profit of Rs. 50,00,000 (before tax).
Calculate the Taxable income of the company for Financial Year 20X-59/X-60, after considering the following facts.
(June 2004, 8 Marks, CA Inter)
a. Fixed Assets schedule of the Company shows the following balances:
Pool WDV as on 20X-59.3.3

© The Institute of Chartered Accountants of Nepal 155


CAP II Paper 7 Income Tax and VAT

A 20,00,000
B 10,00,000
C 12,00,000
D 50,00,000
The Company purchased Furniture of Rs. 50,000 on Falgun 15, 20X-59. It also purchased a vehicle or Rs. 8,00,000 on
Baisakh 1, 20X-60.
b. The Company holds 20% share in ABC & Co. Ltd. It received Rs. 1,00,000 (net of tax) as dividend during the year.
c. The Company has spent Rs. 2,80,000 on the repairs of the Machinery during the year and charged the whole amount to
the Profit and Loss Account.
d. The Company has paid a premium of Rs. 2,50,000 for insurance of its Machinery and charged the whole amount to the
Profit & Loss account. The period covered by the premium is from 20X-60/1/1 to 20X-60/12/30 (one year).
e. The Company has charged Rs. 15,00,000 as depreciation during the year in its books.
Answer

Note: Nepal Taxation system does not allow add-back methodology on tax computation. However, there is no alternative in
this question, therefore, add back method is used.

Assessable Income from Business


Sec.
Particulars Amount Notes
Ref
Net Profit before Tax 5,000,000
Add:
Depreciation Charged in above Profit 1,500,000
Repairs on Machinery charged in above
280,000
profit
Insurance expense charged in above
250,000
profit
Less:
Allowable Depreciation under ITA (1,868,970)
Allowable Repair Cost under ITA (280,000)

© The Institute of Chartered Accountants of Nepal 156


Compilation of Suggested Answers Income Tax

Accrued Insurance Expense 62,500 250,000 divided by 12 multiplied by 3


Dividend from Resident company 92 - Final Withholding

There are no reductions u/s 12, 12Ka and 12Kha. So, this
Assessable Income from Business 4,943,530
amount is taxable income

Working Notes
1. Allowable Depreciation
Pool A Pool B Pool C Pool D Notes/Total
Assuming
Opening Depreciation Base 2,000,000 1,000,000 1,200,000 5,000,000
tax bases
Add: Absorbed Additions
Furniture (two-third of 50,000) 33,333
Vehicle (one-third of Rs. 8,00,000) 266,667
Less: Disposal Proceeds - - - -
Depreciable Basis 2,000,000 1,033,333 1,466,667 5,000,000
Depreciation Rate 6.67% 33.33% 26.67% 20.00%
Depreciation Amount 133,400 344,410 391,160 1,000,000 1,868,970
Manufacturing company is a special industry and hence, depreciation must be computed at accelerated
rate.

2. Allowable Repair
Pool A Pool B Pool C Pool D
7% of Depreciable Basis 140,000 72,333 102,667 350,000
Actual Cost - - - 280,000
Eligible (Minimum of above two) - - - 280,000 280,000

103. Mr. Mahesh is operating a business relating to computer parts and the sales during the year is Rs. 20,00,000. Except the
computer sales, other incomes relating to business are:

© The Institute of Chartered Accountants of Nepal 157


CAP II Paper 7 Income Tax and VAT

a. Net profit by disposal of business assets Rs. 25,000.


b. Prizes received by LG Computer Company as best seller Rs. 50,000.
c. Amounts received instead of only trading of LG Computer parts Rs. 40,000.
d. Amounts received by subletting of some space of room hired for business purpose Rs. 25,000.
e. The total expenditure relating to business including cost of business stock is Rs. 18,30,000.

Mrs. Sita, wife of Mahesh, is working in a bank and her taxable employment income is Rs. 1,10,000, before deduction of
donation. They have selected the option of dealing a couple as a single individual. They provided the gift to a tax-exempt
organization worth Rs. 15,000. Compute the taxable income of Mahesh and his wife jointly for the income year 20X-59/X-
60. (June 2003, 6 Marks, CA Inter)
Answer
Note:
Sec. 50 of Income Tax Act is a beneficial provision and opted by married individuals only when one of the spouses is nonearning
member. When both the spouses earn money subject to income tax, no rational taxpayer elects couple in order to pay more tax.
Therefore, whatever the question is suggesting, this question is dealt in the premise that there is individual assessment by either
of the spouses based on practical assumption.

Students are advised to take bold decisions while solving this type of problems in examination as well.

Assessable Income from Business (of Mr. Mahesh)


Sec.
Particular Amount Note
Ref.
Inclusions
Sales 7 (2) 2,000,000
Net Gain on Disposal of Business Asset 7 (2) 25,000
Prize received in respect of business 7 (2) 50,000
Amount received in respect of business restrictions 7 (2) 40,000
Subletting income 7 (2) 25,000
Total Inclusions 2,140,000
Less: Total Deductions (1,830,000)

© The Institute of Chartered Accountants of Nepal 158


Compilation of Suggested Answers Income Tax

There is no tax obligation, as it is within basic


Assessable Income from Business 310,000
exemption limit

Assessable income from employment of Mrs. Sita is Rs. 110,000 and she is required to pay tax @ 1% on the income

Donation amount is not deductible in either of the spouses' income calculation as the question does not clarify who pays the
amount.

© The Institute of Chartered Accountants of Nepal 159


CAP II Paper 7 Income Tax and VAT

Tax Liability of Specialized Business


104. Air Asia is airlines operation with head office outside Nepal 24is a non-resident person
having branch in Kathmandu, it has recorded Rs. 50 lakhs as income from sale of the
tickets related to the passengers departed from TIA, Kathmandu and it has Rs. 30 lakhs
branch expenses including salary expenses Rs. 20 lakhs. You are required to make tax
assessment for the Income Year 20X-72/X-73 (Dec 2016, 2.5 Marks)
Answer:
As per Sec. 70 of Income Tax Act, 2058; The taxable income of nonresident air transport
operator, water transport operator or chartered service provide for a particular Income Year
shall be the amount received for following activities, except as a result of transshipment:
• Amount received as a result of transport of passengers who embark from Nepal
• Amount received for the transportation of mail, livestock or other tangible assets that
embark from Nepal
The expenses in relation to the generation of income from air transport operation, water
transport operation or chartered service from Nepal with destination outside Nepal is not
deductible. As such, tax is applicable in Rs. 50 Lakhs @ 5%. The expense is not deductible.
The tax liability is Rs. 25 Lakhs.

105. Explain on the taxability and the implication thereon, of the following transactions as
per the Income Tax Act, 2058. (June 2013, 6+4=10 )
a. A Den airline registered in Denmark, having contact office in Nepal and is operating
its airlines business. During Income Year 20X-68/X-69, it has sold the tickets in Nepal
as follows:
i. Sale of tickets from the passengers departing from Nepal - Rs. 50 crores.
ii. Sale of tickets in Nepal, for the passengers departing from country other
than Nepal – Rs. 10 crores.

Answer
As per Sec. 70 of Income Tax Act, 2058; The taxable income of nonresident air
transport operator, water transport operator or chartered service provide for a
particular Income Year shall be the amount received for following activities,
except as a result of transshipment:

a) Amount received as a result of transport of passengers who embark from


Nepal
b) Amount received for the transportation of mail, livestock or other tangible
assets that embark from Nepal

24
Added when compiling this Suggested Answers

© The Institute of Chartered Accountants of Nepal 160


Compilation of Suggested Answers Income Tax

The expenses in relation to the generation of income from air transport operation,
water transport operation or chartered service from Nepal with destination
outside Nepal is not deductible.

i. The amount is taxable, and the tax rate is 5%.


ii. Principally, the amount is not taxable. However, proviso to Sec. 2 (7) of
Schedule 1 of the Act requires payment of tax @ 2%.

b. Singtel Ltd. is a company registered in Singapore. The company, with its objective
to transmit information and storing data, has a communication hub in Nepal
(without any office in Nepal). Through such system, the companies in Europe and
America are storing data and transmitting information. Singtel has received USD 1
million for such services.
Answer
The taxable income of nonresident telecommunication operator (who conducts a
business of transmitting messages by cable, radio, optical fibre, or satellite
communication) shall include any amount received by a person in respect of the
transmission of messages by apparatus established in Nepal, whether or not such
messages originate in Nepal.
Therefore, the amount of USD 1 Million is taxable in Nepal, as Sec. 70 is applicable to
Singtel.

106. Rolls Airways is conducting the airplane services between Kathmandu and Saudi
Arabia. Rolls Airways has a branch office at Kathmandu for the purpose of operation
of its services. The income of Rolls Airways for the month of Shrawan, 20X-67 while
conducting the airplane services and other operations are as below:
SN Particulars Amount (Rs.)
1 Amount received from the passengers departed from 10,000,000
Kathmandu
2 Amount received from the transportation of the goods having 5,000,000
first point entry at Kathmandu
3 Income from the travels from Bhutan and Lhasa having 5,000,000
transit point at Kathmandu
4 Amount received from the rentals of the goods carried from 5,000,000
Kathmandu which were transported from Bhutan
5 Amount received from the services of restaurant operated for 2,500,000
the facility of passengers at Kathmandu Airport

© The Institute of Chartered Accountants of Nepal 161


CAP II Paper 7 Income Tax and VAT

6 Amount received from the services of Ground Handling for 2,500,000


another airlines’ operation
Total Income 30,000,000

The followings are the details of expenditures of Rolls Airways:


SN Particulars Amount (Rs)
7 Expenditures from Air tickets 5,000,000
8 Expenditures against the accommodation and fooding for 5,000,000
crew members
9 Expenditures against ground handling services 3,000,000
10 Expenditure for the operation of restaurant 2,000,000
11 Expenditure for the transportation of passengers from 5,000,000
Bhutan, Lhasa
12 Expenditure for the re-transportation of the goods 5,000,000
transported from Bhutan
Total Expenditure 25,000,000

Note: Rolls Airways has managed the ground handling against 100 flights of its own
and 100 flights of other airlines during Fiscal Year 20X-67/X-68.
Explain about the inclusion of above incomes and allowances of expenditures of a non-
resident person, for the purpose of section 70 of Income Tax Act, 2058 stating the
relevant provisions. (June 2012, 10 Marks)
Answer
Explanation on Taxability of the given amounts
Particulars Amount Taxability
The amount is taxable under Sec. 70, as it
Amount received from the is received by a non-resident air transport
1 passengers departed from 10,000,000 operator in respect of passengers
Kathmandu embarking from Nepal, not being
transshipment
Amount received from the The amount is taxable under Sec. 70, as it
transportation of the goods is received by a non-resident air transport
2 5,000,000
having first point entry at operator in respect of goods embarking
Kathmandu from Nepal, not being transshipment
Income from the travels from The amount is not taxable, as the amount
3 Bhutan and Lhasa having 5,000,000 does not have source in Nepal; since it is
transit point at Kathmandu for the passengers in transit

© The Institute of Chartered Accountants of Nepal 162


Compilation of Suggested Answers Income Tax

Amount received from the


The amount is not taxable, as the amount
rentals of the goods carried
4 5,000,000 does not have source in Nepal; since it is
from Kathmandu which were
for the goods in transit
transported from Bhutan
Amount received from the
This amount is not within purview of Sec.
services of restaurant
70; but within purview of Sec. 7; i.e.
5 operated for the facility of 2,500,000
business income of Permanent
passengers at Kathmandu
Establishment
Airport
This amount is not within purview of Sec.
Amount received from the
70; but within purview of Sec. 7; i.e.
6 services of Ground Handling 2,500,000
business income of Permanent
for other airlines’ operation
Establishment
7 Expenditures from Air tickets 5,000,000 Not deductible under Sec. 70
Expenditures against the
8 accommodation and fooding 5,000,000 Not deductible under Sec. 70
for crew members
Deductible while calculating assessable
Expenditures against ground income of Permanent Establishment.
9 3,000,000
handling services However, not deductible to the extent it is
incurred for flight operation
Deductible while calculating assessable
Expenditure for the income of Permanent Establishment.
10
operation of restaurant 2,000,000 However, not deductible to the extent it is
incurred for flight operation
Expenditure for the
Not deductible under Matching concept,
11 transportation of passengers
5,000,000 as the income is also not taxable in Nepal
from Bhutan, Lhasa
Expenditure for the re-
Not deductible under Matching concept,
12 transportation of the goods
5,000,000 as the income is also not taxable in Nepal
transported from Bhutan

Calculation of Taxable Income under Sec. 70


Amount received from the passengers departed from
10,000,000
Kathmandu
Amount received from the transportation of the goods
5,000,000
having first point entry at Kathmandu
Total Taxable Income 15,000,000
Tax Liability @ 5% 750,000

Calculation of Assessable Income for Permanent Establishment on Other business Income


Particulars Amount
Inclusions

© The Institute of Chartered Accountants of Nepal 163


CAP II Paper 7 Income Tax and VAT

Amount received from the services of restaurant


operated for the facility of passengers at Kathmandu 2,500,000
Airport
Amount received from the services of Ground Handling
2,500,000
for other airlines’ operation
Total Inclusions 5,000,000
Deductions
Cost for 200 flights.
However, 100 flights
Expenditures against ground handling services 1,500,000 are own, therefore
50% expense is not
deductible u/s 70
Expenditure for the operation of restaurant 2,000,000
Total Deductions 3,500,000
Assessable Income from Business 1,500,000

Deduction of General Expenses and Concept of Non-deductibility


107. Smart Co. Ltd. had contributed Rs. 350,000 to a non-approved Provident Fund on
behalf of the employees and has claimed it as expenditure. The Tax Office proposes to
disallow the claim and add to the income. The company argues that since it has already
spent the money, it is a business expenditure spent for the operation of normal business
and it should be fully allowed under Section 13. What should be the correct action
under the provisions of Income Tax Act, 2058? (Dec 2009, 5 Marks)
Answer
The language used in the question is confusing. Therefore, we have two assumptions while
interpreting the question.
Assumption 1: The amount contributed to unapproved retirement fund is additional
contribution by the employer
In this case, the amount is added to the income of the employee while calculating withholding
tax u/s 87 of the Act. The amount so contributed is normal business expense and is deductible
u/s 13 of the Act. The company’s argument is valid in this regard.

Assumption 2: The amount is deducted from the salary paid by the employer and is
contributed as per the request of the employee
In this case, the amount does not qualify as expense of the company, and tax officer’s
argument is valid.

108. Mr. Anup pays an advance amounting to Rs. 10 lakhs in cash to Mr. Binod, who
required it urgently to clear his goods from customs. The same was returned by Mr.
Binod in cash after six months. Discuss the tax consequence of the above transaction
quoting the relevant provisions of Income Tax Act, 2058. (Dec 2009, 5 Marks)

© The Institute of Chartered Accountants of Nepal 164


Compilation of Suggested Answers Income Tax

Answer
Let’s examine the transactions:
a. Anup has made cash payment to Binod. It was required for customs clearance. While
making advances to Binod, the outgoings related to asset of Mr. Anup Rs. 10 lakhs as
outgoings, though made in cash, is not not-deductible u/s 21 (2) of the Act, since it is
“Cash payment under unavoidable circumstances”.
b. While discharging the liability by Mr. Binod to Mr. Anup, the conditions mentioned in Sec.
21 (2) is not satisfied. Therefore, if Mr. Binod’s turnover is more than Rs. 20 Lakhs for
the year, the outgoings on settlement of liability is Nil (cash payment above Rs. 50,000 at
a time) and as such, there is gain on disposal of liability.

109. How will you deal with the followings for the purpose of Income Tax Act, 2058?
(June 2007, 2×1=2, CA Inter)
a. A Company distributes goods worth Rs. 50,000 as free sample.
Answer
The amount is deductible as business expense for distributing company.

b. Entertainment expenses of Rs. 40,000 incurred during the Previous Year


Answer
The amount is not deductible as it does not meet “incurred during the year” criteria
under Sec. 13 of the Act.

110. Advise whether the following expenditure are allowable: (Dec 2005, 1×3=3, CA Inter)
a. A. Co. Ltd. purchased cane from an agriculturist and paid Rs. 1,20,000 in cash
Answer:
Any amount paid in cash to producer of primary agro products even when the produces
are primarily processed is not not-deductible merely because of cash payment.
Therefore, subject to fulfillment of other conditions of Income Tax Act, the amount is
deductible.

b. B. Co. Ltd. had a fire accident and in order to douse the fire and recover whatever
assets could be recovered and employed the people from the nearby villages who were
called to help and for the purpose paid cash Rs. 1,50,000 to Mr. A for distribution to
the various villagers who helped the Company by payment not exceeding Rs. 5000 to
each.
Answer:
Any amount paid in cash under unavoidable circumstances is not not-deductible merely
because of cash payment. Further, cash payment to a person at a time exceeding Rs.
50,000 is not deductible. In this case, there is no such case. Therefore, and subject to
fulfillment of other conditions of Income Tax Act, the amount is deductible.

© The Institute of Chartered Accountants of Nepal 165


CAP II Paper 7 Income Tax and VAT

c. C. Company Ltd. incurred an expenditure of Rs.10 lacs on repairs to Machinery


whose Depreciable value was Rs.200 lacs. But Rs.10 Lacs included purchase of a new
machinery costing Rs. 50,000.
Answer
The repair expense is Rs. 950,000 as cost of new machinery has to be capitalized in Pool
D, at the time which is earlier of purchase date or use date.
As per Sec. 16, repair cost for a pool to the extent 7% of depreciable basis (i.e. 14 lakhs-
7% of Rs. 200 Lakhs) is deductible during the year. Therefore, repair cost is deductible.

111. How will you treat the in the accounts and for the purpose of income tax?
(Dec 2005, 3 Marks, CA Inter)
a. Advertising and Promotional expenses incurred Rs.15 lacs whose benefit is expected
to be available for 5 years
Answer
It is charged as expense during the year when it is incurred.

b. A Company has received a notice for payment of additional excise duty, and penalty
and interest totaling Rs.5 crores. The Company intends to appeal to the authorities.
Answer
It is deposit and an asset to the company. There is no effect of this payment for tax
purpose. When finally settled, the amount is treated as penalty and hence, not deductible
u/s 21 of the Act.

c. A Company receives Rs.50 lacs from Insurance Company for damages to its stock of
raw material and finished goods.
Answer
The amount is included in income during the year of receipt of compensation u/s 31 of
the Act.

112. What are the expenses considered as not allowable as deductible business expenses
under section 21 of the Income Tax Act, 2058? (June 2004, 7 Marks, CA Inter)
Answer
The following expenses are not deductible under Sec. 21 of the Act:
a. Expense of Domestic and Personal Nature
b. Penalty paid against the breaches of law of any country
c. Income Tax paid under Income Tax Act, 2058
d. Expenses incurred to the extent to earn exempt amounts
e. Expense incurred to the extent to collect or earn final withholding payments
f. Cash payment exceeding Rs. 50,000 at a time; when certain conditions are satisfied

© The Institute of Chartered Accountants of Nepal 166


Compilation of Suggested Answers Income Tax

g. Distribution of Profit
h. Payment to labor or employee, who has not obtained Permanent Account Number
i. Payment of any expenses without invoice bearing Permanent Account Number
j. Expense of Capital Nature
Expense of Domestic and Personal Nature and Expense of Capital nature bear meaning
under Sec. 21, which must be written in examination by students considering the marks
allocated.
Further, the matters as to what is considered cash payment, condition when cash payment
makes an expenditure not deductible and exceptions when cash payment is allowed without
any limit should also be written for Examination purpose, considering the marks allocated.

113. What are considered as "Personal Expenses" not allowable as business expenses under
the Income Tax Act and what are the exceptions to the same?
(June 2011, 5 Marks; June 2004, 5 Marks, CA Inter)
Answer
For the purpose of Sec. 21, Expense of Personal Nature means the following:
a. Interest incurred on loan utilized for personal purpose
b. Private expenses of a natural person that includes:
i. The cost incurred in maintaining the natural person, including in providing shelter
as well as meals, refreshment, entertainment, or other leisure activities
ii. The cost incurred with respect to the natural person’s commuting, other than
commuting in the course of conducting a business or investment that does not involve
commuting between the individual's home and a place at which the business or
investment is conducted
iii. The cost incurred in acquiring clothing for the individual, other than clothing that is
not suitable for wearing outside of work
iv. The cost incurred in educating the individual, other than education that is directly
relevant to a business or investment conducted by the person and that does not lead
to a degree or diploma

Exceptions to the above are as follows:


Where a person makes a payment to an individual, or the costs are incurred in making the
payment to the individual, including costs incurred in favour of a third person, except when
any of the following conditions is satisfied:
a. The payment is included in calculating the income of the individual,
b. The individual makes a return payment of an equal market value to the person as a
consideration for the payment, or
c. The amount of the costs is so small as to make it unreasonable to require or
administratively impracticable for the person to account for them (Rule 6 prescribes the

© The Institute of Chartered Accountants of Nepal 167


CAP II Paper 7 Income Tax and VAT

maximum ceiling for such amounts of Rs. 500 at a time. The amounts include expenses
for or reimbursement of tea expenses, stationery expenses, prizes, gifts, emergency
medical facility, or other such payments as specified by IRD).

Interest Expenses and Concept of ATI for Sec. 14 (2)


114. Fewa Pvt. Ltd. has been established by Nepalese promoters under foreign direct
investment. The eighty percent of the total capital has been held by non-resident
persons. In addition, The Pvt. Ltd. has borrowed the amount from the foreign investors.
For the year ended Ashad end, 20X-70, the profit and loss account of the company has
the following transactions. (June 2014, 5 Marks)
Amount in Rs.
Sales 5,000,000
Interest income 75,000
Total income 5,075,000
Expenses
Cost of sales 2,000,000
Administrative expenses 1,100,000
Interest expenses 1,500,000
Pollution control expense 300,000
Total Expenses 4,900,000
Net profit 175,000

a. Interest expense Rs. 1,400,000 out of total has been charged on the borrowed amount
from foreign investors. Answer whether all the above expenses can be claimed under
Income Tax Act, 2058?
b. If these expenses cannot be claimed during the year, what would be the implication?
Answer
Calculation of Eligible Interest Expense
Step 1: Calculation of ATI for the purpose of Sec. 14 (2)
Particulars Amount Note
Sales 5,000,000
cannot be included as per Sec.
14 (2), ATI must be calculated
Interest Income -
without including interest
expense and interest income
Income to be considered 5,000,000
Deductions
Cost of Sales 2,000,000 Eligible under Income Tax Act

© The Institute of Chartered Accountants of Nepal 168


Compilation of Suggested Answers Income Tax

Administrative Expense 1,100,000 Eligible under Income Tax Act


cannot be included as per Sec.
14 (2), ATI must be calculated
Interest Expense -
without including interest
expense and interest income
Actually incurred during the
Pollution Control Cost 300,000 year, as clarified by Income
Tax Directive
Total expense to consider 3,400,000
Adjusted Taxable Income 1,600,000

Step 2: Calculation of Maximum Limit


Maximum limit is sum of interest income and 50% of ATI calculated above
Interest Income 75,000
50% of ATI Computed above 800,000
Maximum Limit 875,000

Step 3: Calculation of Eligible Interest Expense u/s 14 (2)


Lower of actually paid to controlling person or maximum limit as per Step 2
Actually, paid to controlling person 1,400,000
Maximum Limit 875,000
Eligible u/s 14 (2) 875,000

Step 4: Total eligible interest expense


Eligible interest, where Sec. 14 (2) is not Total interest minus paid to
100,000
applicable controlling person
Eligible interest, where Sec. 14 (2) is applicable 875,000
Total Eligible Interest expense 975,000
Ineligible Portion 525,000

Treatment of Ineligible Portion


As per Sec. 14 (3), the amount can be carried forward for claim during next Income Year(s)
and is treated as being paid with regards to next Income Year(s). The carry forward continue
unless there is application of Sec. 57

115. Simple Innovations Pvt. Ltd., a manufacturing company received a tax assessment
order for fiscal year 20X-66/X-67 on 20X-69/8/20 from Inland Revenue Office. Total
tax assessed by tax authority is Rs. 75,00,000 which includes disputed tax of Rs.
54,43,500. Some of the issues stated in the assessment order are as follows: (Dec 2013)
a. Company has capitalized interest cost worth Rs. 19,34,569 (interest till X67.01.10, on
loan taken for machinery) to plant and machinery ready to /put to use in 20X-

© The Institute of Chartered Accountants of Nepal 169


CAP II Paper 7 Income Tax and VAT

67.01.10. As per Section 14(1), interest on loan which is utilized in the same fiscal
year or loan taken for purchase of assets which is utilized in same fiscal year is
allowed as deduction. Thus, interest on assets put to use in fiscal year 20X-66/X-67 is
not allowed to be capitalized and need to be removed from depreciation base.
b. Value of opening stock includes Repair & Maintenance Expense of Rs. 7,56,760 and
Closing Stock includes Repair Expense Rs. 6,54, 650. Same is not allowed and need
to be written back.
Management is not satisfied with the assessment order and seeks your consultancy.
a. Comment on the above issues in light of provisions of Income Tax Act, 2058.
(3+3=6)
Answer
My comments to management are as follows:
With regards to capitalization of interest expense
The argument of tax officer is correct. As per Sec. 14 (1), interest on loan which is utilized
in the same fiscal year or loan taken for purchase of assets which is utilized in same fiscal
year is allowed as deduction. When an asset is put to use for a moment during any Income
Year, the interest expense of whole year is deductible and cannot be capitalized.

With regards to deduction of repair cost as part of Cost of Trading Stock:


The question is not clear about the nature of repair and maintenance expense. So, we
have the following alternatives to this question:
• If it is related to depreciable asset or business asset, the argument of tax officer
is correct; as Sec. 16 deals with repair cost of depreciable asset and repair cost
of business asset is treated as outgoings of such asset
• If it is related to trading stock, the argument of tax officer is incorrect.

b. Company is planning to apply for administrative review. Answer the following in


these regards. (21=2)
i. Time limit to file application for administrative review.
Within 30 days of receipt of Assessment Order u/s 102. Further, 30 days can be
extended by IRD if an application with genuine reason is filed within 7 days of elapse
of first 30 days.

ii. Amount of deposit to be kept for administrative review.


The amount is 100% of undisputed tax amount and one-fourth of disputed tax amount.

© The Institute of Chartered Accountants of Nepal 170


Compilation of Suggested Answers Income Tax

116. From the following Income Statement of a resident entity controlled by exempt
Organization25, compute the taxable income for the financial year 20X-66/X-67.
(June 2010, 5 Marks)
Income
Sales Rs. 7,50,000
Interest Income Rs. 5,000
Total Income Rs. 7,55,000
Expenditure
Cost of sales Rs. 5,00,000
Administrative Expenses Rs. 1,20,000
Interest expenses to controlling (exempt) Rs. 75,000
entity
Depreciation expenses Rs. 20,000
Rs. 7,15,000
Net Profit Rs. 40,000
Answer
Statement of Assessable Income from Business
Particulars Note
Amount
Inclusions
Sales 750,000
Interest Income 5,000
Total Inclusion 755,000
Deduction
Cost of Sales 500,000
Administrative Expenses 120,000
Interest Expense 60,000 Refer W.N. 1
Depreciation 20,000
Total Deductions 700,000
It is taxable income, as there is no reduction
Assessable Income from Business 55,000
u/s 12, 12Ka or 12Kha

W.N. 1. Calculation of Eligible Interest Expense


Step 1: Calculation of ATI for the purpose of Sec. 14 (2)
Particulars Amount Note
Sales 750,000
cannot be included as per Sec. 14 (2), ATI
Interest Income - must be calculated without including interest
expense and interest income

25
The phrase “Exempt Controlled Resident Entity” is changed

© The Institute of Chartered Accountants of Nepal 171


CAP II Paper 7 Income Tax and VAT

Income to be considered 750,000


Deductions
Cost of Sales 500,000 Eligible under Income Tax Act
Administrative Expense 120,000 Eligible under Income Tax Act
cannot be included as per Sec. 14 (2), ATI
Interest Expense - must be calculated without including interest
expense and interest income
Actually incurred during the year, as clarified
Depreciation 20,000
by Income Tax Directive
Total expense to consider 640,000
Adjusted Taxable Income 110,000

Step 2: Calculation of Maximum Limit


Maximum limit is sum of interest income and 50% of ATI calculated above
Interest Income 5,000
50% of ATI Computed above 55,000
Maximum Limit 60,000

Step 3: Calculation of Eligible Interest Expense u/s 14 (2)


Lower of actually paid to controlling person or maximum limit as per Step 2
Actually paid to controlling person 75,000
Maximum Limit 60,000
Eligible u/s 14 (2) 60,000

Step 4: Total eligible interest expense


Eligible interest, where Sec. 14
- Total interest minus paid to controlling person
(2) is not applicable
Eligible interest, where Sec. 14
60,000
(2) is applicable
Total Eligible Interest expense 60,000

117. A company engaged in garment manufacturing, debited to its Profit and Loss Account
a sum of Rs. 90,000, being the interest on loan of Rs. 9,00,000 taken for financing its
expansion scheme. The plant and machinery purchased for the project with the loan
was not received during the year and those were still in transit at the end of the year.
Discuss the admissibility or otherwise of the interest on borrowing.
(June 2006, 5 Marks, CA Inter)
Answer
As per Sec. 14 (1), interest on loan which is utilized in the same fiscal year or loan taken for
purchase of assets which is utilized in same fiscal year is allowed as deduction. When an
asset is put to use for a moment during any Income Year, the interest expense of whole year

© The Institute of Chartered Accountants of Nepal 172


Compilation of Suggested Answers Income Tax

is deductible. However, if the asset is not put to use for whole year, the expense is not
deductible.
As such, the interest of Rs. 90,000 on plant and machinery not put to use during the year is
ineligible for deduction. It forms part of outgoings of plant and machinery.

Cost of Trading Stock


118. Explain in brief the procedure to be followed for the calculation of closing stock as per
Income Tax Act, 2058. Is it in line with the Nepal Accounting Standard?
(Dec 2008, 5 Marks, CA Inter)
Answer
Procedure followed for calculation of value of Closing Stock
Closing stock is valued at lower of cost or Market value.

Cost of Closing stock:


The cost of closing stock is determined using Specific Identification Method, however, when
this method cannot be used, it is determined using First-In-First-Out Cost Formula or
Weighted Average Cost formula.

Cost of Inventory for manufacturing industries


With regards to calculation of cost of inventory in production process, a person following
cash basis may use prime costing or absorption costing; however, the person following
accrual basis must use absorption costing.

In absorption and prime costing, both, the depreciation and repair and improvement cost of
depreciable asset cannot be allocated as cost of inventory.

Resemblance with Nepal Financial Reporting Standard


As per NAS 02- Inventories, inventories are valued at Cost or Net Realizable Value (NRV).
NRV differs from Market value, since NRV is market value less cost to sales.
Further, if NFRS has to be followed, the person can use only absorption costing and not
prime costing while determining the cost of production, and depreciation of production
equipment and other cost may be allocated to cost of inventory.
Apart from the above, valuation of closing stock confirms the valuation principle under
prevailing accounting standard.

119. How do you treat “a Company receives Rs. 10 lacs from Insurance Company for the
damage of its Stock in Transit” for tax purpose? (June 2007, 1 Mark, CA Inter)
Answer

© The Institute of Chartered Accountants of Nepal 173


CAP II Paper 7 Income Tax and VAT

The amount is included in income as per Sec. 31 during the year when the compensation is
received.

120. Explain the methods of stock valuation under the Income Tax Act, 2058.
(Dec 2003, 3 Marks, CA Inter)
Answer
Valuation of Opening Stock
The value of opening stock is same as the value of closing stock of previous year.

Valuation of Closing Stock


Closing stock is valued at lower of cost or Market value.
The cost of closing stock is determined using Specific Identification Method, however, when
this method cannot be used, it is determined using First-In-First-Out Cost Formula or
Weighted Average Cost formula.

With regards to calculation of cost of inventory in production process, a person following


cash basis may use prime costing or absorption costing; however, the person following
accrual basis must use absorption costing.

In absorption and prime costing, both, the depreciation and repair and improvement cost of
depreciable asset cannot be allocated as cost of inventory.

Repair and Improvement Expenses


121. The status of property, plant and equipment and repair and improvement Expenditure
XYZ & Co. during Income Year 20X-68/X-69 is as follows:
Block of Assets Repair and Depreciation basis (Rs.)
Improvement
Expenditure (Rs.)
Building 20,00,000 10,00,00,000
Computer 100,000 8,00,000
Automobile 16,00,000 1,60,00,000
Total 37,00,000 11,68,00,000
Answer the followings mentioning the relevant provisions of Income Tax Act, 2058:
a. Can repair and improvement expenditure of Rs. 3,700,000 be claimed under Income
Tax Act, 2058. If not, compute the amount of repair and improvement expenditure
that can be claimed by XYZ & Co. during Income Year 20X-68/X-69.
b. What is the implication of the amount of repair and improvement expenditure, if
any, which cannot be claimed during Income Year 20X-68/X-69?

© The Institute of Chartered Accountants of Nepal 174


Compilation of Suggested Answers Income Tax

(Dec 2012, 8 Marks)


Answer
Calculation of Eligible Repair and Improvement Cost
Block A Block B Block C Total
Depreciable Basis
Building 10,000,000
Computer 800,000
Automobile 16,000,000
Depreciable Basis 10,000,000 800,000 16,000,000
7% of Depreciable Basis (A) 700,000 56,000 1,120,000
Actual Repair and Improvement Cost (B) 2,000,000 100,000 1,600,000
Eligible (C=Lower of "A" or "B") 700,000 56,000 1,120,000 1,876,000
Excess Cost (B - C) 1,300,000 44,000 480,000 1,824,000

Conclusion
As calculated above, only Rs. 1,876,000 is eligible for deduction u/s 16 of the Act.

Treatment of Excess amount:


The excess cost of Block A, B and C of Rs. 1,300,000, 44,000 and 480,000 respectively are
treated as outgoings for respective block and added to the block to calculate opening
depreciation base for 20X-69/X-70.

122. An Airlines company operating in Nepal undertakes full repairs and maintenance of
aircrafts as per norms and standards prescribed by civil Aviation Authority of Nepal.
The total annual expenditure for such repairs & maintenance stood at Rs. 6.40 crore
which is equivalent to 8% of depreciation base at year end of that Assets Pool. Suggest
the maximum allowable repairs and maintenance as per the Act, quoting the relevant
section. (June 2011, CA Inter, 4 Marks)
Answer
As per proviso to Sec. 16 (2), the overhauling expense of aircraft as per the standard
prescribed by Civil Aviation Authority of Nepal can be claimed without invoking the limit of
7% of depreciable basis.
As such, the expense of Rs. 6.40 can be claimed irrespective of the limit.

Pollution Control Cost


123. The Profit and Loss Account of CBG & Co. for the financial year 20X-59/X-60 shows
the following details: (Dec 2003, 2 Marks, CA Inter)
Sales 500,000.00
Cost of Sales 300,000.00
Donation 20,000.00

© The Institute of Chartered Accountants of Nepal 175


CAP II Paper 7 Income Tax and VAT

Pollution Control Cost 300,000.00


Repairs & Maintenance 10,000.00

How much amount is deductible as Pollution Control Cost in financial year 20X-59/X-
60?
Answer
Calculation of Eligible Pollution Control Cost
Step 1: Calculation of ATI for the purpose of Sec. 17

Particulars Amount Note


Inclusions
Sales 500,000 Eligible under Income Tax Act

Total Inclusions 500,000


Deductions
Cost of Sales 300,000 Eligible under Income Tax Act
Not considerable, as per the
Donation - definition of ATI
Not considerable, as per Income
Pollution Control Cost - Tax Directive
Repairs and Maintenance 100,000 Eligible under Income Tax Act
Total Deductions 400,000
Adjusted Taxable Income 100,000

Step 2: Calculation of Maximum Limit


Maximum Limit 50,000 50% of ATI from all businesses

Step 3: Eligible Amount


Lower of:
Maximum Limit 50,000
Actually Incurred 300,000

Eligible 50,000

Depreciation
124. Horizon Pvt. Ltd. entered into a publication copyright contract on 1st Chaitra, 20X-72
with Florish Pvt. Ltd. to publish a book. Royalty is fixed Rs. 1,200,000 for 2 years and
10 months with maximum copies of 10,000 per year. Find the deductible expenses for
three years for tax return. (Dec 2018, 5 Marks)
Answer

© The Institute of Chartered Accountants of Nepal 176


Compilation of Suggested Answers Income Tax

Life (converted to nearest half year) = 3 years


Depreciation Method is Straight Line
Block E
Particulars 20X- 20X- 20X- 20X-
72/X-73 73/X-74 74/X-75 75/X-76
Opening Depreciation Base - 933,333 533,333 133,333
Absorbed Additions
800,000 -
(2/3rd of Rs. 12 Lakhs)
Disposal Proceeds - -
Depreciable Basis 800,000 933,333 533,333 133,333
Depreciation Rate
33.33% 33.33% 33.33% 33.33%
(100/3%)- SLM
Depreciation Amount 266,667 400,000 400,000 133,333
Unabsorbed Additions 400,000 - -
Opening Depreciation Base for next year 933,333 533,333 133,333 -

Note 1:
a. Depreciation for Year 2 and 3 is one-third of Rs. 12 Lakhs
b. Balance amount is depreciation for Year 4

125. As a tax consultant, you have been enquired by the clients on the depreciation facility
for the purpose of income tax on following cases: (June 2011, 2×2.5=5)

a. Assets, required for power generation for its industry, capitalized by the production-
oriented industries.
Answer
If a person purchases asset for the purpose of power generation, 50% of the cost of such
asset shall be deductible during the year of purchase.

b. If a person who wants to issue the tax invoices using fiscal printer and cash machine.
Answer
In case where fiscal printer or cash machine is purchase, the cost of such fiscal printer
and cash machine shall be deductible during the year of purchase.

126. XYZ Co. Ltd purchased an Accounting Software for Rs. 50,00,000 with the life span of
10 yrs and 5 months. It was installed on 1st Shrawan 20X-65. Also, the Company
purchased Inventory Management Software for Rs. 31,50,000 with the working life of
10 yrs and 6 months. This was put in use in the month of Baisakh 20X-66.
Compute the allowable depreciation for Tax purpose for Financial Year 20X-65/X-66.
(June 2011, 5 Marks)

© The Institute of Chartered Accountants of Nepal 177


CAP II Paper 7 Income Tax and VAT

Answer
Calculation of Depreciation for Income Year 20X-65/X-66
Inventory
Accounting
Management
Particulars Software Total Note
Software
Block E1 Block E2
Opening Depreciation Base - -
100% of Cost in case of
Absorbed Additions 5,000,000 1,050,000 "E1" and one-third of cost
in case of "E2"
Disposal Proceeds - -
Depreciable Basis 5,000,000 1,050,000
Depreciation Rate 9.52% 9.52%
Depreciation Amount 476,190 100,000 576,190

Working Note
1. Depreciation Rate for Block E1

The life is 10 years 5 months, and when converted into nearest half year (using mathematical
rounding off formula), it is 10.5 years. Depreciation Rate is 100/10.5 percent

2. Depreciation Rate for Block E2

The life is 10 years 6 months, and when converted into nearest half year, it is 10.5 years.
Depreciation Rate is 100/10.5 percent

127. M/s Everest Distilleries Ltd. (Liquor manufacturing unit) gives you the following
information pertaining to F.Y. 20X-66/X-67.
Land and Building 1,75,50,000
Machineries 2,70,80,000
Motorcycle 2,50,000
Truck 24,00,000
Computer and Laptop 8,70,000
Office Furniture 2,50,000
Air Conditioner 3,60,000

The following Assets were added during the year.


Addition date
Car Rs. 28,00,000 Ashad 25, 20X-67
Computer & Rs. 3,00,000 Falgun 12, 20X-67
Laptop

© The Institute of Chartered Accountants of Nepal 178


Compilation of Suggested Answers Income Tax

Computer & Rs.1,70,000 Baishak 03, 20X-67


Laptop
Air Conditioner Rs.1,20,000 Ashad 05, 20X-67

During the year Computer for Rs. 2,10,000 (book value Rs. 2,60,000) and Air
conditioner for Rs. 70,000 (book value Rs. 60,000) were also sold.

Note: The cost of acquisition of land is Rs. 35,50,000.


(June 2011, 15 Marks, CA Inter)
a. 26Calculate allowable pool-wise depreciation as per Income Tax Act, 2058.
Answer
Calculation of Depreciation Blocks
A B C D
Opening Depreciation Base 14,000,000 1,480,000 2,650,000 27,080,000
Add: Absorbed Additions - 296,667 933,333 -
Less: Disposal Proceeds - (280,000) - -
Depreciable Basis 14,000,000 1,496,667 3,583,333 27,080,000
Depreciation Rate 5% 25% 20% 15%
Depreciation 700,000 374,167 716,667 4,062,000

26
Question broken down into two sub-questions in order to make it easy to answer

© The Institute of Chartered Accountants of Nepal 179


CAP II Paper 7 Income Tax and VAT

Working Notes
1. Opening Depreciation Base
Blocks
A B C D
Land and Building 17,550,000
Less: Value of Land (3,550,000)
Machineries 27,080,000
Motorcycle 250,000
Truck 2,400,000
Computer and Laptop 870,000
Office Furniture 250,000
Air Conditioner 360,000
Opening Depreciation Base 14,000,000 1,480,000 2,650,000 27,080,000

2. Absorbed Additions
Blocks
A B C D
Until Poush - - - -
From Magh to Chaitra (2/3rd
of Cost)
Computer & Laptop 200,000
From Baisakh to Ashad (one-
third of cost)
Car 933,333
Computer & Laptop 56,667
Air Conditioner 40,000
Absorbed Additions - 296,667 933,333 -

b. Can this company claim 1/3rd additional depreciation being a manufacturing unit?
Answer
This company cannot claim additional depreciation, as it is not a special industry.

128. KMC Pvt. Ltd. had these depreciated balances in ‘Block B’ of assets as on Ashad 31,
20X-66:
Amount (Rs.)
Furniture and Fixtures 50,000
Computers 25,000
Other office Equipment 10,000
Total of Block B 85,000

© The Institute of Chartered Accountants of Nepal 180


Compilation of Suggested Answers Income Tax

The company on Shrawan 10, 20X-66 disposed of the computer for the following
considerations:
Condition I: Sales consideration received was Rs. 100,000.
Condition II: Sales consideration received was Rs. 30,000.

Calculate the taxable gain from the disposal of Computer and balances in the Block for
the purpose of depreciation for the year 20X-66/X-67 in each of the above conditions.
(June 2010, 5 Marks)
Answer
Calculation of Depreciation Block B
Condition I Condition II
Opening Depreciation Base 85,000 85,000
Add: Absorbed Additions - -
Less: Disposal Proceeds (100,000) 30,000
Depreciable Basis (15,000) 115,000
Depreciation Rate 25% 25%
Depreciation - 28,750

Note: When depreciable basis is negative, it is the case of deemed disposal. The balance
of Rs. 15,000 is treated as gain on disposal of depreciable asset and the Pool is
continued at Zero Value

129. On 20X-65 Magh 10, M/s Stunning Traders Ltd. has purchased a license for a
trademark at a cost of Rs. 6,00,000. The useful life of the license is 5 years 2 months.
The stunning traders Ltd. wants to claim the entire amount as an expense in the current
year. As a tax consultant give your opinion on the admissibility of this transaction by
quoting the relevant provision of the Act. (June 2010, 5 Marks, CA Inter)
Answer
M/s Stunning Traders Ltd. cannot claim the entire amount as expense during the year, as the
purchase of trademark is purchase of intangible asset, which is subject to depreciation u/s
19 and Schedule 2 of Income Tax Act, 2058.
The useful life of the asset is 5 years (when rounded off to nearest half year), which gives
depreciation rate of 20% on Straight Line Method.
The depreciation for the year is two-third of Rs. 120,000 [20% of 600,000 multiplied by two-
third as added in Magh], i.e. Rs. 80,000.
The company has to claim depreciation of Rs. 120,000 each for next four years and
remaining Rs. 40,000 will be claimed as depreciation in sixth income year from now.

© The Institute of Chartered Accountants of Nepal 181


CAP II Paper 7 Income Tax and VAT

130. If a person has constructed building on leased land, shall depreciation on that building
is allowable as per the Act? (Dec 2009, 2 marks, CA Inter)
Answer
Yes, as per Income Tax Directive, the depreciation shall be claimed over the lease period.

131. M/s Mechi Cigarette Industries Ltd. (producer of cigarettes) furnished the following
particulars to you pertaining to the income year 20X-64/X-65.
a. Opening balance (WDV) of depreciable assets as on 20X-64/4/1:
Particulars NRs.
Building 10,00,000
Car 6,00,000
Computers 70,000
Office Equipment 1,20,000
Plant & Machinery 8,00,000
Tools 30,000
Repair & Improvement cost capitalized ( block D) 10,000
b. The company has purchased a plant & machinery as on 20X-65/3/15 for
NRs.6,00,000. The company has also purchased a mini bus as on 20X-64/6/25 for
NRs.3,50,000.
c. During the year one old computer having written down value of NRs. 12,500 is sold
for NRs. 7,500. One printer having written down value of NRs. 3,500 became
unusable and the company recovered nothing from it.
d. Repair and improvement expenses of the company during the year are:
Particulars NRs.
Building 90,000
Office Equipment 10,000
Car 75,000
Plant & Machinery 80,000
e. During the year the company has incurred NRs. 5,00,000 on research and
development. However, only NRs. 3,75,000 is allowable deduction for research and
development cost for the income year 20X-64/X-65.
Required: (June 2009, 5 X 3= 15)
a. Classify the assets as per schedule 2 of Income Tax Act, 2058. 5
b. Amount of depreciation for the income year 20X64/X-65 as per schedule 2 of income
Tax Act 2058. 5
c. Amount of opening depreciable basis for the Income Year 20X-65/X-66.

© The Institute of Chartered Accountants of Nepal 182


Compilation of Suggested Answers Income Tax

Answer
Calculation of Depreciation Blocks
A B C D
Opening Depreciation Base (A) 1,000,000 190,000 600,000 840,000
Add: Absorbed Additions (B) - - 350,000 -
Less: Disposal Proceeds ( C) - (7,500) - -
Depreciable Basis (D = A+B-C) 1,000,000 182,500 950,000 840,000
Depreciation Rate (E) 5% 25% 20% 15%
Depreciation (F= D X E) 50,000 45,625 190,000 126,000
Unabsorbed Additions (G) - - - -
Excess Repair and Improvement Cost (H) 20,000 - 8,500 21,200
Excess PCC and R&D Cost (I) 125,000
Opening Depreciation Base for next Year (D-
F+G+H+I) 970,000 136,875 768,500 860,200

Working Notes
1. Opening Depreciation Base & Classification of Asset
Blocks
A B C D
Building 1,000,000
Car 600,000
Computers 70,000
Office Equipment 120,000
Plant & Machinery 800,000
Tools 30,000
Repair & Improvement cost capitalized ( block D) 10,000
Opening Depreciation Base 1,000,000 190,000 600,000 840,000

2. Absorbed Additions
Blocks
A B C D
Until Poush
Minibus 350,000
From Magh to Chaitra (2/3rd of Cost) - - - -
From Baisakh to Ashad (one-third of cost)
Machinery
(Already considered during Previous Year) -
Absorbed Additions - - 350,000 -

© The Institute of Chartered Accountants of Nepal 183


CAP II Paper 7 Income Tax and VAT

3. Calculation of Excess Repair and Improvement Cost


A B C D
7% of Depreciable Basis (A) 70,000 12,775 66,500 58,800
Actual Repair and Improvement Cost (B) 90,000 10,000 75,000 80,000
Eligible (C= Lower of "A" or "B") 70,000 10,000 66,500 58,800
Excess (B-C) 20,000 - 8,500 21,200

132. GTC Pvt. Ltd. had the following balances in block B of depreciable assets (after
charging depreciation for income year 20X-63/X-64)) as on the end of income year 20X-
63/X-64.
Office equipment Rs. 1,50,000
Furniture Rs. 95,000
Repair & improvement exp. to be capitalized in income year 20X- Rs. 25,000
64/X-65
The Company has purchased furniture for Rs. 45,000 on Falgun 15, 20X-64. The entire
block B has been destroyed by a fire occurred as on Chaitra 1, 20X-64 and the Company
has received Rs. 325,000 from an insurance company. Calculate the gain from the
disposal. (Dec 2008, 5 Marks, CA Inter)

Answer:
This is the case of disposal of a pool, as such, balancing charge or terminal depreciation
is calculated without considering absorption during addition of assets
Block B Note
Opening Depreciation Base 270,000 Sum of given figures in Table
Additions during the year 45,000
Disposal Proceeds (325,000)
Terminal Depreciation /(Balancing Charge) (10,000)
The amount of balancing charge sis treated as gain on disposal of depreciable asset.

133. State the businesses on which additional depreciation (one third of the basic rate) is
allowed as per Income Tax Act, 2058. (Dec 2008, 5 Marks, CA Inter)
Answer
One-third additional depreciation is allowed for Pool A, B, C and D (not E) for the following
entity or projects:
a. Cooperatives that are not dealing in tax exempt business under Sec. 11,
b. Projects of entity) involving construction of power houses, generation and transmission
of electricity and where these are carried out by an entity,
c. Public Infrastructure projects of entity that are ultimately be handed over to Government
of Nepal and where these are carried out by an entity,
d. Special industries operated by entity,
e. Entities operating trolley or tram bus,

© The Institute of Chartered Accountants of Nepal 184


Compilation of Suggested Answers Income Tax

f. Entities involved in building and operation of ropeway, cable car or overhead bridge,
and
g. Entities involved in building and operation of road, bridge, tunnel way, subway, railway
and airport.

134. Geeta Air (P) Ltd., an airlines company has purchased a mini air craft. What is the rate
of depreciation applicable for such crafts? (June 2008, 1 Mark, CA Inter)
Answer: 15%

135. How will you deal with the following for the purpose of Income Tax Act, 2058?
(June 2007, 2 Marks, CA Inter)
a. “Purchase of Patent right for Rs. 20 lacs for using the special process for the next 10
years”
Answer:
Depreciation is charged during next 20 years @ 10% in Straight Line Method.

b. Charges paid to technical engineer of Rs. 5 lacs for locating a proper place for
plantation of Machinery.
Answer:
The amount is treated as part of outgoings of plant and machinery.

136. Mr. Ram is the proprietor of HHH Manufacturing, a special Industry. He is an expert
in the field of his business but lacks the knowledge of the provisions of Income Tax Act,
2058 relating to depreciation. He asks you to calculate the amount allowable for
depreciation or the income year 20X-62/X-63 from his manufacturing industry.
Also, advise him if he is eligible for any additional depreciation as per the Income Tax
Act.
Following information is provided to you:
Opening WDV as on 1 Shrawan 20X-62:
Building 15,00,000
Machinery 3,50,000
Office Equipment 80,000
Transactions during the year:
a. Office equipment worth Rs. 50,000 purchased on Chaitra 23, 20X-62.
b. Shipping expenses of Rs. 10,000 paid for the above equipment.
c. Machine, the WDV of which was 53,000 was sold for Rs. 45,000 on Baishak 1, 20X-
63. (June 2006, 5 Marks, CA Inter)

© The Institute of Chartered Accountants of Nepal 185


CAP II Paper 7 Income Tax and VAT

Answer
Advice on additional Depreciation: Additional depreciation is provided to an entity and not
to a natural person. As such, private firm of Mr. Ram, HHH Manufacturing, is not entitled
to additional depreciation.

Calculation of Depreciation Blocks


A B C D
Opening Depreciation Base (A) 1,500,000 80,000 - 350,000
Add: Absorbed Additions (B) - 40,000 - -
Office Equipment (two-third of cost., added in
Chaitra)
Less: Disposal Proceeds ( C) - - - (45,000)
Depreciable Basis (D = A+B-C) 1,500,000 120,000 - 305,000
Depreciation Rate (E) 5% 25% 20% 15%
Depreciation (F= D X E) 75,000 30,000 - 45,750

137. M/s Gandaki Brewery Industries Ltd. furnished the following particulars to you
pertaining to the income year 20X-61/X-62.
a. Opening balance (WDV) of depreciable assets as on 20X-61-4-1.
Particulars Rs.
Building 20,00,000
Car 12,00,000
Computers 1,40,000
Office equipment 2,40,000
Plant & Machinery 16,00,000
Tools 60,000
Repair & improvement cost capitalized (block 20,000
D)
b. The Company has purchased a plant & machinery as on 20X-62/3/15 for Rs.
12,00,000. The Company has also purchased a mini bus as on 20X-61/6/25 for Rs.
7,00,000.
c. During the year one old computer having written down value of Rs. 25,000 is sold for
Rs. 15,000. One printer having written down value of Rs. 7,000 became unusable and
the Company recovered nothing from it.

© The Institute of Chartered Accountants of Nepal 186


Compilation of Suggested Answers Income Tax

d. Repair & improvement expenses of the Company during the year are :
Particulars Rs.
Building 1,80,000
Office equipment 20,000
Car 1,50,000
Plant & Machinery 1,60,000
e. During the year the Company has incurred Rs. 10,00,000 on research and
development. However, only Rs. 7,50,000 is allowable deduction for research and
development cost for the income year 20X-61/X-62.
Required: (Dec 2005, 15 Marks, CA Inter)
a. Classify the assets as per schedule 2 of Income Tax Act 2058.
b. Amount of depreciation for the income year 20X-61/X-62 as per schedule 2 of
Income Tax Act 2058.
c. Amount of depreciable basis for the Income Year 20X-62/X-63.
Answer
Calculation of Depreciation Blocks
A B C D
Opening Depreciation Base (A) 2,000,000 380,000 1,200,000 1,680,000
Add: Absorbed Additions (B) - - 700,000 300,000
Less: Disposal Proceeds ( C) - (15,000) - -
Depreciable Basis (D = A+B-C) 2,000,000 365,000 1,900,000 1,980,000
Depreciation Rate (E) 5% 25% 20% 15%
Depreciation (F= D X E) 100,000 91,250 380,000 297,000
Unabsorbed Additions (G) - - - -
Excess Repair and Improvement Cost (H) 40,000 - 17,000 21,400
Excess PCC and R&D Cost (I) 250,000
Opening Depreciation Base for next Year (D-
F+G+H+I) 1,940,000 273,750 1,537,000 1,954,400

Working Notes
1. Opening Depreciation Base & Classification of Asset
Blocks
A B C D
Building 2,000,000
Car 1,200,000
Computers 140,000
Office Equipment 240,000
Plant & Machinery 1,600,000

© The Institute of Chartered Accountants of Nepal 187


CAP II Paper 7 Income Tax and VAT

Tools 60,000
Repair & Improvement cost capitalized ( block
D) 20,000
Opening Depreciation Base 2,000,000 380,000 1,200,000 1,680,000

2. Absorbed Additions
Blocks
A B C D
Until Poush
Minibus 700,000
From Magh to Chaitra (2/3rd of Cost) - - - -
From Baisakh to Ashad (one-third of cost)
Machinery - 300,000
Absorbed Additions - - 700,000 300,000

3. Calculation of Excess Repair and Improvement Cost


A B C D
7% of Depreciable Basis (A) 140,000 25,550 133,000 138,600
Actual Repair and Improvement Cost (B) 180,000 20,000 150,000 160,000
Eligible (C= Lower of "A" or "B") 140,000 20,000 133,000 138,600
Excess (B-C) 40,000 - 17,000 21,400

138. How will you treat the following in books of accounts and for tax purpose?
(Dec 2005, 3 Marks, CA Inter)
a. Purchase of Patent rights for Rs.25 lacs for using the special process for the next 10
years.
Answer:
Depreciation is charged during next 20 years @ 10% in Straight Line Method.

b. Survey expenses incurred Rs.50 lacs for locating a proper place for putting up a
dam for generation of electricity.
Answer:
The expense is treated as outgoings of dam, which will be added to Pool “A” asset when
put to use.

c. A generator is purchased for Rs.25 lacs as standby to be used when the existing
generator has to be stopped for overhauling. But during the year, there was no
opportunity to put the newly purchased generator to use.
Answer

© The Institute of Chartered Accountants of Nepal 188


Compilation of Suggested Answers Income Tax

Generators are purchased to use during power stoppage and such assets are always in
passive use from the date when it is available for use. Therefore, depreciation can be
claimed on such asset.

139. The ABC Co. Pvt. Ltd. has assets of B category (computer group) worth Rs. 3,20,000
as on 32 Asadh, 20X-59. On Poush 15, the Company purchased a branded computer-
set costing Rs. 150,000 and the company also paid for repair of old computer Rs. 42,000.
During the year one set computer was disposed at Rs. 60,000. Compute the eligible
depreciation for this group of assets and depreciation base for the year 20X-59/X-60
according to provision of Income Tax Act 2058. (June 2003, 4 Marks, CA Inter)
Answer
Particulars Block B Note
Opening Depreciation Base (A) 320,000 Assumed to be after depreciation for the year 20X-58/59
Add: Absorbed Additions (B) 150,000 Purchased in Poush, 100% of cost is absorbed
Less: Disposal Proceeds ( C) (60,000)
Depreciable Basis (D = A+B-C) 410,000
Depreciation Rate (E) 25%
Depreciation (F= D X E) 102,500

© The Institute of Chartered Accountants of Nepal 189


CAP II Paper 7 Income Tax and VAT

Chapter 8: Income from Employment


140. Mr. Shyam is an Engineer working since 1 Jestha, 20X-75 in an Airlines Company. The contract period is for 4 years. The
Company has been charging expenses related to his service. Calculate the tax to be deducted by the Company on his
employment income for the month of Ashwin, 20X-75. (Dec 2018, 10 Marks)
Charging Monthly Expenses:
a) Monthly remuneration Rs. 80,000 (60% is basic salary and the remaining amount is allowance). Festival allowance Rs.
80,000 for the year.
b) The Company contributes 10% of his salary and he also contributes the same amount and is deposited into an approved
provident fund. In addition, the Company deposited Rs. 10,000 per month to Citizen Investment Trust as per his
request.
c) The Company makes a provision for gratuity @ 8.33% on his basic salary.
Facilities Provided:
a) He has been provided a car along with a driver by the Company for both official and personal use. Actual petrol
expenses Rs. 30,000 pertaining to the fiscal year for upto Ashwin end, 20X-75 was reimbursed by the Company and no
such facility shall be provided further.
b) He has been provided a chair for physically disabled people for his convenience; annual maintenance cost Rs. 6,000 is
also borne by the Company.
c) He has got a Samsung S 9 Mobile phone from the Company on 5 Bhadra, 20X-75. Purchase price of the phone was Rs.
90,000. Monthly total telephone expenses amounting to Rs. 1,500 is reimbursed to him. The phone is used equivalent to
33.33 percent for personal calls.
d) The Company is providing free lunch during office hours, the cost of which is Rs. 100 per meal to all employees. He has
been receiving Rs. 150 per meal due to his special work. Assume 252 working days in a year.
e) Annually, the Company pays a premium of Rs. 25,000 against his health insurance.
f) The Company has been providing bonus to its employees, who served at least 6 months in the company. Such bonus is
equivalent to 3 month's basic salary.
Other evidence and information submitted by him to the Company for tax purpose are:

© The Institute of Chartered Accountants of Nepal 190


Compilation of Suggested Answers Income Tax

a) He has paid premiums of Rs. 15,000 and Rs. 7,000 respectively against his and his wife's life insurance policies.
b) He and his wife declared that they are couple in the Income Year 20X-75/X-76 and his wife does not involve in any
income generating activities.
Answer
Method of Withholding of Tax u/s 87
The employer needs to calculate the annual tax liability of the employee based on the presumptive income every month. It deducts
tax already withheld from that annual tax liability, and the result is divided by remaining months for which the tax has to be
withheld to obtain the figure of withholding tax of the month.
Step 1: Calculation of Annual Tax obligation of the employee based on presumption
Assessable Income from Employment
Sec.
Particulars Amount Notes
Ref.
Monthly Remuneration
Basic Salary 8 (2) 576,000 Basic is 60% of Remuneration, and is annualized
Allowances 8 (2) 384,000 Allowances is 40% of Remuneration and is annualized
Employers' Contribution to 10% of his basic salary, annualized, using practical assumption
8 (2) 57,600
Retirement Fund as per prevailing Labour Law
Employment income is computed under cash basis of
Provision for Gratuity 22 - accounting. This amount is not included as the amount is not
received in cash by the employee
0.5% of Khai Paai aaeko Talab, which is interpreted as sum of
basic salary and grade
Vehicle facility 27 2,880
All other costs related to operation of vehicle is deemed to be
covered by the amount so quantified
Chair at Workplace 8 (3) - Serves the proper business purpose of the employer
Gift in relation to employment
8 (2) 90,000
(Samsung S9)

© The Institute of Chartered Accountants of Nepal 191


CAP II Paper 7 Income Tax and VAT

Reimbursement of Personal Expense 8 (2) 6,000 Annualized personal phone call expense reimbursement
Special Lunch 8 (2) 37,800 Lunch to all employees is exempt
Reimbursement of Personal Expense 8 (2) 25,000 Payment of his health insurance premium
Presumptive Bonus 8 (2) 144,000 3 months' basic salary
Assessable Income from Employment 1,323,280

Taxable Income & Balance Taxable Income


Assessable Income from Employment 1,323,280
Assessable Income from Business -
Assessable Income from Investment -
Total Assessable Income 1,323,280
Less:
Reduction u/s 12 -
Reduction u/s 12Kha -
Lower of following:
a. Actual Retirement Contribution across all ARF- Rs. 235200
(57,600*2+10,000*12)
Reduction u/s 63 235,200
b. One-third of Assessable Income- Rs. 441,093
(one third of Rs. 13,23,280)
c. Maximum Rs. 300,000
Taxable Income 1,088,080
Less:
Deduction availed to disabled
225,000 50% of Basic Exemption Limit, and couple assessment
person
Payment of Life Insurance Premium 22,000 Lower of Rs. 25,000 and Rs. 22,000
Payment of Health Insurance Assuming the insurance company is resident, and the limit of
Premium to 20,000 deduction is actual premium paid or Rs. 20,000 whichever is
Resident Insurance Company lower
Balance Taxable Income 821,080

© The Institute of Chartered Accountants of Nepal 192


Compilation of Suggested Answers Income Tax

Presumptive Annual tax Obligation


1st Rs. 450,000 1% 4,500
Next Rs. 100,000 10% 10,000
Next Rs. 200,000 20% 40,000
Balance Rs. 71,080 30% 21,324
Total Obligation for the Year 75,824

Step 2: Subtraction of taxes withheld during Shrawan and Is not possible due to absence of information, so, we assume the
Bhadra withholding of tax is even throughout the year
Step 3: Withholding Tax for the Month of Tax Obligation for the year divided by 12 (assuming even tax
Ashwin 6,319 withholding throughout the year)

141. Shreedhar Kumar is appointed on 20X-66/4/1 as Accounts Officer of Nepal Telecom Limited with salary structure of Rs.
25,000.00 - 500.00(6)-1,000.00(12). He has received interest free home loan Rs. 1,624,000.00 from the employer. Assume
market rate of interest 9% and make other practical assumptions as necessary. Calculate monthly withholding tax to be
deducted from his salary in Income Year 20X-74/X-75. (June 2018, 5 Marks)
Answer
Method of Withholding of Tax u/s 87
The employer needs to calculate the annual tax liability of the employee based on the presumptive income every month. It deducts
tax already withheld from that annual tax liability, and the result is divided by remaining months for which the tax has to be
withheld to obtain the figure of withholding tax of the month.
Step 1: Calculation of Annual Tax obligation of the employee based on presumption
Assessable Income from Employment
Particulars Sec. Ref. Amount Notes
Monthly Remuneration
Basic Salary 8 (2) 300,000 Given basic salary, annualized

© The Institute of Chartered Accountants of Nepal 193


CAP II Paper 7 Income Tax and VAT

Grade 8 (2) 60,000 Refer W.N. 1, annualized


Practical Assumption, as per Labor law, 10% of
Employers' Contribution to Retirement Fund 8 (2) 36,000
basic salary and grade
Rs. 16,24,000 multiplied by (Market Interest
Quantification of concessional loan 27 146,160 Rate- Charged Interest Rate), assuming loan is
utilized throughout the year
Assessable Income from Employment 542,160

Taxable Income & Balance Taxable Income


Assessable Income from Employment 542,160
Assessable Income from Business -
Assessable Income from Investment -
Total Assessable Income 542,160
Less:
Reduction u/s 12 -
Reduction u/s 12Kha -
Lower of following:
a. Actual Retirement Contribution across all
ARF- Rs. 72,000
Reduction u/s 63 180,720 (36,000*2)
b. One-third of Assessable Income- Rs. 180,720
(one third of Rs. 542,160)
c. Maximum Rs. 300,000
It is balance taxable income as there is no
Taxable Income 361,440
deduction under Schedule 1

Presumptive Annual tax Obligation


1st Rs. 361,440 1% 3,614
Total Obligation for the Year 3,614

© The Institute of Chartered Accountants of Nepal 194


Compilation of Suggested Answers Income Tax

Tax Obligation for the year divided by 12


Step 2: Withholding Tax for a month (assuming even tax withholding throughout the
301
year)

Working Note
Calculation of Monthly Grade amount for I.Y. 20X-74/X-75
Income Year No. of Grade Amount per grade Total Grade
20X-66/67 0 -
-
20X-67/68 1 500 500
20X-68/69 2 500 1,000
20X-69/70 3 500 1,500
20X-70/71 4 500 2,000
20X-71/72 5 500 2,500
20X-72/73 6 500 3,000
20X-73/74 7 500 for first six years and 1000 thereafter 4,000
20X-74/75 8 500 for first six years and 1000 thereafter 5,000

142. Mr. Tukaram Bidari joined Star General Stores, a trading firm on 1st Shrawan 20X-70. The following information is
received in relation to his employment income for the Income Year 20X-73/X-74:(Dec 2017, 10 Marks)

a. Pay scale Rs. 22,000 - 1,500 - 40,000.


b. He was paid with festival allowances equal to one months’ salary. Apart from the allowance, the firm provided 1 set
TV as a gift to the best performing staff. The market value of the TV is Rs. 55,000.
c. The firm pays Mr. Bidari Rs. 15,000 p.m. as house rent allowance. Out of the allowance, he pays only Rs. 12,000 p.m.
as house rent and saves Rs. 3,000 p.m.
d. He is entitled to entertainment allowance of Rs. 500 p.m. and annual medical allowance equal to 2% of salary.
e. The firm paid salary of Rs. 6,000 to the cook of Mr. Bidari in his residence. He compensated Rs. 2,000 to the firm on
this account.

© The Institute of Chartered Accountants of Nepal 195


CAP II Paper 7 Income Tax and VAT

f. Appointed in a senior post, he attended 12 meetings and was paid Rs. 2,000 per meeting as meeting allowance. He also
received allowance Rs. 2,000 p.m. for writing minutes of Human Resource Committee of the firm.
g. He was provided meal in the office in an equal term to all employees of the firm. The expenses borne by the firm for
this facility was Rs. 36,000 for entire year i.e. 3,000 p.m.
h. He spent Rs. 20,000 for promotion of firm by hosting dinner to customers. Cost on this account was reimbursed by the
firm to Mr. Bidari.
i. The firm paid Rs. 1,000 p.m. as tuition fees for each of two children of Mr. Bidari. This payment was made directly to
school rather than to Mr. Bidari.
j. During the year, he was paid Rs. 10,000 on account of leave encashment. Further, at the end of the year, his accumulated
leave pay was valued at Rs. 50,000 which was payable on retirement from the firm.
k. Mr. Bidari received a sum of Rs. 30,000 from his previous employer. The payment was made on account of target
commission on sales achieved by him when he was in employment with previous employee.
l. Mr. Bidari paid life insurance premium as follows:
• Rs. 15,000 for his policy of Rs. 5,00,000
• Rs. 10,000 for policy of Rs. 2,50,000 of his son
m. Mr. Bidari met an accident during the financial year and received Rs. 5,00,000 as compensation from insurance
company, out of which he incurred Rs. 1,00,000 for medical treatment. Mr. Bidari had contributed only Rs. 1,50,000 as
insurance premium to insurance company so far.
You are required to calculate income from employment and net tax liability of Mr. Bidari for income earned during FY
20X-73/X-74.
Answer
Assessable Income from Employment
Particulars Sec. Ref. Amount Notes
Monthly Remuneration
Basic Salary 8 (2) 264,000 Given basic salary, annualized
Grade 8 (2) 48,000 Refer W.N. 1, annualized
Festive Allowance 8 (2) 26,000 One months' salary, which includes both basic salary and grade
Gift in relation to employment 8 (2) 55,000 Market value of gift

© The Institute of Chartered Accountants of Nepal 196


Compilation of Suggested Answers Income Tax

House Rent Allowance 8 (2) 180,000 Total amount is included, as it is allowance


Entertainment Allowance 8 (2) 6,000
Medical Allowance 8 (2) 6,240 2% of basic salary and grade
Cook Facility 8 (2) 48,000 Actual Cost borne by employer, which is 4,000 per month
Final withholding, as per the clarification by IRD through Income
Meeting Allowance 92 -
Tax Directives
Minutes Writing Fee 8 (2) 24,000
Meals provided by Office 8 (3) - Exempt under Sec. 8 (3)
Dinner Hosting Cost for Promotion of
8 (3) - Serves proper business purpose of the employer
office
Payment of tuition fee of two children @ 1000 per child for 12
Reimbursement of personal expense 8 (2) 24,000
months
Leave encashed 8 (2) 10,000 Cash basis payment received during the tenure of employment
Accumulated leave payment 22 - Cash basis, as cash is not received, amount is not included
Compensation from previous employer 22, 8 (2) 30,000 Cash basis, and employment includes past employment as well

Any compensation received against injury caused due to personal


Compensation from Insurance
31 - accident is exempt, and the related medical cost cannot be treated
company in respect of accident
as approved medical expense for Medical Tax credit purpose

Assessable Income from Employment 721,240

Taxable Income & Balance Taxable Income


Assessable Income from Employment 721,240
Assessable Income from Business -
Assessable Income from Investment -
Total Assessable Income
721,240 It is taxable income as there is no reduction u/s 12, 12Kha or 63
(Taxable Income)
Less:

© The Institute of Chartered Accountants of Nepal 197


CAP II Paper 7 Income Tax and VAT

Payment of Investment Insurance Own premium to the extent of Rs. 25,000


15,000
premium Son's premium cannot be claimed
Balance Taxable Income 706,240

Calculation of Tax Liability


1st Rs. 400,000 1% 4,000 Individual Assessment
Next Rs. 100,000 10% 10,000
Next Rs. 200,000 20% 40,000
Balance Rs. 6,240 30% 1,872
Total Tax Obligation 55,872

Working Note:
Calculation of Monthly Grade amount for I.Y. 20X-74/X-75
Income Year No. of Grade Amount per grade Total Grade
20X-70/71 0 1,500 -
20X-71/72 1 1,500 1,500
20X-72/73 2 1,500 3,000
20X-73/74 3 1,500 4,000

143. Mohani Sharma was school English teacher in a reputed private school located at remote area (classified as “A” class area
by GON) between Shrawan 20X-72 to Poush 20X-72. She joined an international organization located at Kathmandu
immediately after resigning with effect from 1st Magh 20X-72. Mrs. Sharma has received following salary and benefits
during the income year 20X-72/X-73:
a. Salary & benefit received from private school
Particulars Amount (Rs.)
Basic Salary (Gross) 50,000 p.m.
Dashain Allowance 50,000

© The Institute of Chartered Accountants of Nepal 198


Compilation of Suggested Answers Income Tax

House Rent Allowance 5,000 p.m.


Remote Area Allowance 10,000 p.m.
Leave Encashment 25,000

b. Salary & benefit received from INGO in Kathmandu


Particulars Amount (Rs.)
Basic Salary (Gross) 1,00,000 p.m.
Advance Salary 1 month
House Rent Allowance 25,000 p.m.
Telephone Allowance 5,000 p.m.
Travelling & Daily Allowance (TADA) 10,000/day
School tuition fees of Mrs. Sharma children directly 4,000 p.m.
paid by INGO.

Other information:
a. Mrs. Sharma managed to deposit in retirement fund approved by IRD operated by listed commercial bank in
Kathmandu. She has deposited 4,50,000 in the retirement fund within income year 20X-72/X-73.
b. Mrs. Sharma contributed donation of Rs. 3,00,000 to tax exempt organization.
c. Mrs. Sharma had gone outstation for 25 days to monitor INGO activities. TADA provided to Mrs. Sharma is only to
pay cost of foods & lodging charges. Mrs. Sharma could only submit bills of Rs. 50,000 incurred by her for food and
lodging charges.
d. Life insurance premium paid by Mrs. Sharma Rs. 25,000.
e. Mrs. Sharma was sick and incurred medical expenses of Rs. 5,000 to hospital run by GON.
f. TDS deducted & deposited by school for Mrs. Sharma salary & benefits withdrawn by her up to Poush 2072 was Rs.
63,388 and INGO had deposited 1,00,000 within the end of Ashad 20X-73.
Required: (June 2017, 6+1+3=10)
(a) Calculate taxable income and tax liability of Mrs. Sharma for the income year 20X-72/X-73.

© The Institute of Chartered Accountants of Nepal 199


CAP II Paper 7 Income Tax and VAT

Answer
Statement of Assessable Income from Employment
Sec.
Particulars Amount Note
Ref.
Salary from Private School 8 (2) 300,000 Rs. 50,000 per month for six months
Dashain Allowance from Private
8 (2) 50,000
School
House Rent Allowance from Private
8 (2) 30,000 Rs. 5,000 per month for six months
School
Remote Area Allowance from Private
8 (2) 60,000 Rs. 10,000 per month for six months
School
Final withholding, assuming that the amount is received at
Leave Encashment from Private the time of retirement
8 (3) -
School Alternatively, it can be included in income assuming it was
regular employment payment
Salary from INGO 8 (2) 600,000 Rs. 100,000 per month for six months
Advance Salary from INGO 8 (2) 100,000 Cash basis, assumed not a loan from employer
House Rent Allowance from INGO 8 (2) 150,000 Rs. 25,000 per month for six months
Telephone Allowance from INGO 8 (2) 30,000 Rs. 5,000 per month for six months
Principally Travel and daily allowance serves proper
business purpose of employer and is exempt from tax, as it
is computed so scientifically that there cannot be any
saving to employee. Saving, if any, will be at the cost of
Traveling and Daily allowance 8 (3) -
health or hygiene of employee.
In case, it is made tax avoidance tool, IRD can apply Sec.
35 to collect tax. However, it is subject to interpretation by
court
Reimbursement of Personal Expenses
8 (2) 24,000 School fee of children, Rs. 4,000 per month for six months
from INGO
Assessable Income from
1,344,000
Employment

© The Institute of Chartered Accountants of Nepal 200


Compilation of Suggested Answers Income Tax

Statement of Taxable Income and Balance Taxable Income


Assessable Income from Business -
Assessable Income from Employment 1,344,000
Assessable Income from Investment -
Total Assessable Income 1,344,000
Less: Reductions
Lower of following:
a. 5% of Adjusted Taxable Income- Rs. 67,200
(Assessable income is ATI as there are no any
Under Sec. 12 67,200 adjustments)
b. Actual Gift or Donation to Exempt Organization- Rs.
300,000
c. Maximum Rs. 100,000
Under Sec. 12Kha -
Lower of following:
a. One third of Assessable Income- Rs. 448,000
Under Sec. 63 300,000
b. Actual Contribution among all ARF- Rs. 450,000
c. Maximum Rs. 300,000
Taxable Income 976,800
Less: Deductions under Schedule 1
Residence in Remote Area 25,000 Six months in "A" Category Remote Area
Payment of Investment Insurance
25,000 Lower of actual premium or Rs. 25,000
Premium
Balance Taxable Income 926,800

Statement of Tax Liability


1st Rs. 400,000 1% 4,000 Assuming Individual Assessment
Next Rs. 100,000 10% 10,000
Next Rs. 200,000 20% 40,000

© The Institute of Chartered Accountants of Nepal 201


CAP II Paper 7 Income Tax and VAT

Balance Rs. 251,800 30% 68,040


Total before Tax Credits 122,040
Less: Female Tax Credit 12,204 10% of computed tax liability before all tax credits
Lower of Following:
a. 15% of approved Medical expense- Rs. 750
Less: Medical Tax Credit 750 (Cost incurred in Government hospital for her own
treatment is Rs. 5,000)
b. Maximum Rs. 750
Tax Liability for the Year 109,086

(b) Does Mrs. Sharma require to file income tax return as per the provision of Income Tax Act, 2058.
Answer:
As per Sec. 97 (1), a natural person satisfying all the following conditions, is not required to file income return unless IRD
requires so in writing or through public circular or the natural person’s income exceeds Rs. 40 Lakhs during the Income
Year:
i.The natural person earns income only from employment having source in Nepal,
ii.There is only one employer at a time, and all employers are resident of Nepal, and
iii. The person claims medical tax credit and contribution to approved retirement fund to the extent paid through employer
and does not claim donation
In the given case, first two conditions are satisfied and not the third one as the employee has claimed donation and
contribution to approved retirement fund by way of own contribution, the person is not exempt from filing income return
due to this criterion.
Further, the person has payable tax obligation for the year, does not derive all income from final withholding payments
and does not satisfy conditions of Sec. 97 (1) (Gha) and (Nga), therefore, the person shall file income return.

© The Institute of Chartered Accountants of Nepal 202


Compilation of Suggested Answers Income Tax

(c) Compute amount of TDS need to be deposited as last installment for the salary & benefit withdrawn by Mrs. Sharma
during the income year 20X-72/X-73.
Answer
The information is incomplete regarding computation of TDS during Ashad as the question is not set using practical legal
assumption. If we need to answer it to the best of question, the tax obligation is less than the amount already paid, i.e. there
seems no additional withholding of taxes.

144. Mr. Robin is a retired person from Nepal Bank Limited, now he is receiving pension @ Rs. 30,000 per month and residing
in Kathmandu with his family. For tax purpose, he wants to select a single person. You are required to make tax assessment
for the Income Year 20X-72/X-73 (Dec 2016, 2.5 Marks)
Answer
Total income is Rs. 390,000 (including Dashain expense), which is below the basic exemption limit of Rs. 400,000 (in case of
individual assessment).
Therefore, there is no tax obligation for Mr. Robin during the year.

145. Mr. Donald is an employee of American Embassy located at Kathmandu. He is working since 15 Falgun, 20X-72. He has
received amounting of Rs. 10 lakhs from his part-time consultancy services provided to an educational consultancy Pvt.
Ltd. located at Kathmandu. You are required to make tax assessment for the Income Year 20X-72/X-73.
(Dec 2016, 2.5 Marks)
Answer
The amount from consultancy is not an exempt income, but business income of Mr. Donald. Mr. Donald is resident of Nepal, as
such, the tax liability (assuming single) is as follows:
1st Rs. 400,000 0% 0
Next Rs. 100,000 10% 10,000
Next Rs. 200,000 20% 20,000
Balance Rs. 300,000 30% 90,000
Total 120,000
146. Laxmi Khadka, a married female, took up an employment in Shrawan 20X-72 with an NGO in Khalanga of Jumla District.
The following are the particulars of her income for the financial year 20X-72/X-73:

© The Institute of Chartered Accountants of Nepal 203


CAP II Paper 7 Income Tax and VAT

a. Total salary Rs. 720,000


b. Total allowances Rs. 200,000

NGO provided her an accommodation free of cost in Khalanga for which NGO has to pay the rental charges of Rs. 10,000
per month. The above allowances include total Daily Allowance of Rs. 100,000 paid to her during the year for her official
travel. She is also entitled for Dashain Festival Allowance equivalent to 1 month’s salary.

Compute her taxable income and tax liability for the financial year 20X-72/X-73. (June 2016, 5 Marks)
Answer
Statement of Assessable Income from Employment
Particulars Sec. Ref. Amount Note
Salary 8 (2) 720,000
Allowances for official travel serves proper business
Allowances 8 (2) 100,000 purpose of employer, and thus, exempt as per Sec. 8
(3)
2% of Khai paai aaeko talab, i.e. 2% of basic salary
Accommodation Facility 8 (2) 14,400
and grade
Assumed to be one month Salary (excluding
Festive Allowance 8 (2) 60,000
allowances)

Assessable Income from Employment 894,400

Statement of Taxable Income and Balance Taxable Income


Assessable Income from Business -
Assessable Income from Employment 894,400
Assessable Income from Investment -
Total Assessable Income
894,400 There is no reduction u/s 12, 12Kha and 63
(Taxable Income)
Less: Deductions under Schedule 1

© The Institute of Chartered Accountants of Nepal 204


Compilation of Suggested Answers Income Tax

Jumla is Class B Remote Area, assuming residence


Residence in Remote Area 40,000
throughout the year
Balance Taxable Income 854,400

Statement of Tax Liability


1st Rs. 400,000 1% 4,000 Assuming Individual Assessment
Next Rs. 100,000 10% 10,000
Next Rs. 200,000 20% 40,000
Balance Rs. 154,400 30% 46,320
Tax Liability 100,320

147. Mrs. Indira Dahal was retired on 1st Magh, 20X-71 after completing 22 years of private banking service from Kathmandu.
She submits the following details with respect to her employment for the Income Year 20X-71/X-72.
a. Monthly salary Rs. 35,000
b. Two months' salary is provided for dress and festival, one month for each.
c. She has obtained overtime allowance of Rs. 20,000
d. The bank deducts 10 % from her salary and contributes 20 % of her salary to an approved retirement fund.
e. She has car facility from the office for all purpose. Expenses Rs. 1,000 per month and salary of the driver of Rs. 12,000
per month were paid by the bank.
f. Compensation received as per Employee Rule Rs. 100,000
g. Insurance premium paid by Employer on her behalf, Rs. 25,000 that include Rs. 12,000 for her policy and Rs. 13,000
for her son's policy.
h. The bank makes payment equivalent one-month salary to the employees who work for full 12 months during the years.
i. The bank has provided her with accommodation facility on leased house for which 2 % of salary was deducted. Actual
rent paid by the bank was Rs. 35,000 per year.
j. On the retirement, she has entitled to receive gratuity amount equivalent to one month salary for each year services.
k. The company paid bonus to her equivalent to two month's salary related to the Income Year 20X-71/X-72 on 20th
Aswin 20X-72.

© The Institute of Chartered Accountants of Nepal 205


CAP II Paper 7 Income Tax and VAT

She has only income from this employment and opted for single. You are required to assess income from employment and
her tax liability. (Dec 2015, 10 Marks)
Answer
Statement of Assessable Income from Employment
Sec.
Particulars Amount Note
Ref.
Salary 8 (2) 210,000 Rs. 35,000 per month for six months
Dashain Allowance 8 (2) 35,000
Dress Allowance 8 (2) 35,000
Overtime Allowance 8 (2) 20,000
Employers' contribution to Retirement
8 (2) 21,000
Fund
0.5% of khai paai aaeko talab, i.e. 0.5% of Rs. 210,000
Vehicle Facility 27 1,050 Car operating cost including driver's salary is covered by the
amount so calculated
Compensation related to employment 31 10,000
Insurance premium paid by employer on top of salary and other
Reimbursement of personal expense 8 (2) 25,000
benefits
Additional pay by bank 8 (2) 17,500 Point (h), pro-rata on the basis of time served
Accommodation Facility 27 4,200 2% of khai paai aaeko talab
Gratuity 92 - Final withholding, since it is retirement payment
Bonus 8 (2) 70,000 Cash basis and it is received this year
Assessable Income from Employment 448,750

Statement of Taxable Income and Balance Taxable Income


Assessable Income from Business -
Assessable Income from
448,750
Employment
Assessable Income from Investment -

© The Institute of Chartered Accountants of Nepal 206


Compilation of Suggested Answers Income Tax

Total Assessable Income 448,750


Less: Reduction under Sec. 63
Actual or 1/3rd of Assessable
Income or 42,000 Actual Contribution is Rs. 42,000 (20% of salary)
Rs. 300,000; whichever is lower
Taxable Income 406,750
Less: Deduction under Schedule 1
Payment of Investment Insurance Her own premium only
12,000
Premium Minimum of actual payment or Rs. 25,000
Balance Taxable Income 394,750

Statement of Tax Liability


1st Rs. 394,750 1% 3,975 Individual Assessment

Tax Liability 3,975

148. Smriti works in a private school at monthly salary of Rs. 30,000 for financial year 20X-72/X-73. She is paid Dashain
allowance of Rs. 20,000. Her contribution to approved retirement fund is Rs. 40,000 per annum. She has already paid
medical insurance of Rs. 25,000 and life insurance premium of Rs. 15,000 for the year. She incurs Rs. 3,000 for purpose of
transportation to and fro school from her residence, and also pays Rs. 5,000 for baby care of her young child so that she
could work in school. The school pays Rs. 6,000 school fee of her elder child per month. Calculate the tax liability assume
individual. (Dec 2015, 5 Marks)
Answer
Statement of Assessable Income from Employment
Sec.
Particulars Note
Ref. Amount
Salary 8 (2) 360,000 Rs. 30,000 per month for 12 months
Dashain Allowance 8 (2) 20,000

© The Institute of Chartered Accountants of Nepal 207


CAP II Paper 7 Income Tax and VAT

Transportation expense 21 - Not deductible u/s 21, personal expense


Baby care expense 21 - Not deductible u/s 21, personal expense
Reimbursement of personal expense 8 (2) 72,000 Rs. 6,000 per month for her child's eduction
Assessable Income from Employment 452,000

Statement of Taxable Income and Balance Taxable Income


Assessable Income from Business -
Assessable Income from Employment 452,000
Assessable Income from Investment -
Total Assessable Income 452,000
Less: Reduction under Sec. 63
Actual or 1/3rd of Assessable Income or
20,000
Rs. 300,000; whichever is lower
Taxable Income 432,000
Less: Deduction under Schedule 1
Payment of Investment Insurance
15,000 Minimum of actual payment or Rs. 25,000
Premium
Payment of Health Insurance Premium 20,000 Minimum of actual payment or Rs. 20,000
Balance Taxable Income 397,000

© The Institute of Chartered Accountants of Nepal 208


Compilation of Suggested Answers Income Tax

Statement of Tax Liability


1st Rs. 397,000 1% 3,970 Individual Assessment
Tax Liability 3,970

149. Mr. Robert Wooster, a citizen of USA, is employed in research department of Mokshya Pvt. Ltd., Nepal with effect from 1 st
Poush 20X-71. Due to nature of his job requirements Mr. Wooster has to reside in Nepal. His employer furnishes him pay
slip showing deduction of Rs. 265,000 as withholding Tax under sec. 87 of Income Tax Act, 2058 for financial year 20X-
71/X-72.
Mr. Wooster has obtained following information about remuneration from Mokshya Pvt. Ltd.
Particulars Amount
(Rs.)
Basic salary per month 50,000
Uniform allowance 7,000
Technical allowance per month 60,000
Flat rent of Mr. Wooster paid by employer 60,000
Telephone facility per month 500
Free vehicle facility provided
Compensation paid by current employer to previous employer of Mr. Wooster which had to be paid by Mr. Wooster 200,000
Advance salary for Shrawan 20X-72 and Bhadra 20X-72 100,000
Leave pay 50,000
Value of accumulated leave provision payable at the end of service period 10,000
Tuition fees of son of Mr. Wooster for learning Nepali language paid directly by employer to private tutor 40,000
Air fare of Mr. Wooster from USA to Nepal paid by employer 100,000
Canteen bill paid by employer 40,000
Drivers salary of vehicle provided by employer 100,000

Required: (July 2015, 7+2+1=10)

© The Institute of Chartered Accountants of Nepal 209


CAP II Paper 7 Income Tax and VAT

a. Mr. Wooster now seeks your advice whether withholding Tax deducted by his employer under sec. 87 of Income Tax
Act, 2058 is correct.
Answer
Statement of Assessable Income from Employment
Sec.
Particulars Amount Note
Ref.
Basic Salary 8 (2) 350,000 Rs. 50,000 per month for 7 months (Poush to Ashad)
Uniform Allowance 8 (2) 7,000
Technical Allowance 8 (2) 420,000 Rs. 60,000 per month for 7 months (Poush to Ashad)
2% of Khai paai aaeko talab, assuming that flat is leased by
employer
Flat Rent of Wooster 8 (2) 7,000
If students assume that the rental payment was made directly
to Wooster in cash, the whole amount is included in income
Telephone facility 8 (2) 35,000 Rs. 500 per month for 7 months (Poush to Ashad)
0.5% of Khai paai aaeko talab, i.e. 0.5% of Rs. 350,000
Vehicle Facility 8 (2) 1,750 The driver's salary in last row is deemed to be covered by the
amount so quantified
Compensation to be paid by Wooster to previous employee
Reimbursement of personal expense 8 (2) 200,000
paid by current employer
Advance Salary 22 100,000 Cash basis, and the cash is received this year
Leave pay 22 50,000 Cash basis, and the cash is received this year
Accumulated leave provision 22 - Not yet paid in cash, as such, not included
Reimbursement of personal expense 8 (2) 40,000 Tuition fee of Mr. Wooster reimbursed
Reimbursement of personal expense 8 (2) 100,000 Airfare from USA to Nepal
Assumed that food is provided on similar terms to all
employees
Canteen Bill paid by employer 8 (3) -
Alternatively, if it is assumed that food is provided to him
specifically, the amount shall be included in income
Driver's Salary 27 - Refer Notes on Vehicle facility above

© The Institute of Chartered Accountants of Nepal 210


Compilation of Suggested Answers Income Tax

Assessable Income from


1,310,750
Employment

Statement of Taxable Income and Balance Taxable Income


Assessable Income from Business -
Assessable Income from Employment 1,310,750
Assessable Income from Investment -
Total Assessable Income 1,310,750
Less: Reduction under Sec. 63, 12 or 12Ka -
Taxable Income 1,310,750
Less: Deduction under Schedule 1 -
Balance Taxable Income 1,310,750

Statement of Tax Liability


1st Rs. 400,000 1% 4,000 Individual Assessment
Next Rs. 100,000 10% 10,000
Next Rs. 200,000 20% 40,000
Balance Rs. 610,750 30% 183,225
Tax Liability 237,225

As computed above, the total tax obligation of Mr. Wooster is Rs. 237,225; which is less than that deducted by the accountant.

© The Institute of Chartered Accountants of Nepal 211


CAP II Paper 7 Income Tax and VAT

b. Whether Mr. Wooster is required to file income Tax Return for financial year 2071-72 in Nepal as per Income Tax Act,
2058?
Answer
As per Sec. 97 (1) jointly read with Sec. 97 (2), a person is not required to file income return in the following conditions, unless
otherwise required by IRD in writing or through public circular or where the natural person’s income exceeds Rs. 40 Lakhs
during the Income Year:
a. If the person does not have any payable tax liability,
b. If the person generates income only from final withholding income,
c. If the person satisfies conditions of Sec. 4 (3),
d. If the person is a natural person not operating vehicle paying tax under Sec. 1 (13) of Schedule 1 of the Act through a
private firm, or
e. If a person derives income only from disposal of non-business chargeable asset, desirous of not filing income return
Since condition (c) above is satisfied, Mr. Wooster is not required to file income return.
c. Mr. Wooster contends that as he is a citizen of USA, he is required to pay tax in USA and not in Nepal. Is Mr. Wooster
Correct? If not explain why is he required to pay Taxes in Nepal.
Answer
Mr. Wooster is resident of Nepal, as he has stayed in Nepal for more than 183 days during the Income Year.
As per Sec. 6, a resident person has to pay tax on worldwide income and a nonresident person on income having source in
Nepal.

Though he is citizen of Nepal, however, he is resident of Nepal, as such, he has to pay tax in Nepal on his worldwide Income.
Further, if he were not resident of Nepal and since his employment is in Nepal (which means employment income has source
in Nepal), he is still required to pay tax in Nepal.

© The Institute of Chartered Accountants of Nepal 212


Compilation of Suggested Answers Income Tax

150. Saru Pvt. Ltd., Kathmandu declared a Voluntary Retirement Scheme (VRS) for its staffs during FY 20X-71/X-72.
Provident funds are deposited in CIT, an approved retirement fund. Man Bahadur decided to take VRS with effect from
1st Baishakh 20X-72. The following details are available for income of Man Bahadur for the year.
• Basic Salary till Chaitra 20X-71 Rs. 50,000 per month.
• Monthly Allowance Rs. 30,000 per month.
• Provident Fund contributed by employer 10% of basic salary
An equal amount was contributed by employee.
• Life Insurance premium paid by Man Bahadur for himself Rs. 25,000
• Remote area allowance for working in Category C district for two months Rs. 5,000 per month
• Provident Fund paid by CIT on 15th Baishakh 20X-72 Rs. 12,00,000.
• Retirement payment paid by Saru Pvt. Ltd. as per Voluntary Retirement Scheme was Rs. 20,00,000, paid to Man
Bahadur directly by the company.

Required: (Dec 2014, 7+3)


a. Calculate taxable income and tax liability of Man Bahadur for F.Y. 20X-71/X-72 as per Income from Employment.
Assume individual.
Answer
Statement of Assessable Income from Employment
Particulars Sec. Ref. Amount Note
Basic Salary 8 (2) 450,000 Rs. 50,000 per month for 9 months (until Chaitra)
Monthly Allowance 8 (2) 270,000 Rs. 30,000 per month for 9 months (until Chaitra)
Employer's Contribution to Retirement Fund 8 (2) 45,000 10% of Basic Salary
Remote Area Allowance 8 (2) 10,000 Rs. 5,000 per month for 2 months (until Chaitra)
Assessable Income from Employment 775,000

Statement of Taxable Income and Balance Taxable Income


Assessable Income from Business -
Assessable Income from Employment 775,000

© The Institute of Chartered Accountants of Nepal 213


CAP II Paper 7 Income Tax and VAT

Assessable Income from Investment -


Total Assessable Income 775,000
Less: Reduction under Sec. 63 -
(One-third of Assessable Income, or Actual, Actual is Rs. 90,000; assuming the contribution is
90,000
or Rs. 300,000; whichever is lower) made at Approved Retirement Fund
Taxable Income 685,000
Less: Deduction under Schedule 1
C' Category is entitled to deduct Rs. 30,000 for the
Residence in Remote Area 5,000 year; as the residence is for two months, pro-rated
for two months
Investment Insurance Premium Paid 25,000 Lower of actual payment or Rs. 25,000
Balance Taxable Income 655,000

Statement of Tax Liability


1st Rs. 400,000 1% 4,000 Individual Assessment
Next Rs. 100,000 10% 10,000
Next Rs. 155,000 20% 31,000
Tax Liability 45,000

b. Identify any payments subject to final withholding Tax (Final TDS) and applicable TDS rate and Tax amount.
Answer
Identification of and amount of Final withholding Payments and Tax
Provident Fund by CIT 1,200,000
Gain is calculated by deducting higher of 50% of Rs.
Taxation on Gain @ 5% 30,000 1,200,000 or Rs. 500,000 from Rs. 1,200,000; i.e. Gain is Rs.
600,000
Retirement Payment under VRS 2,000,000
Taxation @ 15% 300,000
Total Final Withholding Taxes 330,000

© The Institute of Chartered Accountants of Nepal 214


Compilation of Suggested Answers Income Tax

151. Mr. Sailesh, a disabled person, is working in Dailekh branch of Nabil Bank Ltd. In Fiscal Year 20X-69/X-70, following are
the transactions;
a. Salary and allowances Rs. 50,000 per month; Dashain allowance Rs. 20,000; Bonus Rs. 40,000.
b. Bank has managed to contribute to Provident Fund (approved) Rs. 24,000 from employer as well as employee side.
c. In addition to the remuneration received from Bank, Mr. Sailesh has also received Rs. 150,000 from Nepal Government
in the form of Pension.
d. He has covered the insurance and paid the insurance premium of Rs. 25,000 during Fiscal Year 20X-69/X-70.
e. He has declared the couple under section 50, in Fiscal Year 20X-69/X-70.
Compute Assessable Income, Taxable Income, Tax liability of Mr. Sailesh for the Fiscal Year 20X-69/X-70 stating relevant
provisions of Income Tax Act, 2058. (June 2014, 10 Marks)
Answer
Statement of Assessable Income from Employment
Particulars Sec. Ref. Amount Note
Salary and allowances 8 (2) 600,000 Rs. 50,000 per month for a year
Dashain Allowance 8 (2) 20,000
Bonus 8 (2) 40,000
Assuming 24,000 is total contribution, 50% is
Employer's Contribution to Retirement Fund 8 (2) 12,000
employer's part
Cash basis, employment includes past employment
Pension from Government of Nepal 8 (2) 150,000
as well
Assessable Income from Employment 822,000

Statement of Taxable Income and Balance Taxable Income


Assessable Income from Business -
Assessable Income from Employment 822,000
Assessable Income from Investment -
Total Assessable Income 822,000

© The Institute of Chartered Accountants of Nepal 215


CAP II Paper 7 Income Tax and VAT

Less: Reduction under Sec. 63 -


(One-third of Assessable Income, or Actual, or
24,000
Rs. 300,000; whichever is lower)

Taxable Income 798,000


Less: Deduction under Schedule 1
Dailekh is in Category "C", deduction of Rs. 30,000
Residence in Remote Area 30,000
assuming he resides there throughout the year
Investment Insurance Premium Paid 25,000 Lower of actual payment or Rs. 25,000
Incapacitated Person 225,000 50% of Basic exemption limit of Rs. 450,000

Balance Taxable Income 518,000

Statement of Tax Liability


Couple Assessment, and other employment income
1st Rs. 450,000 1% 4,500
is above basic exemption limit
Next Rs. 68,000 10% 6,800
Tax Liability 11,300

152. Mr. Sinha is an employee of Surya Moon Corporation Ltd. His financial activities F.Y. 20X-69/X-70 includes the following:
Basic Salary Rs. 40,000 P.M.
Traveling Allowance (From Home to Office and Rs. 4,000 P.M.
Back)
Contribution to approved Provident fund Rs. 4,000 P.M.
Dividend income from Surya Moon Corporation Rs. 10,000
Ltd.

Mr. Sinha also deposited equal amount to the same provident fund.

© The Institute of Chartered Accountants of Nepal 216


Compilation of Suggested Answers Income Tax

On the occasion of 25th Anniversary of the corporation, the corporation organized a National 1 Km. race competition in
association with the Nation Game Association. Mr. Sinha participated in the race and won the first prize worth Rs. 50,000.
Mr. Sinha has a Life Insurance Policy worth Rs. 2,00,000 and annual premium paid is Rs. 15,000.

Calculate the total income and tax liability from the employment for the fiscal year 20X-69/X-70, if he selects to be assessed
as couple. (Dec 2013, 2+5=7)
Answer
Statement of Assessable Income from Employment
Particulars Sec. Ref. Amount Note
Basic Salary 8 (2) 480,000 Rs. 40,000 per month for a year
Travel from home to office and back, does not serve
Reimbursement of Personal Expenses 8 (2) 48,000
proper business purpose of employer
Final withholding assuming distributing company is
Dividend Income 92 -
resident
Employer's Contribution to Retirement Fund 8 (2) 48,000
Windfall gain, and final withholding
If the competition were among the staffs of the
Prize from National 1 KM race 92 -
corporation, the amount would be included in
income
Assessable Income from Employment 576,000

Statement of Taxable Income and Balance Taxable Income


Assessable Income from Business -
Assessable Income from Employment 576,000
Assessable Income from Investment -
Total Assessable Income 576,000
Less: Reduction under Sec. 63 -
(One-third of Assessable Income, or Actual, or
96,000 Total contribution is Rs. 96,000
Rs. 300,000; whichever is lower)

© The Institute of Chartered Accountants of Nepal 217


CAP II Paper 7 Income Tax and VAT

Taxable Income 480,000


Less: Deduction under Schedule 1
Investment Insurance Premium Paid 15,000 Lower of actual payment or Rs. 25,000
Balance Taxable Income 465,000
Statement of Tax Liability
Couple Assessment, and other employment income
1st Rs. 450,000 1% 4,500
is above basic exemption limit
Next Rs. 15,000 10% 1,500
Tax Liability 6,000

153. Green Development Bank Ltd. has appointed Mr. Hira Paudel as CEO for the effective operation of the bank. As per the
TOR with the CEO, annual salary and allowances Rs. 5,000,000 (net of tax) shall be paid to the CEO by the bank. All tax
related to the employment of the CEO shall be borne by the company and paid to Inland Revenue Office. Mr. Hira is to be
assessed as couple and he claimed Rs. 20,000 of insurance premium. Should the bank deduct the tax amount from his
salary? If yes, then mention the relevant provisions of the Act; compute the tax liability of Mr. Hira and deductible salary
and allowances for this payment on the tax assessment of the bank. (June 2013, 5 Marks)
Answer
Gross Amount = Net Amount divided by (1- Tax Rate)
Net Amount is Rs. 4,980,000 (net of insurance premium paid)
The balacing figure is subject to 36% tax
The balance of net amount is 49,80,000 minus net amounts of first, second, third and fourth slab

Tax Tax Net


Particulars Gross Amount Rate amount Amount Note
A B C D= A- C

1st Rs. 450,000 1% 4,500 445,500 5,327,344= 3,409,500


Next Rs. 100,000 10% 10,000 90,000 divided by 0.64

© The Institute of Chartered Accountants of Nepal 218


Compilation of Suggested Answers Income Tax

Next Rs. 200,000 20% 40,000 160,000


Next Rs. 1,250,000 30% 375,000 875,000
Balance Rs. 5,327,344 36% 1,917,844 3,409,500 3,409,500= 4,980,000-445,500-90,000-160,000-87,5000
Total 7,327,344 4,980,000
Insurance
Premium Paid 20,000
Total Gross Salary 7,347,344

154. List out the payments not included in the income from employment under Income Tax Act, 2058. (June 2013, 5 Marks)
OR
What are the payments that are received or deemed to have been received by an employee from the present or past
employer or his associates which are not includible in the income from “Employment” under the Income Tax Act, 2058?
(June 2005, 7 Marks, CA Inter)
Answer
The following amounts are not included in the income from employment:
a. Exempt amounts
b. Final withholding payments
c. Any reimbursements or discharge of expenses satisfying any of the following two conditions:
• That serves proper business purpose of the employer
• That would or otherwise be exempt from inclusion in business or investment income
d. Lunch or tiffin to all employees under similar terms
e. The amount of the costs is so small as to make it unreasonable to require or administratively impracticable for the person to
account for them

155. Mr. Z, retired person from Nepal Government on 1st Sharwan, 20X-68 after 30 years of Service. After retirement, he joined
Kathmandu Bank at Kathmandu on the same day, .i.e. Sharwan, 20X-68 and remained in service till Ashad end 20X-69.
Information about his income are as follows:
S. N. Particulars Nepal government Kathmandu bank
1. Salary Rs. 3,00,000 Per month

© The Institute of Chartered Accountants of Nepal 219


CAP II Paper 7 Income Tax and VAT

2. House rent Rs. 20,000 Per month


allowance
3. Other allowance Rs. 10,000 Per month
4. Provident fund Additional 10 % of
contribution basic salary & grade
5. Pension received Rs. 144,000 during ----------------
the year

Other information:
a. He has been provided a vehicle for personal as well as official use by the Kathmandu bank.
b. Both employers have deducted 10% of salary for provident fund and deposited 20% to the approved retirement fund.
c. He had received all retirement payment in Ashad 20X-68.
d. He has not been provided Dashain allowances during the period.
e. His wife does not have any income in this Income year and they declared they are couple.
Calculate the assessable income, taxable income and tax amount of Mr. Z for the income year. (Dec 2012, 6 Marks)

© The Institute of Chartered Accountants of Nepal 220


Compilation of Suggested Answers Income Tax

Answer
Statement of Assessable Income from Employment
Particulars Sec. Ref. Amount Note
Salary from Kathmandu Bank 8 (2) 3,600,000
House Rent Allowance from Bank 8 (2) 240,000
Other Allowance 8 (2) 120,000
There cannot be contribution to RF while the person
Employer's Contribution to Retirement Fund 8 (2) 360,000
is obtaining pension income
Pension from Government of Nepal 8 (2) 144,000
0.5% of Khai paai aaeko talab, Khai paai aaeko
talab is interpreted as sum of basic salary and
Vehicle Facility 27 18,000
garde derived by the employee from the employer
providing such facility
Assessable Income from Employment 4,482,000

Statement of Taxable Income and Balance Taxable Income


Assessable Income from Business -
Assessable Income from Employment 4,482,000
Assessable Income from Investment -
Total Assessable Income 4,482,000
Less: Reduction under Sec. 63 -
(One-third of Assessable Income, or Actual, or
300,000 Total contribution is Rs. 720,000
Rs. 300,000; whichever is lower)
Taxable Income 4,182,000
Less: Deduction under Schedule 1
Balance Taxable Income 4,182,000

Statement of Tax Liability

© The Institute of Chartered Accountants of Nepal 221


CAP II Paper 7 Income Tax and VAT

Couple Assessment, and other employment income


1st Rs. 450,000 1% 4,500
is above basic exemption limit
Next Rs. 100,000 10% 10,000
Next Rs. 200,000 20% 40,000
Next Rs. 1,250,000 30% 375,000
Balance Rs. 2,182,000 36% 785,520
Tax Liability 1,215,020

156. Mr. Ram Prasad Thapa, a Joint secretary in Nepal Government had worked since 1/4/20X-68 to Poush end 20X-68 as
acting ambassador in USA. Due to his unsatisfactory performance GoN has called him back to Nepal and transferred to
Solukhumbu as Chief District Officer. During the income year 20X-68/X-69 he had taken following amount of salary and
perquisites:
Amount in Rs.
Salary per month 25,000
Dashain allowance 25,000
Contribution to EPF 30,000
(an equal amount of deduction has been made from his salary as his own contribution)
Foreign allowance monthly 50,000
Calculate his taxable income for income year 20X-68/X-69. (Dec 2012, 5 Marks, CA Inter)

Answer
Statement of Assessable Income from Employment
Particulars Sec. Ref. Amount Note
Salary 8 (2) 300,000
Dashain Allowance 8 (2) 25,000
Employer's Contribution to Retirement Fund 8 (2) 30,000
Foreign Allowance 8 (2) 300,000 Six months from Sharawan to Poush
Assessable Income from Employment 655,000

© The Institute of Chartered Accountants of Nepal 222


Compilation of Suggested Answers Income Tax

Statement of Taxable Income and Balance Taxable Income


Assessable Income from Business -
Assessable Income from Employment 655,000
Assessable Income from Investment -
Total Assessable Income 655,000
Less: Reduction under Sec. 63 -
(One-third of Assessable Income, or Actual, or
60,000 Actual Contribution is Rs. 60,000
Rs. 300,000; whichever is lower)
Taxable Income 595,000
Less: Deduction under Schedule 1
Salleri of Solukhumbu district is in Category "D",
Residence in Remote Area 10,000
deduction of Rs. 20,000; pro-rata for six months
Foreign Allowance 225,000 75% of foreign allowance
Balance Taxable Income 360,000

157. Mrs. Sita Nepal is an employee at Manang Branch of ABC Bank Ltd. For income year 20X-68/X-69 she earns Rs. 25,000
per month as salary and allowances. She received Rs. 20,000 as Dashain expenses and Rs. 40,000 against bonus. Bank has
contributed Rs. 24,000 during the year towards contribution to approved retirement fund and by deducting same amount
from her salary, deposited the amount in approved retirement fund. Calculate the taxable income and tax liability of Mrs.
Sita for income year 20X-68/X-69. (Dec 2012, 4+3=7, CA Inter)
Note:
a. Mrs. Sita Nepal opts to be assessed as individual and has no other source of income.
b. Manang falls under class A as per classification given by Income Tax Act.
Answer
Statement of Assessable Income from Employment
Sec.
Particulars Amount Note
Ref.

© The Institute of Chartered Accountants of Nepal 223


CAP II Paper 7 Income Tax and VAT

Salary and Allowance 8 (2) 300,000


Dashain Allowance 8 (2) 20,000
Bonus 8 (2) 40,000
Employer's Contribution to Retirement Fund 8 (2) 24,000
Assessable Income from Employment 384,000
Statement of Taxable Income and Balance Taxable Income
Assessable Income from Business -
Assessable Income from Employment 384,000
Assessable Income from Investment -
Total Assessable Income 384,000
Less: Reduction under Sec. 63 -
(One-third of Assessable Income, or Actual, or Rs.
48,000 Actual Contribution is Rs. 48000
300,000; whichever is lower)
Taxable Income 336,000
Less: Deduction under Schedule 1
Residence in Remote Area 50,000 A' category Remote Area
Balance Taxable Income 286,000

Tax Liability (1%) 2,860

© The Institute of Chartered Accountants of Nepal 224


Compilation of Suggested Answers Income Tax

158. Mr. Bhai Raja is a person working in a Financial Institution as Chief Manager. Besides working in Financial Institution,
he is a professor of Account in Shankar Dev Campus. He generates the following income from the employers in Fiscal Year
20X-67/X-68.
a. Monthly Remuneration NPR 40,000; Dashain Allowance NPR 40,000; Education Allowance NPR 2,000 per month;
Expenditure Allowance NPR 2,000 per month; Bonus NPR 60,000; Remuneration from teaching in Shankar Dev
Campus NPR 6,000 per month.
b. He is provided a car along with driver for his official and personal purpose. Monthly remuneration and allowance of
driver is NPR 8,000.
c. Petrol Expenditure is provided for his car which is NPR 10,000 per month and maintenance expenditure of such car is
NPR 25,000 in Fiscal Year 20X-67/X-68.
d. He is provided a housing facility by the employer.
e. Monthly expenditure of Telephone connected in his resident is NPR 2,000 per month. Out of this 50% is his personal
telephone expenditure.
f. A security guard is being provided in his resident and it is being paid NPR 6,000 per month by the employer. Because of
being used the security guard in his resident, NPR 3,000 is being deducted from his monthly salary.
g. He has used the loan amounting to NPR 10, 00,000 under housing loan facility at the rate of 5% interest. 8% interest
rate is being charged by the institution under similar loan to other borrowers.
h. He has received one-month salary against his house leave during Mangsir, 20X-67.
i. The employer contributes 10% of his drawn salary and the employee same amount from his salary, to approved
provident fund. Also, the employer contributes NPR 20,000 per month to Citizen Investment Fund up on request of
employee.
j. He has covered the insurance of NPR 250,000 and NPR 150,000 of his wife in Rastriya Beema Sansthan. He pays the
premium of NPR 19,000 and NPR 14,000 respectively in that Fiscal Year. Also, he has contributed to tax exempted entity
approved from department as donation amounting to NPR 60,000.
k. He has expensed out NPR 20,000 for his treatment in Teaching Hospital.
l. He and his wife declared that they are couple in the Fiscal Year 20X-67/X-68 and his wife does not have any income in
that Fiscal Year. Compute Assessable Income, Taxable Income and Tax liability of Mr. Bhai Raja for the Fiscal Year
20X-67/X-68.
(Dec 2011, 12 Marks)

© The Institute of Chartered Accountants of Nepal 225


CAP II Paper 7 Income Tax and VAT

Answer
Statement of Assessable Income from Employment
Particulars Sec. Ref. Amount Note
Monthly Remuneration 8 (2) 480,000
Dashain Allowance 8 (2) 40,000
Education Allowance 8 (2) 24,000
Expenditure Allowance 8 (2) 24,000
Bonus 8 (2) 60,000
Remuneration from Shankar Dev Campus 8 (2) 72,000
0.5% of Khai paai aaeko talab, Khai paai aaeko
talab is interpreted as sum of basic salary and garde
derived by the employee from the employer
Vehicle Facility 27 2,400 providing such facility
All other vehicle operation expenses including
driver's salary, fuel, repair, etc. is deemed to be
included in amount so quantified
Driver's Salary 27 - Refer Notes on Vehicle Facility
2% of Khai paai aaeko talab, see notes on vehicle
Accommodation Facility 27 9,600
facility for what constitute khai paai aaeko talab
Personal Telephone Expense 8 (2) 12,000
Actual cost borne by employer per month is Rs.
Security Guard Expenses 8 (2) 36,000
3,000
Loan amount multiplied by differential interest rate,
Loan at Concessional Interest Rate 27 30,000 3% is differential interest rate, Loan is assumed to
be used throughout the year
Leave Encashment 22 40,000 Cash basis amount
10% of monthly remuneration from bank, assuming
Employer's Contribution to Retirement Fund 8 (2) 48,000
Shankar Dev Campus does not provide such amount
Assessable Income from Employment 878,000

© The Institute of Chartered Accountants of Nepal 226


Compilation of Suggested Answers Income Tax

Statement of Taxable Income and Balance Taxable Income


Assessable Income from Business -
Assessable Income from Employment 878,000
Assessable Income from Investment -
Total Assessable Income 878,000
Less: Reduction under Sec. 63 -
(One-third of Assessable Income, or Actual, or Actual Contribution is Rs. 48000+ Rs. 48,000+ Rs.
292,667
Rs. 300,000; whichever is lower) 240,000
Lower of: 5% of ATI or Actual or Rs. 100,000
Less: Donation to Exempt Entity 12 29,267 ATI, in this case = Total Assessable Income minus
Eligible Contribution to ARF
Taxable Income 585,333
Less: Deduction under Schedule 1
Actual premium paid (Rs. 33,000) or Rs. 25,000;
Payment of Investment Insurance Premium 25,000
whichever is lower
Balance Taxable Income 560,333

Statement of Tax Liability


1st Rs. 450,000 1% 4,500 Couple Assessment
Next Rs. 100,000 10% 10,000
Next Rs. 10,333 20% 3,100
Tax Liability 17,600

159. Mr. Yubaraj Thapa, a bank officer, retired from his job in the year 20X-65/X-66. He received Rs. 4,50,000 as retirement
payment. The payment was made through an approved retirement fund. From the Shrawan 1, 20X-66 he is entitled to get
a pension of Rs. 21,000 per month. He also entitled to get one month's pension as dashain bonus. Mr. Yubaraj received
interest of Rs. 50,000 from a fixed deposit from Rastriya Banijya Bank. He has also received a dividend of Rs. 95,000 from
Nabil Bank. He started a consultancy business in the month of Shrawan 20X-66 and earned net profit of Rs. 25,000 in the
year 20X-66/X-67.

© The Institute of Chartered Accountants of Nepal 227


CAP II Paper 7 Income Tax and VAT

His wife Mrs. Lolita is a school teacher in a government school and getting Rs. 10,000 monthly salary. She is also operating
a boutique since last five years and the net profit and the annual sales for the year 20X-66/X-67 are Rs. 1,50,000 and Rs.
12,45,000 respectively. Mrs. Lolita owns a house located at Putalisadak and the rental income of the house is Rs. 50,000 per
month. The house was rented to a commercial bank. She also does the assignment of question paper setting and evaluation
of answers and she received Rs. 1,05,000 during the year 20X-66/X-67.

Mr. Yubaraj and his wife have not selected the option as a couple for tax returns purpose. Calculate the tax liability of Mr.
Yubaraj and Mrs. Lolita and also state their responsibility to file the income tax return for the year 20X-65/X-66 and 20X-
66/X-67. (June 2011, 12 Marks)
Answer
Calculation of Tax Liability of Mr. Yubaraj
Sec.
Particulars Amount Note
Ref.
Pension Income 8 (2) 252,000
Dashain Bonus 8 (2) 21,000
Retirement payment - Considered in Previous Income Year
Final withholding, Considering Rastriya Banijya Bank a
Interest 92 -
resident company

Dividend 92 - Final withholding, Considering Nabil Bank a resident company

Assessable Income from Employment 273,000


Statement of Taxable Income and Balance Taxable Income
Consultancy Business profit, assuming computed as per Income
Assessable Income from Business 25,000
Tax Law
Assessable Income from Employment 273,000
Assessable Income from Investment -

© The Institute of Chartered Accountants of Nepal 228


Compilation of Suggested Answers Income Tax

Total Assessable Income


298,000
(Balance/taxable income)

There is no tax liability, since the amount of balance taxable income falls within the range of basic exemption limit

Calculation of Tax Liability of Mrs. Lolita


Sec.
Particulars Note
Ref. Amount
Assessable Income from
8 (2) 120,000 Salary from School
Employment
Profit from Boutique, assuming computed as per
Income Tax Law
Assessable Income from Business 7 150,000
She cannot pay presumptive tax, as she has employment
income as well
House Rent Income - Exempt, refer definition of RENT
Income from setting question
paper and checking answer 92 - Final withholding
sheets
Total Assessable Income
270,000
(Balance/Taxable Income)

Statement of Tax Liability


1st Rs. 120,000 1% 1,200 There is no 1% tax on business income
160. Mr. Ram Singh is a salaried employee having taxable salary income amounting to Rs. 1,50,000 during the fiscal year 20X-
66/X-67. Calculate the tax liability of Mr. Ram Singh (opting as couple) as per the provisions of Income Tax Act, 2058 for
the fiscal year 20X-66/X-67. Will your answer be different if his earning was from proprietorship business not from salary?
(Dec 2009, 5 Marks)

Answer
The tax liability is 1% of Rs. 150,000 (i.e. Rs. 1,500)

© The Institute of Chartered Accountants of Nepal 229


CAP II Paper 7 Income Tax and VAT

There is no 1% tax on basic exemption limit in case of business income, investment income, pension income or when a natural
person contributes to Contribution based social security Fund or Contribution based Pension Fund.

Therefore, if it were business income, the tax would be nil.

161. What are the deductions, reduction and exemptions permitted under the Income Tax Act, 2058 (including Schedule 1) for
the income year 20X-65/X-66, in the computation of income under the head "employment”? Discuss.
(June 2009, 5 Marks, CA Inter & June 2006, 6 Marks, CA Inter)
Answer
Reductions- There are no specific deductions in respect of employment income. However, a natural person is eligible to reduction
for donation or gift to exempt organization, contribution to Prime Minister God Induced Disaster Relief Fund or National
Reconstruction Fund of Government of Nepal, and Contribution to Approved Retirement Fund while arriving at taxable income
from Total Assessable Income.

Deductions- A natural person is entitled to following deductions under Sec. 1 of Schedule 2 of the Act. Please note that this is
available for all income and not just employment income only:
a. Residence in Remote Area: The deduction is as follows, if a natural person stays at such remote area throughout the year:
Category for Remote Area Amount
A 50,000
B 40,000
C 30,000
D 20,000
E 10,000
b. Facility to an employee of GON deputed in Foreign Diplomatic Mission [Sch. 1- Sec. 1 (6)]
In case a natural person who is an employee of Government of Nepal is deputed in Diplomatic Mission of Nepal situated at
foreign country derives Allowance in the head “Foreign Allowance”, 75% of amount derived in the head “Foreign Allowance”
is deductible from taxable income.

c. Deduction to an incapacitated resident natural person [Sch. 1- Sec. 1 (10)]

© The Institute of Chartered Accountants of Nepal 230


Compilation of Suggested Answers Income Tax

In case a resident natural person is a disabled individual, 50% of Basic Exemption Limit is deductible from taxable income:

d. Deduction for payment Investment Insurance Premium (Sch. 1- Sec. 1- 12)


In case a resident natural person makes payment of investment insurance premium during any Income Year, the lower of
following amount can be deductible from taxable income:
i. Actual amount of Investment Insurance Premium paid, or
ii. Rs. 25,000

e. Deduction for payment of Health Insurance premium [Sec. 1 (16) of Schedule 1]


In case a Resident Natural Person pays Medical/health Insurance Premium to a Resident Insurance Company, the lower of the
following amount can be deducted from the taxable income of that individual:
i. Annual Medical Insurance Premium paid, or
ii. Rs. 20,000

© The Institute of Chartered Accountants of Nepal 231


CAP II Paper 7 Income Tax and VAT

162. Mr. Sudip Sharma, Manager in M/s Jeevan Nirman Bank Ltd. furnishes the following information related to Income Year
20X-63/X-64 (20X-63/4/1 to 20X-64/3/32).
Particulars Rs.
Monthly Basic Salary 55,000
Dashain Allowance 35,000
Bonus 50,000
Children Education Allowance 10,000
Medical Expenses Reimbursement 20,000
Free Meal (Lunch) 10,000
Dress Allowance 15,000

Other information:
a. He resides at the flat provided by the company for which company paid Rs. 5,000 p.m.
b. A car having market value of Rs. 2,000,000 is provided to him by the company. Petrol and driver expenses for running
such car is Rs. 300,000 for the year 20X-63/X-64 is borne by the company.
c. Company provides meal (lunch) to all staff at office, ordered from Bakery Cafe, a restaurant nearby the office at free
of cost in equal basis.
d. His approved medical expenses are Rs. 1,000.
e. He has fixed deposit of Rs. 1,000,000 on Nepal Bank Ltd. for complete year. Interest rate on deposit is 7% p.a. He has
not yet received and bank has not credited interest on his account too.
f. The Bank provides him Housing Loan of Rs. 500,000 on 20X-63/10/1 with interest of 6% p.a. Regular interest rate of
such loan to outsiders is 10% p.a..
g. Mr. Sudip has life insurance policy of Rs. 100,000 and premium Rs. 12,000 is paid by the bank.
h. Unadjusted Medical Tax Credit Rs. 850 carried forward from F.Y. 20X-61/X-62.
Calculate the Tax Payable if he opted as couple. (June 2008, 8 Marks, CA Inter)
Answer
Statement of Assessable Income from Employment

© The Institute of Chartered Accountants of Nepal 232


Compilation of Suggested Answers Income Tax

Particulars Sec. Ref. Amount Note


Monthly Basic Salary 8 (2) 660,000
Dashain Allowance 8 (2) 35,000
Bonus 8 (2) 50,000
Children Education Allowance 8 (2) 10,000
Medical expense reimbursement 8 (2) 20,000
Free Lunch 8 (3) - Provided to all employees under similar terms
Dress Allowance 8 (2) 15,000
Accommodation Facility 27 13,200 2% of Khai paai aaeko talab
0.5% of Khai paai aaeko talab, assuming car is not
gifted to him but is available for his use
Vehicle Facility 27 3,300 All other vehicle operation expenses including driver's
salary, fuel, repair, etc. is deemed to be included in
amount so quantified
Cash basis, as such, the amount is not income as it is
not paid
Interest on Fixed Deposit 22 -
Even if it were income, it would have been final
withholding payment
Loan amount multiplied by differential interest rate, 4%
Loan at Concessional Interest Rate 27 10,000
is differential interest rate, Loan used for six months
Assessable Income from Employment 816,500

Statement of Taxable Income and Balance Taxable Income


Assessable Income from Business -
Assessable Income from Employment 816,500
Assessable Income from Investment -
Total Assessable Income 816,500
Less: Reduction under Sec. 63 -
Taxable Income 816,500
Less: Deduction under Schedule 1

© The Institute of Chartered Accountants of Nepal 233


CAP II Paper 7 Income Tax and VAT

Actual premium paid (Rs. 12,000) or Rs. 25,000;


Payment of Investment Insurance Premium 12,000
whichever is lower
Balance Taxable Income 804,500

Statement of Tax Liability


1st Rs. 450,000 1% 4,500 Couple Assessment
Next Rs. 100,000 10% 10,000
Next Rs. 200,000 20% 40,000
Balance Rs. 54,500 30% 16,350
Tax Liability before Tax Credit 70,850
Lower of:
a. 15% of Approved Medical Expense+ Carried over
Less: Medical Tax Credit 750 from Previous Year
i.e. 15% of Rs. 1,000+850= Rs. 1,000
b. Rs. 750
Tax Liability for the Year 70,100

163. Mr. Neelmani Shakya, who is going to retire from 1st Shrawan 20X-65, submits the following details with respect to his
employment for the income year 20X-64/X-65.
Net Salary received Rs. 2,44,800
Later on, TDS Rs. 43,200 reimbursed by employer
Contribution by employer to retirement fund (approved) Rs. 80,000
His Contribution Rs. 50,000
Compensation received Rs. 40,000
Life Insurance Premium paid by Employer on his behalf, Rs. 20,000
Insured sum of Rs. 2,00,000
Encashment of un-availed leave upto Chaitra 18, 2058 Rs. 50000, after Chaitra 18, 2058 Rs. 70,000
Emergency Medical Treatment paid by Employer Rs. 10,000. But he didn’t claim.
Reimbursement of Tour Expenses during Official visit to Germany Rs. 2,40,000

© The Institute of Chartered Accountants of Nepal 234


Compilation of Suggested Answers Income Tax

Gratuity received
Up to 18 Chaitra, 2058 Rs. 5,00,000
After 18 Chaitra, 2058 Rs. 6,00,000 (approved)
Provident Fund received
Up to Chaitra 18, 2058 Rs. 4,00,000 (unapproved)
After Chaitra 18, 2058 Rs. 5,00,000 (approved)
He also provides following further information not related to his employment:
a. Gain Rs. 1,00,000 on Sale of non-business chargeable asset.
b. Donation to a work fully allowed for deduction Rs. 50,000.
c. Interest of Rs. 20,000 from deposit account from Everest Bank.
Mrs. Shakya works in a school and the couple has elected to assess the income separately.
Required: (Dec 2007, 15 Marks, CA Inter)
a. Assessable Income from Employment
b. Total Assessable Income
c. Tax Payable
Answer
Statement of Assessable Income from Employment
Sec.
Particulars Note
Ref. Amount
Net Salary Received 8 (2) 244,800
TDS Reimbursed 8 (2) 43,200 Reimbursement of Personal expense
Employer's Contribution to Retirement
8 (2) 80,000
Fund
Compensation received 8 (2) 40,000 Assuming related to employment
Leave Encashment 92 - Final withholding, assuming it is paid at the time of retirement
Emergency Medical Treatment paid by
8 (2) 10,000
employer
Reimbursement of Tour Expenses 8 (3) - Serves proper business purpose of employer

© The Institute of Chartered Accountants of Nepal 235


CAP II Paper 7 Income Tax and VAT

Gratuity Received 92 - Retirement payment, final withholding


Provident Fund Received 92 - Retirement payment, final withholding
Interest on Deposit 92 - Final withholding payment, assuming Everest Bank is resident
Assessable Income from Employment 418,000

Statement of Taxable Income and Balance Taxable Income


Assessable Income from Business -
Assessable Income from Employment 418,000
Assessable Income from Investment 100,000
Total Assessable Income 518,000
Lower of:
a. one-third of total assessable income,
Less: Reduction under Sec. 63 130,000
b. Actual Contribution to ARF- Rs. 130,000
c. Maximum Rs. 300,000
Lower of:
a. 5% of ATI, ATI = 518,000- 130,000
Less: Reduction under Sec. 12 19,400
b. Actual Donation to Exempt Entity- Rs. 50,000
c. Maximum Rs. 100,000
Taxable Income 368,600
Less: Deduction under Schedule 1
Payment of Investment Insurance
20,000 Actual premium paid (Rs. 20,000) or Rs. 25,000; whichever is lower
Premium
Balance Taxable Income 348,600

Statement of Tax Liability


1st Rs. 348,600 1% 3,486 Assessable income from employment is more than Rs. 348,600

164. Calculate taxable income and tax amount for the financial year 20X-61/X-62 of Mrs. X, a widow, with no children,
employed by a bank. Explain the tax implication on the followings: (June 2006, 8 Marks, CA Inter)

© The Institute of Chartered Accountants of Nepal 236


Compilation of Suggested Answers Income Tax

a. Provident fund as on the date 18 Chaitra 2058 was Rs. 500 thousand, which was approved retirement fund and total
provident amount of Rs. 1.2 million is received at her retirement on Jestha 30, 20X-62.
b. She gets Rs. 400 thousand from unapproved retirement fund in which her contribution was 300 thousand.
c. She also gets 100 thousand in total in total as medical benefits related F.Y. 20X-55/X-56.
d. She received 10 years pension amount Rs, one million in advances. She has no other income.
e. She also received Rs. 5,000 as widow allowances from government.
Answer
Particulars Tax Implication
Provident fund as on the date 18 Chaitra 2058 Provident fund as on the date 18 Chaitra 2058 was Rs. 500,000 is
was Rs. 500 thousand, which was approved exempt
retirement fund and total provident amount of Out of remaining Rs. 700,000; gain is taxable @5%
Rs. 1.2 million is received at her retirement on Gain= 700,000 - Higher of (500,000 or 50% of 700,000)= 200,000
Jestha 30, 20X-62 Tax = 10,000 and it is final withholding
She gets Rs. 400 thousand from unapproved
Gain of Rs. 100,000 is taxable @ 5%
retirement fund in which her contribution was
Tax = Rs. 5,000 and it is final withholding
300 thousand
She also gets 100 thousand in total in total as Medical Benefit up to Rs. 180,000 accrued until 2058 Chaitra 18 is
medical benefits related F.Y. 20X-55/X-56 exempt, as such, this is exempt
She received 10 years pension amount Rs. one The pension income is taxable as income from employment during the
million in advances. She has no other income year of cash receipt. The tax liability is computed in Rs. 10 Lakhs
She also received Rs. 5,000 as widow
It is exempt as per Sec. 10 of the Act
allowances from government

165. What are the types of incomes that are included under the head of Income from Employment under Sec.8 of the Income
Tax Act, 2058? (June 2005, 8 Marks, CA Inter)
Answer
The amounts to be included are as follows:
a. Payment of Salary, Wages, Leave Encashment, overtime pay, fees, commission, prizes, gifts, bonus and other benefits

© The Institute of Chartered Accountants of Nepal 237


CAP II Paper 7 Income Tax and VAT

b. Payment of Personal Allowances including Dearness allowance, subsistence allowance, rent, entertainment or Transportation
allowance
c. Payments providing any discharge or reimbursement of costs incurred by the individual or an associate of the individual
d. Payments for the individual's agreement to any conditions of the employments
e. Payments for redundancy or loss or termination from employment
f. Retirement Payments & Employer’s Contribution to the Retirement Fund
g. Other payments in respect of employment
h. Payments to be included as per Chapter 6 or 7 of the Act
This includes: Reverse of Amounts including Bad debt, Quantification of Perquisites, Indirect Payments, Amount of
Compensation in relation to Employment, Amount to be included as per the order of IRD in case where income splitting
arrangement is proved

166. Mr. S. Rana received the following amounts during the income year 20X-60/X-61. Calculate his taxable income explaining
with reasons for including or for not including any particular item in the taxable income.
(June 2005, 15 Marks, CA Inter)
Salary Rs.25, 000 per month
Dashain Allowance Rs.15, 000
Dearness Allowance Rs.10, 000 per month
Employer’s Contribution to approved Provident Fund 10% of salary
a. He contributed similar amount to Provident Fund.
b. He retired from service from 1st Mangsir 20X-60. He received a pension of Rs. 5,000/- per month from the date of his
retirement. He joined service on 1st Mangsir, 2020. He also received gratuity of Rs. 600,000 from the employer.
c. He was allowed free accommodation by the employer, who allowed him to stay there till Ashad 20X-61.
d. He received his Provident Fund amount of Rs. 800,000 from the approved Provident Fund on 1st Poush 20X-60. The
balance of Provident Fund amount in his account as on 19th chaitra 2058 was Rs. 600,000.
e. He was also paid a sum of Rs. 10,000 by the Managing Director as his personal token in appreciation of his sincere
services to the employer.
f. He received a sum of Rs. 50,000 as unutilized leave salary as per the rules on 1st Magh, 20X-60.

© The Institute of Chartered Accountants of Nepal 238


Compilation of Suggested Answers Income Tax

g. He was provided one car for both official and personal purpose. Employee allowed him to retain the car with him till
end of Ashad 20X-61.
h. During Shrawan 20X-60, he was admitted to a private hospital for a week and the employer spent Rs.75, 000 on his
medical treatment to the hospital.
i. Other employees presented him with a silver casket engraving costing Rs. 25,000
j. He received a dividend of Rs. 95,000 from various companies.

Answer
Assessable Income from Employment
Sec. Reason for inclusion or non
Particulars Amount Note
Ref inclusion
Salary 8 (2) 100,000 For four months from Shrawan to Kartik Included as per Sec. 8 (2)
Dashain Allowance 8 (2) 15,000 Included as per Sec. 8 (2)
Dearness Allowance 8 (2) 40,000 For four months from Shrawan to Kartik Included as per Sec. 8 (2)
Employer's Contribution to RF 8 (2) 10,000 10% of salary Included as per Sec. 8 (2)
Pension Income 8 (2) 40,000 For eight months from Mangsir to Ashad Included as per Sec. 8 (2)
Final withholding, it is retirement
Gratuity 92 - Not included u/s 8 (3)
payment
There is no clear provision for
quantification of accommodation facility,
when the same person uses the facility in
two different capacity. Therefore, 2% of
khai paai aaeko talab has been taken as
Accommodation Facility 27 2,000 Included as per Sec. 8 (2)
benefit
Ideally, the sum of 0.5% of khai paai
aaeko talab and 25% of Market or actual
rent for eight months would be
appropriate for quantification

© The Institute of Chartered Accountants of Nepal 239


CAP II Paper 7 Income Tax and VAT

Payment from Approved Retirement


Fund
Until Chaitra 18, 2058 - Tax Exempt Not included u/s 8 (3)
Final withholding, it is retirement
After Chaitra 18, 2058 92 - Not included u/s 8 (3)
payment
it is not employment, business or
Gift from Managing Director - investment income Not included u/s 5 and 6
Can be treated as windfall gain income
Final withholding, it is retirement
Sum of unutilized leave 92 - Not included u/s 8 (3)
payment
There is no clear provision for
quantification of vehicle facility, when the
same person uses the facility in two
different capacity. Therefore, 0.5% of khai
paai aaeko talab has been taken as
Vehicle Facility 27 500 benefit, in absence of information Included as per Sec. 8 (2)
regarding market value of vehicle
Ideally, the sum of 0.5% of khai paai
aaeko talab and 1% of Market value of
vehicle for eight months would be
appropriate for quantification
Medical Treatment Reimbursement 8 (2) 75,000 Reimbursement of personal expenses Included as per Sec. 8 (2)
it is not employment, business or
Gift from Colleague - investment income Not included u/s 5 and 6
Can be treated as windfall gain income
Final withholding, assuming all
Dividend Income 92 -
companies are resident
Assessable Income from
No any income from business or
Employment 282,500
investment
(Total Assessable Income)
Less:

© The Institute of Chartered Accountants of Nepal 240


Compilation of Suggested Answers Income Tax

Lower of:
a. one-third of total assessable income,
Contribution to ARF 63 20,000
b. Actual Contribution to ARF- Rs. 20,000
c. Maximum Rs. 300,000
Taxable Income or Balance
262,500
Taxable Income

167. Mr. is employed in an undertaking and gets the following remuneration:


Basic Salary Rs. 36,000
Dearness Allowance 50% of salary
House Rent Allowance Rs. 10,000

Employer takes a house on rent for Mr. D paying a rent of Rs. 20,000/- per month. Out of which he recovers Rs. 10,000
from Mr. D. The employer pays the electricity charges, phone bills and water charges. Personal phone calls are recovered
from Mr. D. Mr. D has taken a Life Insurance Policy for Rs. 5 lakhs and pays a premium at the rate of Rs. 1,250 p.m. He
is entitled to Provident Fund calculated at 10% of his salary and a similar amount is contributed by the employer and paid
into the Citizen Investment Trust. In addition, the employer deposits Rs. 30,000 towards gratuity to the Citizen Investment
Trust. Mr. D owns a car. But the car maintenance expenses are paid by the Company.

Mr. D's wife is employed as a teacher drawing a salary of Rs. 10,000 p.m. She is also entitled to 10% Provident Fund which
is deposited with the Employee Provident Fund. She has also taken an Insurance Policy for Rs. 5,00,000 paying Rs. 12,000
premium per annum.

The school bus takes her to the school in the morning and drops at home in the evening.

The school provides free transport in the school bus. The school collects Rs. 500 per month from the students for the bus
transport.

Advise whether they should submit separate returns and work-out the tax payable. (June 2004, 10 Marks, CA Inter)

© The Institute of Chartered Accountants of Nepal 241


CAP II Paper 7 Income Tax and VAT

Answer:
Assessable Income of Mr. D from employment
Sec.
Particulars Note
Ref Amount
Basic Salary 8 (2) 36,000 Assumed for the whole year
Dearness Allowance 8 (2) 18,000
House Rent Allowance 8 (2) 10,000
Accommodation Facility 27 720 2% of Khai paai aaeko talab
Electricity Charges 27 Not quantifiable due to missing information
Water charges 27 Not quantifiable due to missing information
Telephone charges - Expense for Personal call not reimbursed
Employers' Contribution to PF 8 (2) 3,600 Assuming salary includes basic salary only
Employers' Contribution to PF 8 (2) 30,000 Gratuity deposited in personal account of employee
Car maintenance expense 27 Not quantifiable due to missing information
Assessable Income from
98,320
employment
Less:
Lower of:
a. one-third of total assessable income,
Contribution to Approved RF 63 32,773
b. Actual Contribution to ARF- Rs. 37,200
c. Maximum Rs. 300,000
Taxable Income 65,547
Less:
Payment of Investment Insurance
Sch. 1 15,000 Lower of actual premium of Rs. 25,000
Premium
Balance Taxable Income 50,547

Assessable Income of Mrs. D from employment

© The Institute of Chartered Accountants of Nepal 242


Compilation of Suggested Answers Income Tax

Particulars Sec. Ref Amount Note


Salary 8 (2) 120,000
Employers' Contrbution to PF 8 (2) 12,000
Not included in income of Mrs. D, but the employer is not
Use of School bus 21 -
entitled to claim deduction of such expense
Assessable Income from employment 132,000
Less:
Lower of:
a. one-third of total assessable income,
Contribution to ARF 63 44,000
b. Actual Contribution to ARF- Rs. 130,000
c. Maximum Rs. 300,000
Taxable Income 88,000
Less:
Payment of Investment Insurance Premium Sch. 1 12,000 Lower of actual premium of Rs. 25,000
Balance Taxable Income 76,000

Tax Liability 760 SST Only

Assuming all conditions of Sec. 4 (3) are satisfied <students to quote the provisions for examination purpose, assumed they have
already memorized until this problem is to be solved>, neither of Mr. D and Mrs. D are required to submit income returns.

168. Mr. Ram, a chartered accountant, is working as a finance manager in a company. The company pays him Rs. 50,000 per
month as salary and Rs. 5,000 as helper allowance of which he actually pays Rs. 4,500 to the helper for his duties.
He has made an application for 3 months advance salary in the month of Falgun 20X-59 and the company approved and
paid him 2 months advance salary (excluding allowances) to be deducted from his salary within 6 months starting from 1st
Shrawan, 20X-60.
The company has provided a quarter to Mr. Ram. The company has also provided a car facility for private use of Mr.
Ram.

© The Institute of Chartered Accountants of Nepal 243


CAP II Paper 7 Income Tax and VAT

Mr. Ram has paid Rs. 10,500 for his medical expenses during the year. He has contributed Rs. 35,000 to various institutions
which include Rs. 25,000 given to a Public charitable institution approved as exempt organization by the Inland Revenue
Department.
Mr. Ram has received Rs. 17,000 (net of TDS) as meeting fee during the year. Due to the poor performance of the company,
it faces severe problem and did not pay any salary to its staff in the month of Baisakh 20X-60.
Workout the taxable income from remuneration and the Tax liability of Mr. Ram for the financial year 20X-59/X-60,
assuming he has no other income. (Dec 2003, 6 Marks, CA Inter)

Answer
Assessable Income from Employment
Sec.
Particulars Amount Note
Ref
Cash basis, as such, amount not received for the month of Baisakh is not
Salary 8 (2) 540,000 included in income. It will be included when there is inflow of cash to
Mr. Ram

Rs. 4,500 is reimbursement of cost of employer, i.e. Rs. 4,500 serves


Helper Allowance 8 (2) 5,500 proper business purpose of employer and remaining Rs. 500 per month
is taxable; 11 months as no payment for the month of Baisakh

It is not salary income for Mr. Ram. It is actually a loan and a loan is
Advance Salary -
not an income. Since it is not an income, it is not included in income

Accommodation Facility 27 10,800 2% of khai paai aaeko talan


Vehicle Facility 27 2,700 0.5% of khai paai aaeko talan

Meeting Fee 92 - Final withholding, assuming per meeting fee is less than Rs. 20,000

Dividend Income 92 -

© The Institute of Chartered Accountants of Nepal 244


Compilation of Suggested Answers Income Tax

Assessable Income from


Employment 559,000 No any income from business or investment
(Total Assessable Income)
Less:
Lower of:
Donation to Exempt a. 5% of ATI
63 25,000
Organization b. Actual - Rs. 25,000
c. Maximum Rs. 100,000
Taxable Income 534,000

Tax Liability
1st Rs. 400,000 1% 4,000 Assuming Individual Assessment
Next Rs. 100,000 10% 10,000
Balance Rs. 34,000 20% 6,800
Tax Liability before tax credit 20,800
Min of:
a. 15% of Approved Medical expense- 15% of Rs. 10,500
Les: Medical tax Credit 750
(assuming Rs. 10,500 is approved)
b. Max. Rs. 750
Tax Liability for the year 20,050

169. Mr. Ram is a retired Civil Servant and Pension holder. His wife Mrs. Mohani is a housewife. Ram has received pension
Rs. 10,500 per month for 13 months including Dashain allowance. Calculated the tax liability for the income year 20X-
59/X-60 while Mr. Ram is residing in class (D) location. (June 2003, 6 Marks, CA Inter)
Answer
The total income without considering deduction for having stayed at “D” Category Remote Area is less than the basic exemption
limit. As such, there is no tax obligation of Mr. Ram, in both the assessment whether it is couple assessment or individual
assessment.
Total Balance Taxable Income= Rs. 136,500 minus Rs. 20,000= Rs. 116,500

© The Institute of Chartered Accountants of Nepal 245


CAP II Paper 7 Income Tax and VAT

© The Institute of Chartered Accountants of Nepal 246


Compilation of Suggested Answers Income Tax

Chapter 9: Income from Investment


170. Ms. Anjali Amatya, submitted the following details of income for Income Year 20X-75/X-76.
Particulars Amount
1. House Rental income let out to XYZ Ltd. (Net Rent) (Rs.)
342,000
2. Interest Income from Development Bank in Nepal (Net 85,000
Interest)
3. Interest Income from friends (Gross) 35,000
4. Furniture rental income from Mr. Paras (Gross) 850,000
5. Windfall gain withholding tax not deducted (Gross) 100,000
6. Gift received related to investment (Gross) 225,000
7. Income from natural resources (Net) 8,500
8. Gain recovered from investment insurance (Net) 85,000
9. Recovery of bad debt 200,000
10. Income from sale of listed shares (Net) 277,500

Ms. Amatya submitted the following details of expenses:


Particulars Amount (Rs.)
1. Repairing expenses of house property let out to XYZ Ltd. (Rs.)40,000
(Rs.)
2. Interest expenses paid to borrowed funds to give loan to 9,000
friends depreciation for furniture let out to Mr. Paras
3. Allowable 35,000
4. Interest expenses for borrowed to pay insurance premium 6,500
5. Life insurance premium paid 19,000
6. Expenses paid related to natural resources 9,000
7. Bad debts recovered was not allowed for deduction earlier 80,000
8. Contribution to approved retirement funds 90,000
Your assistance is required to compute her taxable income and tax liability for Income Year 20X-75/X-76.
(June 2019, 10 Marks)

© The Institute of Chartered Accountants of Nepal 247


CAP II Paper 7 Income Tax and VAT

Answer
Assessable Income from Investment
Sec.
Particulars Amount Note
Ref.
Inclusions
As per definition of "Rent", natural person's house rent income
that is not generated through a private firm does not fall under
House Rental Income -
the definition of rent, which means such income is exempt from
income tax
Final Withholding, assuming not generated by private firm of Ms.
Interest income from bank 92 -
Anjali
Interest income from Friend 9 (2) 35,000
Furniture rental income 9 (2) 850,000
Windfall Gain 92 - Final Withholding
Gift received 9 (2) 225,000
Income from natural resource 9 (2) 10,000 Grossed up using TDS rate of 15% [8,500 divided by 0.85]
Included only to such extent that was claimed earlier, assumed
Recovery of bad debt 25 120,000
bad debt was related to investment
Grossed up using advance tax rate of 5% [277,500 divided by
Income from sale of listed shares 9 (2) 292,105
0.95]
Total Inclusions 1,532,105
Less: Deductions
Repair expense of House
21 - As income is exempt, expense is also not deductible
property
Interest Expense 14 9,000
Allowable Depreciation 19 35,000
Interest expense borrowed to Assuming borrowed to pay investment insurance premium, which
21 -
pay insurance premium is deductible when calculation gain from investment insurance

© The Institute of Chartered Accountants of Nepal 248


Compilation of Suggested Answers Income Tax

Expense related to natural resources 13 9,000

Bad debt recovered not allowed A mere information to determine, how much recovery of bad debt
earlier is included in income
Total Deductions 53,000
Assessable Income from
1,479,105
Investment
Less:-
Lower of:
a. one-third of total assessable income,
Contribution to ARF 63 90,000
b. Actual Contribution to ARF- Rs. 90,000
c. Maximum Rs. 300,000
Taxable Income 1,389,105
Less: Payment of Life insurance Sch.
19,000 Lower of actual premium of Rs. 25,000
premium 1
Balance Taxable Income 1,370,105
Balance Taxable Income without
including gain on disposal of 1,078,000
NBCA

Computation of Tax Liability


1st Rs. 400,000 0% -
Next Rs. 100,000 10% 10,000
Next Rs. 200,000 20% 40,000
Next Rs. 378,000 30% 113,400
Balance Rs. 292,105 5% 14,605
Total Tax Liability 178,005

© The Institute of Chartered Accountants of Nepal 249


CAP II Paper 7 Income Tax and VAT

171. Mrs. Kriti is an employee in Nepal Bank Ltd. Besides employment income, she has the following transactions for Income
Year 20X-73/X-74. Sort out the income from investment and tax liability for the year assuming that 0% and 15 10% tax
slabs are already over by income from employment. (June 2018, 10 Marks)
S.
Particulars Amount (Rs)
No.
1 Net gain on disposal of listed shares 100,000.00
2 Purchase of New listed shares 600,000.00
3 Income from Taxi 400,000.00
4 Gain on sale of gold 100,000.00
5 Interest from savings deposit from 'A' class bank 4,000.00
6 Interest from fixed deposit from 'A' class bank 40,000.00
Net gain from disposal of ancestral house (owned and
7 resided since Bikram Sambat 20X-60) 5,000,000.00
8 Royalty from music video 100,000.00
9 House rent 300,000.00
10 Shares transaction commission expenses 5,000.00
11 Salary to taxi driver and other taxi related expenses 200,000.00
12 Music video expenses 20,000.00
13 House repair and maintenance expenses 20,000.00

Answer
Assessable Income from Investment
Sec.
Particulars Amount Note
Ref.
Inclusions
Net Gain on Disposal of Listed Shares 9 (2) 100,000 Income is booked at gross amount
Outgoings of shares yet to be sold, will be considered at the
Purchase of new listed share -
time of disposal of new shares

© The Institute of Chartered Accountants of Nepal 250


Compilation of Suggested Answers Income Tax

It is assumed that the person's business is not in gold and gold


Gain on Sale of Gold - is kept for personal purpose, thus the gain is not taxable since it
is not business, employment or investment income

Interest from Saving Deposit (A Class


92 - Final withholding
Bank)
Interest from Fixed Deposit (A Class
92 - Final withholding
Bank)

Since the house is owned continuously for more than 10 years,


and assuming that the person has resided there for 10 years or
Net gain on disposal of ancestral more; the asset does not fall under the category of Non Business
-
house Chargeable asset.
Since the asset does not fall under any category of asset as
defined by Income Tax Act, the gain is exempt from income tax

Royalty from Music Video 9 (2) 100,000


As per definition of "Rent", natural person's house rent income
that is not generated through a private firm does not fall under
House Rent -
the definition of rent, which means such income is exempt from
income tax
It is assumed that it has already been considered while
Share Transaction Commission
- determining net gain on disposal of listed shares, or it is the
Expense
outgoings for purchase of new listed shares
Total Inclusions 200,000
Less: Deductions
Music Video related expense 13 20,000
House repair and maintenance
21 - As income is exempt, expense is also not deductible
expense
Total Deductions 20,000
Assessable Income from Investment 180,000

© The Institute of Chartered Accountants of Nepal 251


CAP II Paper 7 Income Tax and VAT

Assessable Income from Business


Sec.
Particulars Amount Note
Ref.
Income from Taxi is no more presumptive taxation, and it is a
Income from Taxi 7 (2) 400,000
business income
Salary to Taxi Driver and other taxi
13 200,000
related expenses
Assessable Income from Business 200,000

Total Taxable income subject to tax rate above 10%


Assessable Income from Investment 180,000
Assessable Income from Business 200,000
Less: Income subject to less tax rate (100,000) Gain on disposal of listed shares
Income subject to 20%, 30% or 36%
280,000
tax rate

Computation of Tax Liability


Rs. 200,000 20% 40,000
Rs. 80,000 30% 24,000
Balance Rs. 100,000 5% 5,000
Total remaining Tax Liability 69,000

172. Mr. Dinesh Maharjan, an investor, submitted the following details of income for assessment of his income tax liability for
the assessment year 20X-72/X-73. (Dec 2016, 10 marks)
Particulars Amount in (Rs.)
1. House Rental income let out to ABC Ltd. (Net Rent) 324,000
2. Interest Income from Commercial Bank in Nepal (Net 95,000
Interest)

© The Institute of Chartered Accountants of Nepal 252


Compilation of Suggested Answers Income Tax

3. Interest Income from his friends (Gross) 25,000


4. Furniture rental income from Mr. Krishna (Gross) 900,000
5. Windfall gain withholding tax not deducted (Gross) 200,000
6. Cash dividend (Gross) from investment in shares of ABC 300,000
commercial Bank, the bank was merged with another bank
18 month ahead with the approval of NRB and the
investment was made by Mr. Maharjan before merger.
7. Income from sale of bonus shares (Gross) received from 250,000
investment in shares of DEF commercial Bank, the bank was
merged with another bank 23 month ahead with the approval
of NRB and the investment was made by Mr. Maharjan
before merger.
8. Gift received related to Investment (Gross) 150,000
9. Income from Natural resources (Net) 8,500
10. Gain recovered from Investment Insurance (Net) 95,000
11. Bad debt recovered 100,000
12. Income from sale of listed shares (Net) 190,000

Mr. Maharjan, who is also a married guy, submitted following details of expenses to you for assessment of taxable income,
tax liability & net tax payable for the financial year 20X-72/X-73:
Particulars Amount in (Rs.)
1. Repairing expenses of house property let out to ABC Ltd. 35,000
2. Interest exp. paid to borrowed funds to give loan to friends 10,000
3. Allowable depreciation for furniture’s let out to Mr. Krishna 25,000
4. Interest expenses for borrowed to pay insurance premium 7,500
5. Life Insurance premium paid 18,000

© The Institute of Chartered Accountants of Nepal 253


CAP II Paper 7 Income Tax and VAT

6. Expenses paid related to natural resources 10,000


7. Bad debts recovered was not allowed for deduction earlier 50,000
8. Contribution to retirement funds 100,000

Your assistance is required to compute his taxable income & tax liability for financial year 20X-72/X-73.
Answer
Assessable Income from Investment
Sec.
Particulars Amount Note
Ref.
Inclusions
As per definition of "Rent", natural person's house rent
income that is not generated through a private firm does not
House Rental Income -
fall under the definition of rent, which means such income is
exempt from income tax
Final Withholding, assuming not generated by private firm of
Interest income from bank 92 -
Ms. Anjali
Interest income from Friend 9 (2) 25,000
Furniture rental income 9 (2) 900,000
Windfall Gain 92 - Final Withholding
Cash dividend from a bank that Exempt for two years from the date of merger
47Ka -
was merged 18 months before Out of syllabus for CAP II students
Gain on disposal of shares from a
bank that was merged 23 months Exempt for two years from the date of merger
47Ka -
before, shares held on the date of Out of syllabus for CAP II students
merger
Gift received 9 (2) 150,000
Income from natural resource 9 (2) 10,000 Grossed up using TDS rate of 15% [8,500 divided by 0.85]
Gain from Investment Insurance 92 - Final withholding

© The Institute of Chartered Accountants of Nepal 254


Compilation of Suggested Answers Income Tax

Included only to such extent that was claimed earlier,


Recovery of bad debt 25 50,000
assumed bad debt was related to investment
Grossed up using advance tax rate of 5% [190,000 divided by
Income from sale of listed shares 9 (2) 200,000
0.95]
Total Inclusions 1,335,000
Less: Deductions

Repair expense of House property 21 - As income is exempt, expense is also not deductible

Interest Expense 14 10,000


Allowable Depreciation 19 25,000
Assuming borrowed to pay investment insurance premium,
Interest expense borrowed to pay
21 - which is deductible when calculation gain from investment
insurance premium
insurance
Expense related to natural
13 10,000
resources
Bad debt recovered not allowed A mere information to determine, how much recovery of bad
earlier debt is included in income
Total Deductions 45,000
Assessable Income from Investment 1,290,000
Less:
Lower of:
a. one-third of total assessable income,
Contribution to ARF 63 100,000
b. Actual Contribution to ARF- Rs. 100,000
c. Maximum Rs. 300,000
Taxable Income 1,190,000
Less:
Sch.
Payment of Life insurance premium 18,000 Lower of actual premium of Rs. 25,000
1
Balance Taxable Income 1,172,000

© The Institute of Chartered Accountants of Nepal 255


CAP II Paper 7 Income Tax and VAT

Balance Taxable Income without


972,000
including gain on disposal of NBCA

Computation of Tax Liability


Assuming individual assessment
1st Rs. 400,000 0% -
Married does not mean compulsory couple assessment
Next Rs. 100,000 10% 10,000
Next Rs. 200,000 20% 40,000
Next Rs. 490,000 30% 147,000
Balance Rs. 200,000 5% 10,000
Total Tax Liability 207,000

173. Mr. Oli, a disable person and is residing in Manang District, disclosed his following income and expenditure for the income
year 20X-71/X-72
Income side:
a. Interest on private money lending activities Rs. 127,500 (after TDS)
b. Royalty from natural resources Rs. 306,000 (after TDS)
c. Interest from bank deposit Rs. 190,000
d. Dividend from domestic companies Rs. 95,000
e. Royalty from writing article Rs. 15,000
f. Royalty from book publication Rs. 170,000 (after TDS)
g. Compensation received Rs. 50,000
h. Money found lying in street Rs. 7,500 (net)

Expense side:

a. Interest on private money lending Rs. 2,000

© The Institute of Chartered Accountants of Nepal 256


Compilation of Suggested Answers Income Tax

b. Natural resources Rs. 1,000


c. Royalty from writing article Rs. 100
d. Dividend from domestic companies Rs. 500
e. Legal expenses (income tax appeal) Rs. 5,000
f. Salary to assistant Rs. 30,000
g. Life insurance premium Rs. 15,000 on his own policy amount Rs. 300,000
h. Contribution to approved retirement fund Rs. 50,000
i. Donation to tax exempt organization Rs. 12,000

Compute the tax liability and net tax payable of Mr. Oli for the income year 20X-71/X-72. (Assume him as couple for
calculation) (June 2016, 10 marks)
Answer
Assessable Income from Investment
Sec.
Particulars Note
Ref. Amount
Inclusions
Interest on Private Money Lending
9 (2) 150,000 Grossed up, 127,500 divided by 0.85
Activity
Royalty from Natural Resources 9 (2) 360,000 Grossed up, 306,000 divided by 0.85
Interest from Bank deposit 92 - Final withholding, assuming bank is resident of Nepal
Dividend from resident company 92 - Final withholding
Royalty from book publication 9 (2) 200,000 Grossed up, 170,000 divided by 0.85
Compensation received 31 50,000 Assumed related to investment
As per contract law, it must be produced before law enforcement
authority
Money found lying in street -
If we assume it as income, it is windfall gain and is final
withholding
Total Inclusions 760,000
Deductions

© The Institute of Chartered Accountants of Nepal 257


CAP II Paper 7 Income Tax and VAT

Interest on Private Money Lending


14 2,000
Activity
Natural resources 13 1,000
Dividend from resident company 21 - Collection fee for final withholding payments
Legal expenses 13 5,000 Appeal is legal right
Salary to Assistant 13 30,000 Assuming all incurred for income generating activities
Total Deductions 38,000
Assessable Income from Investment 722,000

Assessable Income from Business


Royalty from writing article 7 (2) 15,000 Service fee
Less: Writing article 13 (100)
Assessable Income from Business 14,900

Total Assessable Income, Taxable Income and Balance Taxable Income


Assessable Income from Investment 9 722,000
Assessable Income from Business 7 14,900
Total Assessable Income 736,900
Less:
Lower of:
a. one-third of total assessable income,
Contribution to ARF 63 50,000
b. Actual Contribution to ARF- Rs. 50,000
c. Maximum Rs. 300,000
Lower of:
a. 5% of ATI, i.e. 5% of Rs. 686,900
Donation 12 12,000
b. Actual Donation to Exempt Entity- Rs. 12,000
c. Maximum Rs. 100,000
Taxable Income 674,900
Less:

© The Institute of Chartered Accountants of Nepal 258


Compilation of Suggested Answers Income Tax

Payment of Life insurance premium Sch. 1 15,000 Lower of actual premium of Rs. 25,000
Balance Taxable Income 659,900

Calculation of Tax Liability


1st Rs. 400,000 0% -
Next Rs. 100,000 10% 10,000
Balance Rs. 159,900 20% 31,980
Tax Liability 41,980

174. The investment income and expenses related to Mr. Narayan for the financial year 20X-71/X-72 are as follows:
Particulars: Amount (Rs.)
A. Income:
House rent income net of TDS 180,000
Bank interest income net of TDS- Nepal Bank Ltd. 190,000
Natural resources payments net of TDS 170,000
Interest income net of TDS from ABC Ltd. 2,125,000
Compensation received from loss of last year investment 25,000
Income from investment insurance net of TDS- Rastriya Beema Sanstha 95,000
Gift related to investment income 50,000
Dividend income net of TDS from Sanima Bank Ltd. 47,500
B. Expenses:
Expenses related to collection of house rent 4,000
Expenses related to natural resources 8,500
Allowable depreciation allowance as per the Act 5,500
Life insurance premium paid to Rastriya Beema Sanstha 22,500
Donation paid to tax exempt organization 40,000

Required: (July 2015, 8+2=10)

© The Institute of Chartered Accountants of Nepal 259


CAP II Paper 7 Income Tax and VAT

a. Compute total Taxable income from investment.


b. Compute net Tax liability for the financial year 20X-71/X-72, assuming Mr. Narayan is a married and had no other
income.
Answer
Assessable Income from Investment
Particulars Sec. Ref. Amount Note
Inclusions
As per definition of "Rent", natural person's house rent income
that is not generated through a private firm does not fall under
House Rent -
the definition of rent, which means such income is exempt from
income tax
Final withholding, assuming not related to business and the
Interest from Banks 92 -
bank is resident of Nepal
Natural Resource Payment 9 (2) 200,000 Grossed up, [=170,000/0.85]
Grossed up, [=2,125,000/0.85] assuming ABC Ltd. is not a
listed company or entity authorized to issue debenture/bond
Interest from ABC Ltd. 9 (2) 2,500,000 Alternatively, if we assume that ABC Ltd. is a listed company or
entity authorized to issue debenture/bond, interest is Final
withholding
Compensation received 31 25,000 Cash basis as per Sec. 31
Final withholding, assuming insurance company is resident of
Income from Investment Insurance 92 -
Nepal
Gift related to investment 9 (2) 50,000
Final withholding, assuming Sanima Bank Ltd. is resident of
Dividend 92 -
Nepal
Total Inclusions 2,775,000
Deductions
House Rent Collection expense 21 As income is exempt, expense is also not deductible
Natural Resource Expense 13 8,500
Allowable Depreciation 19 5,500

© The Institute of Chartered Accountants of Nepal 260


Compilation of Suggested Answers Income Tax

Total Deductions 14,000


Assessable Income from Investment
2,761,000
(Total Assessable Income)
Taxable Income, Balance Taxable Income and Tax Liability
Total Assessable Income 2,761,000
Less:
Lower of:
a. 5% of ATI, i.e. 5% of Rs. 686,900
Donation to Exempt Entity 12 40,000
b. Actual Donation to Exempt Entity- Rs. 40,000
c. Maximum Rs. 100,000
Taxable Income 2,721,000
Less:
Payment of Life insurance premium Sch. 1 22,500 Lower of actual premium of Rs. 25,000
Balance Taxable Income 2,698,500
Tax Liability
1st Rs. 450,000 0% - Assuming Couple Assessment
Next Rs. 100,000 10% 10,000
Next Rs. 200,000 20% 40,000
Next Rs. 1,250,000 30% 375,000
Balance Rs. 698,500 36% 251,460
676,460
Less:
Taxes withheld and paid by agents
Natural Resource Payment 30,000
Interest from ABC Ltd. 375,000
Net Tax Payable 271,460

175. What is investment income? What sorts of revenue is included in investment income? (Dec 2011, CA Inter, 5 Marks)

© The Institute of Chartered Accountants of Nepal 261


CAP II Paper 7 Income Tax and VAT

Answer
“Investment” means the holding of one or more properties or the investment in a property except the following:
a. The use of the property for the private use of the owner, or
b. A business or an employment.
The holding of a non-business chargeable asset is also known as an investment.

Revenue to be included in investment income is as follows:


a. Any dividend, interest, natural resource payment, rent, royalty, gain from investment insurance, gain from unapproved
retirement fund pursuant to Sec. 63 (1), or retirement payment from approved retirement fund,
b. Net gains on disposal of the non-business chargeable assets of the investment of the person calculated under Chapter 8,
c. Where the incomings exceed the remaining value, including the outgoings in respect of the pool of depreciable assets under
Clause (Ka) of Sub-Section 2 of Section 4 of Schedule 2, at the time of disposal of the depreciable assets of the investments
made by the person, the excess amount,
d. Gifts received by the person in respect investment,
e. Retirement payments made in respect of investment, and the retirement contributions along with amounts deposited in the
retirement fund for that person,
f. Amounts derived as consideration for accepting a restriction on the capacity to conduct the investment
g. Other amounts to be included pursuant to Chapter 6 or 7, or Section 56

176. Mr. Y, a married person has the following incomes/ losses and investments for the year ending on 31/3/20X5. Calculate his
taxable income for the financial year 20X4/X5. (Dec 2006, 8 Marks, CA Inter)
Rs.
(a) Interest on debentures 2,000
(b) Interest on bank accounts 10,000
(c) Dividend from companies 20,000
(d) Salary and interest for being a working partner in a firm (share in
firm's income Rs. 50,000 excluded) 64,000
(e) Business loss from proprietary business 81,000

© The Institute of Chartered Accountants of Nepal 262


Compilation of Suggested Answers Income Tax

(f) Rental income from house property fully let-out 18,000


(g) House tax paid for let-out property 500
(h) Interest paid on capital borrowed for purchase of self-occupied
residential flat 18,000
(i) Long-term capital gain on sale of plot 1,60,000
(j) Deposit in CIT Account 5,000
Answer
Assessable Income from Investment
Sec.
Particulars Note
Ref. Amount
Final withholding, assuming debenture issuing company is resident of
Interest on Debenture 92 -
Nepal
Final withholding, assuming bank is resident of Nepal and the interest
Interest on Bank Accounts 92 -
earned has no any connection to business
Final withholding, assuming the dividend paying company is resident
Dividend from Companies 92 -
of Nepal
Final withholding, assuming the dividend paying firm is resident of
Share in firm's income 92 -
Nepal
As per definition of "Rent", natural person's house rent income that is
Rental income from house
- not generated through a private firm does not fall under the definition
property
of rent, which means such income is exempt from income tax
Interest on loan for
- It forms part of outgoings of self-occupied house
construction of private house
Gain on Sale of Land 9 (2) 160,000 Assuming the incomings from land exceeds Rs. 10 Lakhs
Total Inclusions 160,000
Less:
Business loss from Private firm 20 81,000
Assessable Income from
79,000
Investment

© The Institute of Chartered Accountants of Nepal 263


CAP II Paper 7 Income Tax and VAT

Assessable Income from Employment


Salary from Partnership firm 7 (2) 14,000
Assessable Income from
14,000
Employment

Total Assessable Income and Taxable Income


Assessable Income from
79,000
Investment
Assessable Income from
14,000
Employment
Total Assessable Income 93,000
Less:
Lower of:
a. one-third of total assessable income,
Contribution to Approved RF 63 5,000
b. Actual Contribution to ARF- Rs. 5,000
c. Maximum Rs. 300,000
Taxable Income 88,000

© The Institute of Chartered Accountants of Nepal 264


Compilation of Suggested Answers Income Tax

Chapter 10: Tax Credits


177. Discuss the various types of Tax Credits available under Income Tax Act, 2058.
(Dec 2016, 5 Marks)
Answer
The various types of tax credits are:
a. Female Tax credit
Female tax credit, which is equal to 10% of tax liability before all tax credits, is available
to a person when all the following conditions are satisfied:
• The person shall be a female,
• The person shall be resident of Nepal,
• The person shall generate income only from employment, and
• The source of such employment income shall be only from Nepal

b. Medical Tax Credit


Medical tax credit, which is lower of 15% of approved medical expenditure and Rs. 750
every year, shall be available to a resident natural person.
Any excess 15% of approved medical expenditure over Rs. 750 is carried forward to be
claimed during subsequent income year.

c. Foreign Tax Credit


Foreign tax credit is available to a resident person that generates income having source
outside Nepal and pays tax in such source country.
The maximum amount of foreign tax credit shall not exceed the amount determined by
multiplying foreign assessable income by average rate of tax in Nepal, and such
maximum amount shall be calculated separately for each country. Any excess foreign tax
payment during a country is carried forward to be claimed during subsequent income
years.

Foreign Tax Credit


178. Mr. Ram Prasad, Nepali resident with single status for taxation, had submitted the
following details of income from business and investment as self-assessment of his tax
liability for the Income Year 20X-73/X-74:
Amount (Rs.)
Country Assessable Income Tax paid
Nepal 500,000.00 25,000.00
USA 200,000.00 65,000.00
UK 300,000.00 -
Canada 500,000.00 80,000.00
Total 1,500,000.00 170,000.00

© The Institute of Chartered Accountants of Nepal 265


CAP II Paper 7 Income Tax and VAT

Compute any unabsorbed foreign tax credit of Mr. Ram Prasad during the assessment.
(June 2018, 5 Marks)
Answer
Step 1: Calculation of Tax Liability before Foreign Tax Credit

Assessable Income from Nepal 500,000


Assessable Income from USA 200,000
Assessable Income from UK 300,000
Assessable Income from Canada 500,000
Total Assessable Income
(Taxable/Balance Taxable Income) 1,500,000

Tax Liability Note


Assuming there is no
1st Rs. 400,000 0% -
employment income
Next Rs. 100,000 10% 10,000
Next Rs. 200,000 20% 40,000
Balance Rs. 800,000 30% 240,000
Tax Liability before FTC 290,000

Step 2: Calculation of Average Tax rate in Nepal


Tax Liability before Foreign Tax Credit
(A) 290,000
Taxable Income in Nepal (B) 1,500,000
Average Tax Rate in Nepal [A/B * 100] 19.33%

Step 3: Calculation of Eligible Foreign Tax Credit for each country and excess, if any

Particularrs USA UK Canada Total


A Foreign Assessable Income 200,000 300,000 500,000
B Average Rate of Tax in Nepal 19.33% 19.33% 19.33%
C Average Tax Paid in Nepal (A * B) 38,666.67 58,000.00 96,666.67
D Actual Tax Paid in Foreign Country 65000 - 80000
E Excess Taxes of Previous Income Year (s) 0 - 0
F Sum of D & E 65000 0 80000
G Eligible Amount [Lower of "C" or "F"] 38,666.67 - 80,000.00 118,666.67
H Excess to be carried forward [F- G] 26,333.33 - - 26,333.33

© The Institute of Chartered Accountants of Nepal 266


Compilation of Suggested Answers Income Tax

179. What are the possible methods of taking foreign tax credit to avoid double taxation of
resident person? (June 2017, 5 Marks)
Answer
There are two methods:
a. Credit method:
Foreign tax credit is available to a resident person that generates income having source
outside Nepal and pays tax in such source country.
The maximum amount of foreign tax credit shall not exceed the amount determined by
multiplying foreign assessable income by average rate of tax in Nepal, and such
maximum amount shall be calculated separately for each country. Any excess foreign tax
payment during a country is carried forward to be claimed during subsequent income
years.

b. Deduction Method: The tax paid in foreign country can be claimed as deduction, if the
taxpayer so wishes to relinquish its right to use credit method.

180. Mr. Z has a source of income in Nepal and also in more than one foreign country.
During the Fiscal Year 20X-67/X-68, income and tax paid in each foreign country is
given below;
Name of the Country Income (Rs.) Tax Paid (Rs.)
USA 200,000 60,000
Australia 150,000 30,000
UAE 100,000 5,000
Nepal 250,000 -
He is a resident natural person and selected for the couple as tax payers during the
year. Calculate his tax liability during the Fiscal Year 20X-67/X-68.
(June 2012, 5 Marks)
Answer
Step 1: Calculation of Tax Liability before Foreign Tax Credit
Note
Assessable Income from Nepal 250,000
Assessable Income from USA 200,000
Assessable Income from Australia 150,000
Assessable Income from Canada 100,000
Total Assessable Income
(Taxable/Balance Taxable Income) 700,000

© The Institute of Chartered Accountants of Nepal 267


CAP II Paper 7 Income Tax and VAT

Tax Liability
Assuming there is no
1st Rs. 450,000 0% - employment income
Next Rs. 100,000 10% 10,000
Next Rs. 150,000 20% 30,000
Tax Liability before FTC 40,000

Step 2: Calculation of Average Tax rate in Nepal


Tax Liability before Foreign Tax Credit
(A) 40,000
Taxable Income in Nepal (B) 700,000
Average Tax Rate in Nepal [A/B * 100] 5.71%

Step 3: Calculation of Eligible Foreign Tax Credit for each country and
excess, if any

USA Australia UAE Total


A Foreign Assessable Income 200,000 150,000 100,000
B Average Rate of Tax in Nepal 5.71% 5.71% 5.71%
C Average Tax Paid in Nepal (A * B) 11,428.57 8,571.43 5,714.29
D Actual Tax Paid in Foreign Country 60,000 30,000 5,000
E Excess Taxes of Previous Income Year (s) - - -
F Sum of D & E 60,000 30000 5,000
G Eligible Amount [Lower of "C" or "F"] 11,428.57 8,571.43 5,000.00 25,000.00
H Excess to be carried forward [F- G] 48,571.43 21,428.57 - 70,000.00

Step 4: Calculation of Tax Liability after Tax Credit


Tax Liability before Foreign Tax Credit 40,000
Less: Eligible Credit from Step 3 (25,000)
Tax Liability after Foreign Tax Credit 15,000

© The Institute of Chartered Accountants of Nepal 268


Compilation of Suggested Answers Income Tax

181. Mr. A, a resident natural person, is a musician deriving income from concerts
performed from various countries outside Nepal. During Financial Year 20X-66/X-67,
he performed concerts in India. Details of his assessable income and tax paid in the
country as aforesaid where the concerts were given are:
Income Tax

India Rs. 1,000,000 Rs. 300,000


Also, he earned Rs. 5,00,000 in Nepal during the financial year 2066/67.
Assuming that he has chosen to be couple, find his tax liability on his total income and
amount for foreign tax credit available to him under section 71(1) for the income year
20X-66/X-67. (Dec 2010, 10 Marks)
Answer
Assuming there is no employment income
Step 1: Calculation of Tax Liability before Foreign Tax Credit

Assessable Income from Nepal 500,000


Assessable Income from USA 1,000,000
Total Assessable Income
(Taxable/Balance Taxable Income) 1,500,000

Tax Liability
1st Rs. 450,000 0% -
Next Rs. 100,000 10% 10,000
Next Rs. 200,000 20% 40,000
Next Rs. 750,000 30% 225,000
Tax Liability before FTC 275,000

Step 2: Calculation of Average Tax rate in Nepal


Tax Liability before Foreign Tax Credit
(A) 275,000
Taxable Income in Nepal (B) 1,500,000
Average Tax Rate in Nepal [A/B * 100] 18.33%

Step 3: Calculation of Eligible Foreign Tax Credit and excess, if any

India
A Foreign Assessable Income 1,000,000
B Average Rate of Tax in Nepal 18.33%
C Average Tax Paid in Nepal (A * B) 183,333.33
D Actual Tax Paid in Foreign Country 300,000

© The Institute of Chartered Accountants of Nepal 269


CAP II Paper 7 Income Tax and VAT

E Excess Taxes of Previous Income Year (s) -


F Sum of D & E 300000
G Eligible Amount [Lower of "C" or "F"] 183,333.33
H Excess to be carried forward [F- G] 116,666.67
Step 4: Calculation of Tax Liability after Tax Credit
Tax Liability before Foreign Tax Credit 275,000
Less: Eligible Credit from Step 3 (183,333)
Tax Liability after Foreign Tax Credit 91,667

182. In what context “Average Rate of Nepal Tax” is relevant under the Income Tax Act?
Define the same. (Dec 2008, 5 Marks, CA Inter)
Answer:
“Average Rate of Nepal Tax” is relevant in the context of determining maximum limit of
foreign tax credit available to a resident person paying tax in foreign country in income
generated from such country.
It is calculated by dividing tax liability of a person before foreign tax credit by taxable
income for the year of such person.

183. Mr. Bimal Shrestha has a source of income in Nepal as well as foreign countries. His
net income and tax paid in each foreign country during the income year 20X-62/X-63
is given below:
Name of the Country Net Income (Rs.) Tax Paid (Rs.)
UAE 2,00,000 10,000
USA 4,00,000 1,20,000
Australia 3,00,000 60,000
Nepal 5,00,000 -

Mr. Bimal Shrestha is a resident natural person and opted couple status for income tax
purpose. He decided to elect to get benefit of foreign tax credit. Calculate his tax liability
for the income year 20X-62/X-63. (Dec 2006, 8 Marks, CA Inter)
Answer
Step 1: Calculation of Tax Liability before Foreign Tax Credit

Assessable Income from Nepal 500,000


Assessable Income from UAE 200,000
Assessable Income from USA 400,000
Assessable Income from Australia 300,000
Total Assessable Income
(Taxable/Balance Taxable Income) 1,400,000

© The Institute of Chartered Accountants of Nepal 270


Compilation of Suggested Answers Income Tax

Assuming there is no employment income


Tax Liability
1st Rs. 450,000 0% -
Next Rs. 100,000 10% 10,000
Next Rs. 200,000 20% 40,000
Next Rs. 650,000 30% 195,000
Tax Liability before FTC 245,000

Step 2: Calculation of Average Tax rate in Nepal


Tax Liability before Foreign Tax Credit (A) 245,000
Taxable Income in Nepal (B) 1,400,000
Average Tax Rate in Nepal [A/B * 100] 17.50%
Step Calculation of Eligible Foreign Tax Credit for each country and excess, if
3: any
UAE USA Australia Total
A Foreign Assessable Income 200,000 400,000 300,000
B Average Rate of Tax in Nepal 17.50% 17.50% 17.50%
C Average Tax Paid in Nepal (A * B) 35,000 70,000 52,500
D Actual Tax Paid in Foreign Country 10,000 120,000 60,000
E Excess Taxes of Previous Income Year (s) - - -
F Sum of D & E 10,000 120,000 60,000
G Eligible Amount [Lower of "C" or "F"] 10,000 70,000 52,500 132,500
H Excess to be carried forward [F- G] - 50,000 7,500 57,500

184. Mr. A receives Royalty Rs. 25,00,000 from Nepal on which 15% taxes deducted at
source. Mr. A occasionally visits Nepal and during the year 20X-59/X-060 he stays in
Nepal for 100 days. He is a resident of USA and he has to pay tax in USA. Can he claim
the tax he has to pay in USA to be set off against the tax deductible in Nepal and claim
refund as that is the only income that he receives in Nepal? He has to pay a tax at 35%
in the USA in the Royalty income earned. (Dec 2004, 5 Marks, CA Inter)
Answer
Mr. A is non-resident of Nepal since his habitual place is in the USA, he stays in Nepal for
less than 183 days during the Income Year and he is not employee of GoN deputed by GoN
in foreign country, and as per sec. 6, a non-resident has to pay tax on Nepal sourced Income.
As per Sec. 71, a resident person is entitled to foreign tax credit in case the person has paid
tax in foreign county on its foreign sourced income.
As such, Mr. A cannot claim foreign tax credit in Nepal, and he has to pay tax @15% on
royalty income having source in Nepal, which is treated as final withholding u/s 92 of the
Act.

© The Institute of Chartered Accountants of Nepal 271


CAP II Paper 7 Income Tax and VAT

185. Write short notes on “Foreign Tax Credit”. (Dec 2003, 3 Marks, CA Inter)
Foreign tax credit is available to a resident person that generates income having source
outside Nepal and pays tax in such source country.
The maximum amount of foreign tax credit shall not exceed the amount determined by
multiplying foreign assessable income by average rate of tax in Nepal, and such maximum
amount shall be calculated separately for each country.
Any excess foreign tax payment during a country is carried forward to be claimed during
subsequent income years.

© The Institute of Chartered Accountants of Nepal 272


Compilation of Suggested Answers Income Tax

Relevant Matters in Taxable Income

Chapter 11- Tax Accounting: Methods and Timing


186. Munal Trade Link is a various goods supplier. The trade link is using accrual basis of
accounting for the income generated from the business. He gave an application to the
department to change his basis of accounting from accrual to cash in Income Year 20X-
70/X-71. In accordance with the generally accepted accounting principles, 27 Inland
Revenue Department has given the approval to account for his income on cash basis in
Income Year 20X-70/X-71. Some accounting information of the trade link at the end
of 20X-69/X-70 before changing the basis of accounting, was as below:
a. Goods worth of Rs. 75,000 has already been supplied to a retailer in Income Year
20X-69/X-70 but amount has not been received. Advance Rs. 45,000 has been
received from another retailer.
b. Goods worth of Rs. 60,000 has been purchased from a vendor in Income Year 20X-
69/X-70 but amount has not been paid. Advance Rs. 35,000 has been paid to another
vendor.
c. Communications and electricity expenses of Rs. 3,500 were paid on Bhadra, 20X-71.
These expenses were incurred in Jestha and Ashad, 20X-70.

State how you make adjustments of income and expense in FY 20X-70/X-71 in above
conditions? (Dec 2014, 5 marks)
Answer
Particulars Treatment Prior Treatment Conclusion
under Accrual
Method
Goods worth of Rs. The sales had Since cash basis Under Sec. 22 (6), there
75,000 has already to be included was followed, shall be adjustment in
been supplied to a in income u/s the amount income during the year of
retailer in Income Year 7 during I.Y. could not be change in basis of
20X-69/X-70 but 20X-69/X-70 included in accounting so as to avoid
amount has not been income during omission or taking into
received I.Y. 20X-69/X- account more than once.
70 Since it has not been
included in income, as such,
to avoid omission, the
amount has to be included in
income during I.Y. 20X-
70/X-71
Advance Rs. 45,000 has The amount Since cash basis Under Sec. 22 (6), there
been received from has to be was followed, shall be adjustment in

27
The term “Inland Revenue Department” replaces the term used in original question

© The Institute of Chartered Accountants of Nepal 273


CAP II Paper 7 Income Tax and VAT

another retailer in included in the amount was income during the year of
relation to sales income when included in change in basis of
the sales is income during accounting so as to avoid
executed I.Y. 20X-69/X- omission or taking into
70 account more than once.
Since the amount is included
in income already, as such,
there shall be no treatment
of such amount during I.Y.
20X-70/X-71, to avoid
taking into account more
than once
Goods worth of Rs. The amount The amount has No treatment during I.Y.
60,000 has been has to be been 20X-70/X-71
purchased from a considered as considered as
vendor in Income Year cost during cost during
20X-69/X-70 but 20X-69/X-70, 20X-69/X-70,
amount has not been as per Sec. 15 as per Sec. 15
paid
Advance Rs. 45,000 has The amount The amount was No treatment during I.Y.
been received from has to be not treated as 20X-70/X-71
another retailer in treated as
expense u/s 15,
relation to purchase advance since cost of
during I.Y.
trading stock is
20X-69/X-70 determined
under accrual
concept even
under cash
basis of
accounting
Communications and These are Treated as To avoid taking into account
electricity expenses of expenses in expense at the of expense more than once,
Rs. 3,500 were paid on relation to I.Y. time of outflow the amount has to be
Bhadra, 20X-71. These 20X-70/X-71 of resources neglected during I.Y. 20X-
expenses were incurred during I.Y. 20X- 70/X-71
in Jestha and Ashad, 69/X-70
20X-70

187. Standard Chartered Bank has appointed AX Consultancy Pvt. Ltd. as Tax Consultant
on Magh 20, 20X-70 and the effective date of contract is Shrawan 1, 20X-71. Under the
contract, the bank is liable to pay an annual fee of Rs. 500,000. As per the contract, an
advance amount of Rs. 100,000 is to be paid on appointment date (i. e. on Magh 20,
20X-70). AX Consultancy Pvt. Ltd. is in confusion whether the amount of Rs. 100,000
to be included in its income of Income Year 20X-70/X-71 as the appointment letter is
already received during the Income Year. Advice. (Dec 2014, 5 marks)

© The Institute of Chartered Accountants of Nepal 274


Compilation of Suggested Answers Income Tax

Answer
As per Sec. 22 (3), a company is required to record its transaction under accrual basis for
tax purpose.
As per Sec. 2, a company incorporated under Companies Act is a company for tax purpose.
Therefore, AX Consultancy Pvt. Ltd., which is a company for tax law purpose as well, shall
use accrual basis of accounting.
As per Sec. 24, income under accrual basis has to be recognized as and when the right to
receive the payment is established.
In the given case, right to receive payment will be established only when the service is
rendered, i.e. only after Shrawan 1, 20X-71.
Therefore, a liability is recorded when the advance is received on Magh 20, 20X-70, and the
amount becomes taxable only when the service is rendered.

188. Mr. X is engaged in legal consultancy services. Mr. X has maintained his books of
accounts on cash basis till income year 20X-68/X-69. From income year 20X-69/X-70,
he wants to changeover to accrual basis of accounting. Following information from
records of his books of accounts of income year 20X-68/X-69 is abstracted:
a. Service provided but fee not received Rs. 5,00,000 which was not included in the
income.
b. Advance received from customers but service not rendered Rs. 3,00,000. The amount
was included in the income in the income year 20X-68/X-69.
c. Rent for the period 20X-69 Shrawan to 20X-69 Poush paid in Ashad 20X-69. The
amount is included in the expense in the income year 20X-68/X-69.
You are required to advise Mr. X (Dec 2013, 5 Marks)
Particulars Treatment Prior Treatment Conclusion
under Accrual
Method
Service provided but Since right to Since cash basis Under Sec. 22 (6), there
fee not received Rs. receive the was followed, shall be adjustment in
5,00,000 which was not payment was the amount income during the year of
included in the income established could not be change in basis of
during I.Y. included in accounting so as to avoid
20X-68/X-69, income during omission or taking into
the amount I.Y. 20X-68/X- account more than once.
had to be 69 Since it has not been
included included in income, as such,
during same to avoid omission, the
I.Y. amount has to be included in

© The Institute of Chartered Accountants of Nepal 275


CAP II Paper 7 Income Tax and VAT

income during I.Y. 20X-


69/X-70
Advance received from Since right to Since cash basis Under Sec. 22 (6), there
customers but service receive the was followed, shall be adjustment in
not rendered Rs. payment was the amount was income during the year of
3,00,000 established included in change in basis of
during I.Y. income during accounting so as to avoid
20X-69/X-70, I.Y. 20X-68/X- omission or taking into
the amount 69 account more than once.
has to be Since it has already been
included included in income, as such,
during same to avoid taking into account
I.Y. more than once, the amount
shall be ignored in income
during I.Y. 20X-69/X-70
Rent for the period Rent for 20X- Since cash basis Under Sec. 22 (6), there
20X-69 Shrawan to 69 Shrawan to was followed, shall be adjustment in
20X-69 Poush paid in 20X-69 Ashad the amount was income during the year of
Ashad 20X-69 is the expense deducted as change in basis of
of I.Y. 20X- expense during accounting so as to avoid
69/X-70 I.Y. 20X-68/X- omission or taking into
69 account more than once.
Since it has already been
deducted, as such, to avoid
taking into account more
than once, the amount shall
be ignored in income during
I.Y. 20X-69/X-70

189. Mr. Devan Mahara is lawyer. He is using cash basis of accounting for the income
generated from his profession. He gave an application to the department to change his
basis of accounting from cash to accrual in Fiscal Year 20X-67/X-68. In accordance
with the generally accepted accounting principles, Income Tax Department has given
the approval to account for his income on accrual basis in Fiscal Year 20X-67/X-68.
The accounts of Mr. Mahara immediately before changing the basis of accounting, was
as below:
a. Service has already been rendered in FY 20X-66/X-67 but amount has not been
received and hence no income is included in that FY NPR 80,000.
b. In FY 20X-66/X-67 advance has been taken (service has been rendered on FY 20X-
67/X-68) but amount is included in that FY on cash basis though service is still to be
rendered NPR 30,000.

© The Institute of Chartered Accountants of Nepal 276


Compilation of Suggested Answers Income Tax

c. In FY 20X-66/X-67 House Rent from Magh 20X-66 to Poush 20X-67 has been paid
and expense has been claimed in FY 20X-66/X-67 on cash basis of accounting NPR
48,000.
State how you make adjustments of income and expense in FY 20X-67/X-68 in above
conditions? (Dec 2011, 5 Marks)
Answer
Particulars Treatment Prior Treatment Conclusion
under Accrual
Method
Service has already Since right to Since cash basis Under Sec. 22 (6), there
been rendered in FY receive the was followed, shall be adjustment in
20X-66/X-67 but payment was the amount income during the year of
amount has not been established could not be change in basis of
received and hence no during I.Y. included in accounting so as to avoid
income is included in 20X-67/X-68, income during omission or taking into
that FY NPR 80,000 the amount I.Y. 20X-66/X- account more than once.
had to be 67 Since it has not been
included included in income, as such,
during same to avoid omission, the
I.Y. amount has to be included in
income during I.Y. 20X-
67/X-68
Advance received from Since right to Since cash basis Under Sec. 22 (6), there
customers but service receive the was followed, shall be adjustment in
not rendered Rs. payment was the amount was income during the year of
30,000 established included in change in basis of
during I.Y. income during accounting so as to avoid
20X-67/X-78, I.Y. 20X-66/X- omission or taking into
the amount 67 account more than once.
has to be Since it has already been
included included in income, as such,
during same to avoid taking into account
I.Y. more than once, the amount
shall be ignored in income
during I.Y. 20X-67/X-68
Rent for the period Rent for Since cash basis Under Sec. 22 (6), there
20X-66 Magh to 20X- a. Poush was followed, shall be adjustment in
67 Poush paid during 20X-66 to the amount was income during the year of
20X-66/X-67 Ashad deducted as change in basis of
20X-66 is expense during accounting so as to avoid
expense of I.Y. 20X-66/X- omission or taking into
I.Y. 20X- 67 account more than once.
66/X-67 Since it has already been

© The Institute of Chartered Accountants of Nepal 277


CAP II Paper 7 Income Tax and VAT

b. 20X-67 deducted, as such, to avoid


Shrawan taking into account more
to 20X-67 than once, the amount shall
Poush is be ignored in income during
the I.Y. 20X-67/X-68
expense of
I.Y. 20X-
67/X-68

190. What do you understand by method of accounting? How far method of accounting is
relevant in computing the income for tax purpose? (June 2009, 6+4=10, CA Inter)
Answer
Tax accounting deals with the question at what time amounts derived and costs incurred are
recognized for taxation purposes, i.e. when these amounts are to be included in the
calculation of the taxable income.

Mainly there are two types of accounting methods-

i) Cash Basis of Accounting and


ii) Accrual Basis of Accounting.

Under the cash system of accounting, revenue and expenses are recorded only when received
or paid irrespective of the fact whether it relates to the previous year or not. Income under
cash system of accounting is, therefore, excess of receipts over disbursements during the
previous year.

Under the Accrual (Mercantile) system of accounting, income and expenditure are recorded
at the time of their occurrence during the previous year. The profit calculated under the
accrual system is profit actually earned during the previous year, though not necessarily
realized in cash. In other words, where accounts are kept on accrual basis, the profits or
gains are credited though they are not actually realized and entries, thus, made really show
nothing more than an accrual or arising or the said profits at the material time.

Income Tax Act, 2058 prescribes the following methods of accounting for tax purposes:

1) An individual has to account on a cash basis in calculating his income from employment
and investment, i.e. the cash method is to be used by employees for their employment and
investment income (s 22 (2)).
2) A company has to account on an accrual basis (s 22 (3)).
3) Any other taxpayer may choose between the cash or the accrual method for calculating
his income from business or investment unless the IRD prescribes otherwise by notice in
writing (s 22 (4)).

© The Institute of Chartered Accountants of Nepal 278


Compilation of Suggested Answers Income Tax

4) Taxpayers can apply in writing to the IRD for a change in his basis of accounting. The
IRD may then in exceptional cases allow the change if it is necessary to clearly reflect
the taxpayer’s income (u/s 22).

191. Distinguish between Cash basis of Accounting and Accrual Basis of Accounting.
(Dec 2003, 3 Marks, CA Inter)
Answer
Under the cash system of accounting, revenue and expenses are recorded only when received
or paid irrespective of the fact whether it relates to the previous year or not. Income under
cash system of accounting is, therefore, excess of receipts over disbursements during the
previous year.

Under the Accrual (Mercantile) system of accounting, income and expenditure are recoreded
at the time of their occurrence during the previous year. The profit calculated under the
accrual system is profit actually earned during the previous year, though not necessarily
realized in cash. In other words, where accounts are kept on accrual basis, the profits or
gains are credited though they are not actually realized and entries, thus, made really show
nothing more than an accrual or arising or the said profits at the material time.

© The Institute of Chartered Accountants of Nepal 279


CAP II Paper 7 Income Tax and VAT

Chapter 12- Long Term Contract


192. ABC Ltd. has entered into a construction contract with PQR Ltd. with a term of 3
years. The contract price and the total estimated cost are Rs.60 Billion and Rs. 40
Billion respectively in Year1 of the contract, cost amounting to Rs. 10 Billion was
incurred and up to year 2 Rs.25 Billion has been expensed. Calculate the income of
ABC Ltd. to be included in the concerned head of income with respect to long term
contract in Year 1 and Year 2 by also showing the calculation of Cumulative Inclusions
and Cumulative Deductions.
Briefly explain on Long Term Contract, Excluded Contract and Deferred Return.
(Dec 2012, 5+5=10)
Answer

S.N. Particulars Year I Year II


A Initial Total Revenue for the Contract 60,000,000,000 60,000,000,000
B Variation Order - -
C Escalation Clause - -
D Total Revenue for the Contract(A+B+C) 60,000,000,000 60,000,000,000
E Initial Total Estimated Cost of the Contract 40,000,000,000 40,000,000,000
F Increase in estimated cost due to variation order - -
Increase in estimated cost due to cost escalation, not
G
considered elsewhere (G) - -
Total Estimated Cost to complete the Contract
H (E+F+G) 40,000,000,000 40,000,000,000
Actual Cost incurred to Date (Cumulative Cost till
I this year) 10,000,000,000 25,000,000,000
J Percentage Completion (I/H *100) 25.00% 62.50%
K Cumulative Revenue till this year (D * J) 15,000,000,000 37,500,000,000
L Cumulative Profit/(Loss) till this Year (K - I) 5,000,000,000 12,500,000,000
M Profit/(Loss) up to Previous Year - 5,000,000,000
N Profit/(Loss) for the Year (L - M) 5,000,000,000 7,500,000,000

Long term Contract


Long term contract includes the following contract having term more than twelve months:
a. A contract for manufacture, installation, or construction, or, in relation to each, the
performance of related services, or
b. A contract with a deferred return that is not an “excluded contract”.

Excluded Contract
The following deferred return contracts are excluded from the scope of Sec. 26:
a. Any contract consisting of an interest in any entity,
b. Any contract consisting of membership in a retirement fund, or
c. Any contract of investment insurance

© The Institute of Chartered Accountants of Nepal 280


Compilation of Suggested Answers Income Tax

Deferred Return Contract


A contract is a “Deferred Return Contract”, in case party to the contract fails to show the
matters as prescribed by IRD in relation to the estimated profit and estimated loss in every
six months after the commencement of the contract.

193. Torrent Co. Ltd. is a large publicly listed company whose main activity involves
construction contracts. Torrent won a global contract to build a new football stadium
owned by Star Youth Club at a contracted price of Rs. 40,00,000 thousand. This
construction contract was to be completed over the period of three years starting from
financial year 20X-65/X-66.
Initially, the total cost of completing the said stadium was estimated at Rs. 36,00,000
thousand.
Star Youth Club had placed additional variation order in 2nd and 3rd financial years
with the details as given below:
20X-66/X-67 20X-67/X-68
Variation order Rs. 70,000 thousand Rs. 80,000 thousand
Additional Estimated Cost due to Rs. 50,000 thousand Rs. 4,20,000
variation Order thousand

Details of actual cumulative costs over the period of 3 years to build above football
stadium were as follows:
(Rs. In ‘000)
20X-65/X-66 20X-66/X-67 20X-67/X-68
Actual accumulated costs to date 18,00,000 26,28,000 39,86,000
Rectification costs - - 34,000
The rectification costs are the costs incurred in widening access roads to the stadium.
This was the result of an error by Torrent’s architect when he made his initial drawings.

Compute the extracts of taxable income over those three years ending F.Y. 20X-67/X-
68 and amount of losses carried back. (Dec 2011, 15 Marks)
Answer
Assumption: Contract started from Shrawan 1
Particulars 20X-65/X-66 20X-66/X-67 20X-67/X-68
Initial Total Revenue for the Contract (A) 4,000,000,000 4,000,000,000 4,000,000,000
Variation Order (B) - 70,000,000 150,000,000
Escalation Clause (C ) - - -
Total Revenue for the Contract(D=
A+B+C) 4,000,000,000 4,070,000,000 4,150,000,000

© The Institute of Chartered Accountants of Nepal 281


CAP II Paper 7 Income Tax and VAT

Initial Total Estimated Cost of the


Contract (E) 3,600,000,000 3,600,000,000 3,600,000,000
Increase in estimated cost due to variation
order (F) - 50,000,000 470,000,000
Increase in estimated cost due to cost
escalation, not considered elsewhere (G) - - -
Total Estimated Cost to complete the
Contract (H= E+F+G) 3,600,000,000 3,650,000,000 4,070,000,000
Actual Cost incurred to Date (Cumulative
Cost till this year) (I) 1,800,000,000 2,628,000,000 4,020,000,000
Percentage Completion (J= I/H *100) 50.00% 72.00% 100.00%
Cumulative Revenue till this year (K= D *
J) 2,000,000,000 2,930,400,000 4,150,000,000
Cumulative Profit/(Loss) till this Year (L=
K - I) 200,000,000 302,400,000 130,000,000
Profit/(Loss) up to Previous Year (M) - 200,000,000 302,400,000
Profit/(Loss) for the Year (N= L - M) 200,000,000 102,400,000 (172,400,000)

As per Sec. 20 (4), the loss incurred in a long-term contract won through global bidding at
the end or disposal of contract can be set off against the income from same contract
pertaining to previous Income Year(s), subject to the approval from IRD. Therefore, Torrent
Co. can carry back its loss of Year III to set it off against income of Year I and Year II as
instructed by IRD. The taxes paid for income of previous Income Year(s) are subject to refund
u/s 113.

© The Institute of Chartered Accountants of Nepal 282


Compilation of Suggested Answers Income Tax

Chapter 13- Gain on Disposal of Asset and Liability


194. M/s Sonica Pvt. Ltd. having registered office at Kathmandu, exclusively deals in
computer trading wholesale business. It recently extended its branches to all 77 districts
of Nepal. The Company imports computers for wholesale business @ Rs. 85,000 per
computer. After opening the branches, the Company sent the imported computers for
office use one piece to each branch. Is there any tax implication for the above
transaction as per the provision of Income Tax Act, 2058? (Dec 2018, 5 Marks)
Answer
As per Sec. 40 (3) (Gha), when an asset of one form is used in another form, the previous
form of asset is deemed to be disposed just before it is used in new form.

In the given case, the computer that was imported as trading stock has been used as
depreciable asset. Therefore, trading stock is deemed to be disposed and depreciable asset
is deemed to be purchased.

The tax implication would be:


a. The market value of seventy-seven computers would be treated as sales and included in
computing assessable income from business,
b. The cost of such computers is treated as Cost of Trading Stock and are claimed as
deduction u/s 15, and
c. The market value of seventy-seven computers is treated as additions to Block B of
Depreciable asset, and depreciation expense is claimed as deduction under sec. 19 of the
Act.

195. Oriental Pvt. Ltd is dealing with buying, constructing and selling of land and building.
It has the following transactions for one of the projects during the year 20X-73/X-74.
(Dec 2017, 5 Marks)
Land purchase for the project Rs. 50 million
Land purchase for office Rs. 10 million
Housing construction cost (10 houses) Rs. 150 million
Building construction cost for office Rs. 10 million
Land in Stock Rs. 20 million
Housing in Stock Rs. 60 million
Sales of housing (8 houses) Rs. 300 million

Based on the above information, answer the following questions:


a. Calculate gain and loss in this transaction
Answer:

Particulars Trading Stock Depreciable Asset

© The Institute of Chartered Accountants of Nepal 283


CAP II Paper 7 Income Tax and VAT

Incomings
Sales of Housing 300,000,000 -
Outgoings
Cost of Land 50,000,000 10,000,000
Cost of Construction of 10 houses 150,000,000 -
Building Construction for office - 10,000,000
Less: Closing Stock
Land in Stock (20,000,000)
Housing in Stock (60,000,000)
Total Outgoings 120,000,000
Gain on Disposal 180,000,000

b. Whether the company should pay capital gain tax28 or not?


Answer
There is no concept of capital gain tax in Nepal. However, as per Sec. 95Ka, where an
entity sale its land and building, 1.5% of sales value shall be paid as advance tax at the
time of transfer of ownership at land revenue office. The amount of such advance tax
would be Rs. 4,500,000.

c. If the company should pay capital gain tax, how much and where to pay?
Answer
Refer explanation to part (b) above.

d. If the company does not require to pay the capital gain tax, then what and where to
pay tax on the gain of this transaction?
Answer
The company needs to pay advance tax at the rate 1.5% of sales value at the time of
transfer of ownership at land revenue office. Further, it needs to pay corporate tax on its
annual income within three months of the end of Income Year.

196. Mr. Ram has been holding shares of various listed companies of Nepal from primary
and secondary markets. Total cost of the shares is Rs. 2 million. He left for USA for a
year on Aswin 30, 20X-70 but he returned back on Kartik 15, 20X-72. The market value
of the shares was Rs. 3.5 million at the time he became non-resident.
He sold all the shares for Rs. 3.6 million on Kartik end, 20X-73. He has received his
payment from share broker after deducting broker commission Rs. 20,000. Advance
tax is deducted by NEPSE as per Section 95Ka (2). Compute gain and loss under Section

28
There is no concept of Capital Gain Taxation in Nepal; it should be understood as “Collection of Advance tax by
Land Revenue Department” at the time of disposal of land and building.

© The Institute of Chartered Accountants of Nepal 284


Compilation of Suggested Answers Income Tax

40 (3), gain and loss calculation on Kartik end 20X-73 by NEPSE and tax withholding
amount in these situations. (Dec 2017, 5 Marks)
Answer
Mr. Ram became non-resident on Ashad end, 20X-70; when the market value of shares was
Rs. 3.5 Million. The outgoings related to such shares was Rs. 2 Million.

As per Sec. 40 (3) (Cha), when a resident natural person becomes non-resident, he is deemed
to dispose all his asset and liabilities except land and building just before he becomes non-
resident.
As per Sec. 41 (remember syllabus of CAP II does not cover Sec. 41, and this part may be
skipped in examination), such disposal is at market value, that means, Mr. Ram has gain on
deemed disposal of shares of Rs. 1.5 Million just before he becomes non-resident.

Further, when he disposes off his shares as per Sec. 40 (1), the incomings is Rs. 3.6 Million
and the outgoings is Rs. 3.52 Million (3.5 Million as cost for subsequent disposal of shares
as per Sec. 41 and 20,000 as cost to broker), where the gain is Rs. 0.48 Million, on which he
has to pay tax on actual disposal of assets.

Conclusion:
He pays tax on Rs. 1.5 Million as per the application of Sec. 40 (3) (Cha) and he pays tax on
Rs. 0.48 Million at the time of actual disposal of shares.

197. Mr. Apurva Das has 250 shares of Nabil Bank Ltd. from IPO. Bonus shares has been
distributed 300% (100% before Income Tax Act and remaining thereafter) till now. He
became a non-resident for income year 20X-72/X-73. The market value at the time of
introduction of the Act was Rs. 1,800 and at the time of becoming non-resident was Rs.
2,600. Quoting the related criteria for deemed disposal of shares. Advise him on, if any,
tax liability arises on those shares for income year 20X-72/X-73. (June 2017, 5 Marks)
Answer
It is assumed that bonus share’s percentage is expressed in terms of original number
of shares, i.e. total additional bonus share is 750 kittas (250 Kitta before the
implementation of prevailing Act and 500 kittas thereafter)
Where a practical assumption is made, total numbers of shares become more than
1000, as per different assumptions.

As per Sec. 40 (3) (Cha), when a resident natural person becomes non-resident, he is deemed
to dispose all his asset and liabilities except land and building just before he becomes non-
resident.

As per Sec. 41 (remember syllabus of CAP II does not cover Sec. 41, and this part may be
skipped in examination), such disposal is at market value, i.e. shares are deemed to be

© The Institute of Chartered Accountants of Nepal 285


CAP II Paper 7 Income Tax and VAT

disposed at Rs. 2,600 per share (per share assumed). Total incomings from deemed disposal
of shares is Rs. 2.6 Million (1,000 shares @ Rs. 2,600 each)

Outgoings of Shares:
Number of Shares as on implementation of the Act- 250 kittas plus 100% bonus shares= 500
Kittas
Outgoings = 500 * Rs. 1,800= Rs. 900,000 for 500 Kitta
Additional bonus shares, 500 Kittas (200% of 250 Kitta) at Rs. 100 each = Rs. 50,000
Therefore, total outgoings of 1000 kitta shares is Rs. 950,000
Gain on Disposal= Incomings – Outgoings = Rs. 1.65 Million

Tax liability, in respect of this gain, would be 5% of Gain, assuming basic exemption limit is
utilized by other income.

Note to CAP II Students: Students may answer this question in theoretical aspect, since a
part of the question is out of syllabus.

198. Mr. Govinda has incurred a net loss of Rs. 2 lakhs during income year 20X-71/X-72
from sale of shares. In the same year he incurred a net loss of Rs. 3 lakhs from his
stationary business. During the year 20X-71/X-72, he sold his personal land 29 (non-
business chargeable asset) for Rs. 60 lakhs net of brokerage and other expenses. He
purchased the land at Rs. 20 lakhs before 4 years. During the year 20X-70/X-71, he
incurred a taxable loss of Rs. 2 lakhs from the business.

Calculate net taxable income from disposal of the land mentioning the relevant
provisions of Income Tax Act, 2058. (June 2016, 5 Marks)
Answer
Gain on Disposal of Non-Business Chargeable Asset = Rs.40 Lakhs

As per Sec. 20 (2), any loss on one investment during an Income Year can be offset against
the gain from another investment during the same Income Year. Further, Sec. 11 is not
applicable in respect of Gain on Disposal of Non-Business Chargeable asset, as such, the
loss on sale of shares of Rs. 2 lakhs can be offset against the gain on disposal of NBCA, i.e.
net gain after set off of investment loss is Rs. 38 Lakhs.

As per Sec. 20 (1), business loss can be offset against investment income, as such, business
loss of Rs. 3 lakhs can be offset against gain of Rs. 38 Lakhs, this gives us Rs. 35 Lakhs as
income from investment during the Income year.

29
Land cannot be termed personal land, technically, if its non-business chargeable asset

© The Institute of Chartered Accountants of Nepal 286


Compilation of Suggested Answers Income Tax

199. Mr. Ram had purchased land and private building on 20X-65/7/4 for Rs. 2.8 million.
He sold such property for Rs. 5.2 million on 20X-71/8/15. Answer the followings
mentioning the relevant provisions of Income Tax Act/Rules. (Dec 2015, 5 Marks)
a. Calculate capital gain and tax thereon. Can Ram claim such tax as advance tax?
Answer
Incomings from disposal of NBCA = Rs. 5.2 Million
Outgoings of the NBCA = Rs. 2.8 Million
Gain on Disposal of NBCA = Incomings – Outgoings = Rs. 2.4 Million

There is no such concept of capital gain and capital gain tax in Income Tax law, since
the gain on disposal is treated part of investment income.

As per Sec. 95Ka, the gain on disposal of land and building is subject to collection of
advance tax by malpot karyalaya at the rate 2.5% of gain, if the ownership of such land
and building is 5 years or more. In the given case, the collection of advance tax by Malpot
karyalaya is Rs. 60,000.

The amount can be claimed as advance tax.

b. What shall be your answer if such property was purchased on 20X-60/4/15 and Mr.
Ram has been residing continuously?
Answer
As per Sec. 2, the definition of NBCA excludes private building having owned
continuously for 10 years or more and having resided for 10 years or more, either
continuously or intermittently.

Assuming, the building occupies total land area and the land area is within 1 ropani, if
such property was bought on 20Z-60/4/15 and having been resided since then, the private
building is not within the scope of definition of NBCA.

Further, the private building cannot be treated as business asset, trading stock or
depreciable asset as per the definition provided in the Act, as such, the gain on disposal
is exempt from Income Tax.

c. Will your answer be different if such property was purchased on 20X-69/4/15 and
sold for Rs. 0.930 million?
Answer
As per Sec. 2, the definition of NBCA excludes land and/or private building disposed off
for less than Rs. 1 Million.

30
Originally, it was Rs. 2.9 Million, now question modified to make it relevant as per the change in law.

© The Institute of Chartered Accountants of Nepal 287


CAP II Paper 7 Income Tax and VAT

In case the property is sold for Rs. 0.9 Million, the property does not meet the definition
of NBCA.

Further, the property cannot be treated as business asset, trading stock or depreciable
asset as per the definition provided in the Act, as such, the gain on disposal is exempt
from Income Tax.

200. JKL Limited purchased 100 shares of MNO Bank Limited for each Rs. 200 each
including brokerage and other expenses, as on Chaitra 19, 2058, the company was
holding 250 shares of MNO including the shares purchased and bonus shares paid by
MNO. The quoted closing price on Chaitra 18th, 2058 on MNO shares was Rs. 800 each.

Calculate the gain from the disposal of the shares under the circumstances given below:
(Dec 2014, 10 Marks)

Assumption: 100 shares purchased before Chaitra 19, 2058 and total 250 shares were held
on Chaitra 18, 2059

a. On Jestha 15, 20X-71 the company had disposed of the total shares at the rate of Rs.
1,000 each. The brokerage paid was 1.2% of the sales amount.
Answer
Incomings from the disposal = 250 shares * Rs. 1,000 per share= Rs. 250,000
Outgoings =
Shares held on 2058-12-18 = 250 shares * Rs. 800 per share = Rs. 200,00
Brokerage charge on sales = Rs. 250,000 * 1.2% = Rs. 3,000
Total Outgoings = Rs. 203,000
Gain = Incomings – Outgoings = Rs. 47,000

b. On Jestha 15, 20X-71 the company had disposed the total shares at the rate of Rs.
750 each. The brokerage paid was 1.2% of the sales amount.
Answer
Incomings from the disposal = 250 shares * Rs. 750 per share= Rs. 187,500
Outgoings =
Shares held on 2058-12-18 = 250 shares * Rs. 800 per share = Rs. 200,00
Brokerage charge on sales = Rs. 187,500 * 1.2% = Rs. 2,250
Total Outgoings = Rs. 202,250
Gain = Incomings – Outgoings = Rs. 14,750 (loss)

© The Institute of Chartered Accountants of Nepal 288


Compilation of Suggested Answers Income Tax

c. On Falgun 10, 20X-70 the company had received 125 bonus shares for the 250 shares
held by it. On Jestha 15, 20X-71 the company had disposed 250 shares at the rate of
Rs. 750 each. The brokerage paid was 1.2% of the sales amount.
Answer
Per share
Particulars Amount
value
Incomings from Sale of 250 shares 187,500.00 750
Outgoings
Value of 250 shares 200,000.00 800
Value of 125 bonus shares 12,000.00 100
Total Cost 212,000.00
Weighted average cost per share 565.33
Outgoings of sold shares 141,333.33
Brokerage charge to sale shares 2,250.00
Total Outgoings of 250 shares 143,583.33

Gain on disposal 43,916.67

201. Mr. Anil has held 500 shares, at the rate of Rs. 100 per share (par value), of Standard
Chartered Bank Nepal Ltd (listed on Nepal Stock Exchange Ltd). He has requested to
the stock broker to sell those shares held by him. Mr. Anil has purchased those shares
at the rate of Rs. 1,000 per share. He has paid Rs. 4,000 as brokerage commission at the
time of purchase of shares. He sold those shares at the rate of Rs. 7,000 per share and
paid the brokerage commission of Rs. 17,500 to the broker on sale.
a. Compute the gain on shares disposal.
Answer

No. of Per share


Particulars Total
shares value

Incomings from sale 500 7000 3,500,000


Outgoings
Initial Purchase cost 500 1000 500,000
Brokerage cost during
purchase 4,000
Brokerage cost during sales 17,500
Outgoings related to sold
shares 521,500

Gain on Disposal 2,978,500

© The Institute of Chartered Accountants of Nepal 289


CAP II Paper 7 Income Tax and VAT

b. Is there any implication of withholding taxes? If yes, calculate the payment to be


made to Mr. Anil stating relevant provisions of the Income Tax Act, 2058.
Answer
There is no implication of withholding taxes. However, advance tax is collected by Nepal
Stock Exchange Ltd. from the gain on disposal at the rate 5% of gain, as per Sec. 95ka
of the Act.

The amount of advance tax to be paid to Nepal Stock Exchange Ltd. is Rs. 148,925.

c. What will be the implication if the above-mentioned shares were held and sold by an
entity instead of Mr. Anil? (June 2012, 6 Marks)
Answer
If those shares were held and sold by entity:
i. The rate of advance tax collection by Nepal Stock Exchange Ltd. would be 10% of
gain, and the amount of advance tax would be Rs. 297,850.
ii. The amount of gain would be included in income from business of the entity, and
would be taxed at corporate tax rate.

202. Smart Pvt. Ltd. purchased a piece of land on Ashwin 25, 20X-64 for Rs. 10,00,000 and
incurred an expenditure of Rs. 1,50,000 on registration and brokerage. It constructed
a building on the land costing Rs. 25,00,000. The building was ready for use on Shrawan
25, 20X-65. The depreciated value in Block A was Rs. 10,00,000 at the end of year 20X-
64/X-65 excluding the cost of the building under construction. During the year 20X-
65/X-66 the company capitalized Rs. 60,000 as repair and improvement cost in Block
A.

On Shrawan 1, 20X-66, the company sold the land and newly constructed building for
Rs. 40,00,000. The market value of land on that date was Rs. 15,00,000 and that of the
building was Rs. 25,00,000. You are required to calculate taxable gain for the disposal
of land and building for the year 20X-66/X-67. (June 2011, 5 Marks)
Answer
The land is business asset and the building is depreciable asset. Since, all the building in
Pool ‘A’ are not disposed, there is existence of Pool ‘A’ at the end of 20X-66/X-67.

Calculation of Gain on Disposal of Land:

Particulars Amount Notes


Incomings 1,500,000 Market value as on the date of sales

Outgoings

© The Institute of Chartered Accountants of Nepal 290


Compilation of Suggested Answers Income Tax

Purchase cost 1,000,000


Registration Charges 150,000
Total Outgoings 1,150,000

Gain on Disposal 350,000

Calculation of effect of disposal of Building


Particulars 20X-65/X-66 20X-66/X-67 Notes
Opening Depreciation Base 1,000,000 3,385,000
Absorbed Additions 2,500,000 - Added in Ashoj
Disposal Proceeds - (2,500,000)
Depreciable Basis 3,500,000 885,000
Depreciation Rate 5% 5%
Depreciation Amount 175,000 44,250
Unabsorbed Additions - -
Capitalized Repair Cost 60,000 - Given
Opening Depreciation Base for next Year 3,385,000 840,750

Since, the depreciable basis for I.Y. 20X-66/X-67 is positive and there is existence of assets
in the pool, there is no gain on disposal of building.

© The Institute of Chartered Accountants of Nepal 291


CAP II Paper 7 Income Tax and VAT

Chapter 14- Other Matters

Taxpayers’ Right and Taxpayers’ Identification


203. What are the rights of a tax payer as per Income Tax Act, 2058?
(Dec 2011, 4 marks, CA Inter)
OR
As Section 74 (2) of Income Tax Act-2058, briefly mention the rights of taxpayers.
(June 2003, 5 Marks, CA Inter)
Answer:
The following rights are available to a taxpayer:
a. Right to get respectful behavior;
b. Right to receive any information related to tax as per the prevailing Laws;
c. Right to get an opportunity of submitting a proof in one’s own favor in respect of tax
matters;
d. Right to appoint lawyers or auditors for defense; and
e. Right to secrecy in respect of tax matters and to keep it inviolable.

204. What is Permanent Account Number? Explain the provisions relating to Advance
Rulings. (Dec 2003, 1+3, CA Inter)
Answer
Permanent Account Number:
IRD can issue a unique identification number to identify a taxpayer. The unique identification
number is Permanent Account number (interchangeably called Taxpayer’s Identification
number- TPIN), which is issued by IRD to identify a taxpayer under the power conferred by
Sec. 78 (1) of the Act.

Besides IRD, the PAN can also be issued by entity authorized by Department to issue such
number. The list of such authorized department is produced in the website of IRD, and
extracted (as on the date of writing this book) as part of End Note of this book. The authorized
entity shall issue permanent account number by abiding the provisions of Income Tax Act.
But there is restriction on such taxpayer obtaining PAN number from other authorized entity
than IRD to conduct foreign trade, i.e. the taxpayer cannot conduct import or export
transaction for particular period after obtaining PAN.

Provisions related to Advance Rulings


In case a person makes a written application to IRD seeking IRD’s position regarding the
application of this Act with respect to an arrangement proposed or entered into by the
person, IRD under signature of DG may issue, in writing, an advance ruling in this regard.
IRD shall not issue such an advance ruling on the matters under consideration of any court
or decided by a court.

© The Institute of Chartered Accountants of Nepal 292


Compilation of Suggested Answers Income Tax

Under the following circumstances the ruling shall be binding on the every officer of IRD,
on IROs and a civil servant who is authorized to work as tax officer:

The applicant has made a full and true disclosure of all aspects of the arrangement relevant
to the ruling; and
a. The arrangement proceeds in all material respects as described in the application for the
ruling.
b. Where the advance ruling issued by IRD contradicts with a public circular, for the person
who applied for the advance ruling, the advance ruling shall be applicable.

Donations
205. What is the provision under sec 12Ka of Income Tax Act 2058 as regards expenses
incurred towards ancient, religious and cultural heritage conservation and sports
development activities? (Dec 2010, 5 Marks)
Answer
Conditions to claim Expenditure u/s 12Ka
All the following conditions shall be satisfied for any expenditure to be eligible to be claimed
under this Section:
a. The person shall be a Company,
b. The person shall obtain prior approval of IRD to incur such expenditure, and
c. The expenditure shall be incurred for conservation and promotion of ancient, religious
or cultural heritages established in Nepal; and/or construction of public infrastructure
related to the Sports

Eligible Amount for deduction to arrive at Taxable Income


The lower of the following amount is eligible for deduction:
a. Actual expenditure incurred
b. 10% of the Assessable income during the year
c. Rs. 10 Lakhs

Special Provisions related to Natural Person & Entity


206. Write in brief the provisions of Income Tax Act in case of a married resident couple
elect to be treated as a couple. (Dec 2010, 5 Marks)
Answer
A resident natural person and his/her resident spouse (husband or wife) may elect (select)
to be treated as Couple by giving written notification to IRD for any Income year for Tax
purpose.

© The Institute of Chartered Accountants of Nepal 293


CAP II Paper 7 Income Tax and VAT

In case a resident natural person & his/her resident spouse elect to be assessed as Couple
for the income pertaining to any Income Year, the natural person & his/her spouse is jointly
& severally (separately) responsible for payment of tax pertaining to that Income Year.

A Widower or widow is treated as Couple for tax purpose, if s/he has to look after the
dependent.

207. The paid-up share capital of ABC Ltd. consists of 1,50,000 shares of Rs. 100 each. As
of Ashad end 20X-68, the Profit and Loss Appropriation Accounts of the company is
abstracted as below:
Profit and Loss Appropriation Account:
Profit for the year Rs. 10,00,000
Accumulated profit b/d Rs. 15,00,000
Total distributable profit Rs. 25,00,000

The Board of Directors (BOD) of the company proposed to issue bonus share at 10%.
Also, it is proposed that dividend tax on the capitalized profit (sec 88(1)) is to be borne
by company by distributing cash dividend so that shareholders do not have to pay any
tax at source as required under sec. 88(1).

You are required to compute the amount of such cash dividend. (Dec 2011, 5 Marks)
Answer
The dividend tax @ 5% is to be borne by the shareholders. If shareholders receive any
amount, the amount is always 95% of total dividend declared by the company.

In the given case, amount of bonus share is 10% of paid up capital of the company, i.e. 10%
of Rs. 15,000,000 which comes to Rs. 1,500,000.

As per the information in the question, the company does not collect cash from shareholders
in order to pay dividend tax, but is proposing cash dividend in order to fulfill that obligation,
which means, the amount of bonus share is 95% of total dividend (total dividend being sum
of bonus share and cash dividend). When we develop a mathematical formulation:
Total dividend = Bonus share + Cash dividend-------- (i)
Bonus share = 95% of Total Dividend------- (ii)
Equating (ii)-
Rs. 1,500,000 = 95% of Total Dividend
Or, Total Dividend = Rs. 1,500,000/0.95 = Rs. 1,578,947.37

From equation (i), pasting value of total dividend and bonus share gives us cash dividend to
be proposed of Rs. 78,947.37.

© The Institute of Chartered Accountants of Nepal 294


Compilation of Suggested Answers Income Tax

Retirement Funds, Finance Lease, etc.


208. Discuss the taxability of the gain from the amount received from Un-approved
Retirement Fund. (June 2007, 5 Marks, CA Inter)
Answer
The gain amount from unapproved retirement fund is taxed at the rate 5% as per Sec. 88 (2)
and it is final withholding as per Sec. 92, when it is paid by a resident unapproved retirement
fund.

The gain amount is included in calculating investment income of a resident natural person,
when it is paid by a non-resident unapproved retirement fund, and the taxation on such
income is subject to Sec. 1 of Schedule 1 of the Act.

For a non-resident person, gain from non-resident unapproved retirement fund is not taxable
in Nepal as per Sec. 6.

209. State the conditions for finance lease. (June 2006, 3 Marks, CA Inter)
Conditions of Finance Lease: If any of the following conditions is satisfied, a lease is treated
as finance lease for tax purpose:
a. Ownership Transfer test: In case the lease agreement includes a clause that the ownership
of the asset is transferred after the end of lease term
b. Option to purchase asset Test: The lessee has the option to purchase the asset at
predetermined or fixed price at the expiry of lease term
c. Term of lease test: The term of lease is more than 75% of the useful life of the asset,
d. Consumption of value test: The expected market price at the end of lease term is less than
20% of the market price prevailing at the inception of the lease
e. Present value of lease rental test: In case of a lease that is started before the last 25% of
the useful life of the asset, the present value of minimum lease rental is 90% or more of
the market value of the asset prevailing at the inception of the lease term
The discount factor to determine present value of minimum lease rental shall be General
Interest Rate, which is 15% p.a. (2075/76)
f. Specific nature of asset test: The leased assets are of such a specialized nature that only
the lessee can use them without major modifications

© The Institute of Chartered Accountants of Nepal 295


CAP II Paper 7 Income Tax and VAT

Payment of Tax and Tax Return

Chapter 15: Form, Place and Time of Payment

Time of Payment of Tax


210. Mention the timings of payment of taxes under the Income Tax Act, 2058 prescribed in
the following cases: (July 2015, 5×1=5)
a. Deposit of amount of withholding taxes by the person who is required to withhold
Taxes.
Answer:
Within 25 days of end of month, month implies the calendar month as per Nepali
Calendar.

b. Amount of Tax payable by a person who has submitted the Income Tax Return as
per section 96 and who has due to be paid as per assessment made under section 99
Answer
Within due date of filing income return under Sec. 96, which is usually three months of
the end of Income Year

c. Amount of Tax payable under jeopardy assessment under section 100 (2)
Answer
Within such due date that is specified in the notice issued by IRD u/s 102

d. Amount of Tax payable under amended assessment under section 101


Answer
Answer
Within such due date that is specified in the notice issued by IRD u/s 102

e. Installments amount of Tax payable based on estimated Tax


Answer
Within Poush, Chaitra and Ashad, an amount not less than 40%, 70% and 100% of
estimated tax liability respectively on cumulative basis in case of taxpayers who has to
pay tax on installments other than a turnover based tax payer
For turnover based taxpayer, due date is Poush and Ashad.

© The Institute of Chartered Accountants of Nepal 296


Compilation of Suggested Answers Income Tax

Chapter 15: Withholding Taxes and Collection of Advance Tax by Agent

Collection of Advance Tax by Agent


211. Mr. Komal purchased a piece of land at Rs. 30 lakhs. He sold the land at Rs. 45 lakhs.
He paid registration expenses Rs. 2 lakhs for this land. In this case, what would be the
tax implications on the following situations? (Dec 2014, 5 Marks)

The gain is Rs. 13 Lakhs (i.e. 45 lakhs minus 30 Lakhs minus 2 Lakhs)

a. The land was purchased on Chitra 20X-64 and sold it on Magh 20X-70.
Answer
The land is owned for more than 5 years, as such, the tax on gain is 2.5%, to be paid at
Malpot Karyalaya at the time of ownership transfer.

b. The land was purchased on Magh 20X-67 and sold it on Magh 20X-70.
Answer
The land is owned for less than 5 years, as such, the tax on gain is 5%, to be paid at
Malpot Karyalaya at the time of ownership transfer.

c. The land was purchased on Chitra 20X-59 and sold it on Baishak 20X-71.
Answer
The land is owned for more than 5 years, as such, the tax on gain is 2.5%, to be paid at
Malpot Karyalaya at the time of ownership transfer.

d. If selling and buying of the land were completed through a sole shareholder of a Pvt.
Ltd. The shareholder is Mr. Komal.
Answer
The company would have paid advance tax at Malpot karyalaya @ 1.5% of sales value
of land, which is then treated as advance tax when determining the corporate tax liability
of the company.

Withholding Tax
212. What is the applicable withholding tax amount on the following: (June 2019, 5 Marks)
a. Service fees of Rs. 50,000 paid for VAT exempted services.
Answer- 1.5% of Service fee amount
Withholding tax Amount= 1.5% of Rs. 50,000 = Rs. 750

b. ABC Limited paid house rent Rs. 100,000 to landlord Mr. Sharma.
Answer: As per definition of "Rent", natural person's house rent income that is not
generated through a private firm does not fall under the definition of rent, which means

© The Institute of Chartered Accountants of Nepal 297


CAP II Paper 7 Income Tax and VAT

such income is exempt from income tax. As per Sec. 88 (4), there is no withholding tax
requirement on exempt income.

c. MNO Ltd. paid Rs. 50,000 as freight charges to Sulabh Transport Co.
Answer- 2.5%
Withholding tax Amount= 2.5% of Rs. 50,000 = Rs. 1,250

d. Bank of Kathmandu paid Rs. 500,000 as interest in fixed deposit account maintained
by Sagarmatha Mutual Funds.
Answer: As per Sec. 88 (4), there shall be no withholding of taxes when interest or
dividend is paid to a mutual fund.

e. ABC Ltd. provides car worth Rs. 2,500,000 to the winner in the lottery scheme of the
Co.
Answer: As per Sec. 88Ka, the withholding tax rate is 25%, and the amount is 25% of
Rs. 25 Lakhs, i.e Rs. 6.25 lakhs.

213. Write the tax implications on the following cases as per Income Tax Act, 2058 :
(Dec 2018, 5 Marks)
a. Smooth Telecom Pvt. Ltd. deposits all gratuity amounts into an Approved
Retirement Fund. On 30 Bhadra, 20X-75, the Fund paid for gratuity Rs. 30 lakhs to
Mr. Ram upon termination of his job.
Answer
It is assumed that the employers operate employers-single-account in Approved
Retirement Fund in order to deposit gratuity, in which case, TDS is applicable on
payment of gratuity to Ram. Smooth Telecom is withholding agent and it needs to
withhold tax @ 15% of gratuity paid, i.e. Rs. 450,000.

For Ram, it is a final withholding payment.

b. Mr. Shayam owns a truck for providing transportation services. He is paying annual
tax as per Income Tax Act, 2058. During Ashwin 20X-75, he has not issued any
invoice but has earned transportation services fee amounting to Rs. 2 lakhs.
Answer
The income from truck operation is subject to income taxes under the head income from
business.

c. Kathmandu Metropolitan City (KMC) has an agreement with Kalimati Users


Committee for drain construction; the agreement was made on 30 Baishakh, 20X-75.

© The Institute of Chartered Accountants of Nepal 298


Compilation of Suggested Answers Income Tax

KMC paid Rs. 70 lakhs to the Committee on 10 Ashwin, 20X-75 after completion of
the works.
Answer
As per Sec. 89 (3Ka), when any work is operated through Consumer (User) Committee
and where the payment exceeds Rs. 50 Lakhs, tax shall be withheld by payer @ 1.5% of
payment, i.e. Rs. 105,000.

d. An NGO having tax exempt certificate, has booked income from house rent Rs.
500,000 during the year 20X-74/X-75.
Answer
The house rent income is subject to corporate tax of NGO, as it is not an exempt amount
under the conditions of Sec. 10 (Chha).

e. A Pvt. Ltd. has booked as income from sale of Car Rs. 300,000 on Ashwin, 20X-75.
The Company is registered with the objectives of providing transportation services.
Answer
The car is a depreciable asset of A Pvt. Ltd., the amount from sale of car is deducted
while calculating depreciable basis of Block C Assets.

214. Mr. Ramji Paneru retired in 2075 Baishakh end from 30 years of his service with Nepal
Timber Corporation and received lump sum retirement payments as follows:
(June 2018, 5 Marks)
a. Payment from approved retirement fund Rs. 1,675,852.00.
b. Payment from unapproved retirement fund Rs. 675,800.00 to which his contribution
was Rs. 511,256.00. This payment is related only with the period after Income Tax
Act, 2058.
You are required to calculate the tax withholding on these payments along with reason
there for.
Answer
a. To end up with 30 years of service, Mr. Paneru had joined the service from 2055 Jestha
1. That means, he earned retirement benefit from 2055 Jestha 1 to 2058 Chaitra 18 and
thereafter. Out of 360 months, payment in relation to 46.5 months is tax exempt.
Exempt payment = 1,675,852/360*46.5 = 216,464
Taxable Portion= Rs. 1,459,388
Deduction of 50% of taxable portion or Rs. 500,000 whichever is higher
Gain = Rs. 729,694
Tax @ 5% on Gain = Rs. 36,484

© The Institute of Chartered Accountants of Nepal 299


CAP II Paper 7 Income Tax and VAT

b. Amount of Gain from Unapproved Retirement Fund is Rs. 164,544 (675,800-511,256).


Tax is 5% of Gain, i.e. Rs. 8,227.20.

215. Calculate the TDS to be deducted on following cases:– (June 2017, 5 Marks)
a. Mr. Santosh Sharma has bought a house on 20th Asoj 20X-67 for Rs. 90,00,000
including registration expenses and other expenses. Again, he has purchased a plot
of land on 3rd Chaitra 20X-70 for Rs. 1,50,00,000 including registration expenses
and other expenses. He sold both of the property on 22nd Mangsir 20X-73 to Mr.
Binod Sharma for Rs. 3,00,00,000 and Rs. 3,50,00,000 respectively.
Stating the relevant provisions of the Income Tax Act, 2058, advise whether TDS needs
to be deducted or not in the said transaction, and if yes, advise who will deduct the TDS
and amount of TDS to be deducted on the above transaction.
Answer
Withholding of tax is not applicable on disposal of Non-Business Chargeable asset, as
tax withholding is dealt by Sec. 87- 89 of the Act and such sections do not cover gain on
disposal of non-business chargeable asset.

However, Sec. 95Ka, Collection of Advance Tax by Agent, is applicable in such


circumstances, in which case if a natural person disposes his land and building
qualifying as non-business chargeable asset, advance tax must be paid at Malpot
karyalaya at the time of property registration. The rate of advance tax is 5% in case of
ownership period less than 5 years and 2.5% in all other cases.

In the given case, property bought on 20th Ashoj 20X-67 is sold after 5 years, which
means gain on disposal of such property is subject to payment of advance tax @ 2.5%.
Gain amount is Rs. 21,000,000 and payment of advance tax is Rs. 525,000.

Further, property bought 3rd Chaitra 20X-70 is sold before 5 years, which means gain
on disposal of such property is subject to payment of advance tax @ 5%. Gain amount is
Rs. 20,000,000 and payment of advance tax is Rs. 1,000,000.

b. Mr. Shyam Silwal and Everest Holdings Pvt. Ltd. are shareholders of Shree Holdings
Pvt. Ltd. Mr. Shyam Silwal holds 5,000 shares and Everest Holdings Pvt. Ltd. holds
10,000 shares of Shree Holdings Pvt. Ltd. which is 10% and 20% shares of total
shares of Shree Holdings Pvt. Ltd. respectively.
Mr. Shyam Silwal has purchased said share from Mr. Naresh Rana on 25 Mangsir
20X-65 for Rs. 22,50,000. He sold the said share on 25 Poush 20X-73 for Rs.
30,00,000. He has paid interest of Rs. 5,00,000 against loan taken for purchase of
the said share.

© The Institute of Chartered Accountants of Nepal 300


Compilation of Suggested Answers Income Tax

Everest Holdings Pvt. Ltd. is a promoter shareholder of the Shree Holdings Pvt.
Ltd., and has paid Rs. 10,00,000. Everest Holdings Pvt. Ltd. sold the said share on
25 Poush 20X-73 for Rs. 60,00,000. Everest Holdings Pvt. Ltd. has purchased the
said share from their own capital.
Stating the relevant provisions of the Income Tax Act, 2058, advise Shree Holdings
Pvt. Ltd. whether TDS needs to be deducted or not in the said transaction, and if yes,
compute TDS amount to be deducted by Shree Holdings Pvt. Ltd. on the above
transaction.
Answer
Withholding of tax is not applicable on disposal of Non-Business Chargeable asset, as
tax withholding is dealt by Sec. 87- 89 of the Act and such sections do not cover gain on
disposal of non-business chargeable asset.

However, as per Sec. 95Ka, advance tax is applicable on transaction of securities. The
rate of advance tax is 10% of gain in case securities of non-listed entities are held by
resident natural person; and 15% of gain in case securities of non-listed entities are held
by resident entity.

Applicability of Tax
Gain on Disposal for Mr. Shyam Silwal= Incomings – Outgoings = Rs. 30 Lakhs – Rs.
27.50 Lakhs = Rs. 2.5 Lakhs (Interest expense on loan for purchase of shares form part
of outgoings)
Advance tax to be collected by Shree Holdings Pvt. Ltd. = 10% of Gain = Rs. 25,000
(assuming Mr. Shyam is resident, otherwise it is 25% of gain, i.e. Rs. 62,500)

Gain on Disposal for M/s Everest Holdings Pvt. Ltd.= Incomings – Outgoings = Rs. 60
Lakhs – Rs. 10 Lakhs = Rs. 50 Lakhs
Advance tax to be collected by Shree Holdings Pvt. Ltd. = 15% of Gain = Rs. 750,000
(assuming the company is resident, otherwise it is 25% of gain, i.e. Rs. 1,250,000)

216. Calculate tax on source (TDS) amount, if applicable, for 20X-72/X-73 as per Income
Tax Act, 2058. (Dec 2016, 5 Marks)
a. Manhood Pvt. Ltd. has borrowed loan amounting of Rs. 1 crore from Boston
International Pvt. Ltd (Non-resident) and paid interest Rs. 10 lakhs.
Answer
The TDS rate is 15% and the TDS amount is Rs. 150,000 (Sec. 88 (1))

b. Kalika Mohan JV has distributed its profit amounting of Rs. 10 lakhs.


Answer

© The Institute of Chartered Accountants of Nepal 301


CAP II Paper 7 Income Tax and VAT

A joint venture is a company under the definition of Income Tax Act, and when a resident
company distributes dividend, it has to withhold tax at the rate 5% of distributed amount
as per Sec. 88 (2). Therefore, TDS amount is Rs. 50,000.

c. UNICEF Nepal (Tax Exempt Entity) has paid vehicle rent amounting of Rs. 200,000
to the Flexible transport Private Ltd.
Answer
Assuming Flexible Transport is not registered for VAT, UNICEF shall withhold tax @
10% as per Sec. 88 (1). Therefore, UNICEF Nepal shall withhold Rs. 20,000 as tax.

Assuming the vehicle was hired for the purpose of transportation of goods, the
withholding tax would be 2.5% (not registered for VAT) of Rs. 200,000.

Note that, Tax exemption is only related to relief from payment of income taxes, not from
the obligation to withhold taxes.

d. Amount paid by an unapproved retirement fund Rs. 15,00,000 to it’s a contributor.


Total amounting Rs. 12,00,000 has been deposited by the contributor to the
retirement fund during the period of 12 years.
Answer
The gain is Rs. 300,000 and it is subject to Withholding tax @5%, i.e. Rs. 15,000 shall
be withheld by Unapproved Retirement Fund u/s 88 (2).

e. Complete Private Ltd. has paid rental charges amounting of Rs. 1,200,000 to the
Employee Provident Fund in 20X-72/X-73.
Answer
Income of an approved retirement fund is exempt from tax. As per Sec. 88 (4), there shall
be no withholding of taxes while making payments of amounts exempt from tax.
Therefore, there shall be no withholding of taxes.

If the previous version of suggested answer deliberated on withholding of taxes, students


should reason that penalty of violation of Rule 20 of Income Tax Regulation, 2059 is
termination of approval status of an approved retirement fund and taxes cannot be levied
on income of an approved retirement on the ground that Rule 20 is violated, without
terminating its status as ARF.

217. Mr. Pravan has got retirement on Ashad end, 20X-72 from government service after
completing 12 years. The approved and unapproved retirement fund has providing 10
percent interest rate on its contribution. The interest is providing monthly basis.

© The Institute of Chartered Accountants of Nepal 302


Compilation of Suggested Answers Income Tax

He has received statements from these funds, that statements show the following
balances as on first Shrawan, 20X-72.
• Approved fund: 1,500,000 including total interest of Rs. 300,000
• Unapproved fund: Rs. 800,000 including total interest of Rs. 150,000
He came with you for the payment of the balance amount on 30th Ashoj, 20X-72.
Assume you are the finance officer; calculate the TDS on these payments and mention
why have you deducted the TDS? (Dec 2015, 5 Marks)
Answer
a. Payment from Approved Retirement Fund= Rs. 1,500,000
Deduction of 50% of taxable portion or Rs. 500,000 whichever is higher
Gain = Rs. 750,000
Tax @ 5% on Gain = Rs. 37,500
Tax is withheld as per Sec. 88 (1).

b. Amount of Gain from Unapproved Retirement Fund is Rs. 150,000 (interest portion). Tax
is 5% of Gain, i.e. Rs. 7,500. Tax is withheld as per Sec. 88 (2).

218. Mr. Ramesh has been retired from Government of Nepal on 15th Jestha, 20X-71. He
has received the following retirement payments in Ashad, 20X-71. He has not any other
sources of income except salary. Remuneration tax already has been deducted and
deposited. Assume no retirement payment was accrued at the commencement of this Act
in connection with this employment. Calculate the amount of tax that must be paid by
Mr. Ramesh. (Dec 2014, 5 Marks)
Particulars Amount (Rs.)
Payment from GON against accumulated leave and medical Rs. 6,00,000
allowances
Payment from Employee Provident Fund against contribution Rs. 7,00,000
Payment from Citizen Investment Trust against contribution Rs. 5,00,000
Answer
Total Payment from Approved Retirement Fund- Rs. 1,800,000
Deduction of 50% of payment or Rs. 500,000; whichever is higher
Gain = Rs. 900,000
Tax to be paid Mr. Ramesh = 5% of Gain = Rs. 45,000

219. Explain the following with reasons; on tax/withholding taxes and implication thereon,
with reference to Income Tax Act, 2058. (Dec 2013, 5×2=10)
a. Z & Co. has taken loan from M/s Nabil Bank Ltd. amounting Rs. 10 crores for
conducting its business. The company has paid interest at the rate of 10% amounting
to Rs. 1 crore during income year 20X-69/X-70.
Answer

© The Institute of Chartered Accountants of Nepal 303


CAP II Paper 7 Income Tax and VAT

As per Sec. 88 (4), there shall be no withholding of taxes when interest is paid to a
resident banks or financial institutions. Assuming Nabil Bank is resident of Nepal, Z &
Co. shall not withhold tax while making payment of interest.

b. Mr. Gokul has given a contract to Z & engineers, a construction company for
construction of his residential building for Rs. 1 crore. While making payments to Z
& engineers, Mr. Gokul does not deduct any withholding taxes.
Answer
As per Sec. 88 (4), a natural person shall not withhold tax in any payment except when
it is related to operation of business. In the given case, Mr. Gokul is not making payment
in connection to operation of business, as such, there shall be no withholding of taxes.

c. Ms. Rakhi has taken the insurance policy of Rs. 16,00,000 from Rastriya Beema
Sansthan. She pays annual premium of Rs. 80,000. After maturity of the policy, she
is receiving Rs. 24,00,000 from Rastriya Beema Sansthan.
Answer
Assuming that Mr. Rakhi has paid total premium of Rs. 1,600,000 which in turn gives us
gain from investment insurance of Rs. 800,000. Tax withholding @ 5% is applicable on
payment of gain by resident investment insurance company, that means, tax to be
withheld by Rastriya Beema Sansthan is Rs. 40,000 as per Sec. 88 (2).

d. Mr. Smart has won the cash lucky draw of Rs. 1 million from a trading company
under its scheme. He claims that the amount is not subjected to any tax deduction
and wants to receive full payment.
Answer
Winning a lottery is windfall gain, and windfall gain is subject to 25% withholding taxes
as per Sec. 88Ka. As such, trading company shall withhold Rs. 250,000 and pay
remaining balance to Mr. Smart.

e. X & Co. made the following payment to Z & Co., under a contract, for the supply of
materials required for its company. Both the companies are registered under Value
Added Tax (VAT).
Date Amount (Rs.)
20X-69.04.08 25,000
20X-69.04.12 10,000
20X-69.04.13 5,000
20X-69.04.15 10,000
20X-69.04.17 5,000
Since the payments are less than Rs. 50,000, X & Co. claims no deduction of
withholding taxes on such payments.

© The Institute of Chartered Accountants of Nepal 304


Compilation of Suggested Answers Income Tax

Answer
As per Sec. 89, where payment related to a contract exceeds Rs. 50,000 after considering
all payments of moving 11 days, tax shall be withheld @ 1.5% of payment.
In the given case, there is no requirement of withholding taxes on 20X-69 Shrawan 8, 12,
13 and 15 as the payment in moving 11 days is Rs. 25,000, Rs. 35,000, Rs. 40,000 and
Rs. 50,000 respectively.

However, payment exceeds Rs. 50,000 on Shrawan 17, total payment in moving 11 days
is Rs. 55,000 and provision of Sec. 89 is applicable. Since, no payments before Shrawan
17 is subject to withholding taxes, tax shall be withheld on Rs. 55,000 @ 1.5% on
Shrawan 17.

220. X & Co. has received from Y & Co., a resident company, Rs. 10 lakhs as dividend after
deduction of taxes. The profit of X & Co. before inclusion of such dividend income is
Rs. 90 lakhs. X & Co. has decided to declare the dividend distribution to its
shareholders during current fiscal year amounting to Rs. 100 lakhs (including Rs. 10
lakhs of dividend received from Y & Co.). Mr. Bibhu is one of the shareholders of X &
Co. and supposed to receive from Rs. 5 lakhs as dividend. Mention the relevant
provision and calculate the amount of Tax to be deducted while making payment to
Mr. Bibhu. (Dec 2013, 5 Marks)
Answer
As per Sec. 54 (3), there shall be not be applicability of dividend tax on distribution of such
profit, on which dividend tax as per Sec. 54 is already levied.

In the given case, dividend is distributed out of total profit of Rs. 10 Million, of which Rs. 1
Million is dividend received from Y & Co., i.e. 10% of total distributable profit is dividend
received from another resident company subject to application of Sec. 54 (3) and hence, tax
free.

Out of dividend received by Mr. Bibhu, 10% of dividend is tax free, i.e. Rs. 50,000 is tax free.
Dividend tax is applicable on Rs. 450,000 @ 5%. As such, amount of tax to be withheld u/s
88 (2) is Rs. 22,500.

221. Explain the followings with reasons; on tax/withholding taxes and implication thereon,
with reference to the Income Tax Act, 2058. (June 2013, 5×2=10)
a. X & Co., Chartered Accountants, has entered into a contract for tax consultancy
services to Z Ltd. for Rs. 200,000 per month. X & Co. raised VAT invoice for the
month of Shrawan, 2068 amounting to Rs. 226,000 (Rs. 200,000+13%VAT = Rs.
226,000).
Answer

© The Institute of Chartered Accountants of Nepal 305


CAP II Paper 7 Income Tax and VAT

As per Sec. 88 (1), payment of service fee to VAT registered service provider is subject
to withholding tax @ 1.5% of payment. Hence, tax to be withheld in this case is 1.5% of
Rs. 200,000 (VAT is payment to Government and not to service provider), i.e. Rs. 3,000.

b. Mr. Hari, an individual person, owned the furnished building situated at Kalanki,
Kathmandu. He has given on lease the furnished building to a development bank for
Rs. 40,000 per month from Shrawan 20X-68. Mr. Hari is not conducting any business
and has rent income only.
Answer
As per definition of "Rent", natural person's house rent income that is not generated
through a private firm does not fall under the definition of rent, which means such income
is exempt from income tax. As per Sec. 88 (4), there shall not be tax withholding when
an income is exempt from tax.

c. Mr. Krishna is an economist. He published an article on "Current Status of


Economy" in Kantipur daily and received Rs. 6,000. He also teaches, occasionally,
"Economics" in an educational institution for which he received Rs. 10,000 for a class
in Income Year 20X-68/X-69.
Answer
As per Sec. 88 (4), there shall be no withholding of tax in respect of payment by
newspaper in relation to articles published in such newspaper. Therefore, Kantipur daily
shall not withhold tax on honorarium to Mr. Krishna for his article title “Current Status
of Economy”. However, Mr. Krishna’s such income is not exempt from income tax.

Further, occasional lecture fee is service fee and subject to withholding tax @ 15% of
payment. Tax to be withheld by Educational Institution is Rs. 1,500. It is final withholding
payment for Mr. Krishna.

d. Mrs. Sila has opened the account in Paschimanchal Rural Development Bank,
Parbat and deposited Rs. 60,000. The interest rate is 8% per annum and Mrs. Sila
earned interest income of Rs. 4,800 during Income Year 20X-68/X-69.
Answer
As per Sec. 11 (2Ka), income tax is exempt on interest income up to Rs. 25,000 earned
on deposit by a natural person during the income year from a rural development bank
based on Rural Municipality. Assuming, the bank in question is located at Rural
Municipality, the interest income of Mrs. Sita is tax exempt, and is not subject to
withholding tax as per Sec. 88 (4).

e. A & Company Ltd. decided to declare and distribute the cash dividend at the rate of
10% amounting to Rs. 50,000 and bonus shares at the rate of 20% amounting to Rs.

© The Institute of Chartered Accountants of Nepal 306


Compilation of Suggested Answers Income Tax

100,000 out of the profit of Income Year 20X-67/X-68, from its annual general
meeting dated Mangsir 28, 20X-68.
Answer
As per Sec. 88 (2), tax shall be withheld @5% on distribution of profit by the company.
Total tax to be withheld is 5% of Rs. 150,000, i.e. Rs. 7,500.

222. Anubhav & Co. has taken on rent the premises from Buddhi Shree & Co for the
operation of its business which pays monthly rent of Rs. 100,000. Anubhav & Co. is
deducting withholding tax at the rate of 10% amounting to Rs. 10,000 per month and
making payment amounting to Rs. 90,000 per month to Buddhi Shree & Co. While
submitting Income Tax Return, tax liability of Buddhi Shree & Co, to be payable
during Income Year 20X-68/X-69 came to Rs. 200,000. Answer the following question
mentioning the relevant provision of Income Tax Act, 2058. (Dec 2012, 8 Marks)

a. What is the amount of tax to be deposited?


Answer
As per Sec. 93, M/s Buddhi Shree & Co. can utilize taxes withheld by agent and not in
respect of final withholding payment as tax credit while determining the tax payable for
the Income Year. As such, Rs. 120,000 can be offset against the total tax liability of Rs.
200,000 and M/s Buddhi Shree & Co. shall pay additional Rs. 80,000 to discharge its tax
obligation.

b. Will your answer be different if tax liability of Buddhi Shree & Co comes to Rs.
100,000 instead of Rs. 200,000?
Answer
Yes, in that case M/s Buddhi Shree & Co. has more tax credit than the tax obligation and
there would be no additional outflow of cash to discharge tax obligation.

c. What is the implication if Buddhi Shree & Co had not adjusted the amount of
withholding tax paid through Anubhav & Co. while submitting income tax return?
Answer
As per Sec. 93, the amount of withholding taxes shall be adjusted while submitted income
return during the year when the related income is included in the return. If not adjusted,
the amount cannot be adjusted in any other year.
However, the taxpayer has the option to apply for tax refund under the provisions of Sec.
113.

223. Mr. Ram, an employee of a local bank accepted the offer of Voluntary Retirement
Scheme (VRS) as part of the Bank's employee reduction plans. As a VRS package, the
bank has decided to provide 2 months basic salary for every completed year of service.

© The Institute of Chartered Accountants of Nepal 307


CAP II Paper 7 Income Tax and VAT

Mr. Ram has taken retirement as part of VRS scheme. As per the rules of the Bank,
employees serving more than 15 years are entitled for 2 month's basic salary for every
completed year of service as gratuity and employees serving more than 20 years are
entitled for 2.5 month's basic salary for every completed year of service as gratuity. He
has taken retirement in the month of Ashad 20X-68 and at the time of retirement, his
basic salary was Rs. 20,000 per month. His other monthly allowance is Rs. 10,000 per
month. During the year 20X-67/X-68, he also received one month's basic salary as
Dashain Allowance. The bank has also contributed 10% of his basic salary as Provident
Fund contribution. At the time of retirement, his total service period was 25 years. The
gratuity amount accrued and payable to him upto Chaitra 18, 2058 is Rs. 950,000 and
the amount of leave payable to him upto Chaitra 18, 2058 is Rs. 50,000. The number of
days leave accrued after Chaitra 18, 2058 and payable on Ashad 20X-68 is 90 days.
Compute his Income from Remuneration for the year 20X-67/X-68 and also calculate
the amount of Retirement Payments as per the provision of the Act. What rate of
withholding tax the bank should apply for payments of VRS, Gratuity and Leave at the
time of Retirement and also the amount of Tax to be deducted for payments of VRS,
Gratuity and Leave at the time of Retirement?

On the other hand, Mr. Hari is a low skilled worker and he has been terminated from
the company as part of Compulsory Retirement Scheme (CRS) and for this he got Rs.
400,000 from the Bank and also as part of CRS package, he commuted the amount of
Pension and lump sum pension of Rs. 3,00,000 has been paid to him at the time of
Retirement. Compute the withholding tax to be deducted in this case by also mentioning
the withholding rate in both the cases? (June 2012, 10 Marks, CA Inter)
Answer
Assessable Income from Employment Mr. Ram
Particulars Sec. Ref. Amount Notes
Basic Salary 8 (2) 240,000
Allowances 8 (2) 120,000
Dashain Allowance 8 (2) 20,000
Employers Contribution to RF 8 (2) 24,000 10% of basic salary
Assessable Income from Employment 404,000

Tax to be deducted at the time of Retirement of Mr. Ram


Particulars Details Total Pay TDS Rate TDS
Amount
VRS after 25 2 months basic 50 X 20,000= Rs. 15% Rs.
years of service salary each year for 1,000,000 150,000
25 years

© The Institute of Chartered Accountants of Nepal 308


Compilation of Suggested Answers Income Tax

Gratuity after 2.5% months basic 62.5 X 20,000 - 15% Rs. 45,000
Chaitra 19, 2058 salary each year for 950,000 = Rs.
25 years 300,000
Gratuity after 950,000 Exempt -
Chaitra 19, 2058
Leave until 2058 50,000 Exempt -
Chaitra 18
Leave after 2058 90 days~ 3 months 3 X 20,000= Rs. 15% 9,000
Chaitra 18 60,000

Tax to be deducted at the time of Retirement of Mr. Hari


Particulars Total Pay TDS Rate TDS Amount
CRS payment 400,000 15% Rs. 60,000
Lump sum Pension 300,000 15% Rs. 45,000

224. Mention, with reasons, the amount of tax that need to be deducted at source/accounted
for in the books in following cases: (June 2012, 5×2=10 Marks)
a. M Ltd. has entered into an agreement with PC Pvt. Ltd. to conduct an awareness
program on maternal health in 5 districts. The total amount paid by M Ltd. against
such agreement was Rs. 40,000. In addition to such amount, M Ltd. also reimbursed
Rs. 20,000 as expenses on air –ticket, lodging and fooding, daily expenses etc. of staffs
of PC Pvt. Ltd. to PC Pvt. Ltd. as per the terms of the agreement. PC Pvt. Ltd. did
not give VAT invoice.
Answer
Total service fee paid to PC Pvt. Ltd. is Rs. 60,000. As per Sec. 88 (1), service fee is
subject to tax withholding @ 15%, as such, M. Ltd. shall withhold tax amounting to Rs.
9,000.

b. M/S ABC Company Limited has 60 shareholders. The company has distributed
iPhone worth Rs. 38,000 to each shareholder free of cost in FY 20X-68/X-69.
Answer
In case a person makes payment in kind, the market value of such kind is treated as the
amount of payment, as per sec. 27.
Payment in the nature as given in this question, to shareholders is treated as dividend as
per Sec. 53, as such, the distribution of iPhone is dividend. There are two possible way
to collect withholding taxes in the given case:
• If the company collects dividend tax from the shareholders, the amount to be collected
from each shareholder is Rs. 1,900 (5% of Rs. 38,000)
• If the company bears the cost of withholding taxes, the shareholders are deemed to
receive Rs. 38,000 net of TDS; which means, Rs. 38,000 is 95% of total dividend, and

© The Institute of Chartered Accountants of Nepal 309


CAP II Paper 7 Income Tax and VAT

the total dividend comes to Rs. 40,000. The dividend tax would be Rs. 2,000 per
shareholder.

c. Mahesh is constructing a building for his personal residence. He has appointed


Ramesh, an engineer, to take care of the overall construction activities at monthly
remuneration of Rs. 30,000.
Answer
As per Sec. 88 (4), a natural person shall not withhold tax in any payment except when
it is related to operation of business. In the given case, Mr. Mahesh is not making
payment in connection to operation of business, as such; there shall be no withholding
of taxes.

d. MNO Company Pvt. Ltd. has taken loan Rs. 10 lakhs from Hamro Bank Limited @
12% Per annum. MNO Company Pvt. Ltd. has paid loan service charge of Rs. 25,000
and interest Rs. 90,000 during the year to the Hamro Bank Limited.
Answer
As per Sec. 88 (4), there shall be no withholding of taxes when interest is paid to a
resident banks or financial institutions. Assuming Hamro Bank is resident of Nepal, MNO
Co. Pvt. Ltd. shall not withhold tax while making payment of interest.

Interest means any payment in excess of principal in respect of loan obligation, as such;
service charge is also treated as interest.

e. Mr. Ram is invited to deliver lecture on corporate finance by KBC College. KBC
College has paid Rs. 10,000 to Mr. Ram for delivering lectures, Rs. 3,000 for setting
up model questions and Rs. 5,000 for checking the answer-sheets.
Answer
All such activities are subject to withholding tax @ 15% as per Sec. 88 (1). Therefore,
the amount of withholding tax is Rs. 2,700 (15% of Rs. 28,000)

225. Adishree & Co. is conducting business in Pokhara and has taken the office premises on
rent from Mr. Garibdash. According to agreement between them, Adishree & Co. is
required to pay the house owner NPR 50,000 per month. Further, Adishree & Co.has
taken on rent a vehicle for NPR 25,000 from Mr. Daman Thapa for its business purpose.
Mr. Daman Thapa has paid the annual tax as per Section 1 (13) of Schedule 1 of Income
Tax Act.

State whether the TDS as per Income Tax Act, 2058 is to be deducted in above
circumstances. If yes, what is the amount to be deducted as TDS? (Dec 2012, 3 Marks)
Answer

© The Institute of Chartered Accountants of Nepal 310


Compilation of Suggested Answers Income Tax

As per definition of "Rent", natural person's house rent income that is not generated through
a private firm does not fall under the definition of rent, which means such income is exempt
from income tax. As per Sec. 88 (4), there shall not be tax withholding when an income is
exempt from tax. As such, there shall be no withholding tax while making rental payment to
Mr. Garibdash.

As per Sec. 88, rental payment is subject to withholding tax @ 10%, as such; Adishree &
Co. shall withhold tax while making vehicle rental payment to Mr. Daman Thapa is subject
to tax @ 10%.

226. What is the rate of TDS on following payments? Also explain whether such payments
are final withholding or not? (Dec 2011, CA Inter, 3 Marks)
a. Retirement benefit given by Nepal Government or by any approved retirement fund.
Answer: 5% of Gain, Gain is calculated by deducting higher of 50% of payment or Rs.
500,000 from the payment.

b. Commission payment by resident employment Company to a non-resident person


Answer: 5%

c. Rs. 12,000 interest paid by Postal Saving Bank31


Answer: No withholding tax, as interest up to Rs. 25,000 is exempt u/s 11 (2Ka) if the
interest is paid to natural person.

d. Disposal of Non-Business Chargeable Assets (land and house owned since last 5 years
and 6 months) by a natural person.
Answer: 5% of gain, to be paid as advance tax (not a withholding tax) in Malpot
Karyalaya at the time of transfer of ownership.

e. Gain from commodities future trading


Answer: 10% of gain, to be collected as advance tax (not as withholding tax) u/s 95Ka.

f. Dividend given by Cooperative Society having registered office in Banepa


Municipality?
Answer: 5% of Dividend amount.

227. On the occasion of Deepawali 20X-67, Himal Group organized a nationwide song
competition and Mr. Ghanashyam wins a first prize of Rs. 5,00,000. The organizer of
the program wants to deduct TDS @ 25% on the prize amount and pay the balance to
him.

31
Postal Saving Bank, now closed by Government of Nepal

© The Institute of Chartered Accountants of Nepal 311


CAP II Paper 7 Income Tax and VAT

Mr. Ghanashyam was of the view that prize is related to the work of art and TDS is not
applicable on this payment. As a tax consultant, you are required to advice to Mr.
Ghanashyam whether TDS is applicable in this payment or not also mention the
relevant provisions of Income Tax Act 2058. (June 2011, 3 Marks)
Answer
As per Sec. 88Ka, prizes of national or international level up to Rs. 500,000 is exempt from
windfall gain tax, only if it is received in recognition of somebody’s contribution in the field
of sports (or any other seven fields).

In the given case, Mr. Ghanshyam has won the prize in competition and it is not awarded to
recognize his contribution, as such; the prize is subject to windfall gain tax @ 25%.

228. What would be the rate of TDS in case of following payments?


(June 2011, 0.5×12=6, CA Inter)
a. Dividend
Answer: 5%

b. Rent
Answer:
• House rent income of an individual that is not generated through operation of private
firm- exempt
• Vehicle rent paid to VAT registered person- 1.5%
• Transport rent to VAT registered person- 1.5%
• Transport rent to person not registered for VAT purpose- 2.5%
• All other cases than above: 10%

c. Commission
• VAT registered service provider- 1.5%
• By a resident employment agency to non-resident- 5%
• All other cases- 15%

d. Retirement Benefit by approved agency / fund


Answer: 5% of gain

e. Rent paid on Leased Aircrafts


Answer: 10%

f. Interest to Natural Person by Bank & Financial Institutions


Answer: 5% in case it is not related to his/her business and 15% in all other cases

© The Institute of Chartered Accountants of Nepal 312


Compilation of Suggested Answers Income Tax

g. Commission to tax payer registered under VAT


Answer: 1.5%

h. Commission paid by Resident Employer to non-resident Person


Answer: 5%

i. Interest Paid by Person other than Bank & Financial Institution


Answer: 15%

j. Payment of Benefit on Insurance Investment


Answer: 5%

k. Retirement Benefit by unapproved Fund / Agency


Answer: 5%

l. Windfall gains
Answer: 25%

229. Mr. X held 500 shares of ABC Ltd., a Listed Company, of Rs. 100 each. He acquired
those shares through a stockbroker by paying Rs. 700 each, and incurred Rs. 2,800
towards broker commission. He sold all those shares through the broker for Rs. 3,500
each with the brokerage commission of Rs. 8,750. Assuming that Mr. X is a non-
resident, compute the gain amount and withholding tax amount. (June 2010, 5 Marks)
Answer

Per share
Particulars Shares Total (Rs.)
value
Incomings 500 3500 1,750,000
Outgoings
Original Cost 500 700 350,000
Brokerage on Purchase 2,800
Brokerage on Sales 8,750
Total Outgoings 361,550
Gain 1,388,450
Advance tax Collection rate 25%
Advance tax to be collected by NEPSE 347,113

230. M/s A. Ltd has entered into a contract with a party for supply of X items. M/s A. Ltd.
has made the payments to the supplier on following dates:

© The Institute of Chartered Accountants of Nepal 313


CAP II Paper 7 Income Tax and VAT

20X-66/4/8 Rs. 25,000


20X-66/4/12 (morning) Rs. 10,000
20X-66/4/12 (at 2 PM) Rs. 5,000
20X-66/4/12 (at 4 PM) Rs. 10,000
20X-66/4/17 Rs. 5,000
M/s A. Ltd contends that it need not deduct TDS on the payment made to the supplier
since each payment does not exceed Rs. 50,000. Give your opinion on this case.
(June 2010, 5 Marks, CA Inter)
Answer
As per Sec. 89, where payment related to a contract exceeds Rs. 50,000 after considering all
payments of moving 11 days, tax shall be withheld @ 1.5% of payment.

In the given case, there is no requirement of withholding taxes on 20X-66 Shrawan 8 and 12
as the payment in moving 11 days is Rs. 25,000, and Rs. 50,000 respectively.

However, payment exceeds Rs. 50,000 on Shrawan 17, total payment in moving 11 days is
Rs. 55,000 and provision of Sec. 89 is applicable. Since, no payments before Shrawan 17 is
subject to withholding taxes, tax shall be withheld on Rs. 55,000 @ 1.5% on Shrawan 17.

231. Calculate the amount of TDS to be deducted by a payer of the following payments as
per the provisions of the Income Tax Act, 2058. Also state whether the payments are
final withholding payments or not? (Dec 2009, 5 Marks)
a. M/S Lama Construction Pvt. Ltd. has received an invoice for Rs. 3,39,000 including
VAT from M/S Jurist Company Pvt. Ltd. for legal advisory services rendered during
fiscal year 20X-65/X-66.
Answer
The payment of service fee to VAT registered service provider is subject to Withholding
tax @ 1.5% of payment as per Sec. 88 (1). Therefore, Lama Construction shall withhold
tax amounting Rs. 4,500 (1.5% of Rs. 300,000).

This tax is not final withholding for M/s Jurist Company.

b. Amount paid by an approved retirement fund Rs. 15,00,000. The accumulated


principal and the interest thereon as on Chaitra 19, 2058 for the same amount was
Rs. 4,00,000
Answer
The accumulated principal and the interest thereon as on Chaitra 19, 2058 is exempt
from tax, therefore, taxable amount is Rs. 1,100,000.

© The Institute of Chartered Accountants of Nepal 314


Compilation of Suggested Answers Income Tax

Withholding tax rate is 5% of gain, as per Sec. 88 (1). Gain is calculated by deducting
higher of 50% of taxable amount or Rs. 500,000 from taxable amount. Therefore, gain is
Rs. 550,000, and the withholding tax is Rs. 27,500.

The amount is final withholding in the hand of recipient.

c. Amount paid by an unapproved retirement fund Rs. 15,00,000. The accumulated


principal and the interest thereon as on Chaitra 19, 2058 for the same amount was
Rs. 4,00,000.
Total amount deposited by the account holder after Chaitra 19, 2058 to the
retirement fund Rs. 9,00,000.

Answer
The accumulated principal and the interest thereon as on Chaitra 19, 2058 is exempt
from tax, therefore, the total payment amount subject to taxation is Rs. 1,100,000.

Withholding tax rate is 5% of gain, as per Sec. 88 (2). Gain is Rs. 200,000 (Rs. 1,100,000
– Rs. 9,00,000). The withholding tax amount is Rs. 10,000.

The amount is final withholding in the hand of recipient.

d. Binod paid Rs. 50,000 as interest to Ram for loan taken from him. Binod has taken
the loan for his personal purpose.
Answer
As per Sec. 88 (4), a natural person shall not withhold tax in any payment except when
it is related to operation of business. In the given case, Mr. Binod is not making payment
in connection to operation of business, as such; there shall be no withholding of taxes.

However, Mr. Ram has to include the amount as part of investment income and pay tax
as per Income Tax Act.

e. A college is paying Rs. 20,000 to Mr. Ramesh for setting various question papers. Mr.
Ramesh is an employee of the college and getting regular remuneration.
Answer
The amount is to be included as part of employment income computation u/s 87 and tax
shall be withheld accordingly. It is not final withholding for Mr. Ramesh.

232. If a resident Airlines company makes payment to a non-resident person for aircraft
maintenance, at which rate TDS shall be deducted? If such payment is made for
consultancy services, will your answer differ? (Dec 2009, 2 Marks, CA Inter)

© The Institute of Chartered Accountants of Nepal 315


CAP II Paper 7 Income Tax and VAT

Answer
As per Sec. 89 (3), if a resident person makes payment to a non-resident person under any
contract or agreement, tax shall be withheld @ 5%.

Sec. 88 is applicable when a resident person makes payments having source in Nepal. Service
fee is treated as having source in Nepal when the service is rendered to Government of Nepal,
or when the service is rendered in the state of Nepal. If consultancy service is rendered
outside the state of Nepal, there shall be no withholding of tax.

233. Gain on commodity future market trading is subject to withholding tax or not?
(Dec 2009, 2 Marks, CA Inter)
Answer
No, it is subject to collection of advance tax by agent under Sec. 95Ka.

234. The following issues arise in connection with the deduction of tax at source under
Chapter 17 of the Income Tax Act, 2058. Discuss the liability for tax deduction in these
cases:
(June 2009, 8 Marks, CA Inter)
Editor’s Note:
Students should note that the specific requirement of this question is not clear. So, the
answer may be multiple provided logically explained by students.

a. TDS to be deducted in the case of Non-Resident.


Answer
Where a resident person makes payments subject to Sec. 87-89, even when the payment
is made to non-resident, withholding of tax is applicable. These amounts are treated as
final withholding in the hand of non-resident persons.

However, a non-resident person cannot withhold tax u/s 87, 88 and 89. Whether a non-
resident can withhold tax u/s 88Ka is not specifically clear. However, compelling a non-
resident to withhold tax through literal interpretation of tax law is blunder and against
the very premise on which the concept of withholding taxes is introduced in tax law.

b. TDS in the case of co-operative society.


Answer
Where a resident person makes payments subject to Sec. 87-89, even when the payment
is made to non-resident, withholding of tax is applicable. Cooperatives are withholding
agent and shall withhold tax u/s 87-89 of the Act.

c. TDS in case of Contract.

© The Institute of Chartered Accountants of Nepal 316


Compilation of Suggested Answers Income Tax

Answer
Withholding of tax is required in case of contract payments within the scope of Sec. 89.
Sec. 89 does not define what contract mean; however, it clarifies the types of contract on
which withholding of tax is required.

In case payment related to a contract exceeds Rs. 50,000 when the payment is aggregated
over moving 11 days, tax shall be withheld at the rate 1.5% of payments.

There shall be no double withholding of tax or no non-withholding of tax.

d. Advance Tax Collection in case of profit gain in on sale of securities of listed


companies.
Answer
As per Sec. 95Ka, advance tax shall be collected by Nepal Stock Exchange Limited on
gain on sale of securities of listed companies. The rate of such tax is 5% for resident
natural person, 10% for resident entity and 25% for non-resident persons.

235. As per the Loan agreement with the UK based company, AA has been assigned to
provide fund management service to Resident company BB. The BB Company has
earned one million rupees from the investment made during the year. Service fees paid
to AA company for this purpose is hundred thousand rupees.

Provide your opinion based on given facts based on the provisions of the Income Tax
Act, 2058. The answer must also consist of implication as per the Act and opinion/
recommendation. (June 2007, 1.67 Marks, CA Inter)
Answer
AA is a resident entity as it provides service to BB throughout the year, which qualifies AA
as Foreign permanent establishment. The income generated by AA is subject to corporate
tax under Income Tax Act. When BB makes payment of service fee to AA, it has to withhold
tax @ 15%, if AA is not registered for VAT and @ 1.5% where it is registered for VAT.

236. Mechi Noodles Pvt. Ltd. has made payments on various dates in the income year 20X-
61/X-62 to Kali Construction Company Pvt. Ltd. towards work done under a contract
as follows :
Date of Payment Amount (Rs.)
Ashad 11, 20X-62 15,000
Ashad 15, 20X-62 17,000
Ashad 17, 20X-62 3,000
Ashad 19, 20X-62 20,000

© The Institute of Chartered Accountants of Nepal 317


CAP II Paper 7 Income Tax and VAT

Mechi Noodles Pvt. Ltd. claims that it is not liable to deduction of withholding tax.
Examine the correctness of the claim made by the Company.

What would be the position if the payment on Ashad 19, 20X-62 is Rs. 5,000 only.
(June 2006, 5 Marks, CA Inter)
Answer
Part (a):
As per Sec. 89, where payment related to a contract exceeds Rs. 50,000 after considering all
payments of moving 11 days, tax shall be withheld @ 1.5% of payment.

In the given case, there is no requirement of withholding taxes on 20X-62 Ashad 11, 15 and
17 as the payment in moving 11 days is Rs. 15,000, 32,000 and Rs. 35,000 respectively.

However, payment exceeds Rs. 50,000 on Ashad 19, total payment in moving 11 days is Rs.
55,000 and provision of Sec. 89 is applicable. Since, no payments before Ashad 19 since
Ashad 9 is subject to withholding taxes, tax shall be withheld on Rs. 55,000 @ 1.5% on Ashad
19.

Part (b)- what if, question


If the payment on Ashad 19 was Rs. 5,000 only, the total payment over moving 11 days
(Ashad 9- 19) would be Rs. 40,000 only that is less than Rs. 50,000 and hence, withholding
of tax would not be applicable.

237. M/S 'X' Co. Ltd. (Hydro Power Generation Electricity Company) whose total turnover
in F.Y. 20X-59/X-60 is Rs. 150 crores and net profit before tax is Rs. 20 crores. The
company has entered into a contract for Rs. 60 crores with 'Y' Ltd. of India (Registered
in VAT in Nepal) to erect and install one Hydropower project in Nepal. 'Y' Co. Ltd.
has taken few equipments on hire from 'X' Co. Ltd. and paid hire charges to 'X' Co.
Ltd. amounting to Rs. 18 lacs in the same financial year. What rates of TDS will be
applicable on the payment of this hire charges and also state the implications if any, of
value added Tax Act on the said companies?
(Dec 2003, 5 Marks, CA Inter)
Answer
Value Added Tax is applicable on hire charges of equipment. As such, X Co. Ltd. shall raise
a Value Added tax invoice.

Further, while making payment of hire charges , Y. Co. Ltd. shall withhold tax on rental
charges @ 10% as per Sec. 88 (1).

238. State and explain the rates of TDS applicable in the following cases.

© The Institute of Chartered Accountants of Nepal 318


Compilation of Suggested Answers Income Tax

(Dec 2003, 3 Marks, CA Inter)


a. Payment to security service Provider Company under their service contract.
Answer: 1.5% under Sec. 89, if payment exceeds Rs. 50,000 over a period of mocving 11
days.

b. Labour contract with individual for supply of labour to industry and charging
service fee @5% on total labour payment under contract.
Answer: 1.5% under Sec. 89, if payment exceeds Rs. 50,000 over a period of moving 11
days.

c. Royalty paid to Indian company.


Answer: 15%

d. Interest received from Bank.


Answer: Natural person not in connection to business- 5% and all other cases- 15%,
except the interest is exempt from income tax.

239. What are the payments for which TDS becomes the final tax?
(Dec 2003, 2 Marks, CA Inter)
Answer
The following payments are final withholding as per Sec. 92 (1):
a. Dividend paid by resident company or partnership firm.
b. Other than in respect of operation of business, rent with source in Nepal paid to a natural
person in consideration of lands or buildings and fittings and equipment connected with
them.
c. Payment made by a resident person for gains from investment insurance
d. Gains paid by a resident person in consideration of interest in an unapproved retirement
fund
e. The following interest amount paid by bank, financial institution, or any other institution
issuing bonds, or company listed under prevailing laws or cooperatives referred to in
Subsection (3) of Section 88:
i. Payment with source in Nepal made to a natural person and is not in relation of
operation of business,
ii. Payment made to an exempt organization referred to in section 2 (dha)
f. Payments subject to withholding tax under section 87, 88, 88Ka or 89 made to non-
resident persons
g. All types of retirement payments including that made by Government of Nepal or an
approved or unapproved retirement fund (except pension payable on regular basis)
h. Meeting fee to the extent of rupees twenty thousand per meeting, payment in respect of
occasional lecture service, setting up of question paper or checking answer sheet

© The Institute of Chartered Accountants of Nepal 319


CAP II Paper 7 Income Tax and VAT

i. Payment of Windfall Gain


j. Payment of return made to a natural person by a mutual fund
k. Rental Payment received by a natural person in respect of transport service or vehicle
service other than that received from operation of private firm

Installment Tax
240. Big Traders Pvt. Ltd. dealing with electronic items has annual turnover Rs. 7,890,000
and taxable income Rs. 1,780,000 for IY 20X-74/X-75. The details of tax paid by the
company is Rs. 130,000 on Poush end 20X-74 and filed the estimated tax return on 28
Poush 20X-74, paid Rs. 56,000 on Chaitra 27, 20X-74 and a government agency has
withheld tax on behalf of Big Traders during the month of Jestha 20X-75 amounting
Rs. 57,500. Final installment of tax amounting Rs. 46,500 was deposited during Ashad
20X-75. Examine about the compliance of Section 94 by the company.
(June 2019, 5 Marks)
Answer
Editor’s Comment:
Compliance of Sec. 94 cannot be examined through this question, as Sec. 94 is based on the
estimation of income tax to be paid by a person during any Income Year. If there is margin
of error more than 10% on actual tax liability and estimated tax liability, interest under Sec.
118 is applicable. Hence, any question asked to examine the compliance of Sec. 94 is
irrelevant, as the question should be set to examine the provision of Sec. 118 instead of Sec.
94.

When such questions are asked in examination, students should focus their answers on
theoretical aspect of the said sections rather than practical application.

Provision of Sec. 94:


A turnover based taxpayer shall be tax on two installments, by the end of Poush and by the
end of Ashad. The amount to be deposited in first installment is applicable tax on turnover
until Poush 20 and in second installment is applicable tax on total estimated turnover for the
year taking into account the turnover until Ashad 20 after deducting the amount paid in first
installment.

For other taxpayers, who are required to pay tax in installments, tax shall be paid in three
installments as follows:
a. By Poush end- 40% of estimated tax liability
b. By Chaitra end- 70% of estimated tax liability
c. By Ashad end- 100% of estimated tax liability

© The Institute of Chartered Accountants of Nepal 320


Compilation of Suggested Answers Income Tax

In the given case, since the estimated liability is not given, therefore, the compliance of Sec.
94 cannot be examined.
241. Details of annual estimated tax and withholding tax of A & Co. are as follows:
Estimated Tax
Poush end Rs. 10,00,000
Chaitra end Rs. 12,00,000 (Re-estimated)
Ashad end Rs. 12,00,000 (Re-estimated)

Withholding Tax details:


Upto Poush end Rs. 20,000
Magh to Chaitra end Rs. 10,000
Chaitra to Ashad Rs. 15,000
end
Compute the total Advance Tax to be paid for each installment. (June 2011, 5 Marks)
Answer
Already
WHT to be paid
Cumulativ
considered (Cumulat Remaining
Due Date Standard e to be
(Cumulativ ive) to Pay
deposited
e) except
WHT
By Poush
40% of Estimated Tax Liability 400,000 20,000 - 380,000
end
By Chaitra 70% of Estimated Tax Liability,
840,000 30,000 380,000 430,000
end Including re-estimated
By Ashad 100% of Estimated Tax Liability,
Including re-estimated
1,200,000 45,000 810,000 345,000
end

242. Discuss the provision for payment of Income Tax under Installments.
(Dec 2009, 5 Marks, CA Inter)
Answer
A turnover based taxpayer shall be tax on two installments, by the end of Poush and by the
end of Ashad. The amount to be deposited in first installment is applicable tax on turnover
until Poush 20 and in second installment is applicable tax on total estimated turnover for the
year taking into account the turnover until Ashad 20 after deducting the amount paid in first
installment.

For other taxpayers, who are required to pay tax in installments, tax shall be paid in three
installments as follows:
a. By Poush end- 40% of estimated tax liability
b. By Chaitra end- 70% of estimated tax liability

© The Institute of Chartered Accountants of Nepal 321


CAP II Paper 7 Income Tax and VAT

c. By Ashad end- 100% of estimated tax liability

Tax Assessment
243. During the income year 20X-74/X-75, Mr. Anand won DV lottery of America and he
left permanently with his family members on 25th Poush 20X-75. Mr. Anand has
already filed Income Return for income year 20X-74/X-75, however he has not filed
Income return for income year 20X-75/X-76. Mr. Anand has earned Rs. 1,500,000
taxable income up to 25th Poush 20X-75.
Can income tax authority assess the income tax liability of Mr. Anand immediately?
(June 2019, 5 Marks)
Answer
As per Sec. 96 (5), Inland Revenue Department may demand income return for an Income
year or part of income year before the due date of filing return, in any of the following
conditions:
a. Where a person is bankrupt, goes into liquidation or wound up,
b. Where a person leaves Nepal for indefinite period of time (don’t mix up with Value Added
tax, the word there is “leaving Nepal permanently”. There is difference between two
terms)
c. Where is person is ceasing its activity in Nepal, or
d. Where IRD otherwise thinks it appropriate
When such condition arises, IRD has the right to demand income return for part of Income
Year as well. Further, IRD may also conduct jeopardy assessment of tax under Sec. 100.
In the given case, Mr. Anand is leaving Nepal for indefinite period of time. Therefore, IRD
can assess the income tax liability of Mr. Anand immediately as per the power given to it u/s
96 (5) and 100 of the Act.

244. Sitaram Poudel has received a notice from Tax officer on 1st Chaitra 20X-72 for
submission of Income Tax Return for income year 20X-72/X-73 by 1st Baishakh 20X-
73. He has been planning to migrate to Australia. Sitaram Poudel’s contention is he
need not have to submit tax return since he has income from employment only, and due
date of filing income tax return is also end of Asoj.
Stating the relevant provisions of Income Tax Act, 2058, please advise Sitaram Poudel
on the above matter. (June 2017, 5 Marks)

Answer

© The Institute of Chartered Accountants of Nepal 322


Compilation of Suggested Answers Income Tax

As per Sec. 96 (5), Inland Revenue Department may demand income return for an Income
year or part of income year before the due date of filing return, in any of the following
conditions:
a. Where a person is bankrupt, goes into liquidation or wound up,
b. Where a person leaves Nepal for indefinite period of time (don’t mix up with Value Added
tax, the word there is “leaving Nepal permanently”. There is difference between two
terms)
c. Where is person is ceasing its activity in Nepal, or
d. Where IRD otherwise thinks it appropriate
When such condition arises, IRD has the right to demand income return for part of Income
Year as well. Further, IRD may also conduct jeopardy assessment of tax under Sec. 100.
In the given case, Mr. Sitaram is leaving Nepal for indefinite period of time. Therefore, IRD
can assess the income tax liability of Mr. Anand immediately as per the power given to it u/s
96 (5) and 100 of the Act.
Further, provisions of Sec. 97 of Income Tax Act, 2058 are not blanket exemption for relief
in filing income return, but are conditional. If IRD summons a notice in writing requiring a
person to file return, every person is required to file income return regardless of whether
conditions of Sec. 97 (1) are satisfied.
In view of above, Mr. Sitaram has to submit return demanded by IRD.

245. What is the information to be included while issuing a reassessment notice under
section 102 of the Income Tax Act? (Dec 2012, 3 Marks, CA Inter)
Or
What are the matters that should be included in the assessment notice issued by Inland
Revenue Department? (June 2010, 5 Marks)
Answer
The jeopardy assessment order shall include all the following matters:
b) The total tax payable by the taxpayer for the period of assessment and the tax due to him;
c) The method of calculation of the tax liability;
d) The reason of the amended/jeopardy assessment by the IRO;
e) The period within which the tax due is payable; and
f) Where, when and how to appeal against the order if the taxpayer is not satisfied with the
amended assessment.

246. What are the methods of tax assessment as per the Income Tax Act, 2058?
(Dec 2008, 10 Marks, CA Inter)
Answer
a. Self -assessment

© The Institute of Chartered Accountants of Nepal 323


CAP II Paper 7 Income Tax and VAT

Actual Self-Assessment [Sec. 99 (1)]


Where a person files a return of income for an income year disclosing the following
matters, self-assessment is treated as made on the due date for filing the return:
• the income tax payable by the person for the year under section 3 (Ka) and (Kha),
and
• the amount of that tax still to be paid for the year in the amount shown in the return

© The Institute of Chartered Accountants of Nepal 324


Compilation of Suggested Answers Income Tax

Deemed Self-Assessment [Sec. 99 (2)]


Where a person fails to file a return of income for a tax year then, until such time as a
return is filed, an assessment is treated as made on the due date for filing the return as
follows:
a. the income tax payable by the person for the year is equal to the sum of any income
tax withheld from payments derived by the person during the year under Chapter 17
of the Act and any income tax paid by the person by installment for the year under
Chapter 18 of the Act, and
b. There is no tax payable on the assessment.

b. Jeopardy Assessment
Conditions for Jeopardy Assessment [Sec. 96 (5)]
A written notice may be served upon a person by IRD requiring submission of income
return before the due date of filing return for a full Income Year or part of Income year
by the date specified in the notice, in any of the following four conditions:
• Where the person becomes bankrupt, is wound-up, or goes into liquidation,
• Where the person is about to leave Nepal for indefinite period of time,
• Where the person is otherwise about to cease the activity in Nepal, or
• Where IRD otherwise considers it appropriate.

Jeopardy Self-Assessment [Sec. 100 (1)]


Where a written notice is served upon a person to submit income return for an Income
Year in the conditions as above, the person shall make self-assessment of tax pursuant to
Sec. 99 and submit income return by the date specified in the notice.

Jeopardy Assessment by Tax officer [Sec. 100 (2)]


In the circumstances giving rise to jeopardy assessment, instead of requiring a person to
file a return of income, the IRD may, according to the IRD's best judgment, make an
assessment of income for a full Income year or part of income year.

Information to be considered for Jeopardy Assessment [Sec. 100 (2)]


The following information is considered for the jeopardy assessment:
• Assessable income of the taxpayer from business, employment or investment, and the
source of such income,
• Taxable income of the taxpayer during the year and the total amount of tax due to
the taxpayer, and
• In the case of a taxpayer, which is a foreign permanent establishment, the income
repatriated to a foreign country during the period and tax payable on such
repatriation.

© The Institute of Chartered Accountants of Nepal 325


CAP II Paper 7 Income Tax and VAT

Show-cause Notice [Sec. 100 (5)]


A show-cause notice shall be served upon the person with time limit of 7 days to provide
the proof in own defense before the issuance of jeopardy assessment order.

Effect of Jeopardy Assessment [Sec. 100 (3)]


The effect of jeopardy assessment shall be as follows:
• Where the jeopardy assessment is conducted for a full income year, the taxpayer is
not required to submit a self-assessed income return, or
• Where the jeopardy assessment is conducted for a part of income year, the person
shall file an income return for the full income year

Treatment of Tax paid under Jeopardy Assessment [Sec. 100 (4)]


The tax paid by the taxpayer under jeopardy tax assessment is treated as advance tax
paid for the Income Year and the person can claim credit of such tax while filing the
income return of full income year.

c. Amended Assessment
IRD may amend an assessment made under self and jeopardy assessment provisions. The
amendment in assessment is to adjust the assessed person's liability to tax, including any
tax payable on the assessment. The adjustment shall be in a manner consistent with the
intention of the Act and according to IRD’s best judgment.

Number of Amended Assessment [Sec. 100 (2)]


In case IRD thinks it proper to do so, the assessments can be amended again and
according to the IRO’s best judgment for as many times as it thinks appropriate.
Time Limit to make amendment in Assessment [Sec. 100 (3)]
The amended assessment shall be completed within four years of the date mentioned
below, unless there is evidence that assessment is inaccurate by reason of fraud:
• In the case of self-assessment: within four years from the due date for filing the
return; or
• In the case of jeopardy assessment: within four years from the date on which the
notice of assessment is served to the taxpayer under Sec. 102.
• Reassessment of amended assessment: within four years from the date mentioned
above, as the case may be.

Amended Assessment of inaccurate assessment by reason of Fraud [Sec. 101 (4)]


In case a person’s assessment is inaccurate by reason of fraud, the limitation of four
years as above is not applied. The due date of completing amended assessment of
inaccurate assessment by reason of fraud is one year from the date of receipt of
information of fraud.

© The Institute of Chartered Accountants of Nepal 326


Compilation of Suggested Answers Income Tax

Limitation of IRD if the competent court has settled the assessment [Sec. 101 (5)]
IRD cannot amend an assessment to the extent that the assessment has been amended or
reduced pursuant to an order of Revenue Tribunal or any other authorized court unless
the order is vacated.

Show-cause Notice [Sec. 100 (6)]


A show-cause notice shall be served upon a taxpayer providing 15 days’ time limit to
submit clarification and proof in one’s own favor. The show-cause notice shall include
the clear basis for the amendment.

Collection of Tax in Arrears


247. The Inland Revenue Department has issued notice to Mr. Rohit Shrestha, a renowned
business tycoon of Nepal; to pay the due tax liability amounting to Rs. 25,566,725 within
Baisakh 25, 20X-71 and the same notice was also published in the National Level
Newspaper. The notice also contains that if Mr. Shrestha does not pay the tax dues on
or before the specified time limit then appropriate action shall be taken against him.
Mr. Shrestha aggressively trying to arrange the money but he could not be able to pay
the due tax on time. In the meantime, Mr. Shrestha had a prescheduled visit to New
Delhi on Baisakh 26, 20X-71 to attend a marriage ceremony of one of his friend's
daughter and accordingly he went to Tribhuvan International Airport (TIA) on the
schedule time to fly to New Delhi. At the TIA, the immigration staff stopped him to
enter to the aircraft stating that the Inland Revenue Department has given notice to
stop Mr. Rohit Shrestha to going abroad. In the light of the above fact, please state
whether the immigration staff at TIA has got the right to do so? If not, what remedy is
available to Mr. Shrestha for the unauthorized act of the immigration staff as per the
provisions of Income Tax Act, 2058.
(June 2016, 5 Marks)

Answer
As per Sec. 106, when a person becomes a tax defaulter, the related tax office may serve a
notice in writing to the concerned Department of GON to prevent the tax defaulter from
leaving Nepal for a period of not more than 72 hours from the time of the expiry of deadline
mentioned in the notice served to the defaulter to pay due tax amount.

In case the time limit is to be extended, the tax office has to take a prior approval of the
concerned appellate court.

Once the tax defaulter pays the tax or makes an arrangement for payment in satisfaction to
the tax office, the office may withdraw the order.

© The Institute of Chartered Accountants of Nepal 327


CAP II Paper 7 Income Tax and VAT

In the given case, the deadline to pay tax by tax defaulter Mr. Rohit Shrestha is 25th Baisakh
20X-71. As Mr. Shrestha did not make payment of tax dues within the deadline mentioned by
Inland Revenue Department, IRD has the right under Sec. 106 to serve a notice in writing to
the concerned Department of GON to prevent the tax defaulter from leaving Nepal for a
period of not more than 72 hours from the time of the expiry of deadline mentioned; and the
immigration department may bar him from travelling outside Nepal.

248. What are the various actions that can be resorted by the tax office, if a tax payer does
not pay the tax in time as prescribed in the Income Tax Act, 2058?
(June 2009, 5 Marks; CAP II & CA Inter; June 2009, 7 Marks,
CA Inter, June 2004, 5 Marks, CA Inter)
Answer
The various actions are as follows:
a. Creating charge over asset and auctioning such asset to recover dues
b. Collection of tax from officer of the entity
c. Issue an order to prohibit a person from travelling abroad
d. Collection of tax from receiver of any person
e. Collection of tax from the person who owes money to tax defaulter
f. Collection of tax from the agent of Non-resident person
Note: Students are required to elaborate each topic in short based on the marks allocated.

Income Returns
249. Mention whether the return filing is required or not on the following circumstances:
(Dec 2017, 5 Marks)
Answer
In any of the following circumstances, a person is relieved from submitting income returns
unless otherwise required by IRD in writing or through public circular or if the person is a
natural person, his/her income for any income year exceeds Rs. 4 Million:
1. Where a person has no income tax payable under Sec. 3 (Ka) during the Income Year,
2. Where a person’s income consists exclusively of Final Withholding Payments during the
Income Year,
3. In case of a resident natural person, who satisfies all the following conditions:
➢ The person’s income for the year consists exclusively of income from any employment
having a source in Nepal,
➢ The person has only one employment at a time during the year, even if the
employment changes during the year, and each employment is by a resident
employer, and
➢ The person does not claim the following for the Income Year:
✓ a medical tax credit under section 51, other than with respect to medical tax
credit paid through the employer,

© The Institute of Chartered Accountants of Nepal 328


Compilation of Suggested Answers Income Tax

✓ a reduction in taxable income under section 63 (Contribution to Approved


Retirement Fund), other than with respect to retirement contributions paid
through the employer, and
✓ a reduction in taxable income under Sec. 12 (Donation)
4. Where a vehicle owner paying tax pursuant to Sec. 1 (13) of Schedule 1 of the Act is a
natural person not operating such vehicle through a private firm,
5. Where a person generating income only from disposal of non-business chargeable asset
desires not to file return

a. Mr. X, a natural person having employment income of Rs. 4,00,000 in 20X-73/X-74,


has worked under a foreign consultancy firm, i.e. non-resident for Nepalese tax
purpose. Assume no deduction of retirement fund, insurance, remote area allowances
etc.
Answer
Since any of the conditions above is not satisfied, Mr. X has to file income return.

b. Mr. X with Mrs. X, a resident natural person having employment income of Rs. 2.1
million (Rs. 1 million of Mr. X and Rs. 1.1 million of Mrs. X) in the year 20X-73/X-
74. Assume no deduction of retirement fund, insurance, remote area allowances etc.
Answer
If the employer is resident, these persons fulfill condition (3) specified above, and hence,
are relived from filing income return.
If the employer is non-resident, they have to file income return.

c. S Enterprises, Kathmandu registered in Small and Cottage Industries Office is


dealing with VAT exempt goods. It has annual sales Rs. 15,00,000 in 20X-72/X-73 and
Rs. 25,00,000 in 20X-73/X-74. It has assessable incomes Rs. 1,50,000 and Rs. 1,00,000
in 20X-72/X-73 and 20X-73/X-74 respectively.
Answer
Since any of the conditions above is not satisfied, the owner of S Enterprises has to file
income return.

d. V Enterprises Kathmandu registered in Small and Cottage Industries Office is


dealing with VAT exempted and taxable goods. It has annual sales Rs. 5,00,000 in
20X-72/X-73 and Rs. 30,00,000 in 20X-73/X-74. It has assessable income Rs. 1,00,000
and Rs. 5,00,000 in 20X-72/X-73 and 20X-73/X-74 respectively.
Answer
Since any of the conditions above is not satisfied, the owner of V Enterprises has to file
income return.

© The Institute of Chartered Accountants of Nepal 329


CAP II Paper 7 Income Tax and VAT

e. Mrs. Chaulagai, a single natural person has income from various sources like
dividend income Rs. 2,00,000, interest income Rs. 3,00,000 from bank, meeting
allowances Rs. 20,000 and capital gain from sale of shares Rs. 3,00,000.
Answer
Since any of the conditions above is not satisfied, Mr. Chaulagain has to file income return

250. Mr. Naresh Yadav has been working as a Chief Legal Advisor of a bank. During the
income year 20X-72/X-73, he has received remuneration income of Rs. 50,00,000 from
the bank after deducting TDS of Rs. 22,00,000. During the year he has won a lottery of
Rs. 10,00,000. Company deducted TDS of Rs. 2,50,000 on the said lottery and paid him
Rs. 7,50,000. He has income of Rs. 5,00,000 from his agricultural farm. He has also
received dividend of Rs. 5,00,000 on the shares invested by him in an insurance
company. Insurance Company has deducted TDS of Rs. 25,000 on the said amount, and
paid balance Rs. 4,75,000. He has also received bonus share of Rs. 2,00,000 on the shares
invested by him in a Hydro Power Company. Hydro Power Company has deposited
TDS of Rs. 10,000 on the said bonus share.
Discussing the relevant provisions of the Income Tax Act, 2058 determine whether he
is required to file Income Tax Return or not, if yes, prepare his Income Tax Return.
(June 2017, 5 Marks)
Answer
In circumstances as per Sec. 97 (1), a person is relieved from submitting income returns
unless otherwise required by IRD in writing or through public circular or if the person is a
natural person, his/her income for any income year exceeds Rs. 4 Million.
In the given case, the income of Mr. Naresh Yadav has income exceeding Rs. 4 Million during
the Income Year; therefore, he needs to file income return as per Sec. 97 (2).
251. Mention the cases when the income tax returns are not required to be filed as per the
provisions of the Act. Are there any conditions/circumstances in which the department
can order a person to submit a tax return before the time given for filing income tax
return? (Dec 2012, 5 Marks, CA Inter)
Or
When Return of Income is not required to be filed? (June 2006, 2 Marks, CA Inter)
In any of the following circumstances, a person is relieved from submitting income returns
unless otherwise required by IRD in writing or through public circular or if the person is a
natural person, his/her income for any income year exceeds Rs. 4 Million:
1. Where a person has no income tax payable under Sec. 3 (Ka) during the Income Year,
2. Where a person’s income consists exclusively of Final Withholding Payments during the
Income Year,
3. In case of a resident natural person, who satisfies all the following conditions:
a) The person’s income for the year consists exclusively of income from any employment
having a source in Nepal,

© The Institute of Chartered Accountants of Nepal 330


Compilation of Suggested Answers Income Tax

b) The person has only one employment at a time during the year, even if the
employment changes during the year, and each employment is by a resident
employer, and
c) The person does not claim the following for the Income Year:
✓ a medical tax credit under section 51, other than with respect to medical tax
credit paid through the employer,
✓ a reduction in taxable income under section 63 (Contribution to Approved
Retirement Fund), other than with respect to retirement contributions paid
through the employer, and
✓ a reduction in taxable income under Sec. 12 (Donation)
4. Where a vehicle owner paying tax pursuant to Sec. 1 (13) of Schedule 1 of the Act is a
natural person not operating such vehicle through a private firm,
5. Where a person generating income only from disposal of non business chargeable asset
desires not to file return

252. Mrs. Sarita is working as a lecturer in Bidhata Nursing Campus. She is the Director of
National Maternity Hospital in Lamadanda. After deduction of tax in source, during
the income Year 20X2/X3, she has got Rs. 17,000 as meeting allowance. She has one
house that has been provided in rent to Health Clinic and the Health Clinic pays the
rent amount after deduction of tax on source. She has deposited Rs. 5,00,000 in fix
account in Agriculture Development Bank and the gross interest receipt as per bank
statement is Rs. 35,000. Mrs. Sarita is asking regading the filing of return. State your
view regarding the submission of file by Sarita. (June 2003, 3 Marks, CA Inter)
Answer
In any of the following circumstances, a person is relieved from submitting income returns
unless otherwise required by IRD in writing or through public circular or if the person is a
natural person, his/her income for any income year exceeds Rs. 4 Million:
1. Where a person has no income tax payable under Sec. 3 (Ka) during the Income Year,
2. Where a person’s income consists exclusively of Final Withholding Payments during the
Income Year,
3. In case of a resident natural person, who satisfies all the following conditions:
a) The person’s income for the year consists exclusively of income from any employment
having a source in Nepal,
b) The person has only one employment at a time during the year, even if the
employment changes during the year, and each employment is by a resident
employer, and
c) The person does not claim the following for the Income Year:
✓ a medical tax credit under section 51, other than with respect to medical tax
credit paid through the employer,

© The Institute of Chartered Accountants of Nepal 331


CAP II Paper 7 Income Tax and VAT

✓ a reduction in taxable income under section 63 (Contribution to Approved


Retirement Fund), other than with respect to retirement contributions paid
through the employer, and
✓ a reduction in taxable income under Sec. 12 (Donation)
4. Where a vehicle owner paying tax pursuant to Sec. 1 (13) of Schedule 1 of the Act is a
natural person not operating such vehicle through a private firm,
5. Where a person generating income only from disposal of non business chargeable asset
desires not to file return

In the given case, Mr. Sarita does not fulfil the exclusivity criteria as per condition (b) and
(c) above, as she has multiple incomes. Therefore, principally, she needs to file income
return. However, practically IRD has not designed or circulated such forms to require a
natural person to file such return.

True False Questions


253. State whether following statements are true or false with appropriate provision of
Income Tax Act, 2058: (Dec 2015, 5×2=10)
a. Progressive Income Tax rate is only applicable to entire income of resident natural
person.
Answer
True, progressive tax rate is applicable to entire income of resident natural person.

b. If tax-exempt entities are unable to submit their financial statement within the
stipulated time limit, fee of 0.1% per annum on income shall be levied.
Answer
False, there is no such provision specifically to exempt organization.

c. TDS applicable on payment of vehicle hired by resident person to vehicle provider


registered in VAT is 1.5%.
Answer
True

d. Tax exemption has been increased for annual interest income up to of Rs. 25,000
from Rs. 10,000 in the amount deposited to the microcredit institutions, rural
development banks, postal saving bank, and cooperatives in the rural area Rural
Municipality.
Answer
True

© The Institute of Chartered Accountants of Nepal 332


Compilation of Suggested Answers Income Tax

e. Finance Act, 2072 has introduced mandatory requirement to submit income returns
for the natural person with yearly income exceeding Rs. 4 million.
Answer
True

254. State whether following statements are true or false with appropriate provision of
Income Tax Act, 2058: (July 2015, 5×2=10)
a. Any person who fails to pay Tax within prescribed time without reasonable excuse
shall be liable on contravention for a fine of an amount ranging from Rs. 50,000 to
Rs. 80,000 or an imprisonment for a term of not less than one month and not more
than three months or both.
Answer
False, the penalty ranges from Rs. 5,000 to 30,000 or imprisonment from one to six
months or both.

b. In case of person fails to maintain books of account and records as per section 81,
the person is responsible to pay fee higher of following amounts:
i. 0.1 % of turnover or gross receipt during the period for which the person
fails to maintain the accounts and records or
ii. Rs. 1000 per annum.

Answer

True

c. The listed resident company deducted 15% withholding tax while paying interest
having source in Nepal to Tax exempt organization. The Tax exempt organization
claims interest earned by it from listed resident company as final withholding
income.
Answer
True, as per Sec. 88 (1), interest is subject to 15% withholding tax and as per Sec. 92,
interest paid to an exempt entity by a resident listed company is final withholding.

d. In case of gain on the disposal of land & buildings owned by M/S ABC limited which
has been owned for less than 5 years, applicable Tax rate will be 5% and if the
disposed land & buildings has been owned for more than 5 years, applicable tax
rate will be 2.5%.
Answer
False, the gain on disposal of business asset or depreciable assets of an entity is subject
to tax at corporate tax rate applicable to the entity.

© The Institute of Chartered Accountants of Nepal 333


CAP II Paper 7 Income Tax and VAT

e. Cooperatives societies are allowed to claim as expenses an amount equivalent to the


amount set aside in risk bearing fund to the extent of 15% of outstanding loan.
Answer
False, as per Sec. 59 (1Kha), it is to the extent 5% of loan outstanding.

255. State with reason whether the following statements are True or False.
(Dec 2009, 5 Marks)
a. As per the definition of Income Tax Act, 2058 “every entity is a person but every
person is not an entity”.
Answer
True, the definition of person includes both natural person and entity, however, entity
cannot cover natural person, which means all persons are not entity.

b. A non-resident has to pay income tax on income accrued in Nepal only.


Answer
False, there is no concept of accrual of Income in Nepal. Nepal has embraced source
concept and a non-resident has to pay tax in Nepal sourced income only in Nepal.

c. Dividend of a co-operative society is exempt from tax.


Answer
Partially true, since the dividend by cooperatives specified in Sec. 11(2) is exempt and
that by other cooperatives is subject to dividend tax.

© The Institute of Chartered Accountants of Nepal 334


Compilation of Suggested Answers Income Tax

d. An individual has to follow accrual basis of accounting while calculating income


under the head investment.
Answer
False, as per Sec. 22, it shall be cash basis.

e. Meeting allowance shall not be included in the taxable income of an individual.


Answer
True, since it is final withholding payment.

256. Fill in the blanks. (June 2009, 10 Marks)


a. TDS return should be submitted to the department under Sec. 90 (1) of the Act from the
end of every month within …… days.
Answer: 25

b. Tax deducted should be deposited to the account of the government from the end of every
month within …… days.
Answer: 25

c. Interest on deficiency of installments of advance tax payable against installments paid


will be calculated on (daily/month-to-month/per annum) basis.
Answer: month-to-month basis

d. Interest chargeable on deficiency of installments of advance tax payable against


installments paid will be at the rate of …….
Answer: 15%

e. Income tax return under Sec. 99 is to be filed for the year 20X-65/X-66 before …………
Answer: Ashoj end 20X-66 unless otherwise required by Sec. 97, 98 and 100.

f. Estimate of tax to be submitted by a person liable to pay tax in installment is to be


submitted every year up to …… end.
Answer: Poush

g. The certificate of tax deducted at source should be provided by the withholding agent to
the withholdee within 15 days from (Date of deduction / The end of the month in which
deducted/ the end of the financial year).
Answer: the end of the month in which tax is withheld

h. Tax to be deducted from payment to a person carrying on business of letting out


buildings on rent is …………….%

© The Institute of Chartered Accountants of Nepal 335


CAP II Paper 7 Income Tax and VAT

Answer: not required if owned by a natural person

i. Tax to be deducted from the bill of service provider registered under VAT against VAT
bill is ….%
Answer: 1.5%

j. If the total tax to be paid for the year is less than that, no installment tax need be paid is
…….
Answer: Rs. 7,500

257. State with reason whether the following statements are True or False
(Dec 2007, 10×1=10, CA Inter)
a. For the purpose of calculation of the income for an income year from business, any
unrelieved loss of the previous eight years from any business of the same person is
allowed for deduction.
False, the unrelieved loss of last seven years in all cases and twelve years in case of
projects involving building power houses, generation and transmission of electricity and
those doing transactions under Petroleum Act, 2040 is allowed for deduction.

b. Standard interest rate means 15 percent per annum.


True

c. Any income of Government of Nepal is exempt from tax.


True

d. Dividend distributed by the industry established at special economic zone is fully


exempted for seven years from the commencement of its business.
False, the exemption is for first five years.

e. Withholding tax at the rate of 10 percent is applicable on the payment of dividend


by a resident person to non-resident.
False, it is 5%.

f. Income of the King of Nepal is not taxable.


Irrelevant in current scenario

g. Natural person is required to follow cash basis of accounting for the computation
of income from investment.
True

© The Institute of Chartered Accountants of Nepal 336


Compilation of Suggested Answers Income Tax

h. Income of approved retirement fund is taxable.


False

i. Senior citizen allowance paid by the Government of Nepal is taxable.


False, it is exempt under Sec. 10

j. Department is required to furnish decision within 60 days on the application for


advance ruling.
False, it shall be within 45 days.

Short Notes Questions


a) Advance ruling (June 2019, 2.5 Marks)
Answer
In case a person makes a written application to IRD seeking IRD’s position regarding the
application of this Act with respect to an arrangement proposed or entered into by the person,
IRD under signature of DG may issue, in writing, an advance ruling in this regard.

IRD shall not issue such an advance ruling on the matters under consideration of any court or
decided by a court.

Under the following circumstances the ruling shall be binding on the every officer of IRD, on
IROs and a civil servant who is authorized to work as tax officer:

a. The applicant has made a full and true disclosure of all aspects of the arrangement relevant
to the ruling; and
b. The arrangement proceeds in all material respects as described in the application for the
ruling.
Where the advance ruling issued by IRD contradicts with a public circular, for the person who
applied for the advance ruling, the advance ruling shall be applicable

b) Deductible expenses from cash payment


(June 2019, June 2018 and June 2016, 2.5 Marks)
Answer
The cash payments as follows are always deductible, subject to other provisions of the Act:
a. If the turnover of the person is not more than Rs. 20 lakhs during the Income Year
b. Payment made to Government of Nepal, Constitutional bodies, corporate having
ownership of GON, Banks, and Financial institutions.
c. Payment to a farmer or a producer of primary agro products even in the case where the
farmer himself primarily processes the product.
d. Payment of a retirement contribution or a retirement payment.
e. Payment made in such areas where banking services are not available.
An area not having banking services means the area where there are no banking facilities
within a periphery of 10 kilometers.

© The Institute of Chartered Accountants of Nepal 337


CAP II Paper 7 Income Tax and VAT

f. Payment made on the day when banking services are closed or there is unavoidable
compulsion that the payment shall be made in cash.
g. Payment is made through the bank account of the payee.

c) Tax on agricultural income (Dec 2019 & June 2019, 2.5 Marks)
Answer
Agriculture income of a natural person, except derived as follows, is exempt from tax:
a. In case a person generates agriculture income by doing business through registration of
private firm, partnership, company or corporate body.
b. In case the person holds land for industrial purpose as an industrial unit as specified by a
notification from GON and as per the terms of the notification under Section 12 (d) of Land
Act, 2021 generates agriculture income from such land; and
c. An agro-industrial unit holding land for agro- based industries as specified by a
notification from GON and as per the terms of the notification under Section 12 (e) of Land
Act, 2021, generates agriculture income from such land.

d) Approved medical expenses (Dec 2018, 2.5 Marks)


Answer
Medical expenses incurred for the treatment of a natural person in an approved hospital,
nursing home, health center, or by a medical practitioner on the basis of actual invoice
produced, except the following, is approved medical expenditure:
a. Expenses incurred on cosmetic surgery,
b. To the extent the expenses as above is reimbursed form the insurance company against any
medical insurance, or
c. To the extent the medical cost is incurred to treat the injured bodily part of an individual
where the compensation is exempted from tax

e) Deductible research and development expenses (Dec 2018, 2.5 Marks)


Answer
‘Research and development’ cost is the cost incurred by a person for the purpose of further
development of an established business and for improvement of the production process and
the products.

Research and development cost is eligible for deduction, if it satisfies both the following
conditions:
a. The Cost shall be incurred for the operation of business, and
b. The cost shall be incurred during the Income Year.

Research & Development Cost is deductible for Taxation purpose subject to the minimum of
following TWO:
a. Actual Cost incurred during the Income Year [Sec 18 (1)], or
b. 50% of Adjusted Taxable Income from all businesses [Sec 18 (2)]

In case incurred Research and Development Cost cannot be claimed due to the maximum limit
of 50% of Adjusted Taxable Income from all Businesses during the Income Year, such excess

© The Institute of Chartered Accountants of Nepal 338


Compilation of Suggested Answers Income Tax

of incurred R & D Cost over 50% of Adjusted Taxable Income from all Businesses shall be
capitalized in Pool D to arrive at the Opening Depreciation Base for following Income Year.

f) Withholding agent (Dec 2018, 2.5 Marks)


Answer
Withholding Agent means the person who has the obligation to withhold tax pursuant to Sec.
87, 88, 88Ka and 89 of Income Tax Act, 2058.

g) Interest (June 2018 & June 2014, 2.5 Marks)


Answer
Interest means the following payment or gain:
a. A payment made or accrued under a debt obligation that is not a repayment of capital
b. Any gain realized by way of a discount, premium, swap payment, or similar payment under
debt obligation; and
c. The amount treated as Interest under Sec 32 of the Act while receiving/making payments
for using any assets under finance lease or under annuity or installment sales

h) Amended assessment (June 2018, 2.5 Marks)


Answer
IRD may amend an assessment made under self and jeopardy assessment provisions. The
amendment in assessment is to adjust the assessed person's liability to tax, including any tax
payable on the assessment. The adjustment shall be in a manner consistent with the intention
of the Act and according to IRD’s best judgment.

Number of Amended Assessment [Sec. 100 (2)]


In case IRD thinks it proper to do so, the assessments can be amended again and according to
the IRO’s best judgment for as many times as it thinks appropriate.

Time Limit to make amendment in Assessment [Sec. 100 (3)]


The amended assessment shall be completed within four years of the date mentioned below,
unless there is evidence that assessment is inaccurate by reason of fraud:
• In the case of self assessment: within four years from the due date for filing the return; or
• In the case of jeopardy assessment: within four years from the date on which the notice of
assessment is served to the taxpayer under Sec. 102.
• Reassessment of amended assessment: within four years from the date mentioned above,
as the case may be.
Amended Assessment of inaccurate assessment by reason of Fraud [Sec. 101 (4)]
In case a person’s assessment is inaccurate by reason of fraud, the limitation of four years as
above is not applied. The due date of completing amended assessment of inaccurate assessment
by reason of fraud is one year from the date of receipt of information of fraud.
Limitation of IRD if the competent court has settled the assessment [Sec. 101 (5)]

© The Institute of Chartered Accountants of Nepal 339


CAP II Paper 7 Income Tax and VAT

IRD cannot amend an assessment to the extent that the assessment has been amended or
reduced pursuant to an order of Revenue Tribunal or any other authorized court unless the
order is vacated.
Show-cause Notice [Sec. 100 (6)]
A show-cause notice shall be served upon a taxpayer providing 15 days’ time limit to submit
clarification and proof in one’s own favor. The show cause notice shall include the clear basis
for the amendment.

i) Permanent Account Number (June 2018, 2.5 Marks)


Answer
IRD can issue a unique identification number to identify a taxpayer. The unique identification
number is Permanent Account number (interchangeably called Taxpayer’s Identification
number- TPIN), which is issued by IRD to identify a taxpayer under the power conferred by
Sec. 78 (1) of the Act.

Besides IRD, the PAN can also be issued by entity authorized by Department to issue such
number. The list of such authorized department is produced in the website of IRD, and
extracted (as on the date of writing this book) as part of End Note of this book. The authorized
entity shall issue permanent account number by abiding the provisions of Income Tax Act. But
there is restriction on such taxpayer obtaining PAN number from other authorized entity than
IRD to conduct foreign trade, i.e. the taxpayer cannot conduct import or export transaction
for particular period after obtaining PAN

j) Valuation of Closing Stock (Dec 2017, 2.5 Marks)


Answer
Closing stock is valued at lower of cost or Market value.

The cost of closing stock is determined using Specific Identification Method, however, when
this method cannot be used, it is determined using First-In-First-Out Cost Formula or
Weighted Average Cost formula.

With regards to calculation of cost of inventory in production process, a person following cash
basis may use prime costing or absorption costing; however, the person following accrual
basis must use absorption costing.

In absorption and prime costing, both, the depreciation and repair and improvement cost of
depreciable asset cannot be allocated as cost of inventory.

k) Associated Person (Dec 2017, 2.5 Marks)


Answer
“Associated Person” means one or more persons or group of such persons where the
relationship between the two is such that one may reasonably be expected to act in accordance
with the intentions of the other, and the term includes:

© The Institute of Chartered Accountants of Nepal 340


Compilation of Suggested Answers Income Tax

1. an individual and a relative of the individual or


2. any person or his/her partner
3. Foreign Permanent Establishment and the person having ownership over such foreign
permanent establishment, and
4. an entity and a person who
• either alone or together with an associate or associates, and
• whether directly or through one or more interposed entities
controls or may benefit from 50 percent or more of the rights to income or contributed
capital or voting power of the entity
Provided that the following persons are not associated persons:
1. Employee,
2. Persons prescribed by Department as “not an associated person”

l) Tax Payer’s Rights (Dec 2017 & Dec 2014, 2.5 Marks)
Answer
The following rights are available to a taxpayer:
1. Right to get respectful behavior;
2. Right to receive any information related to tax as per the prevailing Laws;
3. Right to get an opportunity of submitting a proof in one’s own favor in respect of tax
matters;
4. Right to appoint lawyers or auditors for defense; and
5. Right to secrecy in respect of tax matters and to keep it inviolable.

m) Self-assessment (Dec 2017, 2.5 Marks)


Answer
Actual Self Assessment [Sec. 99 (1)]
Where a person files a return of income for an income year disclosing the following matters,
self assessment is treated as made on the due date for filing the return:
a. the income tax payable by the person for the year under section 3 (Ka) and (Kha), and
b. the amount of that tax still to be paid for the year in the amount shown in the return

Deemed Self Assessment [Sec. 99 (2)]


Where a person fails to file a return of income for a tax year then, until such time as a return
is filed, an assessment is treated as made on the due date for filing the return as follows:
a. the income tax payable by the person for the year is equal to the sum of any income tax
withheld from payments derived by the person during the year under Chapter 17 of the Act
and any income tax paid by the person by installment for the year under Chapter 18 of the
Act, and
b. There is no tax payable on the assessment.

n) Presumptive Tax for small resident Vendors (Dec 2016, 2.5 Marks)
Answer
If all the following conditions are satisfied, the tax paid as explained below shall be the tax
liability of such person, and the person is relieved from detailed calculation of tax liability:

© The Institute of Chartered Accountants of Nepal 341


CAP II Paper 7 Income Tax and VAT

a. The person shall be a Resident Natural Person,


b. The person shall derive Income only from Business having source in Nepal during the
Income Year,
c. The person shall not claim Medical Tax Credit (as per Sec 51) and Advance tax arising
out of Withholding Taxes (as per Sec 93),
d. The Income from the Business (Sales or Turnover) shall not be more than Rs. 20 Lakhs and
the profit from such business shall not exceed Rs. 2 Lakhs.
e. The person shall elect this Section to be applied in writing, i.e. the person shall make a
written application (a specific form is developed by IRD on which the person shall make a
tick to opt this facility) to IRD to pay presumptive tax.
f. The person shall not be registered for Value Added Tax purpose.

Amount of Tax to be paid [Schedule 1- Section 1 (7)]


Area of conducting Business Amount of Tax (Rs.)
Metropolitan City and Sub Metropolitan City 7,500
Municipality 4,000
Area other than above (VDC) 2,500

o) Presumptive Tax for Public Transport Carrier Business run by Natural Person
(Dec 2016, 2.5 Marks)
Answer
With amendment in Sec. 1 (13) of Schedule 1 of Income Tax Act, 2058, there shall not be
presumptive tax for public transport carrier business run by natural person.

p) Assessable Income (Dec 2016, 2.5 Marks)


Answer
The following income of a person during any Income Year under head business, employment,
investment and windfall gain shall be the Assessable Income of such person subject to Income
Tax Act, 2058:
i. Income of a resident person from business, employment, investment & windfall gain
regardless of the Source of such payments, and
ii. Income of a nonresident person from business, employment, investment & windfall gain
having source in Nepal
The Assessable Income shall not include any amounts exempted u/s 11 or u/s 64 or under both
Sections of the Act.

q) Bad debts recovered (June 2016, 2.5 Marks)


Answer
When there is writing off of debts for accounting purpose and if the write off is not deductible
for tax purpose, the accounting base of debtor decreases with no change in the tax base of
debtor.
When such written off debts are recovered, the accounting impact is charging such recovery
as income, however, since there is tax base of such debtor, there would be a credit to debtor
account for tax purpose, which means the recovery of such bad debts does not have any tax
impact.

© The Institute of Chartered Accountants of Nepal 342


Compilation of Suggested Answers Income Tax

If the write offs are deductible for tax purpose, thereby decreasing the tax base of debtor; the
recovery is treated as income during the year of recovery.

Simple words:
If the bad debt was allowed for tax purpose at the time the related debt was written off, recovery
is treated as income for tax purpose and vice-versa.

r) Entity (Dec 2014, 2.5 Marks)


Answer
Entity means the following organizations:
1. Partnership, Trust or Company
2. Rural Municipality, Urban Municipality or District Coordination Committee
3. Government of Nepal or Province Government or Local Government
4. Public International Organization established under any foreign government or state
government of such foreign government or local bodies of such foreign government or
under any treaty
5. Foreign Permanent Establishment

s) Company (June 2014)


Answer
Company means any company incorporated under prevailing company law and the following
organizations are also treated as company for tax purpose:
1. Any organized institutions established under prevailing law (e.g. ICAN, Tribhuwan
University, Kathmandu University, etc.)
2. Any unorganized association, committee, organization, or society, or group of persons
except private firm and partnership, or trust
3. Partnership firm having 20 or more partners whether registered or not under prevailing
law, Retirement Fund, Cooperatives, Unit Trust, Joint Venture
4. Foreign Company
5. Any foreign organization as prescribed by Director General of IRD

© The Institute of Chartered Accountants of Nepal 343


CAP II Paper 7 Income Tax and VAT

t) Lease (June 2014)


Answer
Lease means temporary right of a person to use any asset other than cash or cash equivalents
of another person and the term includes license, rent agreement, option, royalty agreement or
right of lessee.

u) Investment (June 2014)


Answer
“Investment” means the holding of one or more properties or the investment in a property
except the following:
1. The use of the property for the private use of the owner, or
2. A business or an employment.
However, the holding of a non-business chargeable asset is also known as an investment.

v) Receiver (June 2014)


Answer
As per Clarification Clause to Sec. 108, the Receiver means the following person:
1. A liquidator;
2. A receiver appointed by a court or out of a court in respect of an asset or an entity;
3. A person who has taken the assets in possession in case the asset is mortgaged to him;
4. An executor, administrator, or direct heir of a deceased individual’s estate; or
5. Any person conducting the affairs of an incapacitated individual.

w) Trustee (Dec 2013)


Answer
Trustee means a natural person, guthi or any organized institutions that holds assets in
fiduciary capacity either singly or jointly in association with another natural person, guthi or
other organized institutions and the term includes:
1. Operator or Administrator of property of a deceased natural person
2. Liquidator, Receiver or Trustee
3. Person controlling, directing, managing or preserving asset of an incapacitated person
either in private or authorized capacity
4. Private Enterprises or any person managing assets under private enterprises
5. Any other person acting in the capacity as described in four points above

x) Royalty (Dec 2013)


Answer
‘Royalty’ means any payments for the lease of intangible asset and the term also includes any
payment made for the purpose of following:
1. Use of or right to use copyright, patent, design, model, plan, confidential formula, or
process or trademark;
2. Rendering of technological knowledge;
3. Use of or right to use cinematographic movie, video tape, sound recording, or similar other
mediums, or use of or rendering of industrial, trading or scientific experience;
4. Any type of assistance in a manner to be ancillary to the matters referred to in clauses (a),
(b), or (c) above, or

© The Institute of Chartered Accountants of Nepal 344


Compilation of Suggested Answers Income Tax

5. Complete or partial restriction for the matters referred in (a), (b), or (c) above
But the term ‘royalty’ does not include the payment for natural resources.

y) Permanent Establishment (Dec 2013)


Answer
“Permanent establishment" means a place where a person carries on business whether fully
or partially, and includes-
1. a place where a person is carrying on business partially or fully through an agent, other
than a general agent of independent status acting in the ordinary course of business as
such;
2. a place where a person has, is using, or is installing substantial equipment or substantial
machinery;
3. one or more places of a country where a person provides technical, business or consultancy
services through employee or otherwise for more than 90 days in any 12 months period
4. a place where a person is engaged in a construction, assembly, or installation project for
90 days or more, including a place where a person is conducting supervisory activities in
relation to such a project

z) Investment Insurance (Dec 2013)


Answer
“Investment Insurance” means insurance of any of the following class:

a. insurance where the specified event is the death of an individual who is the insured or an
associated person of the insured;
b. insurance where the specified event is an individual who is the insured or an associated
person of the insured sustaining personal injury or becoming incapacitated in a
particular manner
Provided that: the insurance agreement is expressed to be in effect for at least five years
or without limit of time and is not terminable by the insurer before the expiry of five years
except in special circumstances prescribed in the insurance contract;
c. insurance under which an amount or series of amounts is to become payable to the
insured in the future;
d. Reinsurance of insurance as specified in (a), (b) or (c) above, and
e. Reinsurance of reinsurance as specified in (d) above

aa) Tax (June 2013)


Answer
“Tax” means the tax imposed under this Act; the term also includes the following payments:
1. Expenses incurred and claimed by the Department in respect of claim over asset and
auction sales of a tax debtor pursuant to Sec. 104 (8) (Ka),
2. Amount payable withholding agent or withholdee pursuant to under Sec. 90 or the amount
payable under Sec. 94 in installment or the amount payable by an advance tax collection
agent pursuant to Sec. 95Ka or amount payable after tax assessment pursuant to Sec. 99,
100 and 101,

© The Institute of Chartered Accountants of Nepal 345


CAP II Paper 7 Income Tax and VAT

3. Amounts payable to the Department in respect of tax liability of a third party under Section
107 (2), Section 108 (3) or (4), Section 109 (1), Section 110 (1),
4. Amounts as referred to in Chapter 22 which are to be paid in respect of fees and interest,
and
5. Fines as referred to in Section 129 which are to be paid on the order of the Department.

bb) Debt Claim (June 2013)


Answer
‘Debt Claim’ means the right to receive payment by any person from another person and the
term also includes right to refund of amount advanced to another person, deposit kept in Banks
and Financial Institutions, receivable, debenture, negotiable instruments, bond, rights under
annuity, or right to receive payments under installment sale or finance lease.

cc) Underlying Ownership (June 2013)


Answer
It means the ownership as follows:
a. in relation to an entity, means interests owned in the entity, directly or indirectly through
one or more interposed entities, by individuals or by entities in which no person has an
interest; or
b. in relation to an asset owned by an entity, is determined as though the asset is owned by
the persons having underlying ownership of the entity in proportion to that ownership of
the entity;

dd) Payment (June 2013)


Answer
“Payment” means the following:
a. the transfer by one person of an asset or money to another person or the transfer by another
person of a liability to the one person;
b. the creation by one person of an asset that on creation is owned by another person or the
decrease by one person of a liability owed by another person;
c. the provision by one person of services to another person; and
d. the making available of an asset or money owned by one person for use by another person
or the granting of use of such an asset or money to another person

© The Institute of Chartered Accountants of Nepal 346


Compilation of Suggested Answers Value Added Tax

Part B Value Added Tax


Basic Concepts .............................................................................................................................................. 2
VAT Registration ........................................................................................................................................ 17
Taxable Value ............................................................................................................................................. 35
Reverse Charging ........................................................................................................................................ 51
Input Tax Credit and Computation of VAT Obligation .............................................................................. 54
VAT Refund................................................................................................................................................ 99
VAT Returns and Payment of VAT .......................................................................................................... 115
Accounting Records & Invoices ............................................................................................................... 118
Tax Assessment ........................................................................................................................................ 122
Fees, Interest, Penalties and Circumstance beyond Control ..................................................................... 125
Short Notes Questions............................................................................................................................... 130
True False Question .................................................................................................................................. 143
Court Procedure ........................................................................................................................................ 149

© The Institute of Chartered Accountants of Nepal 1


CAP II Paper 7 Income Tax and VAT

Basic Concepts
1. Hub Trade Pvt. Ltd. is engaged in import and sales of various types of goods from
India, China, Hongkong and America. During Income Year 2075/76, it imported
following goods from various countries:

India INR 100,000 (1 INR = 1.6 NPR)


China RMB 15,000 (1 RMB = 15 NPR)
Hongkong HKD 15,000 (1 HKD = 15 NPR)
America USD 25,000 (1 USD = 111.06 NPR)

• All goods are imported through Calcutta Port through Karkarvitta Customs.
• Custom charged on goods by Kakarvitta Custom is 15%, and VAT is imposed at
normal rate except for some goods imported from China which are 45% VAT
exempted goods.
• Hub Trade Pvt. Ltd. sells the goods to a wholesaler, who in turn sales it to a
distributor and who inturn sales it to the final consumer.
There is following value additions at each level:
• Hub Trade Pvt. Ltd.: Rs. 350,000 administrative expenses and 10% mark up on
sales
• Wholesaler: Rs. 200,000 administrative expense and 10% profit on cost
• Distributor: Rs. 220,000 administrative expenses and 20% mark up on sales

Required: (Dec 2019, 10 Marks)

a. VAT paid by each business at each stage of sales channel


b. Final Cost to consumer for the goods of VAT attractive goods and VAT exempted
goods.
Answer
Calculation for First Chain of Business (Hub Trade Pvt. Ltd.)
Amount (Nrs.)
Foreign Rate of
Imported From Tax
Currency exchange Total Taxable
Exempt
India 100000 1.6 160,000.00 160,000.00
Hong Kong 15000 15 225,000.00 225,000.00
America 25000 111.06 2,776,500.00 2,776,500.00
China 15000 15 225,000.00 123,750.00 101,250.00
Total 3,386,500.00 3,285,250.00 101,250.00
Add: 15% Custom 507,975.00 492,787.50 15,187.50
Total Base of VAT
Calculation on Import 3,894,475.00 3,778,037.50 116,437.50

© The Institute of Chartered Accountants of Nepal 2


Compilation of Suggested Answers Value Added Tax

VAT 13 % paid on import 491,144.88 491,144.88 Exempt


Add : 350,000
administrative expenses
(Note 1) 350,000.00 339,535.66 10,464.34
Cost of sales 4,244,475.00 4,117,573.16 126,901.84
Value Addition (10% margin on sales) (The margin
is 10% on sales price, the cost is 90% of sales) 471,608.33 457,508.13 14,100.20
Sales Price 4,716,083.33 4,575,081.28 141,002.05
VAT on sales 594,760.57 Exempt
Less : VAT paid on
purchase/import 491,144.88
Net VAT Payable 103,615.69
Note 1:Administrative expenses distributed in the ratio of total cost for taxable and tax exempt
goods, as per question the total administrative expense is for sales of total of goods)

Calculation for Second Chain of Business Tax


Total Taxable
(Wholesaler) Exempt
Purchase Cost 4,716,083.33 4,575,081.28 141,002.05
VAT on purchase 0.00 594,760.57 Exempt
Add : administrative expenses (200,000 distributed
in the ratio of purchase cost) 200,000.00 194,020.38 5,979.62
Total Cost 4,916,083.33 4,769,101.66 146,981.67
Add : Value Addition (10% margin on cost) 491,608.33 476,910.17 14,698.17
Sales Price 5,407,691.67 5,246,011.83 161,679.84
Add: 13% VAT on sales 681,981.54 681,981.54 0.00
Less: VAT paid on purchase 0.00 594,760.57 0
Net VAT Payable 681,981.54 87,220.97 0.00

Calculation for Third Chain of Business Tax


Total Taxable
(Distributor) Exempt
Purchase Cost 5,407,691.67 5,246,011.83 161,679.84
VAT on purchase 681,981.54 681,981.54 0.00
Add : administrative expenses (220,000 distributed
in the ratio of purchase cost) 220,000.00 213,422.41 6,577.59
Total Cost 5,627,691.67 5,459,434.24 168,257.43
Add : Value Addition (20% margin on sales) 1,406,922.92 1,364,858.56 42,064.36
Sales Price 7,034,614.58 6,824,292.80 210,321.79
Add: 13% VAT on sales 887,158.06 887,158.06 Exempt
Less: VAT paid on purchase 681,981.54 681,981.54

© The Institute of Chartered Accountants of Nepal 3


CAP II Paper 7 Income Tax and VAT

Net VAT Payable 205,176.53 205,176.53

Calculation for Final Consumer (Final Chain on Tax


Total Taxable
business) Exempt
Purchase Cost 7,034,614.58 6,824,292.80 210,321.79
VAT on purchase 887,158.06 887,158.06 Exempt
Total Cost 7,921,772.65 7,711,450.86 210,321.79

2. Morang Distributers (Pvt.) Ltd is a sole purchaser of VAT attractive goods from the
manufacturer. Per unit cost price of the goods is Rs. 40,000.00. The company
distributes the goods to the wholesaler, wholesaler sells the goods to the retailer. The
company and the wholesaler, each incurs transportation expenses Rs. 500.00 for the
goods and they charge 10 % profit margin to determine the selling price. The
wholesaler provides 5 % discount on selling price to the retailer. The retailer sells the
goods on the same selling price fixed by the wholesaler. The goods are taxable as per
VAT Act, 2052.
Required: (June 2018, 5 Marks)
a. Actual cost with VAT to the final consumer.
b. Total VAT payable to the government.
Ignore the VAT payable by the manufacturer
Answer
Profit margins are expressed in terms of percentage of sales unless otherwise specified in
question
Assumption: The retailer works under commission basis. 5% discount on selling price is
commission to retailer by Wholesaler.
Level of Cost Other Profit Taxable VAT Input VAT
Business Price costs Margin Value Amount VAT payable
credit
Manufacturer - - - 40,000 5,200 Not Not
known known
Morang 40,000 500 45,000 * 40,500/ 5,850 5,200 650
Distributors 0.1= 0.9 =
4,500 45,000
Wholesaler 45,000 500 5,055 45,500/
0.9 =
50,555

© The Institute of Chartered Accountants of Nepal 4


Compilation of Suggested Answers Value Added Tax

3. Nepal Surgical House (Pvt.) Limited is a principal stockiest for Nepal of Medtronic Inc, Hong Kong (MIH). The company
has policy to sell the items to other distributors with 15% markup at landed cost. Medical and surgical equipment
manufactured by MIH are both VAT attractive and VAT exempt. MIH makes the shipment by billing on FOB Price and
import is done through irrevocable letter of credit. The Company provides following information for the month of
Chaitra 20X-74:

a) All items are subject to 5% custom duty under HS code 84.


b) Shipment against the LC established in Falgun 20X-74 having1FOB value USD 126,300.00 (CIF Birgunj) of which
USD 43,500.00 is subject to VAT was released during the month and entire stock was sold on the same month.
c) Of the total import of taxable goods during the month, items having FOB Price USD 26,400.00 was revalued to USD
33,100.00 by the custom authorities.
d) Terms of Payment 20% during the establishment of LC and 80% on release of documents.
e) Effective exchange rate per USD was 102.6, 103.8 & 103.45 during the date of LC establishment, document release
and custom clearance respectively.
f) Previous non-VAT stock having landed cost of Rs. 2,526,200.00 was also sold during the month.
g) The company paid Rs. 146,800.00 inclusive of tax for Birgunj Kathmandu transportation and insurance of the
consignment.

On the basis of above information, prepare VAT statement of Nepal Surgical House (Pvt.) Limited for the month of
Chaitra 20X-74 considering opening VAT credit of Rs. 73,612. (June 2018, 10 Marks)
Answer
Taxable Value as per Sec. 12 in case of Import and determination of VAT paid on Import
Taxable value shall be landed cost up to border, plus any other indirect taxes levied at the Customs Frontier. When Customs officer revises
the valuation of goods, VAT shall be computed based on revised cost. Customs valuation is based on foreign currency and the conversion
rate shall be the rate prevailing on the date of customs clearance

1
The question modified using CIF Birgunj terms instead of FOB Price, as the question lacks information regarding the insurance and
transportation of goods cost outside Nepal.

© The Institute of Chartered Accountants of Nepal 5


CAP II Paper 7 Income Tax and VAT

Particulars CU Total Taxable VAT exempt Notes


CIF value of goods imported in Chaitra USD 126,300 43,500 82,800 Given
CIF Value of goods revalued by Customs
USD 26,400 - Given
Authority
Not revalued value- to be considered for Total CIF value minus CIF value of
USD 17,100 82,800
customs valuation goods for which valuation is revised
Revised Cost of Taxable Goods as per
USD 33,100 -
Customs
CIF value of VAT exempt goods and
Total Revised Cost of the Goods imported USD 50,200 82,800
revised value for taxable Goods
As per Customs Act, 2063; rate of
Conversion Rate 103.45 103.45
customs clearance date shall be taken
Transaction value for Customs purpose NPR 5,193,190 8,565,660
Customs duty @ 5% 259,659.50 428,283
Taxable Value 5,452,849.50 8,993,943
VAT Paid on Import @13% 708,870.44 Exempt

Determination of Selling Price of Goods & Output VAT


Particulars CU Total Taxable VAT exempt Notes
CIF value of goods imported in Chaitra USD 126,300 43,500 82,800 Given
Payment for goods
During LC Establishment (20%) USD 8,700 16,560
At the time of release of documents (80%) USD 34,800 66,240
Conversion into NPR
For payments during LC Establishment NPR 892,620 1,699,056 using conversion rate as on that date
For payments at the time of release of
NPR 3,612,240 6,875,712 using conversion rate as on that date
documents
Add: Customs duty 259,659.50 428,283.00 Calculated as above

© The Institute of Chartered Accountants of Nepal 6


Compilation of Suggested Answers Value Added Tax

Apportioned on the basis of total NPR


Add: Transportation and insurance in Nepal NPR 129,911.50 44,743.87 85,167.64
Cost
Total Landed Cost (as per NFRS) to value
4,809,263.37 9,088,218.64
inventory
Markup @ 15% on Landed Cost 721,389.51 1,363,232.80
Selling Price 5,530,652.87 10,451,451.43
Output VAT on Goods imported during
721,389.51 Exempt
Chaitra

VAT Statement for the month of Chaitra


Output VAT 721,390
Less: Input VAT Credit
Full VAT Credit (708,870) On Import of VAT attractive items
On Transportation Cost (in
proportion of sales)=
Proportionate VAT Credit (5,844)
129,911.50/(Total Sales) * taxable
Sales
Opening VAT Credit (73,612)
VAT Payable/(Receivable) (66,937)

© The Institute of Chartered Accountants of Nepal 7


CAP II Paper 7 Income Tax and VAT

4. A lawyer practicing in Nepal has provided consultancy services to the Legal Firm
situated in United Kingdom. The legal firm situated in United Kingdom has used such
services for the release of persons in USA. Where is place of supply of services as per
Value Added Tax Act/Rules? (Dec 2012, 5 Marks)
Answer
As per Rule 16 of Value Added Tax Regulations, 2058; the place of supply of service is the
place where the benefit of service is received.

In the given circumstances, the benefit of service rendered by a lawyer in Nepal is received
in United Kingdom, no matter where the service is used by the legal firm in United
Kingdom.

Hence, the place of service is United Kingdom.

5. Moonlight Traders is engaged in the business of selling Television sets on installment


basis. As per the policy and contract agreement, forty percent of the Price of the
Television Rs. 20,000, i.e. Rs. 8,000 has to be paid as upfront payment in cash and
thereafter monthly installment of Rs.1000 along with interest 1% per month has to be
paid at the end of the month. Mr. Ram Binod has purchased a television set by paying
Rs. 8000 in cash on 20X-66.09.07. Installment of Magh and Falgun has been paid on
Falgun 05, 20X-66. By mentioning the relevant provision, determine the time of
supply for Magh and Falgun Month installments. (Dec 2012, 4 Marks)
Answer
As per Sec. 6 (3) of Value Added Tax Act, 2052; the time of supply, in case where the goods
or services are supplied so as to obtain the consideration in tranches under an installment
agreement, is earlier of payment of installment or due date for payment of installment.

The time of supply for the given case is as follows:


Particulars Due date Payment date Time of Supply
Installment of Magh Magh end Falgun 5, 20X-66 Magh end
Installment of Falgun Falgun end Falgun 5, 20X-66 Falgun 5, 20X-66

© The Institute of Chartered Accountants of Nepal 8


Compilation of Suggested Answers Value Added Tax

6. Define Electronic Media and Registered Person as per the VAT Act, 2052
(Dec 2012, 2 Marks, CA Inter)
Answer
Electronic Media:
“Electronic Media” means computer, internet, email, facsimile, Electronic cash register,
fiscal printer, or similar approved media and the term includes any other similar media
prescribed by the Department having similar characteristics.

Registered Person:
“Registered person” means any person who is registered to conduct transactions pursuant
to Section 10 or Section 10A or Section 10B.

7. State the place of supply as per Value Added Tax Act, 2052/Value Added Rules, 2053,
in case of: (June 2012, 5 Marks)
a) Sale of movable goods
Answer: the place from where the buyer takes possession of the goods.

b) Immovable goods
Answer: the place where the goods are situated

c) Imported goods
Answer: the customs frontier from where the goods first enter into Nepal

d) Self-consumption
Answer: the place where vendor resides

e) Service
Answer: the place where the benefit of service is received.

8. Cosmos Pvt. Ltd. having registered office in Jawalakhel, Lalitpur transferred some
goods costing Rs. 1,000,000 to its branch in Bangladesh. In the same month, branch in
Bangladesh transferred some goods costing Rs. 500,000 to its head office in Nepal.
Gross Profit margin of head office is 20% and branch is 25%. The profit of branch is
consolidated at the end of each financial year. Find the VAT impact on the above
transactions. (June 2012, 5 Marks)
Answer
In the given case, there are two transactions:
a. Import from branch in Bangladesh (transfer of goods costing Rs. 500,000 to Head
office from Branch in Bangladesh), and

© The Institute of Chartered Accountants of Nepal 9


CAP II Paper 7 Income Tax and VAT

b. Export to the branch in Bangladesh (registered office in Jawalakhel, Lalitpur


transferred some goods costing Rs. 1,000,000 to its branch in Bangladesh)

Therefore, assuming the goods to be exported and imported is VAT-attractive:


a. Cosmos Pvt. Ltd. has to pay VAT @ 13% at Customs Frontier on the value of goods
determined as per Sec. 12 of Value Added Tax Act, 2052 for the goods to be imported
from Bangladesh. The taxable value is landed cost plus customs duty plus excise duty
plus any other indirect taxes applicable at Customs frontier.
b. Cosmos Pvt. Ltd. has to pay VAT @ 0% on goods to be exported.

9. Tax Accountant of Subhashree & Co. contends that the terms “Zero VAT” and “No
VAT” has the same meaning. As a Tax Expert, express your opinion within the
framework of VAT Act, 2052. (Dec 2011, 5 Marks)
Answer
The terms “Zero VAT” and “No VAT” does not have the same meaning. It can be
distinguished as follows:
Basis Zero Rate No VAT
Rate of VAT VAT is levied at Zero Percent VAT is not levied at all
VAT Invoice A VAT Invoice shall be issued as per the VAT Invoice is not required to
format prescribed by VAT regulation be issued for transaction of
such items
Registration The person dealing in Zero rated items The person dealing exclusively
Requirement shall be registered for VAT purpose. in No VAT items is relieved
from registration formality
Other The person dealing in Zero Rated Items The person dealing exclusively
formalities shall observe other formality as per VAT in No VAT items is relieved
as per the Act, such as- maintenance of sales and from all formalities to be
Act purchase register duly certified from Tax observed as per the Act
Officer, submission of VAT return, etc.

10. You are a tax expert. Answer stating the provisions relating to “Time of Supply” as
per Value Added Tax (VAT) Act, 2052, what happens when: (Dec 2011, 10 Marks)
a) A customer takes delivery of the goods from the business place before the supplier
issues invoice.
Answer:
The time of supply of goods is earliest of “issuance of invoice”, or “at the time of
taking possession of the goods by buyer”, or “at the time of receipt of consideration”.

In this case, the time of supply is when the buyer takes possession of the goods.

© The Institute of Chartered Accountants of Nepal 10


Compilation of Suggested Answers Value Added Tax

b) A customer paid NPR 10,000 to a supplier along with a list of certain goods to be
supplied to the customer. As per the list the total goods are worth NPR 12,000
Answer:
The time of supply of goods is earliest of “issuance of invoice”, or “at the time of
taking possession of the goods by buyer”, or “at the time of receipt of consideration”.

Since the payment is advance is not treated as consideration, the time of supply would
be when any of the above event occur.

c) A supplier at Kathmandu receives order for supply goods from a customer from
Pokhara. The supplier packs the goods as per the order, asks a labor to deliver the
goods to the transporter and the transporter gives delivery of the goods to the
party of Pokhara at Pokhara. When the buyer receives the goods, the supplier
issues the invoices.
Answer
The time of supply of goods is earliest of “issuance of invoice”, or “at the time of
taking possession of the goods by buyer”, or “at the time of receipt of consideration”.

In the given case, the buyer is deemed to take possession the goods when the supplier
delivers the goods to the transporter and it is the earliest event, as such, it is the time of
supply.

11. Define “Person”, “Goods” and “Service” as per VAT Act.


(June 2011, 5 Marks, CA Inter)
Answer
Person: “Person” means any natural person, firm, company, association, institution,
partnership firm, cooperative, joint business, religious endowment, or fund; and the
term also includes any government body, any religious organization, charitable trust or
similar other bodies and branches or sub-branches, with or without any profit motive,
involved in Taxable Transactions.

Goods: Goods means both movable and immovable assets.

Service: Service means anything other than goods.

© The Institute of Chartered Accountants of Nepal 11


CAP II Paper 7 Income Tax and VAT

12. How the place of supply of goods and services is determined under the VAT laws?
(June 2009, 7 Marks)
Answer
Place of supply of Goods:
The following places shall be deemed to be the place of supply of goods:
a) In the case of movable goods transferred by sale, the place where such goods were sold
or transferred,
b) In the case of any immovable goods where location can’t be transferred even if change
in ownership , the place where such goods are located,
c) In the case of imported goods, the Customs point in Nepal through which goods are
imported into Nepal,
d) In case any producer or vendor supplies the goods to the self, the place where the
producer or the vendor of such goods resides.

Place of Supply of Services


The place of supply of a service shall be the place where the benefit of that service is
received.

13. Mr. Deshmukh, an importer imported certain goods at NRs. 50,000. No VAT was
paid on its import. The goods passed to the final consumer through a retailer. Both
middlemen incurred NRs. 1,000 each for administration expenses. Both middlemen
charged 15% profit margin on selling price.
Required: (June 2009, 5+3)
a) Cost price to the final consumer
b) VAT payable to the government at each stage
Answer

Level of Cost Other Profit Taxable VAT Input VAT


Business Price costs Margin Value Amount VAT payable
credit
Importer 50,000 1,000 9,000 =51,000/ 7,800 0 7,800
(60,000 * 0.85 =
15%) 60,000
Retailer 60,000 1,000 71,764.71 61,000/ 9,329.41 7,800 1,529.41
* 0.15= 0.85 =
10,764.71 71,764.71

© The Institute of Chartered Accountants of Nepal 12


Compilation of Suggested Answers Value Added Tax

14. What is the value-add taxable transactions? At what point of time and place of the
transaction, the value added tax is liable to be determined and collected and paid to
the Government. (June 2009& 2004, 2+4+4=10, CA Inter)
Answer
Value Added Tax is applicable on following transactions:
a. Supply of goods or services within the state of Nepal,
b. Import of goods or services into the state of Nepal,
c. Export of goods or services from the state of Nepal.

However, where the goods or services are listed in Schedule 1 of the Value Added Tax Act,
there shall not be applicability of Value Added Tax. Further, value added tax is exempt on
transfer of ownership of business to a registered person or a legal heir in the event of death
of owner.

Point of time when VAT is levied:


Supply of goods or services shall be considered to have taken place at the earliest time of
the following times:
(a) When an invoice is issued by a supplier;
(b) In the case of supply of goods, when the acquirer removes or takes possession of the
goods from the supplier's transaction place;
(c) In the case of supply of services, when the service is rendered;
(d) When supplier receives the consideration for goods and services

However, in the following cases, the time of supply shall be as follows:


(a) In the case of services which are continuously provided, namely, telecommunication
services or similar other public services, when the invoice is issued;
(b) Where there is a contractual provision for paying partially the value of goods or
services in more than one day on contractual basis, the supply time shall be the earliest
day on which the payment is made or the day on which the payment is to be made
according to the contract;
(c) In the case of goods or service which are so used as not to be allowed as offset under
this Act, the time when such goods or services are used

Place of Supply
Place of supply of Goods:
The following places shall be deemed to be the place of supply of goods:
a) In the case of movable goods transferred by sale, the place where such goods were sold
or transferred,
b) In the case of any immovable goods where location can’t be transferred even if change
in ownership , the place where such goods are located,

© The Institute of Chartered Accountants of Nepal 13


CAP II Paper 7 Income Tax and VAT

c) In the case of imported goods, the Customs point in Nepal through which goods are
imported into Nepal,
d) In case any producer or vendor supplies the goods to the self, the place where the
producer or the vendor of such goods resides.

Place of Supply of Services


The place of supply of a service shall be the place where the benefit of that service is
received.

15. State whether the following items are liable to VAT or not:
(Dec 2008, 5 Marks, CA Inter)
a) Drinking water supplied by the Nepal Drinking Water Company
Exempt

b) Green Tea leaves sold by the farmer


Exempt

c) Unprocessed tobacco sold by the farmer


Exempt

d) Tea chests imported for packing by a trader for resale


VAT- attractive

e) Medical services provided by private hospitals and clinics


Exempt

f) Carpets made from raw wool imported from New Zealand & Tibet
VAT attractive

g) Cinema Hall tickets


VAT attractive

h) Elevator /Lift imported by an individual from India for putting in High rise
building of residential flats
VAT attractive

i) 11 KV Electric Power supplied by Nepal Electricity Authority to an industry


Exempt

j) 50 Kg Jute bags for packing sugar by a sugar mill


VAT attractive

© The Institute of Chartered Accountants of Nepal 14


Compilation of Suggested Answers Value Added Tax

16. State whether VAT is applicable to the following cases: (June 2008, 10 Marks, CA Inter)
a) Import of water pumps by a dealer in water pumps.
VAT attractive

b) Bulldozer imported by a construction company


Exempt

c) Mineral water
VAT attractive

d) Cut fruits supplied to customer at a restaurant and registered under VAT


VAT Attractive

e) Tractor imported by a trader in Tractors


VAT Attractive

f) Telephone services charged by the telecom company


VAT attractive

g) Electricity charges billed by NEA


Exempt

h) 2 Kg gold brought as personal baggage while coming from a foreign country after
staying there for more than 1 year
Exempt

i) Rent received on letting out residential quarter by an individual


Exempt

j) Rent charged by the hotel for a room occupied by a company for providing
accommodation to the manager of a trading company
VAT attractive

© The Institute of Chartered Accountants of Nepal 15


CAP II Paper 7 Income Tax and VAT

17. State whether the following are liable to Value Added Tax or not and the rate if any:
(June 2007, 3 Marks, CA Inter)
a) A local dairy sells Curd worth Rs. 4,00,000 during the month
Answer
Note: Curd is prepared by using additives such as lemon juice or vinegar. Yogurt is
prepared under natural fermentation process. Therefore, curd is VAT attractive and
Yogurt is exempt from VAT.

In the given case, the taxable value is Rs. 400,000 and is subject to VAT @ 13%.

b) Global Enterprises Organizes a Cultural Program at the National Academy Hall


and collects Rs. 2 Lacs as ticket charges.
Answer
The ticket fee is VAT attractive and the rate is 13%.

c) LBL, a Legal Consultancy Firm, registered to VAT provides legal services to Mr.
X residing outside the Country worth Rs. 50,000. Mr. X does not have any
business, business agent or legal representative in Nepal.
Answer:
It is the case of export of service; therefore, the VAT is applicable @ 0%.

© The Institute of Chartered Accountants of Nepal 16


Compilation of Suggested Answers Value Added Tax

VAT Registration
18. Manaslu Enterprises, Kathmandu is trading in clothes, and has the following
transactions in the last twelve months from the end of Bhadra, 20X-75. The
Enterprises is not registered in VAT.
a. Export sales Rs. 20 lakh (Kartik 20X-74 to Magh 20X-74)
b. Sales in foreign currency from the Shop: equivalent Rs. 10 lakh (Bhadra 20X-75)
c. Sales in local currency Rs. 25 lakh (Ashwin 20X-74 to Bhadra 20X-75)
d. Purchase Rs. 40 lakh
e. Loan from bank Rs. 15 lakh
f. Loan payment Rs. 15 lakh
g. Salary and other expenses Rs. 10 lakh
h. Rent expenses Rs. 2 lakh
i. Telephone expenses Rs. 1 lakh

Mention the VAT implication and registration in the above transaction.


(Dec 2018, 5 Marks)
Answer

Conditions for Registration of VAT


As per Sec. 10 of Value Added Tax Act, 2052 (as amended until the date), the conditions for
registration for VAT are as follows:
(a) In case of a person who conducts following transactions shall register for VAT
purpose:
Transaction of bricks after its production, operation of business related to
alcohol, wine, health club, disco theque, massage therapy, motor parts, electronic
software, customs agent, business of toy, business of scrap,2 trekking, rafting,
ultra- light flight, paragliding, tourist vehicle, crusher, sand mine, slate or stone

(b) In case of a person who conducts following transactions in metropolitan city, sub-
metropolitan city, municipality 3 or other areas prescribed by Inland Revenue
Department shall register for VAT purpose:
business of hardware, sanitary, furniture, fixture, furnishing, automobiles, motor
parts, electronics, marble, educational and legal consultancy, accounting and
auditing services, catering service, party palace, parking service, dry cleaners
using machinery, restaurant with bar, ice cream industry, color lab, boutique,
Tailoring with shirting suiting materials, uniform supplier for educational, health
or other entities

2
Amended by Finance Bill, 2076
3
Amended by Finance Bill, 2076

© The Institute of Chartered Accountants of Nepal 17


CAP II Paper 7 Income Tax and VAT

(c) In case of a person that obtains business loan exceeding Rs. 10 lakhs from banks or
financial institutions.
(d) In case where tax officer, during the time of time of inspection, finds stock more than
the limit prescribed by Inland Revenue Department
(e) In case the taxable turnover of a person exceeds as follows during last 12 months
period:
i. Person dealing in goods- Rs. 50 Lakhs
ii. Person dealing in services- Rs. 20 Lakhs
iii. Person dealing mixed transactions of goods and services- Rs. 20 Lakhs

Turnover means the higher of sales or purchase.

Analysis of Question
The person is dealing in clothes, therefore, condition (a) and (b) above is not satisfied.

With regards to condition (c), the person has obtained business loan from bank, as such, it
has to compulsorily get registered for VAT purpose.

Further, the enterprises’ total taxable turnover for the period of 12 months from Ashoj
20X-74 until Bhadra end 20X-75 is:
Particulars Amount (In Rs.)
Export sales 2,000,000
Sales in Foreign Currency 1,000,000
Sales in local currency 2,500,000
Total sales in last 12 months [A] 5,500,000
Purchase in last twelve months [B] 4,000,000
Turnover [A or B, whichever is higher] 5,500,000
The question does not specify when the threshold of Rs. 50 lakhs exceeded. Therefore, the
exact due date for registration cannot be mentioned.

We assume, the threshold exceeded on Bhadra end, 20X-75; therefore, an application for
VAT registration shall be filed within 30 days of Bhadra end, 20X-75.

VAT shall be collected by the Enterprises once it is registered for VAT purpose.

In case, the turnover threshold of Rs 50 Lakhs exceeded before Bhadra 20X-75, the tax
officer deems that any sales by person after 30 days when the turnover threshold exceeded
is inclusive of VAT.

© The Institute of Chartered Accountants of Nepal 18


Compilation of Suggested Answers Value Added Tax

19. Horizon Enterprise, Kathmandu has wholesale transactions of clothes and rice from
two business houses with same PAN. It has the following transactions for 20X-73
Bhadra to Kartik:

Months Clothes Sales Rs. Rice Sales Rs.


Bhadra 20X-73 1,00,000 3,00,000
Aswin 20X-73 5,00,000 5,00,000
Kartik 20X-73 5,00,000 6,00,000

The enterprise recorded its clothes sales Rs. 31 lakh and rice sales Rs. 20 lakh from
Bhadra 20X-73 to Shrawan 20X-74. Then, it has following sales in 20X-74 Bhadra to
Kartik:
Months Clothes Sales Rs. Rice Sales Rs.
Bhadra 20X-74 10,00,000 3,00,000
Aswin 20X-74 16,00,000 5,00,000
Kartik 20X-74 10,00,000 6,00,000

Tax officer has issued a notice to register the business in VAT on 20X-74/05/10 to the
enterprise. The tax officer has calculated interest, additional fees, and penalties as per
VAT Act. Justify whether the enterprise is required to register in VAT or not. If it is
required to register, then when it has to do so and from when it has to collect tax?
(Dec 2017, 5 Marks)
Answer
Conditions for VAT Registration and whether the conditions are fulfilled as per given
question:
Condition
Condition Type of Business satisfied or
not
Compulsory Registration Transaction of bricks after its production, Not
is required just because of operation of business related to alcohol, satisfied
business, anywhere in wine, health club, disco theque, massage
Nepal therapy, motor parts, electronic software,
customs agent, business of toy, business of
scrap, trekking, rafting, ultra- light flight,
paragliding, tourist vehicle, crusher, sand
mine, slate or stone
Compulsory registration business of hardware, sanitary, furniture, Not
is required, if the business fixture, furnishing, automobiles, motor satisfied
are operated in parts, electronics, marble, educational
Municipality, andlegal consultancy, accounting and

© The Institute of Chartered Accountants of Nepal 19


CAP II Paper 7 Income Tax and VAT

metropolitan or sub- auditing services, catering service, party


metropolitan area or palace, parking service, dry cleaners using
areas prescribed by IRD machinery, restaurant with bar, ice cream
industry, color lab, boutique, Tailoring with
shirting suiting materials, uniform supplier
for educational, health or other entities
Compulsory Registration If loan amount exceeds Rs. 10 Lakhs Not
due to obtaining business satisfied
loan from bank
Compulsory registration where tax officer, during the time of time of N/A, as it is
due to holding of stock inspection, finds stock more than the limit decided by
prescribed by Inland Revenue Department Tax officer
Compulsory Registration For any other person than described above, To be
due to turnover threshold the registration is required when taxable tested
turnover of a person exceeds as follows during below
“Turnover” means higher last 12 months period:
of Sales or purchase a. Person dealing in goods- Rs. 50 Lakhs
amount b. Person dealing in services- Rs. 20 Lakhs
c. Person dealing mixed transactions of
goods and services- Rs. 20 Lakhs

Testing of whether turnover exceeded threshold in any 12 months period between 20X-73
Bhadra to 20X-74 Kartik:
Rice is VAT exempt
Twelve months to Total taxable Total taxable Turnover
Sales Purchase
Shrawan 20X-74 (Clothes) 3,100,000 3,100,000
Bhadra 20X-74 3,100,000+ 3,900,000
{Sales from Bhadra 20X-73 to Shrawan 1,000,000 -
20X-74 minus Sales of Bhadra 20X-73 100,000 =
Not given,
plus Sales of Bhadra 20X-74) 3,900,000
assumed to be
Ashoj 20X-74 3,100,000+ 5,100,000
less than Rs.
{Sales from Bhadra 20X-73 to Shrawan 1,000,000 -
5,000,000
20X-74 minus Sales of Bhadra 20X-73 100,000 +
minus sales of Ashoj 20X-73 plus sales 1,600,000-
of Bhadra 20X-74 plus Sales of Ashoj 500,000 =
20X-74) 5,100,000
Since, taxable turnover for 12 months ending Shrawan 20X-74 is less than Rs. 50 lakhs, the
tax officer’s order for registration on 20X-74 Bhadra 10 is not lawful.

© The Institute of Chartered Accountants of Nepal 20


Compilation of Suggested Answers Value Added Tax

The taxable turnover for the 12 months period from Kartik 20X-73 until Ashoj 20X-74
exceeded Rs. 50 Lakhs. As such, the person has to register for VAT within 30 days of the
end of Ashoj 20X-74.

The person can collect VAT on its sales only after its registration for VAT [as per Sec. 15
(1), an unregistered person cannot collect VAT unless it falls in the category of Sec. 15
(3)and it does not fall in category of Sec. 15 (3)].

20. Namaste Steel Private Limited, a VAT registered Company, has witnessed stiff
downfall in demand of its products, and hence its operation has been shut down since
the past 3 months. The company is considering for closure of the business.
Below is the detail of its assets and liabilities, and VAT receivable payable:
a. As per VAT return filed for the previous VAT period, it has VAT credit of Rs. 1
million.
b. Below is the detail of its stock and capital goods and other assets and liabilities as
on the immediately previous VAT period:
i. Finished Goods Inventory (VAT applicable) – Rs. 5 million (Market Value
Rs. 4 million)
ii. Raw Material (VAT applicable) – Rs. 1 million (Market Value Rs. 1.5
million)
iii. Damaged Goods (VAT applicable) – Rs. 5,00,000 (Market Value Rs.
1,00,000)
iv. Plant and Machinery (VAT applicable) – Rs. 9 million (Market Value Rs. 4
million)
v. Vehicle – 4 Wheeler (VAT applicable) – Rs. 3 million (Market Value Rs. 4
million)
vi. Vehicle – 2 Wheeler (VAT applicable) – Rs. 1,00,000 (Market Value Rs.
1,00,000)
vii. Land – Rs. 20 million (Market Value Rs. 50 million)
viii. Sundry Debtors – Rs. 6 million (Rs. 3 million is bad debts)
ix. Sundry Creditors – Rs. 4 million (all amount payable)
Suggest the management of the company regarding: (Dec 2017, 10 Marks)
(a) The provisions in the Value Added Tax Act for cancellation of registration of
VAT, and
(b) Amount of VAT payable or refundable to the company.
Answer
Part (a): (Suggestion to students- when question asks about provisions- write down
conditions and procedures, both)
Conditions for Cancellation of Registration

© The Institute of Chartered Accountants of Nepal 21


CAP II Paper 7 Income Tax and VAT

a) In the case of a body corporate: where such body corporate is closed, sold or
transferred or ceases to exist due to any reason,
b) In the case of individual ownership; if the owner dies,
c) In the case of a partnership firm; if it is dissolved or where a partner die,
d) In case the registered person ceases to be engaged in taxable transaction,
e) In case a registered person submits Zero Return continuously for twelve months or is a
taxpayer that does not submit Tax Return,
f) Where the taxpayer’s taxable transaction does not exceed Rs. 50 Lakhs in case of
transaction of goods and Rs. 20 lakhs in case of transaction of service or mixed
transaction of goods or services in last twelve months period,
g) Where there is registration in error.

Procedure for Cancellation of Registration


In case any of the conditions as discussed above arise and a person is required to cancel
VAT registration, the registered person or his/her legal representative/heir in his/her
absence shall submit an application in the format as prescribed by VAT Regulation within
30 days of happening of such conditions along with the VAT return and the amount of VAT
payable in front of tax officer.

The registered person shall submit all the relevant documents for tax assessment within 15
days of submission of VAT return.

The tax officer shall assess the of such taxpayer and take decision on the matter of
cancellation within 3 months of the submission of application. The matters of decision
whether to cancel or not shall also be notified to registered person within 3 months of the
application of cancellation.

The registered person is required to submit tax return for another three months until the
decision of tax officer regarding cancellation is served to him.

In case the tax officer fails to decide on matter of cancellation or does not serve
notification with regard to the cancellation of registration within 3 months of application,
the registered person is not required submit any tax return pertaining to periods after the
elapse of such three months.

© The Institute of Chartered Accountants of Nepal 22


Compilation of Suggested Answers Value Added Tax

Effect of Cancellation of Registration


The taxpayer shall assume the following items to be supplied at Market Value at the time
of cancellation of registration and shall assess and collect/levy tax:
a. Stock lying at the time of cancellation on which Input Tax Credit is already enjoyed;
and
b. Capital Items still in use, the VAT paid on purchase of which has already been claimed
as Input Tax Credit

Part (b)- Answer


When there is cancellation of registration, the capital items and stocks on which VAT credit
was already claimed are treated to be supplied at market value. The person shall collect
VAT on such market value and pay such collected VAT.
Whether
Capital goods VAT credit VAT
Market
Particulars or inventory- claimed at payable Remarks
Value
yes or no the time of @ 13%
purchase
Finished Inventory Yes 4,000,000 520,000
Goods
inventory
Raw Inventory Yes 1,500,000 195,000
material
Damaged Inventory Yes 100,000 13,000
Goods
Plant and Capital Goods Yes 4,000,000 520,000
Machinery
Vehicle- 4 Capital goods Yes 4,000,000 520,000
wheeler
Vehicle- 2 Capital goods Yes 100,000 13,000
wheeler
Land Capital Goods No - - Land is VAT exempt
Sundry Not a capital No - - There is no
Debtor goods, nor an applicability of VAT
Sundry inventory - - on debtors and
Creditor creditors, since these
are neither service,
nor goods
Total VAT to be collected as a result of cancellation of Registration 1,781,000
VAT Credit (from previous Tax period) 1,000,000
VAT payable 781,000

© The Institute of Chartered Accountants of Nepal 23


CAP II Paper 7 Income Tax and VAT

21. ABC Trading Company, which is dealing in VAT-attractive goods, has been
submitting return of income mentioning its transactions by understating annual
turnover of less than Rs. 50 lakh to avoid from registering with VAT and file return
on monthly basis. During the course of investigation in the month of Kartik 20X-71,
the tax officer found that the above entity’s transactions stood at more than Rs.50
lakh during the 6 month for the period from Bhadra 20X-70 to Magh 20X-70. The
threshold of Rs. 50 lakh has already crossed and its actual turnover has been Rs. 150
lakh during such 6 months.
By stating the relevant provision of VAT Act, give your opinion on registration in this
case and compute only VAT payable amount ignoring the amount of interest,
additional fees and penalty. (June 2016, 5 Marks)
Answer
As per Section 5Kha of Value Added Tax Act, 2052, in case tax officer identifies that a
person requiring registration as per the Act is conducting business without registration and
reasonably believes so, s/he may demand VAT registration of such person.

In case the person defends the exemption of registration requirement as a result of turnover
of the person for any period of consecutive 12 months not exceeding the threshold (i.e. Rs.
50 lakhs), the onus to prove the same is upon the person on whom the registration is
demanded.

In the given circumstances, the tax officer has the authority to demand VAT registration.
The onus to prove that the turnover is within threshold limit is upon the taxpayer.

As the assessee as aforesaid had already exceeded its threshold of Rs.50 lakh in the month
of Magh 20X-70, the assessee has the obligation to get registered with VAT office within
the following 30 days i.e. Falgun 20X-70 itself, and thereafter to file VAT return on
monthly basis.

In the case of above problem, despite crossing threshold of Rs.50 lakh, the assesse firm has
ignored to get registered, the firm is liable to pay VAT on Rs.150 lakh considering this
figure as VAT inclusive for the period from Falgun 20X-70 to Kartik 20X-71.Also, such
person is liable extra fees, interest and penalty as per sections 19, 26 and 29 respectively.

Computation of VAT payable=Rs.150, 00,000 X 13/ 113=Rs. 17, 25,664

© The Institute of Chartered Accountants of Nepal 24


Compilation of Suggested Answers Value Added Tax

22. Nepal Philanthropic Society is a “Company not Distributing Profit” registered under
the Company Act, 2063. The source of fund of this company comes from membership
fees and donation from the member groups and other outsiders. The company deals
with various goods other than those mentioned in schedule 1 to Value Added Tax Act,
2052. Annual turnover of this company ranges from Rs. 50 lakh to 75 lakh every
year.

Though profits earned by the company are not for the distribution amongst its
members, more than 50% of its profits are utilized towards activities related to the
Corporate Social Responsibilities (CSR) every year.

The members of the company when approached by IRD staffs for VAT registration
contended that the company is not required to need not be registered in VAT. As you
are the tax consultant of the entity you are required to comment this statement by the
company. (July 2015, 5 Marks)
Answer
Registration for Value Added Tax is compulsory when a person conducting taxable
transaction satisfies any of the following five conditions, regardless of whether the person
involves in charitable work or profit-making work:
Condition Type of Business
Compulsory Registration Transaction of bricks after its production, operation of
is required just because of business related to alcohol, wine, health club, disco
business, anywhere in theque, massage therapy, motor parts, electronic
Nepal software, customs agent, business of toy, business of
scrap, trekking, rafting, ultra- light flight, paragliding,
tourist vehicle, crusher, sand mine, slate or stone
Compulsory registration business of hardware, sanitary, furniture, fixture,
is required, if the business furnishing, automobiles, motor parts, electronics, marble,
are operated in educational and legal consultancy, accounting and
Municipality, auditing services, catering service, party palace,
metropolitan or sub- parking service, dry cleaners using machinery,
metropolitan area or restaurant with bar, ice cream industry, color lab,
areas prescribed by IRD boutique, Tailoring with shirting suiting materials,
uniform supplier for educational, health or other entities
Compulsory Registration If loan amount exceeds Rs. 10 Lakhs
due to obtaining business
loan from bank
Compulsory registration where tax officer, during the time of time of inspection,
due to holding of stock finds stock more than the limit prescribed by Inland
Revenue Department

© The Institute of Chartered Accountants of Nepal 25


CAP II Paper 7 Income Tax and VAT

Compulsory Registration For any other person than described above, the registration
due to turnover threshold is required when taxable turnover of a person exceeds as
follows during last 12 months period:
“Turnover” means higher a. Person dealing in goods- Rs. 50 Lakhs
of Sales or purchase b. Person dealing in services- Rs. 20 Lakhs
amount c. Person dealing mixed transactions of goods and
services- Rs. 20 Lakhs

Since, as given, the taxable turnover of goods of the company exceeds Rs. 50 lakhs in a
period of consecutive 12 months, the company is required to register itself for VAT
purpose.

23. Nepali Rice Mill Industries purchases the rice in the husk (Dhan) to produce the rice.
The Chitwan based mill sales its packed rice to the local markets. The company has
the following transactions from Bhadra 20X-71 to Chaitra 20X-71.
Sales Rs. 3,000,000
Dhan purchase Rs. 2,000,000
Machinery purchase Rs. 2,500,000 (Net of VAT)
Factory construction Rs. 5,500,000 (Payment to the Contractor)
State the relevant provisions for VAT implications on this case with references to the
VAT Act, 2052 and VAT Rules, 2053. (July 2015, 5 Marks)
Answer
As per Sec. 10 (3), a person dealing only in such transactions that are not subject to VAT
(i.e. transactions of items listed in Schedule 1 of the Act, is not required to be registered for
VAT. Therefore, M/s Nepali Rice Mill Industries is not required to be registered for VAT.

However, there are two VAT obligations in case of an unregistered person, unless Sec.
12Ka and Sec. 15 (3) is applicable (In this case, Sec. 12K and Sec. 15 (3) is not
applicable):
a. Collection of VAT from itself when it purchases service from a foreign person that is
not registered for VAT purpose in Nepal. The collected VAT shall be deposited within
25 days of the month when the amount has to be collected.

b. Collection of VAT from itself, if it constructs building, apartment, shopping complex or


similar other structures for commercial purpose costing more than 50 Lakhs, and the
contractor who constructs the commercial structure is not registered for VAT purpose.
This provision is applicable for factory building construction as well.
The provision is applicable only if the contractor is not registered for VAT purpose.

© The Institute of Chartered Accountants of Nepal 26


Compilation of Suggested Answers Value Added Tax

Through the analysis of question, condition (a) does not prevail.

As to the prevalence of condition (b), there is construction of factory (for Commercial


purpose- building to be used as fixed asset) that costs Rs. 5,500,000; which is more than 50
Lakhs. The question is not clear regarding the registration status of the contractor. If we
assume that the contractor is a VAT registered person who shall issue VAT invoice, M/s
Nepali Rice Mill must not collect VAT from itself.

However, if we assume that the contractor is not registered for VAT, M/s Nepali Rice Mill
Industries must collect VAT from itself on such costs where VAT is usually applicable and
not paid by M/s Nepali Rice Mill Industries.

24. What are conditions for compulsory registration under the Value Added Tax Act,
2052 and Rules 2053? (June 2014, 10 Marks& June 2010, 5 Marks)
(Dec 2007, 2 Marks, CA Inter)
Answer
Conditions for Compulsory Registration:
(a) In case of a person who conducts following transactions shall register for VAT
purpose:
Transaction of bricks after its production, operation of business related to alcohol,
wine, health club, disco theque, massage therapy, motor parts, electronic software,
customs agent, business of toy, business of scrap, trekking, rafting, ultra- light
flight, paragliding, tourist vehicle, crusher, sand mine, slate or stone

(b) In case of a person who conducts following transactions in metropolitan city, sub-
metropolitan city, municipality or other areas prescribed by Inland Revenue
Department shall register for VAT purpose:
business of hardware, sanitary, furniture, fixture, furnishing, automobiles, motor
parts, electronics, marble, educational and legal consultancy, accounting and
auditing services, catering service, party palace, parking service, dry cleaners
using machinery, restaurant with bar, ice cream industry, color lab, boutique,
Tailoring with shirting suiting materials, uniform supplier for educational, health or
other entities

(c) In case of a person that obtains business loan exceeding Rs. 10 lakhs from banks or
financial institutions.
(d) In case where tax officer, during the time of time of inspection, finds stock more than
the limit prescribed by Inland Revenue Department
(e) In case the taxable turnover of a person exceeds as follows during last 12 months
period:

© The Institute of Chartered Accountants of Nepal 27


CAP II Paper 7 Income Tax and VAT

i. Person dealing in goods- Rs. 50 Lakhs


ii. Person dealing in services- Rs. 20 Lakhs
iii. Person dealing mixed transactions of goods and services- Rs. 20 Lakhs

Turnover means the higher of sales or purchase.

25. National Mobile Hub without being registered under VAT Act is dealing in retail of
various branded mobile sets at NLIC, Kamalpokhari. During the F.Y. 20X-69/X-70,
the firm returned its file to the IRO on 20X-70.06.25 with the following Income
Statement:
Income Statement for F.Y. 20X-69/X-70
Amount
Expenses (Rs.) Income Amount (Rs.)
Opening Stock 175,870 Sales 1,787,654
Purchases 1,576,543 Closing Stock 545,387
Gross Profit C/D 580,628
2,333,041 2,333,041

Salary 91,000 Gross Profit B/D 580,628


Showroom Rent 108,000
Telephone Charges 18,000
Printing & Stationery 5,075
Local Conveyance 2,400
Miscellaneous Expenses 1,556
Net Profit Before Tax 354,597
580,628 580,628

The Tax Officer made an assessment on 20X-70.08.15 demanding a sum of Rs.


445,778 as VAT along with interest and penalty and also ordered to register the firm
under VAT. The proprietor of National Mobile Hub was surprised by the assessment
order and rushed to you. He explained you that though his firm not able to sale the
goods above the threshold limit fixed under VAT Act, 2052, the IRD assessed the
VAT. He feels that the VAT should not be applicable on him. Is the contention of the
proprietor of the firm is correct? (June 2014, 5 Marks)
Answer
Conditions for VAT registration, if any of the following conditions are satisfied:
Condition
Condition Type of Business satisfied or
not

© The Institute of Chartered Accountants of Nepal 28


Compilation of Suggested Answers Value Added Tax

Compulsory Registration Transaction of bricks after its production, Not


is required just because of operation of business related to alcohol, satisfied
business, anywhere in wine, health club, disco theque, massage
Nepal therapy, motor parts, electronic software,
customs agent, business of toy, business of
scrap, trekking, rafting, ultra- light flight,
paragliding, tourist vehicle, crusher, sand
mine, slate or stone
Compulsory registration business of hardware, sanitary, furniture, Not
is required, if the business fixture, furnishing, automobiles, motor satisfied
are operated in parts, electronics, marble, educational and
Municipality, legal consultancy, accounting and auditing
metropolitan or sub- services, catering service, party palace,
metropolitan area or parking service, dry cleaners using
areas prescribed by IRD machinery, restaurant with bar, ice cream
industry, color lab, boutique, Tailoring with
shirting suiting materials, uniform supplier
for educational, health or other entities
Compulsory Registration If loan amount exceeds Rs. 10 Lakhs Not
due to obtaining business satisfied
loan from bank
Compulsory registration where tax officer, during the time of time of N/A, as it is
due to holding of stock inspection, finds stock more than the limit decided by
prescribed by Inland Revenue Department Tax officer
Compulsory Registration For any other person than described above, Not
due to turnover threshold the registration is required when taxable satisfied, as
turnover of a person exceeds as follows during total sales
“Turnover” means higher last 12 months period: or
of Sales or purchase a. Person dealing in goods- Rs. 50 Lakhs purchase is
amount b. Person dealing in services- Rs. 20 Lakhs less than
c. Person dealing mixed transactions of Rs. 20
goods and services- Rs. 20 Lakhs Lakhs

Since, none of the above conditions are satisfied, M/s National Mobile Hub is not required
to get registered for VAT purpose, unless it desires so voluntarily.

The contention of the proprietor is correct.

© The Institute of Chartered Accountants of Nepal 29


CAP II Paper 7 Income Tax and VAT

26. State the conditions, procedures for and effect of cancellation of registration under
Value Added Tax Act/Rules. (Dec 2013, 10 Marks)
Answer
Conditions for Cancellation of Registration
a) In the case of a body corporate: where such body corporate is closed, sold or
transferred or ceases to exist due to any reason,
b) In the case of individual ownership; if the owner dies,
c) In the case of a partnership firm; if it is dissolved or where a partner dies,
d) In case the registered person ceases to be engaged in taxable transaction,
e) In case a registered person submits Zero Return continuously for twelve months or is a
taxpayer that does not submit Tax Return,
f) Where the taxpayer’s taxable transaction does not exceed Rs. 50 Lakhs in case of
transaction of goods and Rs. 20 lakhs in case of transaction of service or mixed
transaction of goods or services in last twelve months period,
g) Where there is registration in error.

Procedure for Cancellation of Registration


In case any of the conditions as discussed above arise and a person is required to cancel
VAT registration, the registered person or his/her legal representative/heir in his/her
absence shall submit an application in the format as prescribed by VAT Regulation within
30 days of happening of such conditions along with the VAT return and the amount of VAT
payable in front of tax officer.

The registered person shall submit all the relevant documents for tax assessment within 15
days of submission of VAT return.

The tax officer shall assess the of such taxpayer and take decision on the matter of
cancellation within 3 months of the submission of application. The matters of decision
whether to cancel or not shall also be notified to registered person within 3 months of the
application of cancellation.

The registered person is required to submit tax return for another three months until the
decision of tax officer regarding cancellation is served to him.

In case the tax officer fails to decide on matter of cancellation or does not serve
notification with regard to the cancellation of registration within 3 months of application,
the registered person is not required submit any tax return pertaining to periods after the
elapse of such three months.

Effect of Cancellation of Registration

© The Institute of Chartered Accountants of Nepal 30


Compilation of Suggested Answers Value Added Tax

The taxpayer shall assume the following items to be supplied at Market Value at the time
of cancellation of registration and shall assess and collect/levy tax:
a. Stock lying at the time of cancellation on which Input Tax Credit is already enjoyed;
and
b. Capital Items still in use, the VAT paid on purchase of which has already been claimed
as Input Tax Credit

27. What are the special provisions relating to Tax Payer Registration under VAT Act,
2052? (Dec 2010, 5 Marks, CA Inter)
Answer
Temporary Registration as organizer or Participant as exhibitor of Fair, Exhibition, etc.
[sec. 10Ka]
Conditions for Temporary Registration of Unregistered Person
The following persons shall get temporary registration:
(a) Any unregistered person desiring to engage in any short-term taxable transactions of
goods or services at fair, show, demonstration, display, exhibition etc., or
(b) The unregistered organizer of such event

Stock Transfer Facility for Existing Registered Person


The person who is already registered and is participating the fair, exhibition, etc. as
exhibitor can transfer goods for transaction to the place of exhibition or fair.

Procedures for Temporary Registration (Rule 7Ka)


Application to be filed along with recommendation from Organizer [Rule 7Ka (1) & Rule
7Ka (2)]
Any unregistered person desiring to engage in any short-term taxable transactions of
goods or services at fair, show, demonstration, display, exhibition etc., shall make an
application along with the following:
(a) the recommendation from the organizer in the concerned Tax Office or Taxpayer’s
Service Office, and
(b) Proof of deposit of amount equal to 2% of estimated revenue from the event

© The Institute of Chartered Accountants of Nepal 31


CAP II Paper 7 Income Tax and VAT

Issuance of Temporary Registration Certificate [Rule 7Ka (3)]


The tax officer at concerned Inland Revenue Office or Taxpayers Service Office shall issue
a temporary registration certificate once the application for registration along with deposit
is received.

Special Provision for Registration of Joint Venture


Where two or more persons form a Joint Venture dealing in taxable transaction for a
specific period of time, such co-venturers shall file an application for the purpose of
registration of Joint Venture at such Tax Office where one of the co-venturer is registered.

The Tax Officer shall provide a Registration Certificate in the format as prescribed where
an application is filed for registration.

Joint Venture registered as above shall cancel its registration upon the completion of
period specified in the agreement forming such Joint Venture.

The co-venturers are jointly and severally responsible to discharge tax obligation of Joint
ventures in which they are involved.

28. Binod & Company, a proprietor engaged in the business of selling battery operated
tempo, its chassis and battery. Its annual turnover of the business was estimated of
Rs. 10 million. Binod & Company not registered with VAT. The advisor of the
company suggested to get registered with VAT or ready to pay penalty as per VAT
Act. Assuming you are the advisor of Binod & Company, what would be your advice
to the company?
Would your answer be different if Binod & Company is engaged in the business of
solar energy equipments and operate under an umbrella of Alternative Resources
Promotion Centre? (Dec 2009, 5 Marks)
Answer
Conditions for VAT registration:
A person must register for VAT, if any of the following conditions is satisfied:
Condition Type of Business
Compulsory Registration Transaction of bricks after its production, operation of
is required just because of business related to alcohol, wine, health club, disco
business, anywhere in theque, massage therapy, motor parts, electronic
Nepal software, customs agent, business of toy, business of
scrap, trekking, rafting, ultra- light flight, paragliding,
tourist vehicle, crusher, sand mine, slate or stone
Compulsory registration business of hardware, sanitary, furniture, fixture,
is required, if the business furnishing, automobiles, motor parts, electronics, marble,

© The Institute of Chartered Accountants of Nepal 32


Compilation of Suggested Answers Value Added Tax

are operated ineducational and legal consultancy, accounting and


Municipality, auditing services, catering service, party palace,
metropolitan or sub- parking service, dry cleaners using machinery,
metropolitan area or restaurant with bar, ice cream industry, color lab,
areas prescribed by IRD boutique, Tailoring with shirting suiting materials,
uniform supplier for educational, health or other entities
Compulsory Registration If loan amount exceeds Rs. 10 Lakhs
due to obtaining business
loan from bank
Compulsory registration where tax officer, during the time of time of inspection,
due to holding of stock finds stock more than the limit prescribed by Inland
Revenue Department
Compulsory Registration For any other person than described above, the registration
due to turnover threshold is required when taxable turnover of a person exceeds as
follows during last 12 months period:
“Turnover” means higher a. Person dealing in goods- Rs. 50 Lakhs
of Sales or purchase b. Person dealing in services- Rs. 20 Lakhs
amount c. Person dealing mixed transactions of goods and
services- Rs. 20 Lakhs

A person can register himself voluntarily with VAT even though his VAT applicable annual
turnover is less than Rs. 5 million.

A person dealing with the goods and services exclusively covered in Schedule 1 cannot
register for VAT [Section 10(3)].

In the given case, Binod & Co. is engaged in the business of selling battery operated
tempo, its chassis and battery and its estimated annual turnover is Rs. 10 million. In
battery operated tempo and chassis, VAT is not applicable as these items are covered in
Schedule 1 and only the battery is the VAT applicable item. If the turnover of battery is Rs.
5 million or more, then only Binod & Company is required to get registered with VAT. The
situation of compulsory registration as suggested by the advisor will arise only when the
turnover of battery will exceed Rs. 5 million.

If Binod & Company was engaged in the following business of solar energy equipment and
operates under an umbrella of Alternative Resources Promotion Centre, it is not required
to get registered with VAT as this item is covered by schedule I of VAT Act:
a. Equipment falling under the subheads 7410.21.00, 7607.19.00, 8403.10.00, 8403.90.00,
8412.80.00, 8419.19.00, 8419.31.00 and 8516.60.00 that can only be operated from
solar heat energy

© The Institute of Chartered Accountants of Nepal 33


CAP II Paper 7 Income Tax and VAT

b. Lithium Ion Battery under subhead 8507.60.00 with capacity less than 12 volt and
ampere that are used in solar tuki.

29. Mr. Shrestha, a small business proprietor not required to be registered to VAT, got
voluntarily registered on 1.4.20X-63. After two years, he now feels that he should not
have gone for registration as it is creating unnecessary burden on his part.
He doesn't know whether he can go for cancellation or not and the procedure he is
required to follow. He approaches you for the advice. Suggest him.
(June 2009, 7 Marks)
Answer
The person can apply for cancellation of registration any time after the registration. The
procedure for cancellation of registration is as follows:

In case any of the conditions as discussed above arise and a person is required to cancel
VAT registration, the registered person or his/her legal representative/heir in his/her
absence shall submit an application in the format as prescribed by VAT Regulation within
30 days of happening of such conditions along with the VAT return and the amount of VAT
payable in front of tax officer.

The registered person shall submit all the relevant documents for tax assessment within 15
days of submission of VAT return.

The tax officer shall assess the of such taxpayer and take decision on the matter of
cancellation within 3 months of the submission of application. The matters of decision
whether to cancel or not shall also be notified to registered person within 3 months of the
application of cancellation.

The registered person is required to submit tax return for another three months until the
decision of tax officer regarding cancellation is served to him.

In case the tax officer fails to decide on matter of cancellation or does not serve
notification with regard to the cancellation of registration within 3 months of application,
the registered person is not required submit any tax return pertaining to periods after the
elapse of such three months.

© The Institute of Chartered Accountants of Nepal 34


Compilation of Suggested Answers Value Added Tax

Taxable Value
30. Mr. Ram Prasai a Vat registered person purchased wood/timber from Gairigaun
Community Forest of Jhapa District for business purposes through auction. The
auction price of woods at the time of auction, release order and (cutting)/saw order
was Rs. 2,100,000, Rs. 2,200,000 and Rs. 2,300,000 respectively. The release order was
issued earlier than auction time and cutting order issued by Gairigaun Community
forest.
Required: (Dec 2019, 5 Marks)
a. Explain the applicability of VAT on sale of wood/timber as per the provision of
VAT Act, 2052?
Answer
Value added tax is applicable on sales of wood by community, private cultivated or
private forest when the wood buyer buys it for commercial purpose. Where there is
applicability of Value added tax, the time of supply is earliest of issuance of cutting
order, issuance of release order or auction (sales) of the wood.

The taxable value of the wood shall be deemed royalty (royalty applicable for the wood
if the wood were sold by national forest) or auction value (sales value is treated as
auction value), whichever is higher.

In case of a national forest, VAT is applicable on sales of wood regardless of purpose


of purchase by buyer. Taxable value and time of supply is as explained above.

b. Compute the liability of VAT amount if any under the provision of VAT Act,
2052?
Answer
To determine the amount of VAT, the taxable value shall be determined. Since taxable
value is higher of auction price (sales value) or deemed royalty, the amount of VAT
cannot be determined unless both the variables are given in any question.
In the given question, there are three auction prices and practically it is impossible to
have three auction prices for the auction of same wood, as the auction price is always
the same. As the royalty amount is missing, VAT amount cannot be determined in this
case.

31. Sussex (Pvt) Ltd, Kathmandu imports goods and directly supplies to the retailers.
From the following information, compute the amount of VAT on sales.
(June 2018, 5 Marks)

Total cost of goods declared by importer Rs. 350,000.00


Valuation for cost of goods by Custom Officer Rs. 400,000.00

© The Institute of Chartered Accountants of Nepal 35


CAP II Paper 7 Income Tax and VAT

Freight & Insurance up to custom point Rs. 17,000.00


Freight from custom point to warehouse Rs. 20,000.00
Freight from warehouse to the retailers Rs. 8,000.00

Import Duty @ 10 %, excise duty @ 5% applicable only on import. Gross profit 10 %


Answer
Calculation of Customs Duty and Excise duty to be paid at the time of import
Cost Component Notes
Amount
When value is revised by Customs
Valuation for Cost of Goods by Customs
400,000 officer, the revised value is considered,
Officer
and not the declared value
Add: Freight and insurance up to Customs 17,000
Transaction Value for Customs purpose 417,000
Customs duty @10% of valuation 41,700
Taxable Value for Excise Duty 458,700
Excise duty @5% 22,935

Note:
Gross profit is usually expressed as a percentage of sales, so, 10% gross profit is deemed to be as
percentage of sales
Determination of Selling price and VAT on Sales
Particulars Notes
Amount
Total Cost of Goods declared by Importer 350,000
Add: Freight and insurance up to Customs 17,000
Add: Freight from Customs point to
20,000
warehouse
This cost is not direct cost, therefore, is
Freight from warehouse to retailers - not considered in determining the profit
margin
Customs duty 41,700
Excise Duty 22,935
Total Cost of Purchase of goods 451,635
Cost is 90% of Sales, i.e. Profit= Cost
Add: Profit Margin (10% of Sales) 50,182
divided by 0.9 multiplied by 0.1
Sales 501,817
VAT @13% on sales 65,236

32. Recondition House Baneshwor dealing in used goods made following Purchase &
sales during Kartik 20X-73:

© The Institute of Chartered Accountants of Nepal 36


Compilation of Suggested Answers Value Added Tax

Item Purchase with VAT (Rs.) Sales without VAT (Rs.)


Sofa 14000 21000
Freeze 32000 43000
Wooden Furniture 45000 38000

Calculate the amount of VAT payable by recondition house. (June 2017, 5 Marks)
Answer

As per Rule 33, the taxable value in case of dealer of second hand goods is the profit on
sales of such goods. Since, input credit is not available on purchase of goods, VAT paid
on purchase is treated as cost while determining profit. If there is loss, there will not be
any taxable value resulting into zero VAT.

Wooden
Particulars Sofa Freeze Furniture
Sales without VAT 21,000 43,000 38,000
Purchase with VAT 14,000 32,000 45,000
Difference between Sales without VAT and Purchase with
VAT 7,000 11,000 (7,000)
Taxable Value (=Profit) 7,000 11,000 -
VAT @13% 910 1,430 -

33. A few months few years ago, due to the blockade in Indo-Nepal border that lasted for
quite a few months, almost all imported goods including cooking fuels were
obstructed. To avoid the shortage of cooking fuel, the User Forest Group (UFG)
decided to cut down community forest trees for timbers to be used as cooking fuel.
The UFG accordingly sold out such timber in the market to be distributed amongst
the households. The sold out timbers fetched Rs. 300,000 meant ultimately to be
distributed to the user community group. Had those timbers been sold out to the state
owned forest, the forest offices would have paid Rs. 400,000 as royalty charges.

Compute the amount of VAT in case: (June 2016, 5 Marks)


a) Timbers are sold through auctions and sale proceeds are distributed amongst the
UFG.
b) Timbers are distributed amongst the UFG for their own domestic usages.

© The Institute of Chartered Accountants of Nepal 37


CAP II Paper 7 Income Tax and VAT

Answer

As per Sec. 12Ka, where a national forest sales wood, VAT shall be levied at such time,
that is earlier of issuance of cutting order, release order or auction. The taxable value shall
be higher of auction value or royalty fixed for such wood.

Where a community forest, private cultivated forest or private forest sells wood for
commercial use by buyer, taxable value shall be higher of sales value (deemed auction
value) or royalty fixed for such wood. Tax shall be collected at the time that is earlier of
issuance of cutting order, release order or sales (deemed auction)

(a) In the given case, timbers are sold for the domestic purpose of buyer. For sales of wood
from community forest, private cultivated forest or private forest to become taxable, the
buyer must purchase it for commercial use. Therefore, VAT is not applicable on sales
wood in the given circumstance.
(b) Where wood are distributed for free, there is commercial transaction and buyer uses it
for domestic purpose, therefore, VAT is not applicable.

34. Discuss the amount to be included in Taxable Value as per section 12 of Value Added
Tax Act, 2052. (Dec 2014, 5 Marks)
Or
How taxable value is determined to levy value Added Tax under Section 12 of VAT
Act? (Dec 2007, 5 Marks, CA Inter)

Answer
General Provision of Taxable Value
The taxable value of goods or services shall be the cash consideration received or
receivable by the supplier (i.e. value of goods or services so received) for the goods or
services where the consideration is payable in cash, after deducting trade discount and
trade rebate.
The following amounts are also included while computing taxable value:
• Transportation cost borne by the supplier at the time of transaction and distribution
expenses and the amount of profit,
• Excise duty, ownership tax and other tax amount except the VAT to be levied under
VAT Act.
For this purpose, “Other Tax Amount” means duty, fees and taxes levied prescribed by
annual Finance Bill.

Specific Provision:
In the following cases, the taxable value shall be as follows:
(a) Goods exchanged under barter system- market value of goods exchanged

© The Institute of Chartered Accountants of Nepal 38


Compilation of Suggested Answers Value Added Tax

(b) In case of import of goods- landed cost up-to border (cost paid to vendor, transit cost
or other cost) plus duty charged by the Customs Officer except VAT
(c) In case of significant under-invoicing- Market value
(d) In case of sales through obtaining part consideration- Market value
(e) Taxable value of wood of national forest- higher of auction price or royalty amount
(f) Taxable value of wood of community forest, private cultivated forest or private forest-
higher of auction price (sales value) or royalty amount (deemed) where the buyer buys
wood for commercial purpose
(g) In case of dealer of used goods- profit, i.e. difference between selling price of goods
and cost of such goods
(h) In case of notified goods sold to unregistered person- trade discount cannot be
deducted while determining taxable value
(i) At the event of cancellation of registration- the stock and capital items on which input
tax credit was already claimed and remaining at the time of cancellation of
registration- deemed to be supplied at market value
(j) Discontinuance of use of such capital goods, on which input tax credit was claimed, for
taxable transaction- market value
(k) In case of shortage of stock at the time of inspection by tax officer- shortage is deemed
to be supplied at market value

35. Apar International Pvt. Ltd. is a Recondition House involved in Purchase/Sale of


Used Vehicle, Motorcycle, TV and Refrigerator. The company purchases the used/old
Vehicle, Motorcycle, TV and Refrigerator and Sale those goods to the consumer by
repairing and improving the same. Company purchased from Mr. Ram residing in
Satungal at Rs. 20 lakhs and after repairing the same sold to Ms. Sunita at Rs. 25
lakhs Apar Ltd. seeks your advise as how much VAT shall he collect on such sales
amount. (Dec 2014, 5 Marks)
Answer
Apar International Pvt. Ltd. is dealer of used goods, as such, the taxable value is difference
between sales value and cost of such goods.

In the given case, Apar International shall collect VAT on Rs. 5 Lakhs @ 13%, i.e. Rs.
65,000; and shall be advised accordingly.

36. The Z Pvt. Ltd. has booked following expenses of imported electronic goods in the
ledger.
(June 2013, 8 Marks)
Cost of material as per invoice Rs. 200,000
Bank charges of L/C (Letter of Credit) Rs. 15,000
Insurance Rs. 5,000

© The Institute of Chartered Accountants of Nepal 39


CAP II Paper 7 Income Tax and VAT

Calcutta port clearing expense Rs. 20,000


Freight up to custom point Rs. 14,000
Fright from custom point to Kathmandu Rs. 7,000
Other tax paid at custom including local development tax Rs. 2,500
Nepal custom expenses including 1 % custom duty Rs. 30,000
Discount receivable Rs. 15,000

Further information:
Discount amount has not been deducted in invoices. The above cost is the gross
purchase.
a) Explain the taxable value in case of imported goods with reference to the
provisions of the Value Added Tax Act, 2052.
b) Compute the Value Added Tax payable at custom point as per above information.
c) Assume that the company has 20 % gross profit. What would be its sales price and
VAT amount?
Answer
(a) Taxable Value in case of Import
Taxable value for the purpose of import shall be landed cost up-to border (cost paid to
vendor, transit cost or other cost) plus duty charged by the Customs Officer except
VAT.

(b) Value Added Tax Payable at Customs Frontier


Particulars Amount Notes
Cost of material as per invoice 200,000
Bank charges of L/C (Letter of Credit) - It is cost incurred inside Nepal
Assumed to be incurred outside
Insurance 5,000 Nepal
Calcutta port clearing expense 20,000
Freight up to custom point 14,000
Fright from custom point to Kathmandu - It is cost incurred inside Nepal
Transaction Value for Customs Purpose 239,000
Customs duty @ 1% 2,390 Given in question
Other tax paid at customs including local
development tax 2,500 Part of taxable value for VAT
Taxable Value for VAT at Customs Frontier 243,890
VAT recoverable by Customs Officer @13% 31,706

© The Institute of Chartered Accountants of Nepal 40


Compilation of Suggested Answers Value Added Tax

(c) Value Added Tax on Sales


Note: Gross profit is usually expressed in terms of percentage of sales, unless otherwise stated
Gross profit is computed by deducting direct costs from sales. Therefore, only the direct costs
are considered in determining selling price
Particulars Amount Notes
Cost of material as per invoice 200,000 Part of direct cost
Bank charges of L/C (Letter of Credit) 15,000 Part of direct cost
Insurance 5,000 Part of direct cost
Calcutta port clearing expense 20,000 Part of direct cost
Freight up to custom point 14,000 Part of direct cost
Fright from custom point to Kathmandu 7,000 Part of direct cost
Other tax paid at custom including local
development tax 2,500 Part of direct cost
Customs duty 2,390 Part of direct cost
Part of direct cost, difference
Nepal custom expenses excluding 1 % custom between 30,000 and customs
duty 27,610 duty
It is cash discount and is not
considered in determing gross
Discount Receivable - profit
Cost of Materials including all direct costs 293,500
= cost divided by 0.8 multiplied
Gross profit @20% of Sales 73,375 by 0.2
Selling price (Taxable Value) 366,875
VAT @13% 47,694

37. ABC Ltd., a VAT registered person, has exported Noodles to a party based in China.
In exchange, ABC Ltd. has received Garments from the Chinese Party. How Taxable
Value of the Noodles is determined in this case as per the provision of the VAT Act,
2052. (Dec 2012, CA Inter, 2 Marks)
Answer
Taxable Value in case of goods supplied under barter system is market value of goods so
exchanged. As such, the market value of noodles or garments shall be the taxable value of
noodles.

38. Royalty for the national wood has been fixed at Rs. 600 per cu. feet. Wood from the
private forest is sold for Rs. 500 per cu. feet. How the taxable value is determined in
this case for the wood of private forest as per the provision of VAT Act? How the
taxable value is determined in case of Wood of national forest?
(Dec 2012, CA Inter, 3 Marks)

Answer

© The Institute of Chartered Accountants of Nepal 41


CAP II Paper 7 Income Tax and VAT

Taxable value of wood bought by buyer for commercial purpose from community forest,
private cultivated forest or private forest shall be higher of follows:
(a) Royalty, if the wood were sold from national forest (i.e. Rs. 600 per cu. Ft.), or
(b) Auction value (i.e. selling price) – i.e. Rs. 500 per cu. Ft.

In the given case, assuming the buyer buys the wood from private forest for commercial
purpose; taxable value is higher of above, i.e. Rs. 600 per cu. Ft.

In case of wood of national forest, taxable value shall be higher of auction price or royalty
value of wood.

39. Sharma & co, a registered person dealing in second hand vehicles has purchased a
second-hand motorcycle for NPR 10,000 plus chargeable VAT on it. The following
expenses are incurred for repair of the motorcycle to make it saleable:
Particulars Amount (NPR) VAT (NPR)
Spare Parts 4,000 520
Labor Charges 2,000 0

The person sells the motorcycle for NPR 20,000. Calculate the taxable value of the
motorcycle as per VAT Act, 2052. (Dec 2011, 5 Marks)
Answer
As per Rule 33, the taxable value in case of dealer of second hand goods are the profit on
sales of such goods. Since, input credit is not available on purchase of goods, VAT paid on
purchase is treated as cost while determining profit. If there is loss, there will not be any
taxable value resulting into zero VAT.

Particulars Details Amount


Sales without VAT 20,000
Purchase with VAT 17,820
Purchase cost with VAT 11,300
Spare parts with VAT 4,520
Labor Charges 2,000
Difference between Sales without VAT and Purchase with
VAT 2,180
Taxable Value (=Profit) 2,180
VAT @13% 283

40. M/s XYZ Drinkers Ltd. has sold a kind of drink to non VAT registered party for
Rs.10,000. As per the retail price published by the same company under the direction

© The Institute of Chartered Accountants of Nepal 42


Compilation of Suggested Answers Value Added Tax

of IRD, the retail price is Rs. 12,000. The company has collected VAT from the party
on the amount of Rs.10,000 as it says it has given trade discount to the party and the
trade discount can be deducted to arrive at the transaction value. But the assessing
officer insists to collect VAT on the published price. As an expert, give your opinion
on this. (June 2011, 5 Marks)
Answer
As per Sec. 14 (6) of Value Added Tax Act, 2052; where Inland Revenue Department
requires to publish retail price for the purpose of sales of any goods, trade discount shall
not be deductible while determining taxable value in case when such goods are sold to
unregistered person.

In the given case, there is IRD’s notification to publish retail price to sell the kind of
drinks, as such, the taxable value shall be the retail price published by
manufacturer/importer while selling such goods to unregistered person.

Therefore, the contention of assessing officer is correct, and the taxable value is Rs.
12,000.

41. How is the taxable value for second hand or used goods computed? Similarly how the
taxable value for wood of national forest, private and community forest is computed?
(June 2011, 5 Marks)
Answer
For the dealer of used goods, the taxable value is difference between sales value and cost
of such goods, i.e. Gross profit amount.

In case of wood of national forest, taxable value shall be higher of auction price or royalty
value of wood.

Taxable value of wood bought by buyer for commercial purpose from community forest,
private cultivated forest or private forest shall be higher of follows:
(a) Royalty, if the wood were sold from national forest, or
(b) Auction value (i.e. selling price)

© The Institute of Chartered Accountants of Nepal 43


CAP II Paper 7 Income Tax and VAT

42. Mention the provisions regarding collection of tax by custom authorities


(June 2010, 5 Marks)
Answer
The Taxable Value of imported goods shall be determined by adding Customs Duty,
countervailing duties, and any other taxes to levied at the time of import otherwise than
VAT on the Importation Value determined by including transportation costs, insurance,
freight, commissions of agents and other persons.

The Customs Officer is responsible to collect VAT except as otherwise specified by


Ministry of Finance in case of import of goods.

The Customs officer may exercise right pursuant to VAT Act and Customs Act for the
collection of VAT on import of goods.

The applicable VAT rate on import of goods shall be equal to the rate applicable for goods
or services supplied within the state of Nepal, which is 13%.

43. Elaborate the provisions relating the market value and its determination
(June 2010, 5 Marks)
Answer
As per Sec. 13 of Value Added Tax Act, 2052; Market value of supplied goods or services
shall be determined on the basis of the fair value of consideration received or receivable in
the transaction between unrelated independent parties at arm’s length under similar
circumstances as to characteristics, quality, quantity of materials, and any other relevant
factors.

As per Rule 22 of Value Added Tax Regulations, 2053; Tax Officer has right to fix the
market price based on the transaction of similar goods. In case a Tax Officer cannot
ascertain market price, the Director General has the right to prescribe the method on the
basis of price of similar goods transacted by different suppliers.

As per Value Added Tax Directive issued by IRD, tax officer may take the following bases
in determination of market value:
(a) Tax officer shall collect taxable value determined by other taxpayers dealing in similar
goods
(b) The officer shall consider the similarity in time, place, date and area while collecting
data.
(c) Comparison shall be made between identical and similar goods. Where there is policy
of discount by the taxpayer, the price before such discount shall be compared.
(d) The seller and buyer shall be independent of each other.

© The Institute of Chartered Accountants of Nepal 44


Compilation of Suggested Answers Value Added Tax

(e) Taxable value shall be determined independently.


(f) The consideration against the supply of compared goods shall be identical or similar.
(g) The below market price sales shall be focused on sales promotion.

44. A company dealing with computer parts sold the goods worth Rs. 25 lacs in the month
of Baisakh 20X-67. It has imported the computer parts on which input VAT has been
paid as per the cost fixed/determined by Custom Offices on CIF basis with details
below:
Cost declared by importer Rs. 200,000
Revised cost fixed by Custom Officer Rs. 250,000
(based on prevailing international price)
Transportation Rs. 25,000
Insurance Rs. 5,000
Import Duty @ 6%
Besides, the credit balances up to Chaitra 20X-66 is Rs. 20,500. In addition, it has
omitted to adjust input tax credit on the purchase of Rs. 30,000 pertaining to the
month of Chaitra 20X-66.

Compute the Input and Output VAT. (June 2010, 10 Marks)

Answer
VAT paid at the time of import

Taxable value in case of import:


All costs incurred on importing goods up to border plus customs duty plus excise duty
levied at customs frontier plus any other indirect taxes levied at Customs Frontier,
except VAT. If Customs officer revises costs up to border, the revised cost shall be
considered for customs valuation and determination of taxable value at the time of
import.

Particulars Amount Note


Since, customs office
Cost declared by importer - revised cost, so revised
cost has to be considered
Revised cost fixed by Custom Officer 250,000
Assumed to be incurred
Transportation 25,000
outside Nepal
Assumed to be incurred
Insurance 5,000
outside Nepal
Transaction Value for Customs purpose 280,000
Import Duty @ 6% 16,800
Other indirect taxes, except VAT, levied at -

© The Institute of Chartered Accountants of Nepal 45


CAP II Paper 7 Income Tax and VAT

Customs Frontier
Taxable Value 296,800
VAT @13% 38,584

Calculation of Output VAT


Particulars Amount
Taxable Sales 2,500,000
VAT @13% 325,000

Total eligible VAT credit for the month of Baisakh


Particulars Amount Note
VAT paid on Import 38,584
Carried forward excess VAT of Chaitra 20,500
VAT credit on purchase
Purchase of Chaitra on which VAT credit was
3,900 can be claimed within
erroneously not claimed
one year of purchase
Eligible VAT Credit 62,984

VAT Payable/(Receivable) for the month of Baisakh


Particulars Amount
Output VAT 325,000
Eligible Input VAT credit 62,984
VAT payable/(Receivable) 262,016

45. How much value added tax is to be collected in the following cases?
(June 2010, 10 Marks, CA Inter)
a) Sale of wood from community forest at Rs. 90,000, governmental royalty if would
be sold the same from national forest would be Rs. 100,000.
Answer
If the wood is bought by buyer for commercial use, taxable value shall be higher of
sales price (deemed auction value) or royalty applicable when the wood was sold by
national forest, therefore, taxable value would be Rs. 100,000 (higher of Rs. 90
thousand and Rs. 1 Lakhs). VAT to be collected by Community Forest would be Rs.
13,000 (13% of taxable value)

If the wood is not bought by buyer for commercial use, there would not be any
applicability of VAT.

b) Export of rice costing Rs. 80,000 at Rs. 100,000.

© The Institute of Chartered Accountants of Nepal 46


Compilation of Suggested Answers Value Added Tax

Answer
Rice is listed in Schedule 1 of Value Added Tax Act, 2052. Therefore, transaction of
rice is exempt from VAT. There will not be applicability of VAT

c) Telephone bill was Rs. 10,000 and delay penalty is Rs. 2,500 levied on settlement.
Answer
Fees, Penalty, etc. of Telecommunication companies does not form part of taxable
value (M/s Saroj Shrestha Vs. Nepal Telecommunication Corporation, Writ Number
3719 of 2056 B.S.)
Therefore, taxable value is Rs. 10,000 and VAT is Rs. 1,300.

d) Second hand goods seller sales Rs. 100,000 goods, cost in total was Rs. 80,000.
Answer
As per Rule 33, the taxable value in case of dealer of second hand goods is the profit on
sales of such goods. Since, input credit is not available on purchase of goods, VAT paid
on purchase is treated as cost while determining profit. If there is loss, there will not be
any taxable value resulting into zero VAT.

Therefore, taxable value is Rs. 20,000 and VAT is Rs. 2,600.

e) Sale of furniture by a trader at Rs 50,000, dealer is not registered in VAT.


Answer
VAT is collected by registered person, except in the circumstances mentioned in Sec. 8
(2), 8 (3), 12Ka and 15 (3). In the given scenario, such circumstances do not prevail. As
such, the furniture trader cannot collect VAT.

46. From the following information, compute the amount of VAT. (Dec 2009, 5 Marks)
Total cost of goods declared by importer Rs. 2,50,000
Revised cost fixed by Custom Officers
(based on prevailing International Price) Rs. 3,00,000
Freight Rs. 10,000
Insurance Rs. 2,000
Import Duty @ 10 %
Answer

Taxable value in case of import:


All costs incurred on importing goods up to border plus customs duty plus excise duty
levied at customs frontier plus any other indirect taxes levied at Customs Frontier, except
VAT. If Customs officer revises costs up to border, the revised cost shall be considered for
customs valuation and determination of taxable value at the time of import.

© The Institute of Chartered Accountants of Nepal 47


CAP II Paper 7 Income Tax and VAT

Particulars Amount Note


Since, customs office revised
Cost declared by importer - cost, so revised cost has to be
considered
Revised cost fixed by Custom Officer 300,000
Assumed to be incurred
Transportation 10,000
outside Nepal
Assumed to be incurred
Insurance 2,000
outside Nepal
Transaction Value for Customs purpose 312,000
Import Duty @ 10% 31,200
Other indirect taxes, except VAT, levied at
-
Customs Frontier
Taxable Value 343,200
VAT @13% 44,616

47. Mr. Ram, a furniture supplier produced some woods from a community forest
through auction and has not paid any Royalty or VAT on the deal. IRD Officer
assessed the transaction and ordered to Mr. Ram to pay VAT on the auctioned price.
Mr. Ram disputed the order and argued that since there was no Royalty amount it
cannot be required to pay VAT. Mr. Ram appoints you as a professional advisor.
Please advice Mr. Ram on the applicability of VAT and whether he is required to pay
the amount on the agreed price. (Dec 2009, 5 Marks)
Answer
As per Sec. 12Ka, where a national forest sales wood, VAT shall be levied at such time,
that is earlier of issuance of cutting order, release order or auction. The taxable value shall
be higher of auction value or royalty fixed for such wood.

Where a community forest, private cultivated forest or private forest sells wood for
commercial use by buyer, taxable value shall be higher of sales value (deemed auction
value) or royalty fixed for such wood. Tax shall be collected at the time that is earlier of
issuance of cutting order, release order or sales (deemed auction).
Since, Mr. Ram has purchased the wood for commercial use; VAT is applicable on the
auction price or royalty applicable on such wood for national forest, whichever is higher.

The provision of Sec. 12Ka requires the seller (i.e. community forest) to collect such VAT,
however, IRD has applied the provision in reverse charging mechanism. That means, Tax
officer will assess and collect VAT from the buyer of wood.

© The Institute of Chartered Accountants of Nepal 48


Compilation of Suggested Answers Value Added Tax

As such, the assessment of tax officer (as per IRD’s circular) is correct. However, Mr. Ram
can still challenge the decision of tax officer, since tax has to be assessed and collected
from Community Forest, not from Ram.

48. Determine Taxable Value in the following cases: (June 2005, 10 Marks, CA Inter)
a) Ex-factory price Rs.78, 000. Freight and insurance up to the buyer’s place, 2000.
Trade Discount 7%, special Discount 10%.
Answer
Ex-Factory Price 78,000
Freight & Insurance 2,000
Total 80,000
Less: Trade Discount @7% 5,600
Less: Special discount @10% (before issuance of invoice) 8,000
Taxable value 66,400

b) Ex-factory price Rs.50 000 Quantity Rebate 5%, Trade Discount 10%
Answer
Ex-factory Price 50,000
Less: Trade Discount @10% 5,000
Less: Quantity Rebate at 5% 2,500
Taxable Value 42,500

c) Ex-factory price Rs.18,000 Quantity Rebate on a previous consignment billed and


delivered in Kartik, 20X-61 Rs. 2,000
Answer
Ex-Factory Price 18,000

d) Goods imported for Rs.100, 000 by paying VAT at 13% at custom point. The
quality deteriorated and could not be used in production. Hence it is to be sold for
Rs. 10,000 as scrap
Answer
The taxable value can be taken as Rs. 10,000; if adequate documentation is maintained.

e) Ex-factory price Rs.25, 000, Trade Discount 10%, Cash Discount 2% commission
5%
Answer
Ex- Factory Price 25, 000
Less: Trade Discount @ 10% 2,500
Taxable value 22, 500

© The Institute of Chartered Accountants of Nepal 49


CAP II Paper 7 Income Tax and VAT

Cash discount provided before the issuance of invoice is deductible while calculating
taxable value.

Commission payable as per contractual obligation will not be reduced from the invoice
and same is available deduction separately.

© The Institute of Chartered Accountants of Nepal 50


Compilation of Suggested Answers Value Added Tax

Reverse Charging
49. Saurav Publication prints the various books and sells to the wholesaler. The Company
itself has constructed a building for its Publication House. The cost of construction
was as follows:
Labour charges Rs. 30,00,000
All Material (Cement, rod etc) purchase cost Rs. 550,000 inclusive VAT.
State the VAT Implications on this case, if applicable with references to VAT Act,
2052 and rules, 2053. (Dec 2014, 5 Marks)
Answer
As per Sec. 8 (3), Saurav Publication shall collection of VAT from itself, if it constructs
building, apartment, shopping complex or similar other structures for commercial purpose
costing more than 50 Lakhs, and the contractor who constructs the commercial structure is
not registered for VAT purpose. This provision is applicable for factory building
construction as well.
Since, the total construction cost does not exceed Rs. 5,000,000; therefore, this provision is
not applicable.

50. "Supertech Infosys Pvt. Ltd." deals in computer & related IT technology business. In
the course of its business, it received service from "Key Technology Corp., Japan”,
against which, Technology Corp. raised bill of Rs. 21.60 lakhs. Management of
Supertech is confused as to whether VAT shall be charged on same.

Advise him as regards to VAT applicable on same and net payment to be send.
Specify the provision of VAT Act, 2052 in this regard. (Ignore other taxes, if
applicable). (Dec 2013, 5 Marks)
Answer
As per Sec. 8 (2) of Value Added Tax Act, 2052 (as amended); a person (whether registered
or not) obtaining service from party outside Nepal that is not registered for VAT purpose in
Nepal, shall collect VAT from itself on taxable value determined applying rules u/s 12 of
the Act at the time, which is earlier of payment for service or receipt of service.

In the given case, Supertech Infosys Pvt. Ltd. obtains service from Japanese company.
Japanese company is not registered for VAT purpose in Nepal. Cash consideration payable
by Supertech Infosys is Rs. 21.60 Lakhs, therefore, it shall collect VAT @13% on Rs. 21.60
lakhs and pay the VAT amount in Revenue account of Nepal Government of Nepal.

It shall pay Rs. 21.60 Lakhs to Japanese company. The VAT shall be collected at such time,
which is earlier of payment for service or obtaining service.

© The Institute of Chartered Accountants of Nepal 51


CAP II Paper 7 Income Tax and VAT

51. Mercantile Pvt. Ltd. is a company providing telecommunication services in Nepal. It


has purchased a consultancy service from a company in US for setting up and
expanding its operation in Karnali zone of Nepal. The company has paid USD 500,000
against such services in the month of Jestha 20X-69. Mercantile Pvt. Ltd. is of the
view that since payment is made to foreign company there is no implication of Value
Added Tax. State your view referring relevant provision of the Value Added Tax Act,
2052. (June 2013, 4 Marks)
Answer
As per Sec. 8 (2) of Value Added Tax Act, 2052 (as amended); a person (whether registered
or not) obtaining service from party outside Nepal that is not registered for VAT purpose in
Nepal, shall collect VAT from itself on taxable value determined applying rules u/s 12 of
the Act at the time, which is earlier of payment for service or receipt of service.

In the given case, Mercantile Pvt. Ltd has obtained service from a company in US. The
company in US is not registered for VAT purpose in Nepal. Therefore, Mercantile Pvt. Ltd.
has the obligation to collect VAT from itself on such service. The VAT shall be collected at
the time, which is earlier of payment date (Jestha 20X-69) or time of receipt of service
(question does not specify the time of receipt of service).

The VAT so collected, shall be paid in revenue account of Government of Nepal within 25
days of the end of month when the VAT was collected (as per Sec. 19).

Based on the above explanation, Mercantile Pvt. Ltd. shall collect VAT @13% on Nepalese
rupee equivalent of USD 500,000 (Conversion rate is buying rate of NRB on the date when
VAT is to be collected).

52. EFG Ltd. has given a contract to Garibdas construction, a party not registered in
VAT, for construction of a Shopping Complex built exclusively for commercial
purposes. The cost of the shopping complex is Rs.55 lacs. By mentioning the relevant
provision of the act and rules, comment on the taxability of the transaction.
(Dec 2012, 4 Marks)
Answer
As per Sec. 8 (3), any person (registered or not) in Nepal engaged in construction of
buildings, apartments, shopping malls or similar constructions for commercial purpose, the
cost of which is more than Rs. 5 millions shall collect VAT on the construction cost from
themselves and deposit it to Revenue Authority in case the construction work is not carried
out through a registered person.

In case of failure to levy tax as such, the tax shall be assessed and recovered from the
owner of such building or structure.

© The Institute of Chartered Accountants of Nepal 52


Compilation of Suggested Answers Value Added Tax

Definition of Commercial purpose


For the purpose of Sec. 8 (3), Commercial Purpose means construction of building,
apartment, shopping complex or similar other structures prescribed by IRD for the purpose
of sales or using such asset by accounting the asset as current or fixed asset.

Conclusion
In the given case, since, M/s EFG Ltd. has given a contract to unregistered contractor and
the value of work of construction of shopping complex for commercial purpose exceeds Rs.
50 Lakhs, M/s EFG Ltd. shall collect VAT on such costs of shopping complex on which
VAT is applicable and VAT is not paid already by the company.

© The Institute of Chartered Accountants of Nepal 53


CAP II Paper 7 Income Tax and VAT

Input Tax Credit and Computation of VAT Obligation


53. X Ltd. was incorporated and registered with VAT. During the month of Kartik 20X-
76 the company had following transaction:
NPR.
Purchases/Expenses:
Purchase of raw materials- 175,000 (for taxable transaction)
Electricity expenses 25,000.00
Cold drinks 12,000.00
Diesel 20,000.00
LP Gas 7,000.00
Printing and stationery 35,000.00
Services availed from UK 100,000.00
Sales
Taxable sales 500,000.00
Exempted sales 25,000.00
Export sales 10,000.00
All purchases, expenses and sales are exclusive of VAT.
The company had opening balance of VAT credit of NPR. 1,150,000

You are required to calculate VAT payable or VAT credit available for the month of
Kartik 20X-76. (Dec 2019, 5 Marks)
Answer
A. Output VAT (or VAT Collected on Sales)
Particulars VAT Note
Amount
Taxable Domestic Sales 500,000 65,000 Tax rate of 13%
Exempted Sales 10,000 - VAT is not applicable
Assumed to be that of taxable
Taxable Export Sales 25,000 -
items, taxed @0%
Total Output VAT for the
65,000
month

B. Eligible Input tax credit on purchases during the month


Eligible
Particulars VAT Amount Notes
Amount
for Credit
Full credit, directly related
Purchase of Raw Materials 175,000 22,750 22,750
to taxable sales
Electricity Expenses 25,000 - - Electricity is exempt item
No VAT credit, beverage
Cold drinks 12,000 1,560 -
item (rule 41)

© The Institute of Chartered Accountants of Nepal 54


Compilation of Suggested Answers Value Added Tax

No VAT credit, Consumption


Diesel 20,000 2,600 -
of petroleum (rule 41)
No VAT credit, Consumption
LP Gas 7,000 910 -
of petroleum (rule 41)
Proportionate VAT Credit, as
Printing and Stationery 35,000 4,550 4,465 the use is for both taxable
and non taxable output
Total Eligible VAT Credit 27,215

C. Input VAT Credit on VAT paid as per Sec. 8 (2), 12Ka and 15 (3)
Eligible
Particulars VAT Amount Notes
Amount
for Credit
Proportionate VAT Credit as
the input's direct relation
with any output cannot be
Services availed from UK 100,000 13,000 12,757 established
It is assumed that VAT is
already paid in respect of
receipt of this service
Total 12,757

D.Opening VAT credit (excess of Previous


month, eligible for set off) 1,150,000

E. Payable/ (Receivable) VAT [A-B-C-D]


(1,124,972)

54. Not Only For Profit Pvt. Ltd., Kathmandu imports luxury cars and track-laying
tractors and sells in Nepal. Following details pertain to Ashwin, 20X-75.
a) The showroom price of luxury car is Rs. 5 million. Two percent Dashain discount
was offered on sale of the cars. 10 cars were sold to VAT registered customers and
6 cars were sold to individual customers not registered in VAT.
b) The showroom price of Track-laying tractors is Rs. 2.5 million. Two percent
Dashain discount was offered on the tractors. 10 tractors were sold to individual
customers who were not registered for VAT.
c) The company offers additional 5% discount for prompt payment. 5 customers
purchasing luxury car and 6 customers purchasing tractor made prompt payment
and benefitted from the discount offer.

© The Institute of Chartered Accountants of Nepal 55


CAP II Paper 7 Income Tax and VAT

d) Import price of luxury cars was Indian Rupees (INR) 2 million per car. Total
transportation expenses upto custom point was INR 200,000 (paid to Indian
transporter) and total insurance premium Rs. 200,000 was paid to an insurance
company in Nepal. 16 cars were imported in the same month and sold as above.
e) Import price of per unit Track-laying tractors was INR 1.5 million. Total
transportation expenses up to custom point were INR 100,000 (paid to Indian
transporter) and total insurance premium Rs. 50,000 was paid to an insurance
company in Nepal. 10 tractors were imported in the same month and sold as
above.
f) During the month, Rs. 90,000 was spent on mobile telephone calls, of which 40%
relates to private calls.
g) On 30 Ashwin 20X-75, office equipment was purchased for Rs. 1,000,000.
h) Custom duty was Rs. 41,376,000 for the luxury cars in total.
i) The company also constructed a showroom in the month of Ashwin, 20X-75. The
total cost of construction was Rs. 6 million, which was constructed by a VAT
unregistered builder.
j) All of the figures are exclusive of VAT and use exchange rate @ Rs. 1.6015 per
INR.

Calculate the amount of VAT payable/receivable by the company for Ashwin 20X-75.
(Dec 2018, 10 Marks)
Answer
Calculation of VAT payable/receivable for the month ended Bhadra 20X-75
Particulars Amount Rs.
Output tax (WN 1) 10,192,000
Less: Input tax for imported cars (WN 2) (12,108,759)
Input tax credit on expenses (WN 3) (130,396)
Net VAT receivable (2,047,155)
Reverse charge (VAT) on construction (WN 4-
to be paid) 780,000

WN 1: Calculation of output tax


Particulars Cars sale Rs. Tractors sale Rs.
Showroom price 5,000,000 2,500,000
Dashain Discount (trade discount) 100,000 50,000
Sales price 4,900,000 2,450,000
Sales to VAT registered Customers (10 cars) 49,000,000 0.00

© The Institute of Chartered Accountants of Nepal 56


Compilation of Suggested Answers Value Added Tax

Sales to Unregistered Customers (6 cars&10


tractors) 29,400,000 24,500,000
Total 78,400,000 24,500,000
Proportionate sales ratio 76.19% 23.81%

Track-laying Tractor is VAT exempt as per schedule 1 of VAT Act, 2052.

WN 2: Calculation of VAT on Import at custom point for Luxury cars:


Particulars Amount
Qty
Import price (16*2,000,000*1.6015) 51,248,000

Transportation expenses IRs. 200,000 * 1.6015 320,300


Insurance premium 200,000
Customs duty 41,376,000
Total taxable value for VAT 93,144,300
VAT paid at custom point 12,108,759
Track-laying tractors are VAT exempt items so no need to calculate input VAT on import

WN 3: Calculation of input tax credit on expenses:


Particulars Amount Rs.
Full tax credit:
Insurance for Car Rs. 200,000 * 13% 26,000
Proportionate tax credit:
Mobile expenses Rs. 90,000*13%* 60%* 76.19% 5,349
Office Equipment Rs. 1,000,000 * 13%*76.19% 99,047
Total 130,396

WN 4: Reverse VAT on construction of structures for commercial use through VAT


Unregistered builders.
As per section 8(3), if a building, apartment or similar structures are constructed for
commercial purposes from VAT-unregistered builders, then VAT should be deposited as
if it was constructed from VAT-registered person.
Total cost of construction 6,000,000
VAT 780,000

© The Institute of Chartered Accountants of Nepal 57


CAP II Paper 7 Income Tax and VAT

55. LUC Group Nepal is an emerging business house established by group of clever
business minds. It carries diversified business activities under same permanent
account number and group motto is halo to hydro. Business activities of the group for
the month of Bhadra 20X-73 is as follows:
a) Account receivable from a client for contract completed, approved and invoiced
during the month Rs. 3,655,434.
b) Sales of cellular mobile phone during the month Rs. 1,255,660 of which amount
Rs. 456,360 was sold to non registered customers.
c) Fruits and vegetable sales during the month Rs. 5,34,600 of which Rs. 1,53,600 was
sales of packed garlic in stock since last 3 months.
d) Deep cycled lid acid battery sales Rs. 1,256,400 out of total Rs. 3,25,600 was sold to
Safa Tempo workshop with recommendation from Ministry of Population &
Environments.
e) Industrial machineries for bakery, confectionary & distillery falling under
Harmonized code 84.39.10.00, 8439.20.00 and 8439.30.00 sold during the month
Rs. 55,24,300
f) The group has following purchase/Import during the month exclusive of VAT,
where applicable
• Cement, Iron Nails, Bricks Rs. 21,22,456
• Cellular Mobile & its parts Rs. 16,56,400
• Battery Rs. 7,85,900
• Tower cranes for construction use Rs. 55,60,300
• Fruits and Vegetable Rs. 4,26,300
• Bakery, confectionary & distillery machineries (falling under Harmonized
code 8439.10.00, 8439.20.00 and 8439.30.00) for trading purpose Rs. 60,65,480
• Statutory & internal audit fees Rs. 4,75,000
• Computer for official use Rs. 3,25,400
• Stationeries Rs. 5,25,600
• Purchase of car for group president Rs. 40,00,000
g) During the same month internal auditor of the group submitted their report to the
management that reveals:
• Purchase book of VAT for the month of Shrawan 20X-72 was over casted by
Rs. 87,500.
• No credit claim for the VAT amount Rs. 56,980 paid to department of roads
for auction purchase of road roller during the month of Kartik 20X-72.
• No adjustment for batteries used for office purpose amounting costing Rs.
45,600 during the month of Jestha 20X-73.

© The Institute of Chartered Accountants of Nepal 58


Compilation of Suggested Answers Value Added Tax

Required: (June 2017, 8+2=10)

i) VAT statement for the month of Bhadra 20X-73 and VAT payable by the group
assuming there was no opening VAT credit.
Answer
I. Calculation of Output VAT

Taxable
Particulars Exempt Total
@13%
Construction Revenue billed
3,234,897 3,234,897
(3655434/1.13)
Sales of Cellular mobile phone 1,255,600 1,255,600
Fruits & Vegetable 153,600 381,000 534,600
Battery Sales 1,256,400 1,256,400
Industrial Machineries 5,524,300 5,524,300
Deemed sale of battery self use during
45,600 45,600
Jestha 20X-73 reported by internal auditor
Total 5,946,097 5,905,300 11,851,397
Tax Rate 13% -
Tax collection 772,993 - 772,993

Ratio of Taxable Sales and Non-taxable Sales

Total Transaction 11,851,397


Taxable Transaction 5,946,097 50.17%
Exempt Transaction 5,905,300 49.83%

II. Calculation of Eligible Input Tax Credit

Partial/
Particulars Full Proportio No Total
nate
Cement, iron, nail and bricks 2,122,456
Cellular mobile 1,656,400
Battery 785,900
Statutory & internal Audit
475,000
fees
Computer for office 325,400
Stationeries 525,600
Car Purchase (4000000*0.4) 1,600,000
Total 4,564,756 2,926,000 -
VAT paid on Purchase 593,418 380,380 -

© The Institute of Chartered Accountants of Nepal 59


CAP II Paper 7 Income Tax and VAT

Credit eligible VAT Ratio Full 50.17% -


Credit eligible VAT amount 593,418 190,845 - 784,263
Note:
Fruits and vegetables, machineries and tower crane are non VAT items listed in
Schedule 1. Therefore, VAT is not paid at the time of purchase

III. Calculation of VAT Payable/(Receivable)

VAT Collected on sales ("I" above) 772,993


VAT Paid on Purchase ("II" above) 784,263
VAT payable/(credit forward) (11,270)
Opening VAT Credit -
Adjusted VAT credit (Road Roller Purchased from DOR during
(56,980)
Kartik 20X-72)
Total VAT credit forwarded for Aswin 20X-73 (68,250)

ii) Discuss about VAT refundable, if any.


Answer
As per Sec. 24 of Value Added Tax Act, 2052, unless there is export sales exceeding
40% of total sales during any month, any excess receivable VAT (i.e. when input VAT
exceeds Output VAT) of a month shall be utilized to set off any payable VAT of next
four months. Any remaining excess receivable VAT after such action for consecutive
four months can be refunded by making an application by the concerned taxpayer
along with the VAT return of fourth month or any months thereafter.

In case of export sales exceeding 40 percent of total sales during any month, the
taxpayer may make an application to refund any excess receivable VAT of such month
along with the VAT return of the same month.

Conclusion
In the given case, there are no export sales. Therefore, VAT can be refunded only after
applying the excess to payable VAT of next four months and if there still remain excess
VAT.

© The Institute of Chartered Accountants of Nepal 60


Compilation of Suggested Answers Value Added Tax

56. ABC Ltd. was incorporated and registered with VAT on 2065. During the month of
Jestha 20X-73 the company had following transaction:
Purchases/Expenses:
Purchase of raw materials Rs. 150,000 (Exclusively used for taxable
transaction)
Electricity Expenses Rs. 125,000
Cold Drinks Rs. 25,000
Motorbike Rs. 250,000
Printing & Stationeries Rs. 50,000
Services availed from America Rs. 150,000
Sales:
Taxable sales -Rs. 500,000
Exempted Sales -Rs. 10,000
Export Sales -Rs. 100,000
All purchases, expenses and sales are exclusive of VAT.
The company had opening balance of VAT credit of Rs. 2,750,000 out of which
2,000,000 is credit prior to financial year 20X-68/X-69.
Required: (Dec 2016, 5+2=7)
a. Vat payable or vat credit available for the month of Jestha 20X-73 without setting
off opening vat credit. Write the due date of filling vat return?
Answer
VAT payable or VAT credit available for the month of Jestha 20X-73 without setting
off opening vat credit
Particulars Amount
A. VAT Collected on Sales (Refer "I" below) 65,000
B. Eligible VAT Credit (Refer "II" below) 57,861
C. VAT Credit on VAT paid under Reverse Charge Principle 19,180
(Refer "III" Below)
D. VAT Payable/(Receivable) [A-B-C] (12,041)

Working Notes
I. VAT Collected on Sales
Particulars VAT Note
Amount
Taxable Domestic Sales 500,000 65,000 Tax rate of 13%
Exempted Sales 25,000 - VAT is not applicable
Assumed to be that of taxable items,
Taxable Export Sales 100,000 -
taxed @0%
Total Output VAT for the month 65,000

© The Institute of Chartered Accountants of Nepal 61


CAP II Paper 7 Income Tax and VAT

Ratio of Taxable Sales to Total Sales


Taxable Sales 600,000
Non Taxable Sales 10,000
Total Sales 610,000
Percentage of Taxable Sales 98.36%

II. Eligible VAT Credit on Purchases during the month


Eligible
Amount
Particulars VAT Notes
Amount for
Credit
Full credit, directly
Purchase of Raw Materials 150,000 19,500 19,500 related to taxable
sales
Electricity is exempt
Electricity Expenses 125,000 - -
item
No VAT credit,
Cold drinks 25,000 3,250 - beverage item (rule
41)
Proportionate VAT
Credit, as the use is
Motorbike 250,000 32,500 31,967
for both taxable and
non taxable output
Proportionate VAT
Credit, as the use is
Printing and Stationery 50,000 6,500 6,393
for both taxable and
non taxable output
Total Eligible VAT Credit 57,861

III. VAT Credit on VAT paid under Reverse Charge Principle as per Sec . 8 (2)
Eligible
Amount
Particulars VAT Notes
Amount for
Credit
Proportionate VAT Credit as
the input's direct relation
with any output cannot be
Services availed
150,000 19,500 19,180 established
from America
It is assumed that VAT is
already paid in respect of
receipt of this service
Total 19,180
Note: X Ltd must have deposited value added tax of Rs. 19,500 in respect of services
availed from UK, in order for it to be able to claim credit of such VAT

© The Institute of Chartered Accountants of Nepal 62


Compilation of Suggested Answers Value Added Tax

Due date of filing VAT Return


VAT return must be filed within 25 days of end of tax period.

b. Is it wise to shut down the business of company based on the information provided
by the company?
Answer
It is not wise to shut down the business of company because the company has huge
balance of VAT credit which is not eligible to get refund. VAT refund is permissible
under section 24 of Value Added Tax Act, 2052 to registered person. The refund
application shall be filed within three years of the end of tax period. The company had
huge balance of VAT credit generated before three years; hence, advised to continue
their business and claim vat set off from future transaction.

57. Sujha Pvt. Ltd. imports the Honda cars and track-laying tractors and sells in Nepal.
It has the following transactions and information relevant to the VAT calculation in
the month of Bhadra, 20X-73.
a) Honda cars sales invoices of Rs. 30 million were issued to VAT registered
customers and invoices of Rs. l0 million were issued to individual customers (Not
registered in VAT).
b) Track-laying tractors sales invoices totaling Rs. 10 million were issued to
individual customers that were not registered for VAT.
c) The company offers a 5% discount for prompt payment.
d) Import price of Honda cars was IRs. 15 million, transportation expense upto
custom point was IRs 100,000 (Paid to Indian transporter) and insurance
premium Rs. 100,000 paid to Nepal insurance company. One of the imported cars
valued costing Rs. 50 lakhs were used itself for the pickup and drop of its
executive director.
e) Import price of Track-laying tractors was IRs. 5 million, transportation expense
up to custom point was IRs 50,000 (Paid to Indian transporter) and insurance
premium Rs. 50,000 paid to Nepal insurance company.
f) During the month, Rs. 30,000 was spent on mobile telephone calls, of which 40%
relates to private calls.
g) On 30 Bhadra, an office equipment was purchased for Rs. 7,00,000.
h) Custom duty was 80% in the case of Honda cars and 1 % in case of tractors.
i) All of the figures except IRs are exclusive of VAT.

Calculate the amount of VAT payable/receivable by the company for the month
ended Bhadra 20X-73. (Dec 2016, 10 Marks)

© The Institute of Chartered Accountants of Nepal 63


CAP II Paper 7 Income Tax and VAT

Answer
Calculation of VAT payable/receivable for the month ended Bhadra 20X-73
Particulars Amount Rs.
Output tax (WN 1) 5,850,000
Less: Input tax for imported cars (WN 2) (5,676,840)
Input tax credit on expenses (WN 3) (89,371)
Input tax on deemed sales (WN 4) (212,732)
Net VAT receivable 128,943

WN 1: Calculation of output tax


Particulars Cars sale Tractors VAT
Rs. sale Rs. amount
Rs.
Sales to VAT registered Customers 30 million --- 3,900,000
Sales to Unregistered Customers 10 million 10 million 1,300,000
Deemed Sales- Owned Consumption 5 million --- 650,000
Total 45 million 10 million 5,850,000
Proportionate sales ratio 81.82 % 18.18 %
Note 1: Track-laying Tractor is VAT exempt as per schedule 1 of VAT Act, 2052.
Note 2: Prompt Payment discount is deemed to be given after issuance of invoice.
Note 3: As per Value Added Tax Directive, 2069 (Updated 2076), VAT shall be collected at
cost at the time of self-use of goods. Self-use includes using the asset of one form into
another form.

WN 2: Calculation of VAT on Import at custom point for Honda cars:


Particulars Amount
Rs.
Import price IRs. 15 million * 1.6015 (Marks shall be 24,000,000
allowed even if 1.6 rate is used taking the
level of students)
Transportation expenses IRs. 100,000 * 1.6015 160,000
Insurance premium 100,000
Total value for custom 24,260,000
80 % custom duty 19,408,000
Total taxable value for 43,668,000
VAT
VAT paid at custom point 5,676,840
Track-laying tractors are vat exempt items. Therefore, there is no need to calculate input
vat on import.

© The Institute of Chartered Accountants of Nepal 64


Compilation of Suggested Answers Value Added Tax

WN 3: Calculation of input tax credit on expenses:


Particulars Amount Rs.
Full tax credit:
Insurance for Car Rs. 100,000 * 13% 13,000
Proportionate tax credit:
Mobile expenses Rs. 30,000*13%* 60%* 81.82% 1,915
Office Equipment Rs. 7,00,000 * 13%*81.82% 74,456
Total 89,371

WN 4: VAT credit on deemed sale (for own use in business)


VAT Sale invoice is issued whether it is sold to others or use for business, so, input tax for
the car is included in import and collection is included in output (Sales to VAT registers).
Input can be claimed for the car as follows:

Debit Adjustment for own used car


Cost price Rs. 5,000,000
VAT Rs. 650,000
Eligible for input Rs. 212,732 (650,000*40%*81.82%)

58. Ustab publication house is VAT registered firm, it prints the books and copy. It has
the following transactions excluding VAT for the month of Falgun, 2072.
Particulars Cost/ Sales Price Rs.
Purchase
Printing machine 30,000,000
Car for official purpose 3,000,000
Car for directors household purpose 2,600,000
Petrol to run generator set 500,000
Petrol to run car 200,000
Diesel for transport on truck 400,000
Papers 5,000,000 purchas
Motorcycle for official purpose 280,000
Advertisement 200,000
Royalty paid to the writers 2,500,000
Sales
Copy 6,000,000
Stationary 4,000,000
Books 5,000,000
Patro (Calendar) 1,000,000

© The Institute of Chartered Accountants of Nepal 65


CAP II Paper 7 Income Tax and VAT

You are required to calculate the amount of input tax credit and VAT to be collected
on sales. If Input tax credit is not set off against the collection, when it can claim for
refund? (June 2016, 10 Marks)

Answer
A. Output VAT (or VAT Collected on Sales)
Particulars Amount VAT Note
Copy 6,000,000 780,000 Tax rate of 13%
Stationery 4,000,000 520,000 Tax rate of 13%
Books 5,000,000 - VAT exempt
Patro 1,000,000 - VAT exempt
Total Output VAT for the month 1,300,000

B. Eligible VAT Credit on Purchases during the month


Eligible
VAT Amount
Particulars Amount Notes
paid for
Credit
Proportionate VAT
Credit, as the use is
Printing Machine 30,000,000 3,900,000 2,437,500
for both taxable and
non taxable output
Partial VAT credit
@ 40% of VAT
paid, and since the
person deals both in
Car for official purpose 3,000,000 390,000 97,500 taxable and non
taxable items,
proportionate credit
shall also be
claimed
No VAT credit, not
Car for directors household
2,600,000 338,000 - related to taxable
purpose
transaction
No VAT Credit,
Petrol to run generator 500,000 65,000 - consumption of
petroleum
No VAT Credit,
Petrol to run car 200,000 26,000 - consumption of
petroleum
No VAT Credit,
Diesel to Transport on truck 400,000 52,000 - consumption of
petroleum
Papers 5,000,000 650,000 406,250 Proportionate VAT

© The Institute of Chartered Accountants of Nepal 66


Compilation of Suggested Answers Value Added Tax

Motorcycle for official Credit, as the use is


280,000 36,400 22,750
purpose for both taxable and
Advertisement 200,000 26,000 16,250 non taxable output
Royalty paid to writers 2,500,000 - - VAT exempt
Total Eligible VAT Credit 2,980,250

C. Payable/ (Receivable VAT)

Payable/ (Receivable VAT) [A-B] (1,680,250)

Working Note
1. Ratio of Taxable Sales (For Proportionate VAT Credit)
Taxable Sales 10,000,000
Non Taxable Sales 6,000,000
Total Sales 16,000,000
Percentage of Taxable Sales 62.50%

59. Liza enterprise, a dealer of Toyota cars in Nepal, imported 5 cars for the Value @
US $ 50,000 on 12 Jestha, 20X-72. The payment was made through the Maya Bank
Ltd. on exchange rate of 1 US$ = Rs. 103.60. Others information are as follows:
a) Transportation cost upto Calcutta, India : @ US $ 2,000 (Payment made to
transporter @ Rs. 102.40 per US $)
b) Marine insurance paid to Nepal Insurance Company Ltd. Rs. 33,900
c) Transportation cost from Calcutta to Birgunj Customs Office IRs. 24,000
d) Driver's salary for the transportation from Birgunj to Kathmandu @ Rs. 6,000
e) Petrol cost from Birgunj to Kathmandu Rs. 9,040 with tax invoice
f) The custom duty is 80 %, ignore the excise duty and other taxes, duties
g) The company sells the cars adding 12 % on above cost.
h) Opening VAT account: Rs. 34,000 payable and it has no opening stocks.
i) The exchange rate published by Nepal Rastra bank on the day of clearing the cars
at custom: Buying rate is Rs. 102.40 per US $ and selling rate is Rs. 103.00 per
US$
You are required to calculate the VAT payable/receivable on the following situations:
(Ignore Import Service Fee): (Dec 2015, 6+4=10)

a. The Enterprise has used a car for own business and sold the remaining cars
during the month.
Answer
Output VAT for 4 cars 5,651,049 (13% of 10867402)*4
Input VAT for 4 cars (WN 1 & 2) 5,029,148 (12,56,507+780)*4

© The Institute of Chartered Accountants of Nepal 67


CAP II Paper 7 Income Tax and VAT

Input VAT for a car 502,915 {(12,56,507+780)*40%


VAT Payable (Output-input) 118,986

Note:
VAT paid on petrol cannot be claimed as credit

b. The Enterprise sold 4 cars and one remains in stock. Round up the fraction.
Answer
Output VAT for 4 cars (WN 2) 5,651,049 (13% of 10867402)*4
Input VAT for 5 cars (WN 1 & 2) 6,286,435 (12,56,507+780)*5
VAT receivable (Input-Output) 635,386

Note:
VAT paid on petrol cannot be claimed as credit

Working Notes

WN 1
Calculation of per unit VAT paid at Custom
Amount
Particulars
(Rs.)
Imported cost 5,150,000 (50,000*103)
Insurance 6,000 (33,900/5=6,780/1.13)
Transportation up to Calcutta 206,000 (2000*103)
(24,000/5=
Transportation Calcutta to custom 7,687
4,800*1.6015)
Transaction value 5,369,687
Custom duty @ 80 % 4,295,750
Taxable value 9,665,437
VAT @ 13 % 1,256,507

WN 2
Calculation of per unit cost, sales price and VAT
Particulars Amount (Rs). VAT (Rs.)
Imported cost (50,000*103.60) 5,180,000
Insurance (33,900/5=6,780/1.13) 6,000 780
Transportation up to Calcutta (2000*103) 206,000
Transportation Calcutta to custom
7,687
(24,000/5= 4,800*1.6015)
Custom Duty and VAT (WN 1) 4,295,750 1,256,507
Driver's salary 6,000

© The Institute of Chartered Accountants of Nepal 68


Compilation of Suggested Answers Value Added Tax

Petrol cost (9,040/5=1,808/1.13) 1,600 208


Total Cost 9,703,037 1,257,495
Profit addition (12 % on 97,03,037) 1,164,365
Selling price per unit 10,867,402
Output VAT per Car 1,412,762

60. M/S Prakarti Limited is a VAT registered company and registered with company
Registrar's Office in Nepal. The company is producing equal number of both vat
exempted & vat attractive goods on each day of production. The company has
following transaction during the month of Baishakh, 20X-72:
Opening VAT credit Rs. 125,000
Sales:
Vat exempted sales Rs. 4,000,000
Vat attractive sales Rs. 6,000,000
The company exports 59% of VAT attractive and VAT exempted sales
Purchases of raw materials Rs. 6,000,000
Other office expenses incurred Rs. 450,000
Cost of soft drinks for office party Rs. 200,000
Purchase of car Rs. 2,500,000
Purchase of two-wheeler Rs. 225,000
Purchase of petrol for car & two-wheeler Rs. 185,000
The given purchases & expenses are excluding applicable vat. Other office expenses
include one invoice of Rs. 100,000 pertaining to the period of Chaitra 20X-70.
Required: (Dec 2015, 2+3+4+1=10)
a) Compute the VAT payable on sales, VAT paid on purchase & expenses incurred
by the company and VAT credit available to the company.
Answer
VAT Payable (i.e. collected) on Sales
Particulars Total Value VAT Collected Note
VAT Attractive Sales 6,000,000
Domestic 2,460,000 319,800 13 Percent
Export 3,540,000 - Zero percent
VAT Exempt Sales 4,000,000
Domestic 1,640,000 - Exempt
Export 2,360,000 - Exempt
Total Output VAT 319,800

VAT paid on Purchases and Expenses


Particulars Total Value VAT Paid Note
Purchases of raw materials 6,000,000 780,000 Assumed

© The Institute of Chartered Accountants of Nepal 69


CAP II Paper 7 Income Tax and VAT

Other office expenses incurred during the month 350,000 45,500 that
Other office expenses incurred during Chaitra purchase is
100,000 made from
20X-70 13,000
Cost of soft drinks for office party 200,000 26,000 registered
person
Purchase of car 2,500,000 325,000
Purchase of two wheeler 225,000 29,250
Purchase of petrol for car & two wheeler 185,000 24,050
VAT Paid on Purchases 1,242,800

VAT Credit available to the Company


Ratio of Taxable Sales to Total Sales is 60%
VAT Eligible
Particulars Note
Paid Credit
Purchases of raw materials 780,000 468,000 Proportionate Credit
Other office expenses incurred during the month 45,500 27,300 Proportionate Credit
The date from purchase
Other office expenses incurred during
13,000 - exceeded one year, and
Chaitra 20X-70
hence, no credit
Cost of soft drinks for office party 26,000 - No VAT Credit
40% of VAT paid
assuming used for
administrative purpose,
Purchase of car 325,000 78,000
and to the extent of
taxable sales to total
sales
Purchase of two wheeler 29,250 17,550 Proportionate Credit
Purchase of petrol for car & two wheeler 24,050 - No VAT Credit
Eligible VAT Credit 590,850

© The Institute of Chartered Accountants of Nepal 70


Compilation of Suggested Answers Value Added Tax

b) Compute VAT refundable if any under the provision of VAT Act, 2052.
Answer
Particulars Amount
Total Output VAT 319,800
Eligible VAT Credit 590,850
Opening VAT Credit 125,000
VAT Payable/ (Receivable) (396,050)

61. OLIZ Trading Pvt. Ltd. trades in various vatable goods. The company sold goods on
credit for 60 days whereas creditors provide credit facility for 50 days. The
company’s receivables on account of taxable goods sold is Rs. 3,390,000 for the month
of Kartik 20X-71. Input Tax credit of previous month is Rs. 21,000.
Details of taxable purchase of the company in the month of Kartik are as follows:
a) Total payable to Supplier for the Taxable purchase of the trading goods- Rs.
2,260,000
b) Total payable for computer and printer purchased Rs. 67, 800
c) Payable made for motorcycle purchased Rs. 226,000
d) Payable made to Hotel Rs. 24, 860 for dinner party organized for party meeting.
Find out the VAT amount paid on purchases and collected on sales. Also, workout
amount of allowable input Tax credit and VAT payable. (July 2015, 10 Marks)
Answer
Calculation of Output VAT
Particulars Total Value VAT Note
including VAT Collected
Receivable on account of Taxable 3,390,000 390,000 = 3,390,000/1.13
Goods *0.13
Total Output VAT 390,000
Calculation of Eligible Input Tax Credit
Particulars Total Value VAT Eligible Note
including VAT Paid VAT Credit

Payable to Supplier for 2,260,000 260,000 260,000 As the credit period is


the Taxable purchase 50 days, hence, total
of the trading goods purchase is payable
Payable for computer 67,800 7,800 7,800
and printer purchased
Payable made for 226,000 26,000 26,000
motorcycle purchased
Payable on account of 24,860 2,860 - Entertainment
dinner party expense, no Credit
Total Eligible Input Tax Credit 293,800

© The Institute of Chartered Accountants of Nepal 71


CAP II Paper 7 Income Tax and VAT

Calculation of VAT Payable/(Receivable)


Particulars Amount
Total Output VAT 390,000
Total Eligible Input Tax Credit 293,800
Opening VAT Credit 21,000
VAT Payable/(Receivable) 75,200

62. Cycle Tyre Co. Ltd. of Biratnagar imports raw materials from India for
manufacturing of BLAST brand of bicycle wheels. It sells its product both in Nepal
and India. During Magh, finance controller of the company resigned and the
company has not paid any amount of VAT nor has filed monthly VAT return for
Magh and Falgun and you are appointed as finance officer of the company during
Chaitra.

The details of VAT related transaction of company are:


Month Magh Falgun
Particulars Taxable VAT Taxable VAT
Opening VAT credit 20,000
Domestic-purchase raw materials 21,000 2,730 30,000 3,900
Import of raw materials 280,000 36,400 400,000 52,000
Raw materials consumed out of domestic
purchased 18,900 2,457 27,000 3,510
Raw materials consumed out of imports 252,000 32,760 360,000 46,800
Diesel consumed 22,400 2,912 32,000 4,160
Diesel purchased 63,000 8,190 90,000 11,700
Payment to labour contractor for supply of
workers 36,400 4,732 52,000 6,760
Consumables used from stock 14,000 1,820 20,000 2,600
R&D service procured from Germany
(Unregistered) 300,000
Value of finished goods produced 900,000 1,200,000
Sales domestic 810,000 345,600
Sales export - 734,400
Required: (July 2015, 10 Marks)
Prepare a memo to be forwarded to the Board of Directors of the company referring
the relevant provisions of VAT Act, amount indicating VAT payable, refund
receivable and amount of VAT credit available for each month.

© The Institute of Chartered Accountants of Nepal 72


Compilation of Suggested Answers Value Added Tax

Answer
Office Memo

Subject: Internal Memorandum

The Board of Directors,


Cycle Tyre Co. Ltd.

Based on the attached VAT computation sheet, the company owes Rs. 41,438 as VAT
obligation payable to Government of Nepal during the month of Magh. In addition to Rs.
41,438; the company also owes Rs. 39,000 as VAT payable in respect of purchase of
service from outside Nepal under reverse charging principle as per Sec. 8 (2) of Value
Added Tax Act, 2052 as soon as possible.

Further, the company has receivable VAT of Rs. 56,732 during the month of Chaitra.

The detailed computation of VAT payable, refund receivable and amount of VAT credit
available for each month is presented in Annex 1, Annex 2 and Annex 3 below.

Annex 1: Calculation of Payable/(Receivable) VAT


Particulars Magh Falgun
Total Output VAT for the month 105,300 44,928
Total Eligible Input Tax Credit for the purchase during month 43,862 62,660
Opening VAT credit 20,000 -
Input Tax credit on R&D service procured from Germany 39,000
VAT Payable/(Receivable) 41,438 (56,732)

Annex 2: Calculation of Output VAT


Magh Falgun
Particulars Note
Value VAT Value VAT
Sales domestic 810,000 105,300 345,600 44,928 @13%
Sales export - - 734,400 - @0%
Output VAT is
collected on
Value of finished goods produced 900,000 - 1,200,000 -
Sales and not on
Production
Total Output VAT for the month 105,300 44,928

W.N.3: Calculation of Eligible Input Tax Credit on Purchases made during the month,
ignoring any VAT paid u/s 8 (2), 12Ka or 15 (3)

© The Institute of Chartered Accountants of Nepal 73


CAP II Paper 7 Income Tax and VAT

Magh Falgun
Particulars Note
Value VAT Value VAT
Domestic- purchase raw
21,000 2,730 30,000 3,900 Full VAT credit
materials
Import of raw materials 280,000 36,400 400,000 52,000 Full VAT credit
Input Credit is
Raw materials consumed out claimed on purchase
18,900 - 27,000 -
of domestic purchased and not on
consumption
Input Credit is
Raw materials consumed out claimed on purchase
252,000 - 360,000 -
of imports and not on
consumption
Input Credit is
claimed on purchase
Diesel consumed 22,400 - 32,000 -
and not on
consumption
Diesel purchased 63,000 - 90,000 - No VAT Credit
Payment to labour
contractor for supply of 36,400 4,732 52,000 6,760 Full VAT credit
workers
Input Credit is
Consumables used from claimed on purchase
14,000 - 20,000 -
stock and not on
consumption
Total Eligible Input Tax Credit for the
43,862 62,660
purchase during month

63. M/s RK Trading Private Limited is a Value Added Tax Registered company. It is
engaged in the business of importing wines & liquor and selling in local market. The
company is in confusion to take tax credit on procurement of wines and liquor as per
Value Added Tax Act/Rules, Advise. What will be your answer if the company does
not have such primary business? (Dec 2014, 5 Marks)
Answer
As per Rule 41, there shall be no VAT credit on consumption of liquor, even if the person
deals in taxable transaction otherwise than in such circumstances when the main business
of the person is to deal in liquor.

© The Institute of Chartered Accountants of Nepal 74


Compilation of Suggested Answers Value Added Tax

Conclusion
Since the main business of M/s RK Trading Private Limited is to deal in liquor items, the
person is eligible to claim full VAT credit on the VAT paid on import of wines and liquor.

If M/s RK Trading Private Limited does not sell wines and liquor, VAT paid on such wines
and liquor cannot be claimed as input tax credit.

64. Ilam Tea Pvt. Ltd. is a tea processing and producing industry, located in Ilam. It sells
the tea within Nepal. It has following transaction in the month of Bhadra, 20X-71.
Particulars Transactions Amount
(Rs.)
Sales:-
VAT registered party 20,00,000
VAT Non-registered party 5,00,000
Purchase:-
Wages and others that are VAT 12,00,000
exempt
Compute the VAT amount payable/receivable by the Tea Industry with reference to
the VAT Act, 2052 for the Month. Assume no VAT on opening. (Dec 2014, 5 Marks)

Answer
Output VAT 325,000
(13% of 25 Lakhs)
Eligible Input VAT credit -
(As all purchases are VAT exempt)
Payable VAT for the month 325,000

65. JivanJyoti Gases Industries Pvt. Ltd. is engaged in manufacturing of Oxygen &
Nitrogen Gases. The transactions of the industry for the month of Kartik, 20X-70 is as
follows:
Sales Details (Excluding VAT)
Oxygen Nitrogen Gases
Sold to Date
Gases (in Rs.) (In Rs.)
Teaching Hospital 20X-70.07.02 50,000 -
Ram Metal Works 20X-70.07.05 40,000 25,000
B & B Hospital 20X-70.07.10 55,000 -
Ghorahi Cement Industries 20X-70.07.14 125,000 250,000
Sine Hydro Corporation 20X-70.07.25 150,000 30,000
Total Sales for the month 420,000 305,000

© The Institute of Chartered Accountants of Nepal 75


CAP II Paper 7 Income Tax and VAT

Purchase Details(Excluding VAT)


Amount
Particulars
( in Rs.)
Diesel purchase for Generator
Running 90,000
Telephone Chagres 18,000
Purchase of Vehicle for General
Manager 2,250,000

The Industries also paid the consultancy fee amounting to IRS 50,000 to Sigma
Consultancy, New Delhi, India.
VAT receivable as per Ashwin, 20X-70 VAT Return is NRS 28,000.
You are requested to compute the VAT collected on Sales, VAT Paid on Purchase and
Net VAT payable/receivable for the Kartik end 20X-70. (June 2014, 10 Marks)
Answer
Calculation of Output VAT
Particulars Sales Value Output VAT Reason
Sales of Oxygen 420,000 - Exempt
Sales of Nitrogen 305,000 39,650 @13%
Total Output VAT 39,650
Taxable Sales 42.07%
Non-taxable sales 57.93%

Calculation of Eligible Input Tax Credit


Eligible
Amount VAT
Particulars VAT Notes
(in Rs.) paid
Credit
Diesel purchase for Generator No VAT credit
Running 90,000 11,700 - item
Proportionate
Telephone Chagres 18,000 2,340 984 credit
40% of VAT paid
Purchase of Vehicle for General in proportion of
Manager 2,250,000 292,500 49,222 taxable sales
Total Eligible Input VAT 50,206

© The Institute of Chartered Accountants of Nepal 76


Compilation of Suggested Answers Value Added Tax

Calculation of Net VAT Payable or Receivable


Total Output VAT 39,650
Total Eligible Input VAT 50,206
VAT Payable/(Receivable) (10,556)

66. AMP Wire Industries Pvt. Ltd. purchased the following in the month of Kartik 20X-
70.
Particulars VAT Paid (Rs.)
Used photocopy machine 1,30,000
Brand new Fax machine 2,600
Used generator set 3,90,000
Import of Brand new generator set 5,20,000

Required:
Calculate the VAT amount to be claimed for offset. (Dec 2013, 5 Marks)

Answer
AMP Wire Industries Pvt. Ltd. deals in taxable transaction, therefore, there shall be full
VAT credit on VAT paid on purchases that are intended to be used for taxable output. As
such, the amount to be claimed for set off is as follows:
Particulars VAT paid
Used photocopy machine 130,000
Brand new Fax machine 2,600
Used generator set 390,000
Import of Brand new generator set 520,000
Total 1,042,600

67. EFG Garment Pvt. Ltd. had the following transaction in the month of Shrawan 20X-
70. Calculate the VAT payable/receivable from the information below.
(Dec 2013, 10 Marks)
Particulars Amount (Rs.)
Local sale 50,00,000
Export sale 1,00,00,000
Purchase of clothes, Stitching, Packing Materials, loose tools for 81,00,000
machineries
Special packing for export 4,00,000
Payment of consultancy charges abroad 5,00,000
Purchase of bus for staff transportation 20,00,000
Purchase of motorcycle of hire purchase 4,00,000
Telephone expenses 76,000

© The Institute of Chartered Accountants of Nepal 77


CAP II Paper 7 Income Tax and VAT

Purchase of diesel for generator 80,000


Purchase of diesel for bus 24,000
Purchase of petrol for motorcycle 30,000
Purchase of computers 90,000
Purchase of soft drinks 12,000

Additional information:
Opening VAT receivable for the month was Rs. 91,560. Diesel for bus for Rs. 16,000
and soft drinks for Rs. 6,000 was purchased through abbreviated tax invoice. Items
above are exclusive of VAT.
Answer
A. Output VAT (or VAT Collected on Sales)
Particulars Amount VAT Note
Domestic Sales 5,000,000 650,000 @ 13%
Export Sales 10,000,000 - @ 0%
Total Output VAT for the month 650,000

B. Eligible Input tax credit on purchases during the month


Eligible
VAT Amount
Particulars Amount Notes
paid for
Credit
Purchase of clothes, Stitching, Packing 8,100,000 Full VAT
Materials, loose tools for machineries 1,053,000 1,053,000
credit
Special packing for export 400,000 Full VAT
52,000 52,000
credit
40% of VAT
paid can be
claimed as
credit, as the
Purchase of bus for staff transportation 2,000,000 260,000 104,000
bus is used
for
administrative
purpose
Purchase of motorcycle of hire purchase 400,000 Full VAT
52,000 52,000
credit
Telephone expenses 76,000 Full VAT
9,880 9,880
credit
Purchase of diesel for generator 80,000 No VAT
10,400 -
Credit
Purchase of diesel for bus 24,000 No VAT
3,120 -
Credit
Purchase of petrol for motorcycle 30,000 3,900 - No VAT

© The Institute of Chartered Accountants of Nepal 78


Compilation of Suggested Answers Value Added Tax

Credit
Purchase of computers 90,000 Full VAT
11,700 11,700
credit
Purchase of soft drinks 12,000 No VAT
1,560 -
Credit
Total Eligible VAT Credit
1,282,580
C. Input VAT Credit on VAT paid as per Sec. 8 (2), 12Ka and 15 (3)
Eligible
Particulars Amount VAT Amount Notes
for Credit
Assuming that the VAT in
Payment of Consultancy
500,000 65,000 65,000 respect of this service is
Charges abroad
already paid
Total 65,000

D. Opening VAT credit (excess of Previous month,


eligible for set off) -

E. Payable/ (Receivable VAT)


[A-B-C-D] (697,580)

68. Gramin Krishi Sewa Pvt. Ltd. sales vegetables and fruits. During the month of
Baishakh 20X-70 following sales and purchase made. Compute the net Value Added
Tax payable under the Value Added Tax Act, 2052. (June 2013, 8 Marks)
Items Purchase (Rs.) Sales (Rs.)
Green Tea 50,000 57,500
Barley 40,000 48,000
Garlic 30,000 37,500
Black Tea 80,000 100,000
Bhakti Fish 150,000 175,000
Millet 45,000 49,500
Paneer 70,000 78,400
Curd 60,000 64,375
Telephone Bill 5,000
Stationery Purchase 20,000
Answer
Calculation of Output VAT
Sales VAT
Particulars Reason
Value amount
Green Tea 57,500 7,475 Taxable, & @13%

© The Institute of Chartered Accountants of Nepal 79


CAP II Paper 7 Income Tax and VAT

Barley 48,000 - VAT exempt


Garlic 37,500 4,875 Taxable, & @13%
Black Tea 100,000 13,000 Taxable, & @13%
Bhakti Fish 175,000 22,750 Taxable, & @13%
Millet 49,500 - VAT exempt
Paneer 78,400 10,192 Taxable, & @13%
Curd 64,375 8,369 Taxable, & @13%
Total Output VAT 66,661

Taxable Sales 512,775 84.02%


Non-taxable Sales 97,500 15.98%
610,275

Calculation of Eligible Input Tax Credit


Purchase Eligible
Particulars VAT paid Reason
Value Credit
Green Tea 50,000 6,500 6,500 Full VAT Credit
Barley 40,000 - - VAT Exempt
Garlic 30,000 3,900 3,900 Full VAT Credit
Black Tea 80,000 10,400 10,400 Full VAT Credit
Bhakti Fish 150,000 19,500 19,500 Full VAT Credit
Millet 45,000 - - VAT Exempt
Paneer 70,000 9,100 9,100 Full VAT Credit
Curd 60,000 7,800 7,800 Full VAT Credit
Telephone Bill 5,000 650 546 Proportionate Credit
Stationery Purchase 20,000 2,600 2,185 Proportionate Credit
Total Eligible VAT Credit 59,931

Calculation of VAT payable or Receivable


Particulars Amount
Total Output VAT 66,661
Total Eligible VAT Credit 59,931
VAT Payable/(Receivable) 6,730

69. There are some cases where Value Added Tax (VAT) paid on purchase is not allowed-
no credit, even if output of registered person is VAT attractive. Mention those cases as
prescribed under Value Added Tax/Rules (Dec 2012, 5 Marks)
Answer

© The Institute of Chartered Accountants of Nepal 80


Compilation of Suggested Answers Value Added Tax

The circumstances when Value Added Tax paid on purchase is not allowed, even if output
of registered person is VAT attractive are as follows:
(a) In case there is no proper Value Added Tax invoice (other than abbreviated tax
invoice) to substantiate VAT paid except when VAT is paid under import (where import
documents substitute VAT invoice), u/s 8 (2), 12Ka and 15 (3).
(b) In case such purchase is listed as below and the main business of the person is not
dealing in the items listed below:
(i) Entertainment expense
(ii) Beverages
(iii)Liquor
(iv) Petroleum (includes diesel, liquefied petroleum gases and petrol)

70. M/s Kiran Distributors is engaged in the wholesale business of various types of
noodles. The firm is registered in VAT. The firm has taken a loan of Rs. 10 lacs from
M/s Saraswati Bank for purchasing a delivery van. Bank has paid the total amount
for the purchase of Van but has kept the ownership of van in the name of the bank till
the loan is fully repaid by Kiran Distributors. Kiran has to pay Rs. 25,000 per month
towards the principal and interest to the Bank as part of their loan agreement. Rs.
130,000 was paid as VAT during the purchase of van. By mentioning the relevant
provision of the Act, can Kiran Distributors claim the input tax credit?
(Dec 2012, 4 Marks)
Answer
As per section 17(5ka) of Value Added Tax Act, 2052, if a tax payer purchases or imports
capital goods under a loan agreement under financial lease, then input tax paid can be
claimed by the concerned tax payer.

In this case, although the legal title still vests with the bank, the firm can take the input tax
credit as per section 17(5ka).

71. PQR Ltd's Stock was damaged by fire on 20X-68.05.20. PQR Ltd. enquires on taking
the input tax credit paid on the lost stock. By citing the relevant provisions of the act
and rules, advise PQR Ltd. (Dec 2012, 3 Marks, CA Inter)
Answer
Nepal adopts invoice-credit method. That means, a taxpayer claims input tax credit on
every purchase that are intended to be used for taxable output. When the related stock is
lost due to any reason, the taxpayer shall comply with provisions of VAT law to get the
requisite support from tax officer to legitimatize the credit claimed at the time of purchase
of such stocks.
The related provision for the same is mentioned in Rule 39Ka of Value Added Tax
Regulation, 2053, which is extracted as under:

© The Institute of Chartered Accountants of Nepal 81


CAP II Paper 7 Income Tax and VAT

Application for Input Tax Credit & Time Limit for such application
Where there is loss of goods by fire, theft, accident, accidental damages, or terror and it
compels a person to write off the goods (assets) or sale such goods at lower selling rate,
the person shall make an application in writing to respective Inland Revenue Office along
with evidence within 30 days of happening of such events.

Investigation and permission to claim as input tax credit


The tax office shall investigate the matter and finalize the quantum of tax credits to be
allowed. On the basis of such investigation, the tax office may allow the taxpayer to claim
Input Tax Credit of VAT paid on such assets (goods).

Right to claim Input tax Credit without confirmation from Tax Officer
In case the goods are insured, the tax office may allow the taxpayer to claim Input Tax
Credit on such goods to the extent of compensation paid by the Insurance Company,
without any investigation from tax officer.

Conclusion
In the given case, since the stock is lost by fire, procedure of Rule 39Ka shall be followed
to obtain support from tax officer to legitimatize the credit claim made by PQR Ltd. at the
time of purchase of such stock.
For this, PQR Ltd. shall file an application within 30 days of 20X-68.05.20, along with the
substantiating documents, after which the tax officer will investigate the matter and
legitimize the credit claimed by PQR Ltd. as per Rule 39Ka.

72. Mercantile Limited is providing service in the field of information technology and its
sales in the month of Chaitra 20X-67 was Rs. 5 million. Out of total sales Rs. 3 million
was export. The company has got some service from Star Inc. registered and working
in USA. Cost paid for the services was Rs. 2 million and the company paid Rs. 260,000
in IRO as reverse charging payment. Total purchases for the month of Chaitra were
Rs. 1 million and it has also incurred the following expenditures:
(a) Telephone charges Rs. 10,000
(b) Petrol for vehicle Rs. 5,000
(c) Printing and stationery Rs. 5,000
The company has taken VAT bill for all the above purchases. You are required to
calculate the actual amount of VAT payable by Mercantile Limited for the month of
Chaitra 20X-67. (June 2012, 5 Marks)

© The Institute of Chartered Accountants of Nepal 82


Compilation of Suggested Answers Value Added Tax

Answer
A. Output VAT (or VAT Collected on Sales)
Particulars Amount VAT Note
Domestic Sales 2,000,000 260,000 Tax rate of 13%
Export Sales 3,000,000 - Tax rate of 0%
Total Output VAT for the month 260,000

B. Eligible Input tax credit on purchases during the month


Eligible
Amount
Particulars Amount VAT Notes
for
Credit
Purchase 1,000,000 130,000 130,000 Full VAT credit
Telephone Charges 10,000 1,300 1,300 Full VAT credit
No VAT Credit,
Petrol for vehicle 5,000 650 - consumption of
petroleum
Printing and
5,000 650 650 Full VAT credit
stationery
Total Eligible VAT
131,950
Credit

C. Eligible Input Tax Credit on Purchases during last 12 months, where the credit
claim was erroneously omitted
Eligible
Particulars Amount VAT Amount Notes
for Credit

D. Input VAT Credit on VAT paid as per Sec. 8 (2), 12Ka and 15 (3)
Eligible
Particulars Amount VAT Amount Notes
for Credit
The VAT in
respect of this
service is already
Service from Star Inc.,
2,000,000 260,000 260,000 paid and
USA
therefore, the
amount can be
claimed as credit
Total
260,000

© The Institute of Chartered Accountants of Nepal 83


CAP II Paper 7 Income Tax and VAT

E. Opening VAT credit (excess of Previous month,


-
eligible for set off)

F. Payable/ (Receivable VAT)


(131,950)
[A-B-C-D-E]

73. Satellite Telecom Pvt. Ltd., a telecom operator company in Nepal, seeks your advice
whether it can take credit of the VAT paid in the following transactions.
(June 2012, 5 Marks)
a) VAT paid Rs. 18,000 for the purchase of petrol to be used for vehicle operation.
Answer
No, since the main business of Satellite Telecom Pvt. Ltd. is not dealing in petrol and
there shall be no VAT credit in respect of purchase of petrol by any person whose
main business is not dealing in petrol.

b) VAT paid at Birgunj Customs Office Rs. 1,000,000 while importing the telecom
equipments from China.
Answer
Full VAT credit, as the intended use of input is to generate taxable output, which
shall be demonstrated through use later.

c) The company deposited VAT amount Rs. 175,000 in Inland Revenue Office on
the service charges payable to satellite bandwidth provider and want to take
VAT credit of this whole amount.
Answer
Assuming the amount is paid under reverse charging principle as per Sec. 8 (2) of the
Act, the amount is eligible for VAT credit.

d) The company purchases beverage in the occasion of annual day and paid VAT
Rs. 10,000 on this bill.
Answer
No, since the main business of Satellite Telecom Pvt. Ltd. is not dealing in beverage
and there shall be no VAT credit in respect of purchase of beverage by any person
whose main business is not dealing in beverage.

e) VAT paid Rs. 13,000 for the purchase of petrol to be used for the operation of
generators as BTS sites.
Answer

© The Institute of Chartered Accountants of Nepal 84


Compilation of Suggested Answers Value Added Tax

No, since the main business of Satellite Telecom Pvt. Ltd. is not dealing in petrol and
there shall be no VAT credit in respect of purchase of petrol by any person whose
main business is not dealing in petrol.

74. XYZ Pvt. Ltd. imports/purchases the taxable raw materials such as Iron Ingots and
Steel Plates to manufacture them into different furniture items as well as agriculture
tools.
Details of total imports excluding Input VAT are as follows:
Cost of Iron Ingots declared by XYZ Pvt. Ltd. Rs. 3,00,000
Revised cost fixed by Customs Officer Rs. 3,50,000
Freight Rs. 50,000
Insurance Rs. 5,000
Import Duty @ 6%
In addition, it has also purchased following raw materials:
Steel plates (exclusive of VAT) Rs. 1,00,000
Woods (exclusive of VAT) Rs. 35,000

The products manufactured out of the above raw materials are sold out with the
details as given below:
Agriculture tools (VAT exempt items) Rs. 2,50,000
Furniture Rs. 3,00,000
Find the ratio between taxable and non taxable sales and calculate the amount of
Input VAT that XYZ Pvt. Ltd. is entitled to claim under sec 17 (3) of VAT Act.
(June 2011, 10 Marks)
Answer
Ratio of Taxable and Non-taxable Sales
Particulars Amount Ratio Calculation
Taxable Sales 300,000 54.55% =300,000 divided by 550,000 multiplied by 100
Non-taxable sales 250,000 45.45% =250,000 divided by 550,000 multiplied by 100
Total Sales 550,000 100%

Calculation of Output VAT


Particulars Sales Amount VAT Amount Reason
Taxable Sales 300,000 39,000 @13% on domestic sales
Non-taxable sales 250,000 - Exempt
Total Output VAT 39,000

Calculation of Eligible Input VAT credit

© The Institute of Chartered Accountants of Nepal 85


CAP II Paper 7 Income Tax and VAT

Eligible
VAT
Particulars Amount VAT Reason
paid
amount
Import (Refer W.N. 1 below) 429,300 55,809 30,441 Proportionate credit in
Steel Plates 100,000 13,000 7,091 proportion of taxable sales
Woods 35,000 4,550 2,482 to total sales
Total Eligible Input Tax Credit 40,014

Calculation of Payable or Receivable VAT for the month


Particulars Amount
Total Output VAT 39,000
Total Eligible Input Tax Credit 40,014
Payable /(Receivable) VAT for the month (1,014)

Working Note 1: VAT paid on Import


Particulars Amount
Revised cost fixed by Customs Officer 350,000
Freight 50,000
Insurance 5,000
Transaction Value for Import duty 405,000
Import Duty @ 6% 24,300
Taxable Value for VAT 429,300
VAT @13% 55,809

75. Sharma & Sharma Co. has been a self employed VAT registered trader since 2064
B.S, and is in the process of completing the VAT return for the month ended 31
Baisakh 20X-68. The following information is relevant to the completion of the VAT
return:
a) Sales invoices totaling Rs. 44,00,000 (excluding VAT) were issued to VAT
registered customers in respect of VAT sales. The firm offers a 5% discount for
prompt payment.
b) Sales invoices totaling Rs. 16,92,000 were issued to customers that were not
registered for VAT. Of this figure, Rs. 51,700 was in respect of zero-rated VAT
sales.
c) During the month of Baisakh 20X-68, goods amounted to Rs. 11,20,000 were
purchased. Of this figure, Rs. 80,000 was used for Mr. Sharma’s private purpose.
d) On 15 Baisakh 20X-68, a Toyota Hilux vehicle was purchased for Rs. 24,00,000.
The cost is inclusive of VAT.
e) During the month ended 31 Baisakh 20X-68, Rs. 40,000 was spent on mobile
telephone calls, of which 30% relates to private calls.

© The Institute of Chartered Accountants of Nepal 86


Compilation of Suggested Answers Value Added Tax

f) On 17 Baisakh 20X-68, an office equipment was purchased for Rs. 6,00,000. The
purchase was partly financed by a bank loan of Rs. 5,00,000.
Unless stated otherwise, all of the figures are exclusive of VAT.
Calculate the amount of VAT payable by Sharma & Sharma Co. for the month ended
31 Baisakh 20X-68. (June 2011, 10 Marks)
Answer
Calculation of Output VAT
Particulars Amount
Sales VAT Registered Customers 4,400,000
Less: Discount at the time of sales 220,000
Taxable Value 4,180,000
VAT @13% 543,400

Particulars Amount Reason


Sales to Customers not registered for VAT 1,692,000 Assuming excluding VAT
Less: Zero rated sales 51,700
Taxable Value for non-zero rated sales 1,640,300
VAT @13% 213,239

Total Output VAT


Particulars Amount Reason
VAT collected from Registered Customers 543,400
VAT collected from Unregistered Customers 213,239
VAT collected on Zero rated sales - 51,700 multiply by Zero
Total Output VAT 756,639

Calculation of Eligible Input VAT Credit on Purchases during the month


VAT Eligible
Particulars Amount Reason
paid Credit
There shall not be input tax
credit on purchases consumed
Purchase (excluding
1,120,000 145,600 135,200 for personal purpose.
VAT)
Amount of credit = Rs.
10,40,000 * 13%
VAT paid is 13 % of (2400000
divided by 1.13)
Purchase of Toyota
40% of VAT paid is eligible to
Hilux (Including 2,400,000 276,106 110,442
claim credit, assuming the
VAT)
vehicle is used for personal
(administrative) purpose
Telephone call No VAT credit with regards to
40,000 5,200 3,640
(excluding VAT) personal call, VAT credit is

© The Institute of Chartered Accountants of Nepal 87


CAP II Paper 7 Income Tax and VAT

equal to 70% of VAT paid

VAT credit is availabe to the


Office Equipment 600,000 78,000 78,000 lessee in respect of goods under
finance lease
Eligible VAT Credit 327,282

Calculation of VAT Payable/(Receivable)


Particulars Amount
Output VAT 756,639
Less: Eligible Input VAT credit 327,282
VAT Payable/(Receivable) 429,357

76. Samal Ltd. has following transactions during the month. Find the amount of VAT
payable/receivable for the coming month: (June 2011, 5 Marks, CA Inter)
Purchase (excluding VAT) 10,00,000
Salary for the month 1,00,000
Purchase of a car with VAT 11,30,000
Purchase of Office Supplies (excluding 1,10,000
VAT)
Sales 20,00,000
Office supplies include Rs. 10,000 purchased under abbreviated tax invoice.
50% of the sales represent export sales.
Answer
Calculation of Output VAT
VAT
Particulars Amount Reason
collected
Domestic Sales 1,000,000 130,000 VAT @13% on Domestic Taxable Sales
Export Sales 1,000,000 - VAT @ 0% on Export Taxable Sales
Total Output VAT 130,000

© The Institute of Chartered Accountants of Nepal 88


Compilation of Suggested Answers Value Added Tax

Calculation of Eligible Input VAT credit


Amount Eligible
Amount VAT
Particulars excluding VAT Reason
(Rs.) paid
VAT credit
Purchase (excluding VAT) 1,000,000 1,000,000 130,000 130,000 Full VAT credit
Salary for the month 100,000 100,000 Exempt - Exempt
40% of VAT
paid, assuming
car is used for
Purchase of car with VAT 1,130,000 1,000,000 130,000 52,000
personal
(administrative)
purpose
Purchase of office supplies
(excluding VAT)
There cannot be
tax credit using
In abbreviated tax invoice 10,000 10,000 1,300 -
abbreviated tax
invoice
With Proper VAT invoice 100,000 100,000 13,000 13,000
Full VAT credit
Total Eligible Tax Credit 195,000

Calculation of VAT payable or Receivable


VAT payable/(Receivable)
Output VAT 130,000
Less: Input VAT (195.000)
VAT Payable/(Receivable) (65,000)

77. Fat Limited has following transactions during the month of Baisakh, 20X-67, find the
amount of VAT credit available for the month of Jestha:
Amount (Rs.)
Opening Credit available 12,000
Purchases net of VAT 10,00,000
Salary for the month 1,50,000
Electricity expenses 10,000
Telephone expenses with VAT 13,560
Fuel expenses with VAT 28,250
Purchase of car with VAT 11,30,000
Purchase of office supplies net of VAT 1,50,000
Total sales for the month 15,00,000

© The Institute of Chartered Accountants of Nepal 89


CAP II Paper 7 Income Tax and VAT

Additional information:
All the sales of the company were VAT applicable. Out of office supplies, Rs. 20,000
was purchased by obtaining abbreviated tax invoice. In purchases, Rs. 375,000 is
import of raw material and customs office has valued these goods Rs. 450,000. Fuel
expenses consists expenses for petrol used for office vehicle. (Dec 2010, 6 Marks)
Answer
VAT credit available for the month
Amount Amount
VAT Eligible
Particulars (Rs.)- excluding Reason
paid VAT
Given VAT
credit
Purchases net of VAT
Import where Customs Officer Full VAT credit,
375,000 375,000 58,500 58,500
revised Value Note (1) below
Other purchases 625,000 625,000 81,250 81,250 Full VAT credit
Salary for the month 150,000 150,000 Exempt - Exempt
Electricity expenses 10,000 10,000 Exempt - Exempt
Telephone expenses with VAT 13,560 12,000 1,560 1,560 Full VAT credit
No VAT credit,
as the input is
petrol and the
Fuel expenses with VAT 28,250 25,000 3,250 - person's main
business is not
dealing in
petrol
40% of VAT
paid, assuming
car is used for
Purchase of car with VAT 1,130,000 1,000,000 130,000 52,000
personal
(administrative)
purpose
Purchase of office supplies net
of VAT
There cannot be
tax credit using
In abbreviated tax invoice 20,000 20,000 2,600 -
abbreviated tax
invoice
With Proper VAT invoice 130,000 130,000 16,900 16,900 Full VAT credit
Total Eligible Tax Credit 210,210
Add: Opening VAT Credit 12,000
Total Credit Available for the month 222,210

Note 1:

© The Institute of Chartered Accountants of Nepal 90


Compilation of Suggested Answers Value Added Tax

VAT paid on import of Rs. 375,000 is 13% of Rs. 450,000 as VAT is collected by Customs
officer on revised cost, assuming the taxable value is Rs. 450,000.

78. Goodwill Ltd. has purchased a motorcycle from a VAT registered dealer and paid Rs.
10,000 as VAT. Goodwill Ltd. wants to take input credit of Rs. 10,000 as per section
17 of the Act. But the tax officer contends that as per rule 41 of VAT Rules, only 40%
of the VAT paid i.e. Rs. 4,000 can be claimed. Give your opinion with regards to this
transaction by citing the provision of Section 17 and Rule 41.
(Dec 2010, 5 Marks, CA Inter)
Answer
Partial VAT Credit:
As per Rule 41 of Value Added Tax Regulation, 2053, input Tax credit on purchase of
automobile by a registered person dealing wholly in taxable items can be claimed to the
extent of 40% of Value Added Tax paid on such purchase, if the automobile is used for
personal (administrative use) purpose.
The term "automobiles" means automobiles having three or more wheels and that runs in
road.

Analysis of Fact in the Question


The item that has been purchased is motorcycle. A motorcycle has two wheels; therefore, it
does not fall in the category of "automobiles" within the meaning of Rule 41 of Value
Added Tax Regulation, 2053.

Conclusion
The contention of tax officer is incorrect. Full VAT credit is allowed in respect of purchase
of motorcycle by a registered person dealing only in taxable items, since motorcycle does
not fall in the category of "automobiles" within the meaning of Rule 41.

79. Explain the following with reference to VAT Act (Dec 2009, 10 Marks)
a) VAT credit on the damaged goods.
Answer
Application for Input Tax Credit & Time Limit for such application
Where there is loss of goods by fire, theft, accident, accidental damages, or terror and
it compels a person to write off the goods (assets) or sale such goods at lower selling
rate, the person shall make an application in writing to respective Inland Revenue
Office along with evidence within 30 days of happening of such events.

Investigation and permission to claim as input tax credit

© The Institute of Chartered Accountants of Nepal 91


CAP II Paper 7 Income Tax and VAT

The tax office shall investigate the matter and finalize the quantum of tax credits to be
allowed. On the basis of such investigation, the tax office may allow the taxpayer to
claim Input Tax Credit of VAT paid on such assets (goods).

Right to claim Input tax Credit without confirmation from Tax Officer
In case the goods is insured, the tax office may allow the taxpayer to claim Input Tax
Credit on such goods to the extent of compensation paid by the Insurance Company,
without any investigation from tax officer.

b) VAT credit facility in case of used goods


Answer
The VAT paid on purchase of used goods (second hand goods) as follows shall not be
eligible for VAT credit:
(i) Purchased from unregistered persons
(ii) Purchased from registered person and where such goods are used for personal
purpose

80. A manufacturer purchases the taxable raw materials such as iron and steel goods. He
manufactures iron and steel goods including metal office furniture and metal fencing.
He also makes agriculture implements such as hoes and rakes. These supplies are
exempt from VAT. Details of his purchases and sales are given below:
Purchase and Overheads Rs.
Iron and Steel goods 100,000
Woods 40,000
Overheads (telephone and electricity) 10,000
Overheads (stationery and furniture) 25,000
Machine Parts 35,000
Total purchase excluding VAT 210,000

Sales
Agriculture Implements 100,000
Steel Appliances 100,000
Office Furniture and Fencing 300,000
Total sales excluding VAT 500,000

You are required to find the ratio between taxable and non-taxable sales and calculate
the value of Input Tax Credit. (Dec 2009, 5 Marks)

© The Institute of Chartered Accountants of Nepal 92


Compilation of Suggested Answers Value Added Tax

Answer
Ratio of Taxable and Non-taxable Sales
Non Taxable Sales
Agriculture Implements 100,000
Taxable Sales
Steel Appliances 100,000
Office Furniture and Fencing 300,000
Total Taxable Sales 400,000

Percentage of Non-taxable sales 20%


(Total Non-taxable sales divided by total sales)

Percentage of Taxable Sales 80%


(Total Taxable sales divided by total sales)

Value of Input Tax credit


All input items are used to produce taxable and non-taxable output, which means, there
shall be proportionate VAT credit on all input items, if VAT has been paid.
Purchase Eligible
cost Input
Particulars VAT paid Note
excluding Tax
VAT credit
Iron and Steel goods 100,000 13,000 10,400
Woods 40,000 5,200 4,160 Proportionate VAT credit,
Overheads (stationery in proportion of taxable
and furniture) 25,000 3,250 2,600 sales to total sales
Machine Parts 35,000 4,550 3,640
Check note 1 below, and
proportionate credit on
Telephone 5,000 650 520 sales ratio
VAT exempt, check note 1
Electricity 5,000 - - below
Eligible VAT credit 21,320

Note 1:
There is combined total of telephone and electricity expense. Telephone expense is VAT
attractive and electricity expense is exempt from VAT. Therefore, it is assumed that the
composition of telephone expense and electricity expense is fifty-fifty.

© The Institute of Chartered Accountants of Nepal 93


CAP II Paper 7 Income Tax and VAT

81. Write short note on Partial Credit on VAT (Dec 2009, 2 Marks, CA Inter)
Or
Explain 'no-tax credit' and 'partial tax credit' under the VAT Act.
(June 2009, 5 Marks)
Or
What are the items in which no input tax credit is allowed under Rule 41 of the VAT
Regulations (Dec 2006, 4 Marks, CA Inter)
Answer
Partial VAT Credit:
Input Tax credit on purchase of automobile by a registered person dealing wholly in
taxable items can be claimed to the extent of 40% of Value Added Tax paid on such
purchase, if the automobile is used for personal (administrative use) purpose.
The term "automobiles" means automobiles having three or more wheels and that runs in
road.

No VAT credit:
In any of the following circumstances, VAT paid on purchases cannot be claimed as input
tax credit:
(a) If the intended use of purchase is to generate non-taxable output,
(b) In case the purchase is made by non-registered person,
(c) In case there is no proper Value Added Tax invoice (other than abbreviated tax
invoice) to substantiate VAT paid except when VAT is paid under import (where import
documents substitute VAT invoice), u/s 8 (2), 12Ka and 15 (3).
(d) In case such purchase is listed as below and the main business of the registered person
is not dealing in the items listed below:
(i) Entertainment expense
(ii) Beverages
(iii)Liquor
(iv) Petroleum (includes diesel, liquefied petroleum gases and petrol)

82. Write about the treatment of VAT paid on purchases made by the following:
(Dec 2007, 3 Marks, CA Inter)
a) Registered Person
Answer
In case a registered person pays VAT on purchases, the VAT may be claimed as
follows:
i. Full VAT credit, if the intended sales of products from purchase is taxable.
ii. Proportionate Tax credit, if the intended sales of products from purchase is both
taxable and non-taxable.

© The Institute of Chartered Accountants of Nepal 94


Compilation of Suggested Answers Value Added Tax

iii. Partial VAT credit, if the purchase is automobiles and is intended to be used for
administrative purpose, when the person deals wholly in taxable items. If he
deals both in taxable and non-taxable items, the applicability is partial credit
cum proportionate credit.
iv. No VAT Credit, if the purchase is entertainment, beverage, liquor, diesel, petrol
or liquefied petroleum gases and the person's main business does not include
transaction of those items.

b) Unregistered Person
Answer
The VAT paid cannot be treated as credit and forms part of cost of purchase.

c) Person producing VAT Exempt goods


Answer
VAT paid is treated as cost of purchase.

83. From the following figures of M/s Light Life Pvt. Ltd., calculate the VAT payable or
receivable for Magh 20X-63. (June 2007, 7 Marks, CA Inter)
The Sales during the month:
Oil Rs. 5,00,000
Rice Rs. 2,00,000
Scraps Rs. 1,00,000

Total import during the month is as follows:


Particulars (Amount in Rs.)
Oil Rice
Invoice Value 4,00,000 2,50,000
Transportation etc. up to boarder 25,000 7,000
Custom Duty and Other Duties 40,000 13,000
Transportation etc. expenses from boarder to godown 20,000 10,000
Local Purchase from a VAT Registered Firm 2,50,000 80,000

Other administrative expenses for which VAT was charged, as follows:


Stationery Items 3,000
Telephone Bills 8,000
Hotel Bills 6,000

During the month, the Company had purchased a Van for transportation of goods for
Rs. 10,00,000. The Van has been financed by a Finance Company and hence the
invoice is in the name of the Finance Company.

© The Institute of Chartered Accountants of Nepal 95


CAP II Paper 7 Income Tax and VAT

Answer
Calculation of Output VAT
Particulars Amount Output VAT Reason
Oil 5,00,000 65,000 Taxable @13%
Rice 2,00,000 - VAT exempt
Scraps 1,00,000 13,000 taxable @13%
Total 78,000

Ratio of Taxable and Non-Taxable Sales


Taxable Sales 600,000 75%
Non-Taxable Sales 200,000 25%

Calculation of eligible Input Tax Credit


Particulars Amount VAT Eligible Reason
Paid Input VAT
Import of Oil (Note 60,450 60,450 Full VAT credit
1 below)
Local purchase of 250,000 32,500 32,500 Full VAT Credit
oil
Stationery Items 3,000 390 292.50
Proportionate credit, to the extent
of 75% of VAT paid
Telephone bill 8,000 1,040 780 Proportionate credit, to the extent
of 75% of VAT paid
Hotel bill 6,000 780 - No VAT Credit, entertainment
expense
Purchase of Van 1,000,000 130,000 97,500 Proportionate VAT credit to the
extent of 75% of paid VAT.
VAT credit is claimed by Lessee
when an asset is purchased under
finance lease arrangement
Total Eligible VAT Credit 191,522.50

Calculation of VAT payable or Receivable


Total Output VAT 78,000
Eligible Input VAT credit 191,522.50
VAT Payable/(Receivable) (113,522.50)

Note:
(1) VAT paid on import of oil at Customs Frontier
Particulars Amount

© The Institute of Chartered Accountants of Nepal 96


Compilation of Suggested Answers Value Added Tax

Invoice Value 400,000


Transportation etc. up to boarder 25,000
Custom Duty and Other Duties 40,000
Taxable Value 465,000
VAT @13% 60,450

(2) There is no applicability of VAT on import and domestic purchase of rice.

84. A dealer manufactured goods under exempt under schedule I worth Rs. 5 Crores out
of goods purchased partly VAT exempt and partly goods liable to VAT. VAT exempt
goods used were Rs. 3 Crore were exempted. He claims are zero and he should get full
credit for VAT on exported goods. Consider his claim to set off the VAT paid by him
on the inputs. (Dec 2004, CA Inter)
Answer
As per Sec. 5 (3) of Value Added Tax Act, 2053, no VAT shall be levied on goods and
services listed in Schedule 1 of the Act. The VAT paid on purchase of goods utilized to
produce VAT exempt output cannot be claimed as credit and such VAT cannot be refunded
as well. Though the VAT exempt goods or services are exported, there shall not be
applicability of VAT.

As such, there cannot be refund of VAT when the exempt goods are exported.

85. What are the provisions relating to the deduction and adjustment of Value Added
Tax? (Dec 2004, CA Inter)
Answer
In case a registered person pays VAT on purchases, the VAT may be claimed as follows:
(a) Full VAT credit, if the intended sales of products from purchase is taxable.
(b) Proportionate Tax credit, if the intended sales of products from purchase is both
taxable and non-taxable.
(c) Partial VAT credit, if the purchase is automobiles and is intended to be used for
administrative purpose, when the person deals wholly in taxable items. If he deals both
in taxable and non-taxable items, the applicability is partial credit cum proportionate
credit.
(d) No VAT Credit, if the purchase is entertainment, beverage, liquor, diesel, petrol or
liquefied petroleum gases and the person's main business does not include transaction
of those items.

VAT credit can be claimed even without VAT invoice in case of import of goods (based on
import documents) and in case of VAT paid under Sec. 8 (2), 12ka and 15 (3) [with

© The Institute of Chartered Accountants of Nepal 97


CAP II Paper 7 Income Tax and VAT

documents proving VAT has been paid). In case of assets purchased under finance lease,
lessee can claim input VAT credit even though the legal title is vested on lessor.

© The Institute of Chartered Accountants of Nepal 98


Compilation of Suggested Answers Value Added Tax

VAT Refund
86. Banmali Traders has following sales & purchase transactions exclusive of VAT for
the following months of 20X-75:

Export (% of
Month Sales (Rs.) Purchase (Rs.)
Total Sales)
Baishakh 500,000 600,000 43%
Jeshtha 825,000 785,000 65%
Ashadh 680,000 675,000 38%
Shrawan 450,000 500,000 40%
Bhadra 300,000 275,000 55%
Ashwin 400,000 500,000 45%
Kartik 600,000 490,000 25%

How much refund can Banmali Traders claim for? (Dec 2018, 5 Marks)
Answer
As per Sec. 24 of Value Added Tax Act, 2052, unless there is export sales exceeding 40% of
total sales during any month, any excess receivable VAT (i.e. when input VAT exceeds
Output VAT) of a month shall be utilized to set off any payable VAT of next four months.
Any remaining excess receivable VAT after such action for consecutive four months can be
refunded by making an application by the concerned taxpayer along with the VAT return of
fourth month or any months thereafter.

In case of export sales exceeding 40 percent of total sales during any month, the taxpayer
may make an application to refund any excess receivable VAT of such month along with the
VAT return of the same month.

Fact of the case


In the given case, the output VAT , input VAT and excess VAT receivable for the respective
month are as follows:
Export Payable/(
Sales Purchase (% of Domestic Output Input Receivable)
Month
(Rs.) (Rs.) Total Sales VAT VAT VAT for
Sales) the month

Baishakh 500,000 600,000 43% 285,000 37,050 78,000 (40,950)


Jeshtha 825,000 785,000 65% 288,750 37,538 102,050 (64,513)
Ashadh 680,000 675,000 38% 421,600 54,808 87,750 (32,942)
Shrawan 450,000 500,000 40% 270,000 35,100 65,000 (29,900)
Bhadra 300,000 275,000 55% 135,000 17,550 35,750 (18,200)

© The Institute of Chartered Accountants of Nepal 99


CAP II Paper 7 Income Tax and VAT

Ashwin 400,000 500,000 45% 220,000 28,600 65,000 (36,400)


Kartik 600,000 490,000 25% 450,000 58,500 63,700 (5,200)

Domestic Sales is computed by using the formula: Total Sales * Percentage of Domestic
Sales
Domestic sales is taxed @13%
Export sales is taxed @0%, therefore, the total VAT collected on domestic sales is the
amount of output VAT.

Conclusion
It is assumed that refund application is filed, as and when the condition for filing of
application is fulfilled.
Month Conclusion
Since, export sales exceed 40% of total sales, M/s Banmali Traders may file
Baishakh
an application for VAT refund along with the VAT return of Baisakh itself.
Since, export sales exceed 40% of total sales, M/s Banmali Traders may file
Jeshtha
an application for VAT refund along with the VAT return of Jestha itself
Since, the export sales do not exceed 40% of total sales during the month, the
excess VAT shall be utilized to set off any payable VAT of Shrawan, Bhadra,
Ashadh Ashwin and Kartik. If there is any excess after that, in the given case, the
whole amount of Rs. 32942 is excess, M/s Banmali may file refund
application along with the VAT return of Kartik.
Since, the export sales do not exceed 40% of total sales during the month, the
excess VAT shall be utilized to set off any payable VAT of Bhadra, Ashwin,
Shrawan Kartik and Mangsir. If there is any excess after such treatment, the
remaining excess amount can be claimed as refund by filing an application
along with VAT return of Mangsir.
Since, export sales exceed 40% of total sales, M/s Banmali Traders may file
Bhadra
an application for VAT refund along with the VAT return of Bhadra itself
Since, export sales exceed 40% of total sales, M/s Banmali Traders may file
Ashwin
an application for VAT refund along with the VAT return of Ashwin itself
Since, the export sales do not exceed 40% of total sales during the month, the
excess VAT shall be utilized to set off any payable VAT of Mangsir, Poush,
Kartik Magh and Falgun. If there is any excess after such treatment, the remaining
excess amount can be claimed as refund by filing an application along with
VAT return of Falgun.

© The Institute of Chartered Accountants of Nepal 100


Compilation of Suggested Answers Value Added Tax

87. Silver Imports and Exports, Kathmandu has the following transactions of garments
and ginger:
Particulars Jestha, 20X- Ashadh, 20X- Shrawan 20X-74
74 (Rs.) 74 (Rs.) (Rs.)
Garments Export 3,00,000 6,00,000 10,00,000
Garments Local Sales 7,00,000 6,00,000 5,00,000
Ginger Export 1,00,000 3,00,000 4,00,000
Ginger Local Sales 4,00,000 5,00,000 6,00,000
Garment Purchase 8,00,000 7,00,000 10,00,000
Ginger Purchase 4,00,000 7,00,000 9,00,000

The company has no previous debit/credit balance in its VAT return. The Company
needs your advice to get refund of the VAT amount for these months.
(Dec 2017, 5 Marks)
Answer
As per Sec. 24 of Value Added Tax Act, 2052, unless there is export sales exceeding 40% of
total sales during any month, any excess receivable VAT (i.e. when input VAT exceeds
Output VAT) of a month shall be utilized to set off any payable VAT of next four months.
Any remaining excess receivable VAT after such action for consecutive four months can be
refunded by making an application by the concerned taxpayer along with the VAT return of
fourth month or any months thereafter.

In case of export sales exceeding 40 percent of total sales during any month, the taxpayer
may make an application to refund any excess receivable VAT of such month along with the
VAT return of the same month.

Fact of the case


In the given case, the output VAT , input VAT and excess VAT receivable for the respective
month are as follows:
Particulars Notes Jestha Ashad Shrawan

Garments Export Taxable @ 0% 300,000 600,000 1,000,000


Garments Local Sales Taxable @13% 700,000 600,000 500,000
Ginger Export VAT Exempt 100,000 300,000 400,000
Ginger Local Sales VAT Exempt 400,000 500,000 600,000
Garment Purchase VAT paid @13% 800,000 700,000 1,000,000
Ginger Purchase VAT Exempt 400,000 700,000 900,000

© The Institute of Chartered Accountants of Nepal 101


CAP II Paper 7 Income Tax and VAT

Output VAT
Particulars Notes Jestha Ashad Shrawan
Garments Export Taxable @ 0% - - -
Garments Local Sales Taxable @13% 91,000 78,000 65,000
Ginger Export VAT Exempt - - -
Ginger Local Sales VAT Exempt - - -
Total Output VAT 91,000 78,000 65,000

Input VAT
Particulars Notes Jestha Ashad Shrawan
Garment Purchase VAT paid @13% 104,000 91,000 130,000
Ginger Purchase VAT Exempt - - -
Total Input VAT 104,000 91,000 130,000
Excess VAT
Particulars Jestha Ashad Shrawan
Total Output VAT 91,000 78,000 65,000
Total Input VAT 104,000 91,000 130,000
Payable/(Receivable) VAT for the month (13,000) (13,000) (65,000)

There is excess Receivable VAT during each of Jestha, Ashad and Shrawan.
The percentage of export sales to total sales during Jestha, Ashad and Shrawan are as
follows:
Percentage of Export Sales
Particulars Jestha Ashad Shrawan
Export Sales
(Sum of Export sales of garment and ginger) 400,000 900,000 1,400,000
Total Sales 1,500,000 2,000,000 2,500,000
Percentage of Export Sales 26.67% 45.00% 56.00%

Conclusion
As the export sales during Jestha 20X-74 is less than 40% of total sales, M/s Silver Imports
and Exports shall utilize the excess VAT of Rs. 13,000 during the month to set off any
payable VAT of Ashad, Shrawan, Bhadra and Ashwin. If the amount remains excess even
after such utilization, M/s Silver Imports & Exports may file a refund application along
with VAT return of Ashwin 20X-74.

The export sales for Ashad and Shrawan exceeds 40% of total sales, therefore, M/s Silver
Imports & Exports can file a refund application to obtain refund of Rs. 13,000 of Ashad
20X-74 along with VAT return of Ashad 20X-74 and Rs. 65,000 of Shrawan 20X-74 along
with VAT return of Shrawan 20X-74.

© The Institute of Chartered Accountants of Nepal 102


Compilation of Suggested Answers Value Added Tax

88. ABC Oil & Ghee industry having factory at Lalitpur, Nepal producing Mustard oil,
Vanaspati ghee and other processed cooking oil in Nepal as given below. The sales of
ABC Oil & Ghee industry to VAT registered and unregistered person are in the
ration of 40% and 60% respectively. The ABC Oil & Ghee industry had following
sales transaction during the month of Falgun, 20X-73:

S.N. Particulars Sales


1. Mustard oil- (Domestic mustard seeds) 6,00,000
2. Mustard oil- (Imported mustard seeds) 4,00,000
3. Vanaspati Ghee- (Domestically produced) 4,00,000
4. Vanaspati Ghee- (Imported) 6,00,000
5. Other processed cooking oil 10,00,000
Total 30,00,000
Whereas XYZ oil industry having factory at Bhaktapur import processed edible
cooking oil in bulk quantities and refill it in small packages before sale. The XYZ oil
industry had following sales transaction during the month of Falgun, 20X-73:
S.N. Particulars Sales
1. Refilled small oil packages (Imported 10,00,000
processed edible cooking oil in bulk quantities)
Total 30,00,000

Required: (June 2017, 5 Marks)


Compute the amount of VAT refund available, if any, to ABC Oil & Ghee industry
and XYZ Oil Industry with reference to the provision of schedule 1 of VAT Act.
Answer
As per Sec. 24 of Value Added Tax Act, 2052, unless there is export sales exceeding 40% of
total sales during any month, any excess receivable VAT (i.e. when input VAT exceeds
Output VAT) of a month shall be utilized to set off any payable VAT of next four months.
Any remaining excess receivable VAT after such action for consecutive four months can be
refunded by making an application by the concerned taxpayer along with the VAT return of
fourth month or any months thereafter.

In case of export sales exceeding 40 percent of total sales during any month, the taxpayer
may make an application to refund any excess receivable VAT of such month along with the
VAT return of the same month.

© The Institute of Chartered Accountants of Nepal 103


CAP II Paper 7 Income Tax and VAT

Conclusion:
In the given case, there is no excess receivable VAT (i.e. no information regarding input
VAT is given), Sec. 24 does not apply. There is no question regarding refund of VAT.

89. Sweets Ltd. which is not registered in VAT purchased 1,000 KG of cheese @
1,130/KG with VAT from Kathmandu Dairy. Subsequently Sweets limited sold 50%
of cheese to individual customers. You, as a professional expert advice, Sweets limited
whether the company is eligible to claim VAT refund and limitation there, if any, as
per the provision of Value Added Tax Act, 2052. (Dec 2016, 5 Marks)
Answer
As per Sec. 24 of Value Added Tax Act, 2052, unless there is export sales exceeding 40% of
total sales during any month, any excess receivable VAT (i.e. when input VAT exceeds
Output VAT) of a month shall be utilized to set off any payable VAT of next four months.
Any remaining excess receivable VAT after such action for consecutive four months can be
refunded by making an application by the concerned taxpayer along with the VAT return of
fourth month or any months thereafter.

In case of export sales exceeding 40 percent of total sales during any month, the taxpayer
may make an application to refund any excess receivable VAT of such month along with the
VAT return of the same month.

Conclusion:
In the given case, there is no excess receivable VAT (i.e. no information regarding input
VAT is given), Sec. 24 does not apply. There is no question regarding refund of VAT.

90. Answer the followings with reference to VAT Act/Rules (June 2017, 4×2.5=10)
a) State the provisions on input tax credit on VAT paid on the lost goods
Answer
Application for Input Tax Credit & Time Limit for such application
In case the loss of asset by fire, theft, accident, accidental damages, terror, or riot
compels a person to write off the goods (assets) or sale it at lower selling rate, the
person shall make an application in writing to respective Inland Revenue Office along
with evidence within 30 days of happening of such events.

Investigation and permission to claim as input tax credit


The tax office shall investigate the matter and finalize the quantum of tax credits to be
allowed. On the basis of such investigation, the tax office may allow the taxpayer to
claim Input Tax Credit of VAT paid on such assets (goods).

Right to claim Input tax Credit without confirmation from Tax Officer

© The Institute of Chartered Accountants of Nepal 104


Compilation of Suggested Answers Value Added Tax

In case the asset is insured, the tax office may allow the taxpayer to claim Input Tax
Credit on such goods to the extent of compensation paid by the Insurance Company,
without any investigation from tax officer.

b) Advise about the threshold of procurement for goods, service, and construction
contract that a public institution requires to procure mandatorily from VAT
registered party.
Answer
Rs. 500,000

c) Can an unregistered person collect VAT?


Answer
The person who is not registered as per the provisions of VAT Act cannot collect VAT,
except in following circumstances:
i. Where the law specifically requires collection of VAT under Reverse Charging
Mechanism
a. In case of import of service pursuant to Sec. 8 (2), when the buyer shall
collect VAT from itself
b. In case of construction of building, apartment, etc. pursuant to Sec. 8 (3),
when the buyer shall collect VAT from itself
ii. Where the law specifically requires collection of VAT by Unregistered persons
a. In case of supply of wood pursuant to Sec. 12Ka, when the national forest,
community forest, private cultivated forest and private forest shall collect VAT
even when they are not registered for VAT
b. In case of taxable supply of goods or services by persons specified in Sec. 15
(3), even if they are not registered for VAT

d) Enumerate the transactions that are VAT attracted.


Answer
Value Added Tax shall be collected in following transactions, except when such goods
or services are listed in Schedule 1 of the Act:
i. Supply of goods or services in Nepal,
ii. Import of goods or services in Nepal, or
iii. Export of goods or services from Nepal.

91. Horizon Pvt. Ltd. located at Chitawan, has following taxable sales and purchases
without VAT in 20X-72. Can the Pvt. Ltd. claim the refund in KartikBhadra return?
If yes, how much? (Dec 2015, 5 Marks)
Month Sales (Rs.) Purchase (Rs.)
Baishakh 400,000 600,000

© The Institute of Chartered Accountants of Nepal 105


CAP II Paper 7 Income Tax and VAT

Jestha 300,000 400,000


Ashadh 500,000 550,000
Shrawan 600,000 700,000
Bhadra 400,000 450,000
Aswin 800,000 700,000
Kartik 700,000 500,000
Answer
As per Sec. 24 of Value Added Tax Act, 2052, unless there is export sales exceeding 40% of
total sales during any month, any excess receivable VAT (i.e. when input VAT exceeds
Output VAT) of a month shall be utilized to set off any payable VAT of next four months.
Any remaining excess receivable VAT after such action for consecutive four months can be
refunded by making an application by the concerned taxpayer along with the VAT return of
fourth month or any months thereafter.

In case of export sales exceeding 40 percent of total sales during any month, the taxpayer
may make an application to refund any excess receivable VAT of such month along with the
VAT return of the same month.

Conclusion
In the given case, there are no export sales; therefore, M/s Horizon Pvt. Ltd. shall have to
utilize excess Receivable VAT of a month to set off payable VAT of next four months.
Payable
Output Input /(Excess) Cumulative
Month Sales Purchase
VAT VAT VAT for Excess
the month
Baishakh 400,000 52,000 600,000 78,000 (26,000) (26,000)
Jestha 300,000 39,000 400,000 52,000 (13,000) (39,000)
Ashadh 500,000 65,000 550,000 71,500 (6,500) (45,500)
Shrawan 600,000 78,000 700,000 91,000 (13,000) (58,500)
Bhadra 400,000 52,000 450,000 58,500 (6,500) (65,000)
Aswin 800,000 104,000 700,000 91,000 13,000
Kartik 700,000 91,000 500,000 65,000 26,000

The excess VAT of Baisakh will be utilized to set off VAT payable, if any, of Jestha to
Bhadra. Since, there is no any payable VAT for such months; the company may file a
refund application to obtain refund of Rs, 26,000 belonging to Baisakh along with VAT
return of Bhadra 20X-72.
Similarly, excess VAT of Jestha will be utilized to set off VAT payable, if any, of Ashad to
Ashwin. Since, there is no any payable VAT for Ashad, Shrawan and Bhadra, but there is
payable VAT for Ashwin. The excess VAT of Rs. 13,000 will be utilized to set off payable

© The Institute of Chartered Accountants of Nepal 106


Compilation of Suggested Answers Value Added Tax

VAT of Ashwin of Rs. 13,000 resulting into nil excess in Jestha. Excess of Jestha cannot be
refunded if the company applied refund for the excess of Baisakh along with return of
Bhadra 20X-72.

92. Mr. John entered to Nepal via Birgunj with his family on 1st Aswin 20X-72 and visited
so many places in Nepal. The detail of few expenses incurred in Nepal by Mr. John &
his family members are as follows:
a) Amount paid to tour operator for family tour package amounted to Rs. 339,000
including Rs. 39,000 applicable VAT.
b) Mr. John purchase one laptop costing Rs. 100,000 and paid applicable VAT on it.
c) Mr. John Paid to Hotel in Kathmandu for cost of lodging, foods & other services
Rs. 226,000 including Rs. 26,000 VAT.
d) Mr. John with his family visited orphanage home in Kathmandu and donated a
fridge costing Rs. 50,000 and paid applicable VAT on it.
At the time of departure from Tribhuvan International Airport, Kathmandu, Mr.
John submitted all VAT invoices to the counter of IRD and Mr. John is carrying the
laptop purchased in Nepal along with all his other belongings.

Advise Mr. John how much VAT refund he will get. (Dec 2015, 5 Marks)
Answer
As per Sec. 25Ka of Value Added Tax Act, 2052, in case all the following conditions are
satisfied, a foreign tourist is entitled to refund of Value Added Tax paid in Nepal:
(a) The foreign tourist visiting in Nepal shall leave Nepal via air route,
(b) The value added tax is refunded only in respect of purchase of goods (and not services),
(c) The foreign tourist shall accompany such goods with himself while leaving Nepal, i.e.
goods cannot be gifted in Nepal and it shall be taken away by the tourist; and
(d) The cost of such goods shall be more than Rs. 25,000.

Service Fee on such Refund


The revenue authority shall deduct 3% of refundable amount as service charge.

Conclusion
In the given case, Mr. John leaves Nepal via air route. Therefore, the applicability of VAT
refund is given as below:
Particulars Applicability Reason
of refund
Amount paid to tour No Since, the procured item is not goods
operator for family tour
package
Purchase one laptop, Mr. Yes Since, the cost of goods (i.e. laptop) exceeds

© The Institute of Chartered Accountants of Nepal 107


CAP II Paper 7 Income Tax and VAT

John is carrying the laptop Rs. 25,000 and Mr. John is accompanying
purchased in Nepal along laptop with him while leaving Nepal
with all his other belongings
Paid to Hotel in Kathmandu No Since, the procured item is not goods
for cost of lodging, foods &
other services
Donated a fridge to an No Though the cost of goods (i.e. fridge)
orphanage home in exceeds Rs. 25,000; however, Mr. John is
Kathmandu not taking away goods with him

93. Healthy Oil Industries is manufacturing Mustard Oil. During shortage, the industry
also, imports oil and sell it to its customers to maintain its market. The VAT accounts
of Healthy Oil Industries. Showed the following data for fiscal year 2069/70.

Particulars Trading of Mustard Oil Manufacturing


(import purchase and sales) of Mustard Oil
A. VAT collected on sales
i From VAT registered party 17,00,000 18,00,000

ii From Non-VAT registered 6,00,000 1,90,000


party
B. VAT paid on purchase 15,00,000 14,00,000

Calculate the VAT amount which Healthy Oil Industries is facilitated to get refund
from Inland Revenue Department. (Dec 2013, 3 Marks)
Answer
As per Sec. 24 of Value Added Tax Act, 2052, unless there is export sales exceeding 40% of
total sales during any month, any excess receivable VAT (i.e. when input VAT exceeds
Output VAT) of a month shall be utilized to set off any payable VAT of next four months.
Any remaining excess receivable VAT after such action for consecutive four months can be
refunded by making an application by the concerned taxpayer along with the VAT return of
fourth month or any months thereafter.

In case of export sales exceeding 40 percent of total sales during any month, the taxpayer
may make an application to refund any excess receivable VAT of such month along with the
VAT return of the same month.

Conclusion:
In the given case, there is no excess receivable VAT (i.e. output VAT exceeds input VAT),
Sec. 24 does not apply. There is no question regarding refund of VAT.

© The Institute of Chartered Accountants of Nepal 108


Compilation of Suggested Answers Value Added Tax

94. Swasthya Flour Mill Pvt. Ltd. is a Maida producing industry. It is a complete
manufacturing unit, and does not involve in local/import purchase of Maida. For
fiscal year 20X-69/X-70, it had following VAT transactions:
Particulars VAT Amount (Rs.)
A. VAT collected on sales
i. From VAT registered party 15,00,000
ii. From Non-VAT registered party 2,00,000
B. VAT paid on purchase 13,00,000
What amount of VAT refunds is Swasthya Flour Mill Pvt. Ltd. facilitated?
(Dec 2013, 3 Marks)
Answer
As per Sec. 24 of Value Added Tax Act, 2052, unless there is export sales exceeding 40% of
total sales during any month, any excess receivable VAT (i.e. when input VAT exceeds
Output VAT) of a month shall be utilized to set off any payable VAT of next four months.
Any remaining excess receivable VAT after such action for consecutive four months can be
refunded by making an application by the concerned taxpayer along with the VAT return of
fourth month or any months thereafter.

In case of export sales exceeding 40 percent of total sales during any month, the taxpayer
may make an application to refund any excess receivable VAT of such month along with the
VAT return of the same month.

Conclusion:
In the given case, there is no excess receivable VAT (i.e. output VAT exceeds input VAT),
Sec. 24 does not apply. There is no question regarding refund of VAT.

95. State the provisions on Value Added Tax (VAT) refund under Value Added Tax Act,
2052. (Dec 2012& Dec 2011, 5 Marks)
Or
State the provisions regarding the refund of VAT excess paid.
(Dec 2008, 3 Marks, CA Inter)
Answer
As per Sec. 24 of Value Added Tax Act, 2052, unless there is export sales exceeding 40% of
total sales during any month, any excess receivable VAT (i.e. when input VAT exceeds
Output VAT) of a month shall be utilized to set off any payable VAT of next four months.
Any remaining excess receivable VAT after such action for consecutive four months can be
refunded by making an application by the concerned taxpayer along with the VAT return of
fourth month or any months thereafter.

© The Institute of Chartered Accountants of Nepal 109


CAP II Paper 7 Income Tax and VAT

In case of export sales exceeding 40 percent of total sales during any month, the taxpayer
may make an application to refund any excess receivable VAT of such month along with the
VAT return of the same month.

Where a person makes an application for refund of excess VAT, if the VAT refund is due to
the reason of export sales exceeding 40% of total sales during any months, VAT shall be
refunded within 30 days of application and within 60 days in all other cases.

In case of failure to refund within such specified days, interest shall be paid @15% per
annum for the period beginning from the date of expiry of due date and ending on the day
of refund.

96. Mr. Cristiano Messi, a foreign tourist, visited Nepal for a month and is now returning
to his home country Japan. He has purchased many goods in Nepal and paid VAT on
such purchase. He seeks your advice on refund of VAT on paid by him on purchase
and expenses/costs associated with such refund. (June 2012, 5 Marks)
Answer
As per Sec. 25Ka of Value Added Tax Act, 2052, in case all the following conditions are
satisfied, a foreign tourist is entitled to refund of Value Added Tax paid in Nepal:
(a) The foreign tourist visiting in Nepal shall leave Nepal via air route,
(b) The value added tax is refunded only in respect of purchase of goods (and not services),
(c) The foreign tourist shall accompany such goods with himself while leaving Nepal, i.e.
goods cannot be gifted in Nepal and it shall be taken away by the tourist; and
(d) The cost of such goods shall be more than Rs. 25,000.

Service Fee on such Refund


The revenue authority shall deduct 3% of refundable amount as service charge.

Conclusion:
In the given case, if all the above conditions are satisfied, Mr. Cristiano Messi is entitled to
refund of VAT on such goods. The cost for Mr. Messi is 3% of refundable amount.

97. Ginger Cellular Ltd., a cellular mobile Phone Set producing domestic enterprise, is
engaged in producing and selling of cellular mobile phone sets. As a tax expert,
Ginger Ltd. seeks your advice on the refund of VAT paid on purchase of its raw
materials. Give your opinion on this. (Dec 2010, 5 Marks)
Answer
As per Sec. 24 of Value Added Tax Act, 2052, unless there is export sales exceeding 40% of
total sales during any month, any excess receivable VAT (i.e. when input VAT exceeds
Output VAT) of a month shall be utilized to set off any payable VAT of next four months.

© The Institute of Chartered Accountants of Nepal 110


Compilation of Suggested Answers Value Added Tax

Any remaining excess receivable VAT after such action for consecutive four months can be
refunded by making an application by the concerned taxpayer along with the VAT return of
fourth month or any months thereafter.

In case of export sales exceeding 40 percent of total sales during any month, the taxpayer
may make an application to refund any excess receivable VAT of such month along with the
VAT return of the same month.

The same provision is applicable to M/s Ginger Cellular Ltd.

98. A dealer manufactured goods worth Rs. 10 million which were exempt under schedule
I of VAT Act out of the goods purchased partly VAT exempt and partly goods liable
to VAT. VAT exempt goods amounting to Rs. 7 million were used for the production
of goods which were exported. The Dealer claims that all the exports were zero rated
and he should get full credit for VAT on exempted goods. Consider his claim and to
set off the VAT paid by him on the inputs. (Dec 2010, 4 Marks)
Answer
As per Sec. 5 (3) of Value Added Tax Act, 2052, no VAT shall be levied on items listed in
Schedule 1 of the Act. Value Added Tax paid in purchases used to produce such non-
taxable output can neither be claimed as input tax credit nor be refunded.

There shall not be applicability of VAT on domestic or export sales of such items, which
are listed in Schedule1 of the Act.

Therefore, the dealer cannot claim input tax credit or refund of VAT in respect of VAT paid
on input that are used to produce non-taxable output.

© The Institute of Chartered Accountants of Nepal 111


CAP II Paper 7 Income Tax and VAT

99. What are the conditions for refund of VAT to the foreign tourist? Answer citing the
provisions of the Act. (June 2010& Dec 2009, 5 Marks)
Answer
As per Sec. 25Ka of Value Added Tax Act, 2052, in case all the following conditions are
satisfied, a foreign tourist is entitled to refund of Value Added Tax paid in Nepal:
(a) The foreign tourist visiting in Nepal shall leave Nepal via air route,
(b) The value added tax is refunded only in respect of purchase of goods (and not services),
(c) The foreign tourist shall accompany such goods with himself while leaving Nepal, i.e.
goods cannot be gifted in Nepal and it shall be taken away by the tourist; and
(d) The cost of such goods shall be more than Rs. 25,000.

Service Fee on such Refund


The revenue authority shall deduct 3% of refundable amount as service charge.

100. ABC Export House has a credit of Rs. 200,000 in the first month representing 30% of
its total sales during that month. However, it has a credit of Rs. 600,000 representing
60% of its total sales in the following month.

How does ABC Export House claim refund if it does not always have exports over 50
40% of total sales? (June 2010, 5 Marks)
Answer
As per Sec. 24 of Value Added Tax Act, 2052, unless there is export sales exceeding 40% of
total sales during any month, any excess receivable VAT (i.e. when input VAT exceeds
Output VAT) of a month shall be utilized to set off any payable VAT of next four months.
Any remaining excess receivable VAT after such action for consecutive four months can be
refunded by making an application by the concerned taxpayer along with the VAT return of
fourth month or any months thereafter.

In case of export sales exceeding 40 percent of total sales during any month, the taxpayer
may make an application to refund any excess receivable VAT of such month along with the
VAT return of the same month.

Conclusion
In the given case, the person's total export sales do not exceed 40% of total sales during
first month, therefore, the excess receivable VAT of such month shall be utilized to set off
any payable VAT of next four months. Any remaining excess receivable VAT after such
action for consecutive four months can be refunded by making an application by the
concerned taxpayer along with the VAT return of fourth month or any months thereafter.

© The Institute of Chartered Accountants of Nepal 112


Compilation of Suggested Answers Value Added Tax

Further, the person's export sales exceed 40% of total sales in the second month; therefore,
he can make an application to claim refund of excess receivable VAT of same month along
with VAT return of same month.

101. A Foreign diplomat to Nepal (recognized by foreign ministry in Nepal) during his stay
of last 5 years in Nepal had purchased goods. He has various invoices / Tax Invoices
and seeks your advice whether he can get refund of VAT on purchases made by him
during the stay. Being the tax consultant put forward your advises as per the VAT
Act & Rules. Is there any form designed by IRD for such purpose?
(Dec 2009, 3 Marks, CA Inter)
Answer
As per Sec. 25, a foreign diplomat can avail refund facility on VAT paid by him/her on
consumption in Nepal, if all the following conditions are satisfied:
(a) The person is accredited as diplomatic personby Ministry of Foreign Affairs,
(b) The country represented by the diplomatic person provides refund facility to Nepali
diplomats in their country, i.e. refund is availed on the basis of reciprocity,
(c) The diplomat makes an application within 3 years of the purchase, and
(d) The purchase, i.e. individual invoice, belonging to refund application shall not be less
than Rs. 10,000.
In the given case, the foreign diplomat can make an application for refund, if he satisfies
all the above conditions.

Form designed by IRD


Yes, a form is prescribed by Government of Nepal in Schedule 17 of Value Added Tax
Regulations, 2053.

Further issues of refund:


Such refunds should normally be made within 30 days of application. Amount exceeding
Rs. 20,000 shall be refunded by way of depositing in beneficiary's bank account only.

102. Mr. Shyam, a businessman, submits his VAT return for the month of Falgun
according to which total sales for the month was NRs. 1,000,000 and purchase was
NRs. 600,000. Out of the total sales, NRs. 650,000 was export to Tibet (Non L/C). He
had no previous debit/credit balance in his VAT return. He claims the refund of the
excess tax paid on purchase on 25th of Chaitra.
State the time and amount of claim he is entitled to. Also mention the documents (if
any) he requires to claim the refund. (June 2009, 5 Marks)
Answer

© The Institute of Chartered Accountants of Nepal 113


CAP II Paper 7 Income Tax and VAT

As per Sec. 24 (3) of Value Added Tax Act, 2052, a person may make an application for
refund of VAT of any month with the VAT return of same month in case the person's export
sales is more than 40% of total sales during the same month.
In the given question, the percentage of export sales is 65%, which is more than 40% of
total sales. Therefore, Mr. Shyam may make an application for VAT refund along with the
VAT return of Falgun.
The documents required to be submitted are as follows:
(a) Copy of agreement between buyer & seller.
(b) Copy of Pragyapan Patra of export.
(c) Copy of Pragyapan Patra of import.
(d) Copy of Payment Certificate

© The Institute of Chartered Accountants of Nepal 114


Compilation of Suggested Answers Value Added Tax

VAT Returns and Payment of VAT


103. PQR Pvt. Ltd., a company registered under VAT and submitting the return on
monthly basis. It has submitted the tax return on Jestha 25, 20X-72 for the month of
Baisakh 20X-72. Also, the amount of tax was Rs. 100,000 and it has paid via 'Account
Payee' Cheque dated Jestha 25, 20X-72. The Cheque got cleared on Jestha 26, 20X-72.
Is it considered as payment of tax within due date as pet Act/Rules?

Will your answer be the same if the company paid via 'Account Payee' Cheque
guaranteed by bank (good for payment Cheque) and the Cheque got cleared on
Jestha 27, 20X-72? (Dec 2015, 5 Marks)
Answer
As per Sec. 19 (7) of Value Added Tax Act, 2052, in case a person makes payment of VAT
through electronic means or account payee cheque, the date when the pay order was issues
in respect of use of electronic means and the date when cheque was submitted to revenue
authority in respect of good for payment cheque shall be considered as the date of
payment.

In the given case, since the payment was not made through electronic means or good for
payment cheque, the date of payment of VAT is the date when the amount is cleared by the
respective bank, i.e. Jestha 26, 20X-72.

In the contrary, the cheque was marked "Good for payment", i.e. guaranteed by bank, the
date of payment would be the date when the cheque was deposited with revenue authority.
That means, although the cheque was cleared on 27th Jestha 20X-72, the date of payment
by PQR Pvt. Ltd. is 25th Jestha 20X-72.

104. MNP Pvt. Ltd. is a company voluntarily registered to VAT on 1stShrawan, 2067.
Details of his transactions are as follows:

Fiscal Year (FY) 20X-68/X-69 Amount (Rs.)


Shrawan- Kartik 550,000
Mangsir- Falgun 500,000
Chaitra- Ashadh 930,000
1,980,000
Fiscal Year 20X-69/X-70
Shrawan- Kartik 400,000
Mangsir- Falgun 680,000
Chaitra- Ashadh 590,000
1,670,000

© The Institute of Chartered Accountants of Nepal 115


CAP II Paper 7 Income Tax and VAT

The company is submitting VAT return on four monthly bases, since heit is
voluntarily registered and the turnover of any F/Y doesn't exceed Rs. 20 lakhs, it's
claiming that it need not have to submit its VAT return on monthly basis. Comment
on the company's contention with reference to Value Added Tax Act/Rules.
(June 2014, 5 Marks)
Answer
As per Sec. 11 (1) (Cha), the VAT registration of a person who is registered voluntarily is
cancelled automatically by revenue authority if the turnover of such person in last twelve
months does not exceed the threshold of Rs. 50 lakhs in case he is dealing only in goods
and Rs. 20 lakhs in all other cases. If the person wishes to retain the registration status, an
application along with the proof of submission of VAT returns in twelfth month out of last
twelve months shall be filed.
There is no provision in VAT law barring a person to apply for cancellation of VAT
registration.
Therefore, if M/s MNP Pvt. Ltd. wishes to cancel registration; it can do so by filing an
application in this regard.

105. Explain about the “Tax Periods” under the Value Added Tax Act/Rules
(June 2013, 5 Marks)
Answer
The tax period, as per VAT law, are as follows:
i. Monthly Tax Period (Every Nepalese Calendar Month as Tax Period)
Every registered person, other than those who opt for bimonthly or tri-annual tax
period as per the regulation shall follow monthly tax period for VAT accounting.
ii. Tri-annual Tax period
The following registered persons are eligible to choose trimester tax period:
a. Brick Industry
b. Newspaper publication house
c. Newspaper and electronic publication and transmission house
d. Hotel
e. Tourism Business
f. Cinema Hall
g. Transport business\
These businesses may make an application in front of tax officer to keep their VAT
accounting on trimester basis. Where such application is filed, tax officer must cause to
use trimester tax period for VAT purpose.
It is upon the tax officer to decide the beginning and end of trimester tax period.
iii. Different tax period
In case of a registered person that has maintained its accounts using software cannot
generate the required report in Nepalese Calendar system, it may file an application to

© The Institute of Chartered Accountants of Nepal 116


Compilation of Suggested Answers Value Added Tax

Tax Officer to adopt different tax period with same tax period. If the Tax Office thinks it
appropriate, the person may be granted a different tax period.
iv. First Tax period
First tax period of a registered person shall be period of day of registration to end of
concerned tax period.
Readers should note that this is not a separate tax period, but a provision to clarify that
the tax period may begin at the middle of any month but shall always end in the end of
month. This also clarifies that the tax periods are uniform.
v. No-tax period
In case where any registered person applying cancellation of registration is not
prompted with the decision of tax officer within three months from the date of filing of
application, the person is not required to file any tax return after the elapse of such
three months.

106. ABC Ltd, a foreign party got a contract from an organization in Nepal. Before the
start of the work, they have registered themselves with VAT. As a tax professional,
they enquire you on the provision for payment of VAT as per the VAT Act. Advice
ABC Ltd. (Dec 2012,4 Marks)
Answer
Provision regarding payment of VAT (Section 19)
(a) A taxpayer shall pay tax for each tax period within twenty five days of the end of the tax
period.
(b) If a taxpayer does not pay the tax within the time limit specified under (a), an extra
charge of ten percent per annum shall be imposed on the tax due.
(c) If a taxpayer applies to the Director General for the exemption of the additional
charges as above stating the reason that the failure to make a timely payment was
caused by circumstances beyond the taxpayer's control, the Director General may, if he
finds the reason reasonable, exempt such charges.
(d) The additional charges and the interest pursuant to Section 26 shall be charged from
the date on which the tax first became due.
(e) If a tax officer makes tax assessment under section 20 and finds that the amount of tax
to be collected from the tax payer in a tax period is less than the amount he is entitled
for refund, then extra charge and interest shall not collected in that tax period.
(f) Tax can be paid through the electronic means or Good for Payment Cheque issued by
the Bank. In such a case, the day tax office receives the payment through electronic
means or good for payment cheque shall be deemed as the day when the tax is
collected.
(g) Interest shall not be collected on Interest, Extra charge and penalties.

© The Institute of Chartered Accountants of Nepal 117


CAP II Paper 7 Income Tax and VAT

Accounting Records & Invoices


107. Finance Manager of ABC Hardware is worrying about the price adjustment with its
supplier. He had heard that he can do the price adjustment through the use of
Debit/Credit Note. You are requested to advise him regarding the content of
Debit/Credit Note as per VAT Rules.
(Dec 2014, 5 Marks)
Or
What are Debit / Credit Note? (Dec 2012, 4 Marks)
Or
Discuss the provision of Debit and Credit Note in VAT Rules. (Dec 2010, 5 Marks)
Answer
A person shall issue debit note or credit note to change the value of goods in issued
invoice.
Credit Note is issued to credit the debit balance of a debtor, i.e. the person issuing invoice
issues a credit note and vice-versa. Credit note is a crucial matter in tax accounting as
many countries has policy of using pre-printed credit notes issued by government. But,
such rule has not been in effect in Nepal.
Debit or credit note include following information:
a. Serial Number of the debit or credit note,
b. Date of issue,
c. Name, address and PAN of the supplier,
d. Recipient's name, address, and PAN if a registered person,
e. Serial number and date of the tax invoice concerned,
f. Particulars of the goods or services and reason of issuing to credit or debit,
g. Amount credited or debited,
h. Tax amount credited or debited.

108. What are the records to be maintained by a registered person dealing in used or
second hand materials? How the tax is assessed in such case? Answer with reference
to the Value Added Tax Rules, 2053. (June 2013, 5Marks)
OR
What are the records to be maintained by registered dealer dealing in used or second
hand materials? (Dec 2010, 5 Marks)
Answer
The Dealers of USED GOODS shall keep separate records for each deal of used goods. In
case a registered person dealing used goods is found not to have satisfactorily maintained
the records, Tax Officer may impose VAT on the total selling price of the goods sold by
such taxpayer, and the tax officer may issue a written order requiring them to pay such tax
along with the next tax return.

© The Institute of Chartered Accountants of Nepal 118


Compilation of Suggested Answers Value Added Tax

The following are the matters to be maintained in Sales and Purchase Book of Dealer of
Used Goods:
(a) In relation to Purchase
i. Date of purchase
ii. Information as to the complete details of the goods
iii. Purchase price excluding VAT
iv. Tax Rate
v. Tax Amount
vi. Total Amount paid
(b) In relation to Sales
i. Date of Sales
ii. Selling Price excluding VAT
iii. Difference between Selling Price & Purchase Price
iv. Rate of Tax
v. Tax Amount
vi. Total Amount Received

109. What are the records that are to be maintained by the tax payer as per rule 23 of
VAT rules, 2053? (Dec 2012, 5 Marks)
OR
What are the types of accounts and records to be kept by a taxpayer under VAT Act,
2052? What are their contents? (June 2010, 5 Marks)
Answer
A taxpayer has to maintain the following records:
(a) VAT Account
The format of VAT account prescribed in Schedule 7 of Regulation. It is summary of
purchase or import and sale or export including input VAT and Output VAT.
(b) Purchase book
According to Form prescribed in Schedule 8 of Regulation, registered person and
unregistered person shall record all transaction of purchase and import in this book.
This record is in mandatory use. In Purchase book, purchases on which input tax
credits (even overheads and consumables) is claimed shall be recorded as purchase.
Goods or services without VAT or with VAT, for which input tax credit is not allowed,
are recorded as exempted purchase.
(c) Sales book
According to Form prescribed in Schedule 9 of Regulation, registered person and
unregistered person shall record all transaction of sale and export in this book. This
record is in mandatory use.

© The Institute of Chartered Accountants of Nepal 119


CAP II Paper 7 Income Tax and VAT

(d) Other Records


Apart from above mentioned statutory VAT books, other documents (or computerized
records if allowed by Tax Officer) to be kept by a person are as follows as per Rule 23:
(i) Records regarding transactions, cash, etc
(ii) Copies of Tax Invoices and Abbreviated Tax Invoices
(iii)Tax Invoices received on purchases
(iv) Documents regarding import and export
(v) Records and copies of Debit or Credit Notes
(e) Register of free or sample goods as per Rule 24
A registered person shall maintain separate records for any received or distributed free
goods or samples. The Rule is silence in regards of records for free goods or sample
received or distributed by an unregistered person.

110. Explain the term “Abbreviated Tax Invoice” as per Value Added Tax Act, 2052.
(June 2012, 5 Marks)
Answer
A retailer may issue abbreviated tax invoice after an expressed written permission from tax
officer. An application must be filed by the retailer to obtain prior approval of tax officer.
Tax officer has the right to accept or deny such permission.

The retailer cannot issue an abbreviated tax invoice in respect of sales exceeding Rs.
10,000 including VAT and other duties. It is the duty of the retailer to issue normal VAT
invoice, if the buyer asks for the normal tax invoice.

The buyer cannot claim input tax credit through the use of Abbreviated Tax Invoice.

The retailer may not show the name of each goods or rate or VAT amount in the issued
abbreviated tax invoice. However, the particulars of individual goods shall be explicitly
mentioned.

The person issuing Abbreviated Tax Invoice shall maintain following records, in the event
of failure of which the permission to issue such invoice may be scrapped:
(a) Copy of original invoice,
(b) In case of transaction through maintaining copy of till roll, the total of the transaction
amount, and
(c) Record of each transaction including VAT.

© The Institute of Chartered Accountants of Nepal 120


Compilation of Suggested Answers Value Added Tax

111. List occasions when purchase and sales books are required to be certified by the VAT
authorities (June 2009, 3 Marks)
Answer
The sales and purchase book shall be certified by VAT authorities in following occasions:
(a) At the time when the registered person submits an application to the office for the
certification;
(b) During the period of tax audit or examination, or
(c) At the time of inspection.

112. What is the minimum information that must be accompanied in the VAT invoice
issued? What consequences will follow in the case of the required particulars is not
properly given while issuing invoice? (June 2009, 5 Marks, CA Inter)
Answer
As per rule 17 of the Value Added Regulation, 2053 all the sellers conducting transactions
of taxable goods or service shall issue a tax invoice stating the name, address, permanent
account number, date, serial number and particulars of the goods (model, brand,
Harmonized number) .
In the case of the tax invoice issued without giving full description, tax shall be collected on
the basis of prevailing highest market price. (Rule 17(4) of the Value Added Tax Rules,
2053 as amended by seventh amendment 2062/04/01).
Similarly, in the case of purchaser also the input tax credit under rules 39 shall not be
granted.

© The Institute of Chartered Accountants of Nepal 121


CAP II Paper 7 Income Tax and VAT

Tax Assessment
113. Explain the provisions of Tax Assessment by Tax Officers (Dec 2009, 5 Marks)
(Dec 2009, 3 Marks, CA Inter)
Answer
Conditions for VAT assessment by Tax Officer:
Tax Officer can make tax assessment under any of the following grounds and conditions:
a) In case VAT return is not submitted within the time limit;
b) In case of filing of an incomplete or erroneous return;
c) In case of filing of a fraudulent return;
d) In case the Tax Officer has a reason to believe that the amount of tax is understated or
otherwise incorrect;
e) In case the Tax Officer has a reason to believe that the taxpayer has caused under-
invoicing;
f) In case of supply to group companies through under-invoicing;
g) In case person requiring registration operates business without registration;
h) In case of sales without issuing invoices;
i) In case an unregistered person collects the tax;
j) In case a person does not deposit VAT to be levied u/s 8 (2) or 8 (3) under the principle
of reverse charging; or
k) In case a person does not levy VAT on goods that are left to be used for taxable
transaction for which input tax credit was enjoyed as per Sec. 17 (4)
Assessment by tax officer is risk-based sampling based on above conditions

Bases of the assessment


The Tax Officer may make such tax assessment on one or more of the following bases:
a) Proof of transactions;
b) A tax audit report on transactions submitted by the concerned Tax Officer; and
c) Tax paid on a similar transaction by another person.

Burden of proof
The burden of proof lies on Tax Officer for reasons of assessment.

Time limit for assessment


Assessment by Tax Officer needs to be finalized within four years from the date of filing of
tax return. If the stipulated time expires and the tax officer fails to assess tax, the return so
filed by taxpayer shall be considered to be true and valid.
But, in case of fraudulent return, reassessment tenure is unlimited.

© The Institute of Chartered Accountants of Nepal 122


Compilation of Suggested Answers Value Added Tax

Show cause Notice


The tax officer shall issue a Preliminary Assessment Order (PAO) after evaluating the
basis as described above and determining additional price, if any and send the notice to the
taxpayer for it to be able to defend the allegations made by tax officer providing a time
limit of 15 days from the receipt of such notice.
In case the Tax Officer is satisfied with the reply on PAO, the points raised by the tax
officer shall be omitted and the tax officer shall issue a Final Assessment order considering
the replies of taxpayer on PAO considered unsatisfactory.

Power of Inland Revenue Department in relation to Assessment pending or completed by


Tax Officer
In case where, before the assessment of tax is complete, it appears from the information
received by Director General any action relating to the assessment of tax is about to be
taken in irregular way or has been irregular, the Director General may by executing the
memorandum (Parcha Khada gari) clearly setting out the reasons, give direction to the
concerned tax officer to make reassessment of tax or order any other tax officer to do that
act.

In case where it appears that the tax liability has decreased because of the tax assessment
made by the tax officer recklessly or with the malafide intention, the Director General may
order amendment in such tax assessment order within 4 years after the date of initial tax
assessment.

© The Institute of Chartered Accountants of Nepal 123


CAP II Paper 7 Income Tax and VAT

114. Healthy Bottlers Pvt. Ltd. is a manufacturer of glass bottle packaged beverages. The
Company supplies packaged bottles of beverage, billing the selling price of the
beverage. In addition to that, it gets certain amount from the dealers as deposit for the
glass bottles supplied to them. When the dealer returns the empty bottles, the amount
of deposit is refunded. A tax officer, during his inspection of the company, found that
bottles worth NRs. 200,000 were not in physical stock. He made tax assessment under
section 20 of the VAT Act and treated the shortage of bottles as sold by the company.
The company had proved that the bottles were in stock with various dealers in due
course of return. However, the tax officer did not accept the contention and charged
tax and penalty on the deemed sales of the bottles. Critically examine the contention
of the tax officer.
(June 2009, 5 Marks)
Answer
Healthy Bottlers Pvt. Ltd. is selling beverages only. When beverage is consumed by
consumer, the empty bottle is returned to the producer. Thus, the supply of empty bottles on
deposit does not constitute transaction as per VAT Act, 2052.

Section 2 (g) of the Act, defined the term supply which means act of selling, exchanging
and delivering any goods or services, or the act of granting permission thereto or of
contract thereof for a consideration. In this case the supply of bottle is not for
consideration and so it is not treated as supply for transaction purpose.

It was held in Pepsi Cola Nepal Pvt. Ltd. vs. Inland Revenue Department; the Revenue
Tribunal has interpreted that the bottles would be treated as sold in case of breakage or
lost by the dealer and when the producer charging for such breakage or shortage to the
dealer. The Tribunal has pointed out Section 12 (8) of the Act, according to which deposit
amount could be treated as sales when it is adjusted against consideration receivable for
supply of goods or services.

That is why; the shortage of bottles at factory premises cannot be treated as sold in case it
is proved that the bottles ate lying with the dealers.

© The Institute of Chartered Accountants of Nepal 124


Compilation of Suggested Answers Value Added Tax

Fees, Interest, Penalties and Circumstance beyond Control


115. As per the provisions of Value Added Tax Act, the tax officer has to complete the
assessment within 4 years from date of submission of VAT return. A VAT Registrant
destroyed the VAT records on expiry of above-mentioned period. On inspection, the
tax officer imposed a fine under section 29 stating the reason that the company did
not produced the record for his inspection. The company wants to apply for
administrative review and seeks your advice in this regard. Please give your advice.
(Dec 2015, 5 Marks)
Answer
Section 20(4) of Value Added Tax Act, 2052 provides for the assessment by tax officer
within the prescribed time limit of four years from the date of filing of the tax return.
However, the taxpayer shall maintain VAT records for six years. This is the responsibility
of the taxpayer to maintain the records for the period mentioned in the Act.

The taxpayer in the question destroyed the records before the time limit the tax officer has
rightly imposed penalty under the Act. The Tax officer is correct in his action and there is
no meaning in filing for administrative review.

116. Mr. Krishna K.C is a proprietor of KKC & Co. which is a VAT registered firm. Due
to the occurrence of destructive Earthquake on Baisakh 12, 2072, Mr. K.C could not
deposit VAT amount of Rs. 10 lakh collected by the goods sold by his firm for the
month of Chaitra 2071.
IRD vide public notice has extended the VAT submission date from 2072/01/25 to
2072/02/07. Finally, Mr. K.C has been successful to deposit the said VAT amount on
Jestha 08, 2072.
The Tax officer intends to levy additional fee of 10% p.a. due to failure to pay VAT
even within the extended time of 2072/02/07. Since the failure to pay VAT was due to
the circumstances beyond his control, suggest Mr. K.C to obtain waiver from paying
such extra charges on total VAT amount due. (July 2015, 5 Marks)
Answer
Additional fees @ 10% p.a. on payable VAT obligation, as per Sec. 19 (2), can be waived
by Director General in the event of circumstances beyond control.

In case there is circumstances beyond control as a result of fire, earthquake or similar


other natural disaster, a period of 30 days from the date of happening of such event is
treated as circumstances beyond control.

In order to obtain waiver, the concerned taxpayer shall file an application for waiver
within 30 days of the due date of payment of VAT in front of Director General.

© The Institute of Chartered Accountants of Nepal 125


CAP II Paper 7 Income Tax and VAT

Since, Mr. Krishna has filed an application for waiver within 30 days of due date of
payment of VAT of Chaitra 2071, Director General may waive additional duty. For this,
tax shall be paid after 30th day of 12th Baisakh 2072.

117. What shall be the fine and penalty chargeable under the following situations as per
VAT Act, 2052? (Dec 2013, 2×2=4)
a) Late payment of VAT amount
Answer
Fees @ 10% per annum on payable VAT and interest @ 15% per annum on payable
VAT.

b) Tax plate not kept/misplaced


Answer
Rs. 1,000 every time

118. Describe the circumstances beyond the control under the Value Added Tax Act, 2052.
(June 2013, 10 Marks& Dec 2011, 5 Marks)
Answer
Director General may waive additional fees @10% per annum applicable on payable VAT,
in case when a person fails to make payment of VAT by due date.

In case of any of the following circumstances, the circumstances are treated as


circumstances beyond control:
Due date for
Period to which the due
filing Other formalities,
Conditions date is extended (i.e. tax
Waiver if any
payment due date)
Application
In case of incapacity of Within 7 days after the
a person as a result of person recovers from
illness illness
30 days of
In case the person
expiry of due Within 7 days of the
responsible to pay tax
date for completion of such death
has to perform death
payment of ritual A
ritual
tax recommendation
In case the woman
Within 35 days of giving letter of
responsible to pay tax
birth to a child concerned local
gives birth to a child
level must be
In case of death,
submitted
insanity or absconding 35 days of
Within 7 days of receiving
of person responsible happening of
such application
to pay tax and the such event
rightful beneficiary or

© The Institute of Chartered Accountants of Nepal 126


Compilation of Suggested Answers Value Added Tax

legal representative
In case of failure to
reach Tax Office as a
result of blockage of Within 7 days after the
road due to flood, road is resumed
landslide, snowfall or
similar other reasons
In case of failure to A
reach tax office due to recommendation
complete blockage of 30 days of letter of
means of expiry of due concerned local
The next day when the
transportation date for level where the
transportation is resumed
payment of complete
tax blockage of road
has happened
must be submitted
In case of creation of A
circumstances beyond recommendation
control as a result of Within 30 days of the letter of
fire, earthquake or happening of such event concerned local
similar other natural level must be
disaster submitted
Due Date for application for Waiver of Fees (Rule 36)

The concerned taxpayer shall file an application for waiver of fees within 30 days of the
expiry of due date of payment of tax in relation to the period to which the Circumstances
beyond control is related.

Where it is related to absconding or death of a person, the application must be filed within
35 days of happening of such event.

119. Mention the penalties prescribed under Value Added Tax Act, 2052 on the following
infringements: (June 2012, 5 Marks)
a) Failure to issue a tax invoice and in case of transportation of goods out of the area
specified by Inland Revenue Department, worth more than Rs. 10,000 without
accompanied with a tax invoice.
Answer
Rs. 10,000 every time

b) Registration as mentioned in section 10(1) and (2) related infringement.


Answer
Rs. 20,000 every time

© The Institute of Chartered Accountants of Nepal 127


CAP II Paper 7 Income Tax and VAT

c) Failure to display the tax board as per VAT Act/Rules.


Answer
Rs. 1,000 every time

d) An unregistered person issuing an invoice or documents showing collection of tax.


Answer
100% of tax so collected or imprisonment up to Six months or both.

e) Tax officer finding that a taxpayer has reduced tax liability by making
infringement of any provisions of this act or rules there-under.
Answer
The tax officer may charge, following a procedure set by Inland Revenue Department, a
penalty up to 25% of the tax payable [section 29(ka1)]

Section 29(2) is also applicable in this case and the tax officer may charge a fine of
100% of the tax evaded amount.

120. What are the penalties mentioned in section 29 of the act on the following
infringements:
(June 2011, 5×1=5)
a) Registration as mentioned in section 10(1) and (2) related infringement.
Answer
Rs. 20,000 every time.

b) An unregistered person issuing an invoice or documents showing collection of tax.


Answer
100% of tax so collected or imprisonment up to Six months or both.

c) On obstruction in inspection by a tax officer.


Answer
Rs. 20,000 every time

d) On infringement of the VAT act and the rules.


Answer
Rs. 1,000 every time.

e) To erase and edit the data in software of approved computer Billing System.
Answer
Rs. 500,000

© The Institute of Chartered Accountants of Nepal 128


Compilation of Suggested Answers Value Added Tax

121. Ms. Amita is a proprietor of a VAT registered firm. For the month of Shrawan 20X-
67, she has collected VAT of Rs. 200,000 which she has to deposit within 25 th Bhadra.
But on Bhadra 10,20X-67, she has delivered a baby boy due to that she could not
deposit the VAT amount within Bhadra 25. The tax officer wants to levy additional
fee of 10% per annum on her. She has submitted an application to Director General
IRD on Bhadra 26, 20X-67 for waiver of additional fee levied by the tax officer as the
non-payment of VAT was due to the circumstances beyond her control. Can DG
waive that additional fee levy imposed by the tax officer? Give your opinion
(Dec 2010, 5 Marks)
Answer
Additional fees @ 10% p.a. on payable VAT obligation, as per Sec. 19 (2), can be waived
by Director General in the event of circumstances beyond control.

In case a woman taxpayer who gives birth to a child, a period of 35 days from the date of
delivery is treated as circumstances beyond control.

In order to obtain waiver, the concerned taxpayer shall file an application for waiver
within 30 days of the due date of payment of VAT in front of Director General.

Since, Ms. Amita has filed an application for waiver within 30 days of due date of payment
of VAT of Shrawan 20X-67, Director General may waive additional duty. For this, tax shall
be paid after 35th day of 10th Bhadra 20X-67.

122. Calculate the fine payable by a person registered under VAT on delayed in
submitting monthly VAT return by 5 days. (Dec 2009, 2 Marks, CA Inter)
Details of Monthly Sales are as under
Taxable - 7,000,000
Exempted - 3,000,000
Answer
If a person fails to submit Value Added Tax Return, higher of following amount shall be
imposed as penalty [u/s 29 (1) (Ja)]:
(a) 0.05% per day of payable VAT obligation, or
(b) Rs. 1,000 per VAT period

In the given question, the question lacks the information as to the payable VAT obligation
for the month, therefore, penalty cannot be computed precisely.

© The Institute of Chartered Accountants of Nepal 129


CAP II Paper 7 Income Tax and VAT

Short Notes Questions


123. Write Short Notes on following:
(a) Tax officer (Dec 2019, 2.5 Marks)
Answer
“Tax officer” means any Tax Officer or Chief Tax Officer or Chief Tax Administrator
appointed by the Government of Nepal and the term includes Section Officer or Director
or Deputy Director General of the Department or any other Officer designated by
Government of Nepal who is empowered to use the power of a tax officer in accordance
with the provisions of Value Added Tax Act, 2052.

(b) Electronic invoice (Dec 2019, 2.5 Marks)


Answer
A taxpayer may issue invoice electronically with prior approval from the Department.The
Department may publish a public notice to cause the taxpayers prescribed in the notice
for compulsory issuance of electronic invoice and order to link the electronic medium of
the taxpayer to Central Bill Monitoring System of Department.

Department shall develop and implement a procedure in relation to integrity and security
of software or equipment that issue electronic invoicing. The concerned Producer,
distributor and user shall comply such procedures.

(c) Refund of VAT paid on purchase by foreign tourist (Dec 2019, 2.5 Marks)
Answer
Where a foreign tourist on a visit to Nepal returning from Nepal via air route purchases
goods worth more than Rs. 25,000 and where such goods are accompanied by the tourist
at the time of departure from Nepal, tax paid on such goods shall be refunded as per the
procedures prescribed by the Department.
An amount to the extent of 3% of the refundable amount shall be deducted at the time of
refund as service fee.

(d) Suspension of transaction of registered person (Dec 2019, 2.5 Marks)


Answer
Where a registered person commits any offences as specified in Section 29 of Value
Added Tax Act, 2052 for two or more times, the Director General may order the tax
officer to suspend such person's place of transaction up to 7 days so that the transactions
are not carried out.

© The Institute of Chartered Accountants of Nepal 130


Compilation of Suggested Answers Value Added Tax

(e) Market Value (Dec 2018& June 2016, 2.5 Marks)


Answer
The market value of a supply of goods or services at a given date shall be the
consideration in money which the supply would generally fetch if supplied in similar
circumstances at that date, being a supply freely offered and made between persons who
are not related persons.
While determining the Market Value, Tax Officer shall determine the market value based
after studying the price and transaction of similar other registered seller dealing in
similar goods.
Where the market value cannot be determined by tax officer, the Director General shall
determine the market value on the basis of information received from the registered
suppliers dealing in similar goods.

(f) Collection of tax from other than registered person (Dec 2018, 2.5 Marks)
Answer
Value Added tax is collected from both registered and unregistered persons by a
registered person while supplying taxable goods or services as per Sec. 5 of Value Added
Tax Act, 2052. Under the circumstances as mentioned in Sec. 15 (3), any unregistered
supplier specified therein shall collect value added tax on transaction of taxable goods or
services.
An unregistered recipient of service from foreign party shall levy and collect VAT under
reverse charging principle, if the foreign party is not registered in Nepal.

Further, an unregistered owner of building, apartments, shopping complex built for


commercial purpose shall levy and collect VAT under reverse charging principle, if the
cost of such structures exceed Rs. 50 Lakhs and the builder of such structure is
unregistered person under Sec. 8 (3).

(g) Zero rated and VAT exempted goods/services (Dec 2018, 2.5 Marks)
Answer
Basis Zero Rate No VAT
Rate of VAT VAT is levied at Zero Percent VAT is not levied at all
VAT Invoice A VAT Invoice shall be issued as VAT Invoice is not required to be
per the format prescribed by VAT issued for transaction of such
regulation items
VAT credit Full VAT credit, which means there No VAT credit, which means, if
on purchases will not be any load of value added value added tax is paid at the time
(input) tax on output of purchase, such value added tax
will become part of output
Registration The person dealing in Zero rated The person dealing exclusively in
Requirement items shall be registered for VAT No VAT items is relieved from

© The Institute of Chartered Accountants of Nepal 131


CAP II Paper 7 Income Tax and VAT

purpose. registration formality


Other The person dealing in Zero Rated The person dealing exclusively in
formalities as Items shall observe other formality No VAT items is relieved from all
per the Act as per VAT Act, such as- formalities to be observed as per
maintenance of sales and purchase the Act
register duly certified from Tax
Officer, submission of VAT return,
etc.

(h) Valuation at market price by tax officer (Dec 2018, 2.5 Marks)
Answer
In the following circumstances, the tax officer uses market value to determine taxable
value of any goods or services:
i. When goods or services are exchanged in consideration of goods or services,
ii. When a person supplies goods or services obtaining partial consideration,
iii. When there is sufficient reason to prove that the supplier is doing under-invoicing,
iv. When a person cancels its registration, the goods and capital items on which it has
taken input tax credit,
v. When a person stops using a capital item on which it has taken input tax credit in
taxable transaction, or
vi. When a tax officer finds there is shortage of stock during physical inspection, and
appropriate legal procedure as per Rule 39Ka is not fulfilled or there is no
appropriate documentation to support the shortage

(i) Timing of goods supplied (June 2018, 2.5 Marks)


Answer
In case of supply of goods, earliest of following shall be time for supply under Sec. 6(2):
i. Date of issuance of invoice, or
ii. Date of possession of goods/ date of removal of goods by the recipient from the
supplier’s business place, or
iii. Date of receiving consideration by the supplier
In case of application of one or more conditions at once as mentioned above, the time of
supply shall be determined on subjective basis as prescribed by Director General.

(j) Person (June 2018 & Dec 2016, 2.5 Marks)


Answer
“Person” means any natural person, firm, company, association, institution, partnership
firm, cooperative, joint business, religious endowment, or fund; and the term also
includes any government body, any religious organization, charitable trust or similar

© The Institute of Chartered Accountants of Nepal 132


Compilation of Suggested Answers Value Added Tax

other bodies and branches or sub-branches, with or without any profit motive, involved
in Taxable Transactions.

(k) Proportionate credit (June 2018& June 2016, 2.5 Marks)


Answer
In case a person deals both in VAT attractive items and Exempted Items, the following
principle shall be applicable:
i. VAT paid on purchase that is specifically identifiable to VAT attractive items is
allowed at full while claiming Input VAT Credit.
ii. VAT paid on purchase that is specifically identifiable to Exempted Goods is not
eligible for any Tax Credit.
iii. VAT paid on such goods or services that are not directly identifiable to either of the
Exempted Items or Tax attracting Items shall be eligible for Proportionate Tax
Credit.
The proportion shall be determined on the basis of specific proportion of usage of input
for taxable and nontaxable output. If the proportion cannot be determined on the basis of
direct relation between taxable and nontaxable output, the credit shall be claimed in the
proportion of Taxable Sales & Exempted Sales of the period. The VAT attributable in the
proportion of Taxable Sales to Total Sales during the period shall be eligible for Input
Tax Credit, whereas that attributable to Exempted Sales is not allowed for set off.

(l) VAT refund to diplomats (June 2018, 2.5 Marks)


Answer
The following diplomats or persons entitled to diplomatic privileges are eligible to make
an application for refund:
i. VAT paid in Nepal by diplomatic persons privileged on a reciprocal basis from
Ministry of Foreign Affairs, or
ii. VAT paid in Nepal by person having diplomatic privileges engaged in Regional or
International Organization or missions.
These persons shall be entitled to refund of VAT paid by them to the extent of their own
consumption.
The application for refund shall be made directly to Inland Revenue Department, along
with the recommendation letter from Ministry of Foreign Affairs.
The tax shall not be refunded for a purchase of less than Rs. 10,000 at a time.
The refund application shall be filed within three years of such transactions.

© The Institute of Chartered Accountants of Nepal 133


CAP II Paper 7 Income Tax and VAT

(m)Circumstances beyond Control (Dec 2017, 2.5 Marks)


Answer
The following circumstances shall be deemed as Circumstances beyond Control:
i. In case of incapacity of person required to pay tax due to falling ill, up-to 7- days of
the date of recovery,
ii. In case the person required to pay tax is in death ritual; up to 7- days of the end of
the such ritual,
iii. In case a woman required to pay tax delivers a child, up to 35 days of the date of
delivery,
iv. In case the person required to pay tax dies or becomes insane or disappears and his/
her heir or guardian submits an application within 35- days of the date of such
incident, up to 7 days of the receipt of application,
v. In circumstances where the person required to pay tax has not been able to come to
the tax office because of the closure of the path due to floods, landslides or other
similar reasons, up to 7 days of the opening of the road,
vi. In cases where person cannot come due to transport stoppage, up to the next day of
the end of such stoppage.
vii. In circumstances beyond control created by the act of god-like fire, earth-quake,
within 30- days from such calamity.
Where additional time limit is required to be requested due to circumstances beyond
control referred to in clauses (i) , (ii) , (iii) , (iv) , (v) , and (vii), the recommendation of
the concerned rural municipality or urban municipality shall be submitted.
While requesting for an additional time –limit due to the circumstance referred to in
clause (vi), the recommendation of the rural municipality or urban municipality
concerned with the place or stoppage of means of transport has taken place, shall be
submitted.

(n) Purchase of Under Invoiced Goods (Dec 2017, 2.5 Marks)


Answer
Where the tax officer suspects there is under-invoicing in case of supply of goods or
services, he has two options as follows:
i. Taxable value shall be re-determined based on market value and VAT shall be
assessed by tax officer, and
ii. buy or cause to buy such goods at the price mentioned in the invoice

Where a person shows the sales of goods at value lower than the prevailing market price
through an under-invoicing scheme, the tax officer may suspend the further transaction
of similar remaining stock of goods and may buy or cause to buy such goods at the price
mentioned in the invoice.

© The Institute of Chartered Accountants of Nepal 134


Compilation of Suggested Answers Value Added Tax

Where a person maintaining such stock of goods deny to sell such goods at under-
invoiced price, the tax officer may seize such goods and compute the value of goods
taking the base of under-invoiced price and make payment to the person as and when the
person desires to receive it.
The goods acquired or cause to be acquired as above may be sold or cause to be sold at
such price and process as determined by the Director General.

(o) Taxable Value for Imported Goods (Dec 2017, 2.5 Marks)
Answer
Taxable value for the purpose of import shall be landed cost up-to border (cost paid to
vendor, transit cost or other cost) plus duty charged by the Customs Officer except VAT,
which is Transaction Value determined for the purpose of Customs Duty u/s 13, 14, and
15 of Customs Act plus Customs Duty plus Excise Duty plus any other duty applicable at
customs frontier.

(p) Tax Period (Dec 2017&Dec 2016, 2.5 Marks)


Answer
The tax period, as per VAT law, are as follows:
i. Monthly Tax Period (Every Nepalese Calendar Month as Tax Period)
Every registered person, other than those who opt for bimonthly or tri-annual tax
period as per the regulation shall follow monthly tax period for VAT accounting.
ii. Tri-annual Tax period
The following registered persons are eligible to choose trimester tax period:
a. Brick Industry
b. Newspaper publication house
c. Newspaper and electronic publication and transmission house
d. Hotel
e. Tourism Business
f. Cinema Hall
g. Transport business
These businesses may make an application in front of tax officer to keep their VAT
accounting on trimester basis. Where such application is filed, tax officer must cause
to use trimester tax period for VAT purpose.
It is upon the tax officer to decide the beginning and end of trimester tax period.
iii. Different tax period
In case of a registered person that has maintained its accounts using software cannot
generate the required report in Nepalese Calendar system, it may file an application
to Tax Officer to adopt different tax period with same tax period. If the Tax Office
thinks it appropriate, the person may be granted a different tax period.
iv. First Tax period

© The Institute of Chartered Accountants of Nepal 135


CAP II Paper 7 Income Tax and VAT

First tax period of a registered person shall be period of day of registration to end of
concerned tax period.
Readers should note that this is not a separate tax period, but a provision to clarify
that the tax period may begin at the middle of any month but shall always end in the
end of month. This also clarifies that the tax periods are uniform.
v. No-tax period
In case where any registered person applying cancellation of registration is not
prompted with the decision of tax officer within three months from the date of filing of
application, the person is not required to file any tax return after the elapse of such
three months.

(q) Reverse Charge (Dec 2016, 2.5 Marks)


Answer
As per Value Added Tax Act, 2052, there are two specific circumstances when VAT is
levied under reverse charging principle:
i. Import of Service [Sec. 8 (2)]
Any person, whether registered or not, in Nepal receiving service from person outside
Nepal (the person who is not registered for VAT purpose in Nepal) shall pay VAT for
that service.
VAT is levied on taxable value determined by applying Sec. 12 of the Act.
VAT shall be collected by the service recipient at such time, which is earlier of
following:
(a) When the service is rendered by the service provider, or
(b) When the payment in respect of service is made
When the service recipient pays VAT under this provision in tax office, the tax so paid
can be claimed as input tax credit even without any VAT invoice, subject to other
provisions of the Act.

ii. Construction by awarding contract to unregistered person [Sec. 8 (3)]


Any person (registered or not) in Nepal engaged in construction of buildings,
apartments, shopping malls or similar constructions for commercial purpose, the cost
of which is more than Rs. 5 millions shall collect VAT on the construction cost from
themselves and deposit it to Revenue Authority in case the construction work is not
carried out through a registered person.

For this purpose, Commercial Purpose means construction of building, apartment,


shopping complex or similar other structures prescribed by IRD for the purpose of
sales or using such asset by accounting the asset as current or fixed asset.

(r) Conditions for zero rate of Tax (Dec 2016, 2.5 Marks)

© The Institute of Chartered Accountants of Nepal 136


Compilation of Suggested Answers Value Added Tax

Answer
The conditions for Zero rate of Tax is as follows:
i. Goods exported from Nepal: Where the supply of goods is proved as follows:
a) Goods exported outside Nepal; or
b) Goods shipped as stores on an international flight, the destination of which is
outside Nepal; or
c) Goods put on board in an international flight, the destination of which is outside
Nepal, for retail sale or supply or consumption;
ii. Services to be supplied to Persons outside Nepal:
a) A supply of services by a person residing in Nepal to a person outside Nepal that
has no business transaction, business representative or legally recognized agent
in Nepal.
b) A supply of goods or services by a person who is residing and is registered in
Nepal to a person who is residing outside Nepal.
iii. Goods or services imported by a person or mission enjoying diplomatic privileges
and a person serving in a diplomatic mission enjoying tariff/duty privileges, on the
recommendation of the Ministry of Foreign affairs, Government of Nepal.
iv. Where any previous treaty or agreement provides for the sales tax exemption on
imports, and local purchase is made from the registered taxpayers, on the
recommendation of the concerned project, the facility of zero rates shall be provided
on such supplies, so long as the treaty or agreement is in effect.
v. Sales of the raw materials and goods manufactured to industries established in
Special Economic Zones pursuant to the laws in force.
vi. Where, on recommendation of the Alternative Energy Promotion Centre, the battery
used in the equipment generating energy from the solar power is produced by any
domestic industry and is to be supplied by that industry, the facility of the zero rate
shall be provided to that industry on recommendation of Alternative Energy
Promotion Center and in accordance with the procedure specified by the Department.
vii. Where any machinery, equipment, tools and their spare parts , penstock pipes, or iron
sheets used in making thereof required for hydropower projects are produced by any
domestic industry and are to be supplied by that industry, the facility of zero rate
shall be provided to that industry on that transaction, on the recommendation of the
Alternative Energy Promotion Centre, in the case of the project that is operated with
the approval of that center, and on the recommendation of the Department of
Electricity Development, in the case of the one other than that operated with the
approval of Alternative Energy Promotion Center in accordance with the procedures
specified by the Department.
viii. The Value Added Tax paid on raw materials used for the manufacturing of
painting, handicrafts, carving and similar other handicrafts by a industrialist in
Cottage and Small Scale Industry within Nepal are exported through an approved

© The Institute of Chartered Accountants of Nepal 137


CAP II Paper 7 Income Tax and VAT

Export Trading House of Nepal shall be refunded after fulfilling the procedures
specified by the Inland Revenue Department.
ix. The Value Added Tax paid on the import or local purchase of scooters used by
People with Disabilities shall, if such scooters are registered in their name in the
Office of the Transport Management, be refunded on the recommendation of the
Ministry of Women, Children and Social Welfare, or the Chief District Officer of the
concerned district, and in accordance with the procedures as specified by the
Department. If such goods are sold to any persons other than the People with
disabilities, the refunded Value Added Tax shall be recovered.
x. In case any machinery, equipment and construction materials as per approved master
list required for the purpose of operation of projects under bilateral or multilateral
treaty that have been concluded with prior approval of Ministry of Finance in terms
of tax exemption and that are manufactured by domestic industries, if supplied to
projects directly or through contractor of such projects, such supply can be treated as
zero rated supply as per the procedure prescribed by the Department.

(s) Temporary registration for VAT (June 2016, 2.5 Marks& Dec 2011, 5 Marks)
Answer
i. Temporary Registration as organizer or Participant as exhibitor of Fair,
Exhibition, etc. [sec. 10Ka]
Any fair, exhibition, etc. shall be organized by a registered person. Any person who
wants to take part as exhibitor in such event shall also be registered person, unless
the transaction is exempt from VAT.
In case any unregistered person desires to participate as exhibitor, there shall be
temporary registration specifically for the purpose of conducting taxable transaction
in such event.
Existing registered person can transfer goods for transaction to the place of
exhibition or fair.
Any unregistered person desiring to engage in any short-term taxable transactions of
goods or services at fair, show, demonstration, display, exhibition etc., shall make an
application along with the following:
(a) the recommendation from the organizer in the concerned Tax Office or Taxpayer’s
Service Office, and
(b) Proof of deposit of amount equal to 2% of estimated revenue from the event
The tax officer at concerned Inland Revenue Office or Taxpayers Service Office shall
issue a temporary registration certificate once the application for registration along
with deposit is received.

ii. Special Provisions related to VAT Registration of Joint Venture (Sec. 10Kha)

© The Institute of Chartered Accountants of Nepal 138


Compilation of Suggested Answers Value Added Tax

Where two or more persons form a Joint Venture dealing in taxable transaction for a
specific period of time, such co-venturers shall file an application for the purpose of
registration of Joint Venture at such Tax Office where one of the co-venturer is
registered.
The Tax Officer shall provide a Registration Certificate in the format as prescribed
where an application is filed pursuant to Sub-Section (1).
Joint Venture registered pursuant to Sub-Section (1) shall cancel its registration upon
the completion of period specified in the agreement forming such Joint Venture.
The co-venturers are jointly and severally responsible to discharge tax obligation of
Joint ventures in which they are involved.

(t) Transfer of business (June 2016, 2.5 Marks)


Answer
Under either of the following two conditions, value added tax is not be applicable on the
transfer of ownership of a business:
(i) When a registered person transfers its business to any other registered person; or
(ii) A business is transferred to any inheritor after the death of an owner.

Formalities to obtain the exemption


Where the transfer as above is agreed between the buyer/seller or agreed by the
inheritor, the parties shall fill up a form (Annex 4 of VAT regulation) and submit it to
concerned Inland Revenue Office/ Taxpayers’ Service Office within seven days of such
transfer.

Responsibility of Transferee to maintain safe custody of records as per Law and to bear
liabilities of Tax on assessments of Transactions conducted before such Transfer
In case there is transfer of ownership of business or industries that is registered or
required to be registered under law, the transferee shall bear the tax obligation in
relation to the period before the transfer was affected.

The transferee is also responsible to safely retain the accounts, books of accounts, and
documents in relation to the period before or after such transfer of industry or business
so transferred until such period as prescribed (i.e. six years from the end of tax period)

Where the transaction has been transferred, the rights, power and obligations of the
transferor shall be transferred to the transferee.

The Tax Officer may, summon both the parties related to transfer in his/her presence and
give them necessary instructions in relation to the obligations to be fulfilled by them

© The Institute of Chartered Accountants of Nepal 139


CAP II Paper 7 Income Tax and VAT

under the Act and these rules. It shall be the duty of both the parties to follow the
directions so given

(u) Input Tax Credit (Dec 2014, 5 Marks)


Answer
The input tax credit, as per Value Added Tax Act, 2052, follows invoice-credit method. In
respect of domestic purchase, there shall be proper value added tax invoice, not an
abbreviated tax invoice to claim input tax credit.

Input tax credit in respect of import, is allowed based on import documents and vouchers
proving payment of VAT at Customs Frontier.

Similarly, input tax credit in respect of VAT paid as per Sec . 8 (2), 12Ka and 15 (3), is
allowed based on vouchers proving payment of VAT.

There are four different types of Tax Credit:


(i) Full Tax Credit
Where an input is wholly used for the purpose of taxable output, tax paid for such
input can be claimed as input tax credit.

(ii) Proportionate Tax Credit


Where the use of input in production of output cannot be determined specifically, i.e.
the input has been used both for taxable output and non-taxable output, VAT paid on
such input to the extent input is used for taxable output can be claimed as input tax
credit.

The ratio of credit claim is taxable sales to total sales.

(iii)Partial Tax Credit


40% of VAT paid on purchase of automobiles used for administrative purpose by VAT
registered person dealing only in taxable items can be claimed as VAT credit.

(iv) No Tax Credit


There shall not be any VAT credit on input, in any of the following circumstances
except when the person's main business is dealing in items mentioned in (b) to (e)
below:
a. In case output (Intended or actual sales) is exempted from VAT as per Schedule 1,
VAT paid on purchase is not allowed for credit as per Sec. 5(3).
b. VAT paid on Entertainment expense is not allowed for credit
c. VAT paid for consumption of beverages (soft drink, juice or similar).

© The Institute of Chartered Accountants of Nepal 140


Compilation of Suggested Answers Value Added Tax

d. VAT paid for consumption of liquor items (beer, wine, whiskey, or similar).
e. VAT paid for consumption of diesel, petrol or LPG

(v) Applicability of VAT on import of goods and services (Dec 2011, 5 Marks)
Or
Levy of VAT on Import (Dec 2006, 3 Marks, CA Inter)
Answer
VAT is applicable on import of goods or services.

VAT on Import of Goods:


In case of import of goods, respective customs office shall collect VAT on taxable value.

Taxable value for the purpose of import shall be landed cost up-to border (cost paid to
vendor, transit cost or other cost) plus duty charged by the Customs Officer except VAT,
which is Transaction Value determined for the purpose of Customs Duty u/s 13, 14, and
15 of Customs Act plus Customs Duty plus Excise Duty plus any other duty applicable at
customs frontier.

VAT on Import of Service:


If a person imports service, VAT shall be paid under reverse-charging mechanism as per
Sec. 8 (2).
Any person, whether registered or not, in Nepal receiving service from person outside
Nepal (the person who is not registered for VAT purpose in Nepal) shall pay VAT for that
service.
VAT is levied on taxable value determined by applying Sec. 12 of the Act.
VAT shall be collected by the service recipient at such time, which is earlier of following:
(a) When the service is rendered by the service provider, or
(b) When the payment in respect of service is made
When the service recipient pays VAT under this provision in tax office, the tax so paid
can be claimed as input tax credit even without any VAT invoice, subject to other
provisions of the Act.

(w) Electronic Cash Register (Dec 2008, 1 Mark, CA Inter)


Answer
A taxpayer may issue invoice electronically with prior approval from the Department.The
Department may publish a public notice to cause the taxpayers prescribed in the notice
for compulsory issuance of electronic invoice and order to link the electronic medium of
the taxpayer to Central Bill Monitoring System of Department.

© The Institute of Chartered Accountants of Nepal 141


CAP II Paper 7 Income Tax and VAT

Department shall develop and implement a procedure in relation to integrity and security
of software or equipment that issue electronic invoicing. The concerned Producer,
distributor and user shall comply such procedures.

(x) Non-Filer of VAT Return (Dec 2008, 1 Mark, CA Inter)


Answer
As per sub-section (8) of Section 17 of VAT Act, 2052, in case of any person registered
under the VAT do not file VAT return for more than 6 months, then any carry forward
VAT credit may be withheld and the registration may be suspended.

(y) Unjust Enrichment (Dec 2008, 1 Mark, CA Inter)


Answer
In the case of indirect taxes, such as VAT, the burden of tax is borne ultimately by the
customer. The seller of the goods shifts the burden of tax to the consumer. Thus, it could
happen that having already transferred the burden of tax to the buyer, a seller of the
goods or service may lodge claim for refund of duty and refund if granted results in
double benefits to the seller. This is known as unjust enrichment. Any person who has
collected the tax in the nature of indirect taxes and shifted it to the customer, must be
paid to the government and if found refundable, it should be refunded to the customer
and the seller cannot retain it with him.

(z) VAT on sale of used goods (Dec 2006, 3 Marks, CA Inter)


Answer
Taxable value for persons dealing in secondhand or used materials is the difference of
sales amount and the cost of sales that includes the VAT paid on it.
Taxable value in case of used goods (for dealer of used goods:
Selling value of the goods- Cost of goods (where VAT paid on such costs is part of cost)
Further, if a registered person not a dealer of used goods sells used goods, the taxable
value shall be cash consideration received or receivable after due consideration of trade
discount and trade rebate.

© The Institute of Chartered Accountants of Nepal 142


Compilation of Suggested Answers Value Added Tax

True False Question


124. Answer the followings with reference to Value Added Tax Act/Rules, whether the
statements are correct or not, with reasons. (July 2015, 5×1=5)
a) While filing an appeal to Revenue Tribunal, the undisputed amount of the
assessed Tax due shall have to be deposited and fifty percent of the amount of the
Tax in dispute plus the amount of fine shall have to be deposited.
True

b) In case the person has not made the accounts available for inspection under
section 16 (1), the penalty amount is Rs. 5,000 for each time.
False, the fine is Rs. 20,000 every time.

c) The color of tax registration number plate for registered person shall be green, for
unregistered person doing Taxable transaction shall be yellow and for person
dealing in tax exempted goods and services shall be white.
False, the color of VAT plate shall be as follows:
Persons Color of Alphabet Color of background
VAT registered person White green
Person dealing in taxable transaction, Black Yellow
but not registered due to turnover
threshold
Person dealing in Tax Exempt Items Red White

d) Nepal Government is not required to collect Value Added Tax even if it engages in
supply of goods or services which attract VAT.
False, it has to collect VAT on supply of taxable goods or services as per Sec. 15 (3).

e) No Tax is to be levied for a supply of service by a person residing in Nepal to a


person outside Nepal who has no business transaction in Nepal.
Answer
False, the VAT rate is Zero percent. VAT shall be levied.

125. Explain whether the following statements are true or false with reasons:(Dec 2014, 5×1=5)
a) A person shall apply for the close of taxable transaction in the format prescribed
in Schedule 10 of VAT Regulation, 2053.
False, the form is prescribed in Schedule 11.

© The Institute of Chartered Accountants of Nepal 143


CAP II Paper 7 Income Tax and VAT

b) A person shall be imposed fine of Rs. 10,000 at each time in case of refusal for the
inspection of books of account by the Tax officer.
False, the penalty to be imposed is Rs. 20,000 every time.

c) The Tax paid by a diplomatic body or diplomat on the purchase of taxable goods
or services shall not be refunded if the amount purchase is less than Rs. 5,000 at
one time.
False, the amount of purchase shall not be less than Rs. 10,000.

d) Tax Officer may assess Tax as per section 20 if Kathmandu District Development
Coordination Committee collects the tax on taxable transactions and deposit in its
internal revenue account.
False, as per Sec. 15 (3), it is the duty of Kathmandu District Coordination Committee
to collect VAT on any taxable supplies even without VAT registration.

e) A foreign tourist can take refund of VAT on the purchase of goods amounting to
Rs. 15,000 or more.
False, the amount is more than Rs. 25,000.

126. State with reasons whether the following statements are true or false with reference to
Value Added Tax Act/Rules. (June 2014, 5×2=10)
a) 'No VAT' and 'Zero VAT' have the same meaning as VAT in both the cases is
zero.
False, it is due to the following reason:
Basis Zero Rate No VAT
Rate of VAT VAT is levied at Zero Percent VAT is not levied at all
VAT Invoice A VAT Invoice shall be issued VAT Invoice is not required to be
as per the format prescribed by issued for transaction of such items
VAT regulation
VAT credit Full VAT credit, which means No VAT credit, which means, if
on there will not be any load of value added tax is paid at the time of
purchases value added tax on output purchase, such value added tax will
(input) become part of output
Registration The person dealing in Zero The person dealing exclusively in No
Requirement rated items shall be registered VAT items is relieved from
for VAT purpose. registration formality
Other The person dealing in Zero The person dealing exclusively in No
formalities Rated Items shall observe other VAT items is relieved from all
as per the formality as per VAT Act, such formalities to be observed as per the
Act as, maintenance of sales or Act
purchase register duly certified

© The Institute of Chartered Accountants of Nepal 144


Compilation of Suggested Answers Value Added Tax

from Tax Officer, submission of


VAT return, etc.

b) In the case where there is provision of a contract for paying partly the value of
goods or services in more than one day on an installment basis, the time of supply
shall be the date of payment.
False, as per Sec. 6 (3), the time of supply shall be earlier of following:
• Date of payment, or
• Date of due date of installment

c) Roy & Co. is a VAT registered firm engaged in the business of importing
passengers' car and selling them in local market. The firm is claiming full input
tax credit on purchase of such cars.
True, since the person's main business is dealing in sales and purchase of cars.

d) If a taxpayer fails to submit tax return as per section 18, the penalty imposed is
Rs. 10,000 per month.
False, penalty to be imposed shall be higher of the following amount:
• 0.05% per day of VAT due, or
• Rs. 1,000 per tax period

e) A separate record for purchase and sale shall be maintained for the used goods
which have purchase price more than Rs. 20,000.
False, such record shall be maintained for purchase of any value.

127. Identify the following questions as True or False with reason:


(June 2012, 10 Marks, CA Inter)

a) In F.Y. 2068/69 Value Added Tax will be imposed and recovered as per Finance
Act, 2068.
True, as per Value Added Tax Act, 2052 as amended by Finance Act of each year.

b) VAT is not levied on tea leaf.


True.

© The Institute of Chartered Accountants of Nepal 145


CAP II Paper 7 Income Tax and VAT

c) To make administrative review, the applicant should place one third of disputed
amount as bank guarantee.
False, firstly the amount is one-fourth of disputed amount and 100% of undisputed
amount and secondly, the amount shall be deposited.

d) Rs. 500 thousand penalty will be charged if a person who has taken approval for
computer billing erases or alters the data using another software.
True.

e) Interest rate for the purpose of VAT is 10 p.c.


False, it is 15 percent.

128. State with reasons whether the followings are True or False.
(Dec 2011, 5×2=10, CA Inter)
a) Public corporation who does not transact dealing in vatable VAT attractive goods
and services is not required to be registered with VAT, but if it sells vatable VAT
attractive goods and services, it should collect the VAT and deposit into
government revenue account.
Answer
True, it is as per Sec. 15 (3).

b) If any assessee goes for administrative review, he should deposit one third of
disputed assessed amount as bank guarantee.
Answer
False, the amount is one-fourth of disputed tax amount and 100% of undisputed tax
amount in cash.

c) If any taxpayer does not pay the self declared VAT within 25 days of next month
of particular tax period, he should pay 15% p.a. as additional fee on unpaid VAT
amount.
Answer
False, the rate for additional fee is 10% p.a. as per Sec. 19 (2).

d) In Nepal, prevailing VAT Rate is 13 % and 0 %.


Answer
True, as per Sec. 7, 13% VAT is applied in domestic transaction of VAT attractive items
and 0% VAT is applicable on items listed in Schedule 2.

e) Abbreviated tax invoice is issued as format prescribed in annexure 6 of VAT


Rules, 2053.

© The Institute of Chartered Accountants of Nepal 146


Compilation of Suggested Answers Value Added Tax

Answer
True

129. State with reason whether the following statements are True or False.
(Dec 2010, 10 Marks)
a) A person shall apply for the registration of VAT in the format prescribed in
Schedule – 3 of VAT Regulation, 2053.
Answer
True, the format is prescribed in Schedule 1 of Value Added Tax Regulation, 2053.

b) A shopkeeper can issue abbreviated tax invoice for the transaction upto Rs.10,000.
Answer
True, only if the shopkeeper obtains approval from tax officer and he is a retailer.

c) A registered person shall preserve all the VAT records upto 6 years.
Answer
True

d) A separate record for purchase and sale shall be maintained for the used goods
which have purchase price more than Rs. 20,000.
Answer
False, such record shall be maintained for each goods.

e) Government organizations shall take VAT bill purchasing goods and services
above Rs. 5,000.
Answer
False, as per Rule 56 of Value Added Tax Regulation, 2053, the value is Rs. 20,000.

f) The rate of VAT for the import of goods and services will be same as VAT
applicable for the purchase of goods and services in Nepal.
Answer
True

g) In the case of purchase and sale of land and building, VAT is not applicable.
Answer
True, it is listed in Group 12 of Value Added Tax Act, 2052.

h) In the case of import of goods from USA, the place of transaction is USA.
Answer
False, as per Rule 15, place of supply is in customs frontier.

© The Institute of Chartered Accountants of Nepal 147


CAP II Paper 7 Income Tax and VAT

i) A foreign tourist can take refund of VAT on the purchase of goods amounting to
Rs. 15,000 or more.
Answer
False, the amount shall be more than Rs. 25,000 as per Sec. 25Ka of Value Added Tax
Act, 2052.

j) VAT will not applicable in cable car service.


Answer
False

130. What will be the consequences under VAT Act in the following case?
(June 2004, 5 Marks, CA Inter)
a) Mr. A fails to register his business within the stipulated time mentioned in Sec. 10
of the VAT Act.
Answer
Penalty under Sec. 29 shall be levied @ NRs. 20,000 every time.

b) B, registered in VAT issue the tax invoice and collect tax.


Answer
The person shall pay payable VAT and submit VAT return within 25 days of end of tax
period.

c) C, not registered in VAT, issue the tax invoice and collects tax.
Answer
Tax officer shall recover VAT so collected along with additional amount equal to 100%
of VAT so collected.

d) D, registered in VAT does not use certified purchase and sales books.
Answer
Penalty shall be imposed @ Rs. 10,000.

e) E, registered in VAT fails to file return and pay tax in due time.
Answer
For failure to file VAT return, penalty shall be imposed which is higher of following:
i. 0.05% per day of VAT payable, or
ii. Rs. 1,000 per tax period
For failure to pay VAT by the due date, penalty shall be imposed as follows:
i. Fees @ 10% p.a. on such payable VAT amount, and
ii. Interest @ 15% p.a. on such payable VAT amount

© The Institute of Chartered Accountants of Nepal 148


Compilation of Suggested Answers Value Added Tax

Court Procedure
131. In the course of VAT refund audit of M/S Wipro Ltd., an export-oriented
undertaking, the tax officer issued notice under section 20 of the Value Added Tax
Act, 2052 and Rule 29 of the Value Added Tax Regulation, 2053 disallowing certain
input tax credit on the ground that it is unrelated to the business. What are the
remedies available to the company? Advice (June 2012, 5 Marks)
Or
Describe the procedure of Administrative Review in Value Added Tax.
(June 2011, 5 Marks)
Answer
The company may apply for administrative review; the procedure is as per follows:

Application for Review & Time Limit [Sec. 31Ka (1)]


Every person who is dissatisfied with the tax officer’s decision of assessment of tax shall
make an application for administrative review at Inland Revenue Department within 30
days of receipt of the Final Assessment Order.

Extension of Time Limit for Application [Sec. 31Ka (2)]


In case the taxpayer cannot file application within 30 days of receipt of Final Assessment
Order, the tax payer may file an application for extension of time limit to file Review
Application within 7 days of expiry of such 30 days.

After receiving such extension application, the IRD may extend the deadline for Review
Application for additional 30 days counted from the expiry of first deadline of 30 days, i.e.
the maximum time limit to file a review application is 60 days from the date of receipt of
notice.

Formality to File Application [Sec. 31Ka (6) & 31Ka (8)]


The taxpayer shall deposit 100% of undisputed tax amount and one-fourth of disputed tax
amount before filing the review application. Such deposit shall not be refunded unless the
final verdict on the issue is declared.

In case the taxpayer has not submitted any VAT return in connection to the dispute
involved in Review Application, the taxpayer shall file such return prior to submission of
Review Application. (Rule 34)

Effect of Application [Sec. 31Ka (3)]


The Director General shall examine the documents and other proofs. In case the matters
raised by the taxpayer is true while examining documents and proofs, the DG shall nullify

© The Institute of Chartered Accountants of Nepal 149


CAP II Paper 7 Income Tax and VAT

the decision of the tax officer and order for reassessment to the concerned tax officer or
any other officer.

Time Limit to Decide on Review Application [Sec. 31Ka (4) & 31Ka (5)]
The Department must decide on Review application within 60 days of filing of Review
Application. In case of failure to do so, the taxpayer may deem the application be rejected
and choose to appeal in Revenue Tribunal.

© The Institute of Chartered Accountants of Nepal 150

You might also like